Vous êtes sur la page 1sur 322

TABLE OF CONTENTS FOR SYSTEMIC PATHOLOGY

NOTES
p rCDared bv Edward GoliaD. M.D.
Subject Pages

Hematology: RB C disorders 1- 11
Hemato logy: WBC disorders 11- 14
Hematology: Lympb nodel plasma cell 15- 18
Hematology: Coagulation disorders 19-26
Hematology: Immunohematology 27- 31
Cardiovascular patholo gy 32- 77
Respiratory pnthol og)' 78- 97
Gastrointestinal patho logy 98- 11 7
HepatobiJiary/pancreas patbology 11 8-134
IGdDcylL UTlMale patbology 135-154
FemalelBreast pathology 155-176
Endocrine pathology 177-203
MusculoskeletaVSoft tissue pathology 204-223
SkiD pathology 224- 238
Central nervous sys tem pathology 239- 263
Analomy USMLE lopie. 264-266
Mic.obiology USMLE lopie. 267-269
'Biochemistry USMLE lopic. 270-271
'Pharmacology USMLE lopic. 272- 276
Physiology USMLE topic. 277
Comprehensive pathology cxam/answers 1-48
GOLJAN HIGH YIELD NOTES FOR U ML E TEP I ~
NOle: This maltri.lb copy rich led. All rlChu rescrved. No pari of Ihls publitllion rna)' be repruduud in any
form or by any mttans. electronic or mecha nital, Inc:luding photocopy, ncordlnlit, or any i"formulo" stonge
aDd rtlrle,'.' sySlem. wtlhoul permission In \4'rl1lng fro m the pUbllJhu (Edward F. Coljan. M.D.)

Abbrcvlations common ly uud : AD • autosomal dominanl. AR - autosomal rcccsshc, COD -- cause


of death. DJ • diagnOSIS. 1 • most common, MCC = most common cause, raO: - panial pressure of
oxygen in arterial blood. PO - peripheral blood, R..x = ueatmcnt. aO: - oxygen saturatio n o r ancrial
blood. XR - sex-linked ree s ive

Systemic Pathology
RBC dlsonl ...
,,
Pmp~.nd blood ptmo_ .. U8MLE:
1. byperscgmcnlcd neutrophll- fo lateIB ,! deficiency
2. microcytic hypochromic ct lls-
A. RBes Ihot hove increased pallor can be assumed [0 be mic rocytic
D. iron deficiency
C. anemia of chronic disease (ACO)
D. thalassemia
E. sideroblDstic anemia
J. sickle cell sickle cell anemia not trait ha\'e sickle cells in the pcriphelllJ blood
~. targft cellJ-
A. bullseye
B. alcoholic liver disease
C. hemoglobinopathy
5. spbuocytes-
A. II al area of lIor
B. congenital spherocytosIS
C. ABO hemolytic disease of nc"-born
6. lear drop-
A. myelofibrosis
B. thlliassemias
7. Howell JoUy botly- spleen surgically removed or dysfun ctional spleen 8S in Hb S diseuse,
8. phatclel- small , red, Iln ucleate ce ll
9. Iymphocyt b!tlCk dot with a thin ri m of cytoplasm
10. Auer rod- myeloblast with immature nucleus and splinter-like structures in the cytoplasm
II . IIrypicallympbocYIC:-
A. big cell with abundant sl}' blue cytoplasm
B. infectious monolCMV/foxoplasmosis/viral hepatitis/phenytoin
12. eosipophU-
A. large ffii granules that do not cover the nucleus: same: color as RaCs
B. type I hypersensitivity
C. invasive helminthic infections
13. basophil-
A. large purple granules that do cover the nuc leus: granules darker than RBCs
B. mye loproliferat ive diseases
14. rouleau- RBCs wim stack orcoins effect: increased sedi mentation rale
15. scbulocytes-
A. fragmented RBCs seen in microangiopath ic hemolytic anemias
B. thrombotic thrombocytopenic purpura
C. hemolytic uremic syndrome
D. DIe
16. reticulocyte-
A. specia l supravita l slain: thin filamen ts represent RNA
8. increased in:
( 1) hemolytic anemias
(Z) 5- 7 day, ,tI" blood loss.
(3) after Rx of iron deficiency
(4) B I~/fo late deficiency
17. Heioz bodies-
A. spec ial sta in: large blue inclu sions with invo lvement of the RBC mcmbmne
B. G6PD deficiency
18. toane basophi lic stippli ng-
A. routine stain: pers istent ribosomes
B. Pb poisoning
iJIOIie marrow-pIcfllG"'-'·o"."'·O"'SMU:<
I. '!!'!PIoIIludc ..........-
A. all the cells appear big due to large.• immarure nuc lei: e.g .. giant band neulrophils
B. Bil/ foiale deficienc),
2. ~brwil- marrow is composed of fibrous tissue: large cells represent megakaryocytes
3. apJaltk: aaemJa- empty marrow with predominantly fat. and islands ofiymphocytes
4. aabipk Dlyekulla-- plasmablasts with bright blue c)1oplasm. eccentric nuclei. perinuclear
hal o
5. diIpd Ildeniblut-
A. Prussian blue slain for iron: ring of blue around the nucleus of II nonnobLllSt
B. indlcate a defect in heme synthesis: s ideroblastic anemias due to alcohol. pyridoxine.
Pb poisoning
Erythropoiesis:
1. dcfinition-
A. product ion of RBCs in the bone marrow
8. dependent on erythropoietin (EPO) synthesized in the kidneys
2. nimuli for EPO-
A. low Pa02: hypoxemia
B. anemia <7 gmIdL
C. left shifted oxygen dissociation curve (DOC)
3. peripberal blood (PB) reticulocytes-
A. manifestation of EPO stimulated marrow
B. relicu locytcs require 24 hours to become mature RBCs
C. reliculoc)'tes have RNA filaments: identified ",im supravita l stai ns

2
D. reticulocyte counl IJ the most cost effective method of dctcmnin lna whether the
marrow is reapondinl appropriately to anemia: coiled effecllve erylbropol Is
E. reticulocyte count i.s normally reported as a percentile: normal O.5- I.S%
F. redcalocYk cou.nl It (irs. cornc:trd (or the delr'ft of I nem ia (if PR'St:nt) wuh the:
roltowin fonnula:
el) COfftC"tcd re'l.lculocya.e count =- (patiC1l1 Hct , 4S) ~ micukxyte count. ~~ .. 5
rqnnenlS iii< nonnaI Hct
(1) example: patient's Hct - 15% ReticuJ~yte count -~. Cortftlcd count - J-"
(15145>9%-3%)
C. corncttd tOUI' >3% is touidCftd a good rftPOaH 10 aJlem ia
( I) blood los >7 cia)' (tel:.. marrow-S-7 cIa)"O respond)
(2) hanol).ic anemias
(3) Rx of iron dertcicncy with iron
(4) Rx ors" deficiency with Bu
eS) RJC of rolale deficiency with folate
H. correcud rellculocyte coual <2-/_ is considered II poor n' pnofie (something is
wrong in the bone marrow):
(I) Iron doficlency
(2) anemia or chronic: disease (ACO)
(3) B,,/fol.,. d.ficicncy
(4) aplastic: anemia
(3) blood loss < I week
ilU IDd .,.....p.o.. or uemlu;
I. B" lrolo •• dtll<le0<r- Slos.ltu
2. Bu ddiducy- posterior colum:nflateraJ corticospmal trlCl SIp
l. rnrt.lH 1. lo~rsDCe
4. d)'tpo..
S. kfa" OUIpUt r.Uare- decreased viscosity of blood lerA'en total pcriphenl resistance
6. kollooydtlo (.poo. oollt)-iroo ckflCiency

3
~ Classific:ativn or Ane mia on Ihe HaJis or Mean C nrpuKular Volume (Me V) ~
Microcytic Anemias (MeV <80 J.lIn)
..;, • Iron deficiency (most common)
. ' to ..... .Anemia orchronic di sease
• • Thnlllsscmia
• Sideroblnstic anemia (least common)
Macrocytic AnemiaJ (Mev > 100 .... m))
I'
• Folate deficiency (most cOl11mon)
• Bn: (cobalamin) deficiency
Normocytic Anemias (MeV SO- I00 ....m1)
-7 i)l.ftttO.ri/1\~ C¥i!I!<!o.J Rtf}' ('c.,* ;!>
,..m i,,/.1'fqttt ;. ../1";5 jl><('-
Corrected rtli enual < 2ltJa Corrected reli Dunl > % Corrected rdic coo t > J ./.

Intrinsic: ROC defeet (MAU) EdrinJic ROC delctl

Acute blood los$ «5- 7 days) • Membrane defects • Autoimmune hemolytic anemias
Early iron deliciency Congenilal spherocytosislcl liplOCytosis • MicfOllngiopRthic hemolytic anemia
Aplastic anemia Paroxysmal nocturnal hemoglobinuria (PNH) • Blood loss > I week
Anemia nf chronic disease • Abnonnal hemoglobins
Renal discase Sickle cell discase/varianls
• Deficient enzymes
Glucose 6 phosphate dehydrogenase deficiency (most common)
PyruvBle kinase deficic!1cy

I. Int rinsic hemolytic anemia: something is slruclurally nr biochemically wrong wilh Ihe RBC
2. [ltrinsic hemolylic anemia : RBC is siruciurally normal but somelhing outside the RBC is destroying it
Microcylic IInemll.l': ee Tab l ey~chcmatic5
fr IroDstudies:

Iron

JOO

200

100

A B C D
I. crum Iron-
bar A represcnu the Donnal
D. area an black is iron bound ( 0 transferrin: am)\\ wilh sohd lane
C. height of the bar is Ihe unnsfenin Ic\'cl' arro" w ith intcrrupted hne
D. iron concentnlltlDn is delcnnined b} stripping it orr of the transfcmn molecules \\ ith
acid' iron in bar A - 100
2. TID ls measurtd b)' adding Irnn 10 tbe palit.Dt JpKi mtD 10 order 10 fiU up alla\'1IiI:able
blndfD& tH 01 traDsferrin
A. Iron bound to all the uansfmin sites is then mctlSUJ'ed TlBC an bar A - 100
B. there IS an inverse relationship ofU'aDSfc:rrin with femlln sun In the bone marro\'
( I) dec reased ferriun Siores, increases II\cr synthesLs of tnan ferrm. tncrcues TIn
(%) IOCrta.scd ferritin storcs. dec.reases liver syntheslJ of lransfcrrin decreases TmC
3. Y. sa luration b II ca lcu lalion: Y. saturotion of Iron - ironfllBC :\ 100 · 100/300 x 100::
In. ~ I f
~. Sf'rum ferrilln - ~ sr fl~ b-..,+ I CON ;JtCf# y .\-
A. fcrrilin (storage fonn of iron) is found in oli lissues (panicu larl) In hepal ocyu~s) and in
Inltcrophnges (largest amount) in th e sp lecnlbone mnrrow: Ihere Is " Vl'l")' 501011 "mounl
of circ ulnling ferritin that close ly correitucs w\l h the amount of storage iron that is
aVAilAble for Hb synthesis
O. ferritin is lov. in Iron ddicienc), high in ACD. and high in iron overload diuase

J/ Iron dendenc - bar B


A.
B.
serum iron low' -20
TIBC high ' - 500 (iron store' .... decreased)
I C. ,. salufllion low: 4".
6. Hem ll of chronic dinan- bar C
scrum Iron 10\\ . -20
B. TIBC low: -200 (Iron stores are increosed in ACD)
C. ..~ Sltur&lIon 10"" 20%
D. "rum (emun is hIgh
. / 1' las RiA
T ron!lrerrin-Iron com plex /"
L-. ~ Transferrin (reused)
-------FI"!,.-..;;;:::::::",..;;;;;--------------- Normoblasl membrane
Normobl9.5t cytosol
0rrf~~ Iron I 7'hnluuem;a de/u/
Globi n cha in 'Ylllb.'i" ~",

Ribosome -+ ~~-~
Iron deficiency defeci
ACDdtf~ct

Milochondria Alcohol (mitochondria ;son) n, 13, 6, Y glohin Ch AiM


+

i
Sucdnyl CoA (TeA cycle) Heme --------'1k-+ Heme --+ Hemoglobin
+ lib A (2« 2m
Glydn. 0 '-' \ r<rrOC.+lat... FIb Ad2n2B)
. (Q~' , uc.)
n
':f.
1" AmlnoleVlllinic ocid sy nllmse
(r Ie I~g ~n2yme)
Iron Lead poisoning
+
lib F (2a 2y)

lflUlint id Proloporpbyrin Note: f heme - J.11 Al.A ~y ntbJt


A Amino C\fulinic acid (ALA) .! hcmc - t 6 AL.A s),n lhllS€!
~ Aruinolevulinic acid deh drase Lead nisonin

Porphohilinogen - - -•• UrollOrphobilinogen - - -... Coproporphobilinogen

TeA = tricarboxylic ~dd cycle


0 , = [lyrido1joe
ACD "" anemia n(chronic innammation
Microc\'tic Anemias
Anemia I'atb£weoesis Causes Clinical "~inilinl!3 Trea.tment laborntol"\' Tests
Iron deficiency • .l.. Hh syQthe~ • Blecd ing- • MC overall calise , • • Plummer~ Vinson • rerrous sulr- • Sernm iroo- ./..
sis- J. heme usually gastrointestinal (0 1) syndrome- * csophagen l nlel gluconate • TIIlC- t
• thrombocyf~ synthesis • Prematurity- 4 loss of iron ellch web, "achlorhydria,' gloss- • slools turn • % Saturalioo- J,
osis in chnmic • Decrease in day fetus is not in-utero. " blood ilis, " intestinal malabsorp- hlack • Serum ferritin- J.
cases iron stores t Inss frolll ph lebotomy lion • RDW-' i. * on ly
• microcytic liver synthes is • Ncwhorn- bleeding Meckel' s • Stages or development in microcytic anemia
and DonDO- of transfer- .6 mths-2 years of age- nulritiCJ- suecessinn- • no anemia:
eyfie c.ells- rin- t lolal nal : milk baby iron stores -+ ferritin -+ !
reason for i inm binding • ChiTd - hleeding Meckel 's iron. t TInC, ./.. % saturation.
RDW capacil Y: t • Pregn:mcy ~ • nel loss of 500 mg • anemia: nonnocytic-+ mi c~
Tine if iron supplements are not taken .• rocytic
lactation depletes iron
• Female <SO Yr.i- • menorrhagia:
> 80 mlloss/pcriod
• Male < 50 yrs- PUD: duodenal
ulcer Me
• MalefFemalc >50 yn;- 01 bleed:
RIO colon cancer
Anemia of • J.. lib ~ynthe- • Cb ronic innamm:dion- • micro- • Fib is rarely < 9 gm/dL • Rx underly~ • Serum iron- !
chmnic inn- sis- Fe blocked bial. • rhcunl310id arlhritis ing di~easc 'TlBC- ~
nllllllntifm in macroptJages • Mnlignancy- AC D is Me • % Saturation-l
(ACIJ) • i iron storcs anemia • Serum furitin- t
J. liver synthe- • nuw- nomlnl
sis of tranfert-
in- J. lola I iron
binding capac-
J!y; J. TIBC
Thol «(1 Dnd 11) • J. lib synthesis- AR diseascs . • a-thal- ' Asians,. Afri- .. alp-thol min- • fron stutJies-normal
.. o,-thal- .. J.
a -glnbin elulin synthesi$: gene delet- can-Americans or refJ lIin~ no in mild varieties
.. lear drop ions on eh romostllne 16 . • P. 5. and y globin chain .. Il-thal- .. Africnn-Ameri· R• .. RUC c:oun1- • norm-
('cll" ,yuIlle."", are nonna l.• HbA (2a1211), HbA, (2a12S). enn, .. Halialls.' Greeks .. ~(!yere that alIa t. . very charne-
.. target ce.1I5 and Il bf' (2aJ2y) arc all equAlly reduced .. Ttrrn~ - minor, inlemlediit. requirt'.5 lrans f- Icristic
.. I-gene delelinn-!;:ilent carritr nod major connole degree of u~io ns- danger .. RDW- normal
• 2-gene dch:liQn-" a-11m! lrail." ."rrican-American s(werity of iron o"e rl o~d .. Ub phoresi!- • i!:
and Asian Iypes art: slightly difrerenl .. Se~rre Ihnl does nQt mani- thai minor: nonnal, •
• 3 ~gene deleti(ln- • I-Ibl-l disease: • Hbli == 4 a~glo- ft.'" al birtb- • r-g lobin a.-thai (3~gell~ ~ell or
bin chai us• • hemolytic anemia chai ns in I-IbF arc present.. • HbH d;'j"a",: IlbH t, •
• 4-gene dtlelion- • Hb Ran's disease: +I-I b Ban's = p-globin chains have nOI a-thai {4-geoc del} or
4 y- globin chains, t sponlaneolls abortions (hydrops been synlhes ized yet lib Bart's disease: I-Ib
[cia lis) .• t incidence of chori(lcarcinol1l3 Bart 's t . . . B-Ihal min~
. 13-11101- • J. p.glohill chain synthesis on ch(o~ Qf: l' in HbA, and IlbF
S~lme II, • point mUlalilln produces n..splicing defecL
• a., 6, and y globin cbsio! are syn lhesi'le- • T
synthesis of HbA~•• no p-globin chain synth-
esis is designAted pO, • some j3-gl11bin chAi n synthesis
is designated p'
• i1-lhol minor (fJ/P')- mild proh..-clive effect against
falcipaniOl moloria .
• f}- Ihol nt~jor (Cooley's an ~m ia, pUlp· )- hcmo--
lytic anemia
• lluctlitary pcrsislence of 11bF- · variant ('tr p~lha l
nolec! in Afflcan~Ame r ica n s• • absenll) and S chain
synthesis: n Md 'T chRins unile 10 form IlbF. which
I persists inlo adu lt life, • no anemia
- --
Slrlerohln!llc •! IIh synlhcsi5 Causes: • ri CA ror paine a nd plas- • Dlnll!rt'aprol- • OTood Ph levels- •
anemia: lA.ad • I'b dena tures • C hild-' eftting old Pb-based Itr nAL t . • best screen and
(Pb) poi. onin~ en7)'rnc!- • .&rm: paint • lA.d I:ollc- • abdomina l • [ UTA (:onfimllfory lest
l'hclalasc: hinds Fe • Adull- • working in Of living pit in•• 11b visible 011 x-rays • Urine 6-A1.A- t
• coa r1't bas ... lu proIClJ1Qrphyrin , .....-. bonet)' f.ctory, • poIttl}' • rerfphen l neuropalhy- • o ROC PEJ'- t
phitic:: stippling • ~1..6 dc:hxdmsc: paintel) • moonshine: JIb radia~ wrist drop: radial nerve, • • Serum fron - t
• rine,r d s ld er- c OIl\'er1~ Ii-ami no- I01S c law hand: ulnar nerve oTIDC- ~
oltltuh: In bon e levulinie acid 10 • Enet phalopalhy- • eerelr • ~. S.lurallon- t
ma rrow porphobilinogen • • ral edema. • convulsions • Suum (crri l in- f
ribonuclease: can- • Luning d isabilitirs • Done l - rays- depos-
nOI degrade ribos- • Bonc- Pb deposits in ils in epiphyses
om.. coarr.e bas- cpiphy5is: visible on x-rays
ophilic stippl ing • PrnJJma l rtul lubtdar FEP - free erythrocyte
• Iron oyrrload- - addosb- - Pb denatures protoporphyrin
Fe enlers miroc ho- proxi mullubule cells:
ndrin and CRnnot IItphrotox ic. - Fancon i
exit , • t Fe stores 5yndrome
J. li ver synthesis o r
transferrin: ~ TID'"'
Siderob listie · ! li b sya the-sis- • Alcoh oli~m- - Mi,&, - • DCf'C! nds on the CIll5e of • Depends on • Strum Iroo- t
anC!mitlJ derect in heme a lcohol is a mitochondriAl thC! a nemia t he ca use of Ihe o TIBC- !
• rin~ , ider- synlhcsu poison: iron en ·lers anemia • -I. Saluralion- f
Iobla.•', in bune • Iro n onrload in mitochondria but Cl lUlOl exit • Strum ferriti n- T
marn,,"
- mi loc hon-dril • Pyrid01ine (8,) ddicJency-
• isoniazid Rx in TB .• n, is
corae·tor for ALA synthase in
• Bone marrow- ring-
cd sideroblasts required
for Ox
fi rst heme synthesis reaction;
glycine + suce- iny l e oA 6
aminolcvulinic acid
• Pb poiJonlng
• MyelodYliplaJlit
syndrom ts- pre lcukemia
I ~ndtornes
1. Iron overload dlnases- bar 0
A. examples:
( I) sldtroblastic anemias: Pb poisoning
(2) hemochromatosis
(3) hemos derosis
B. k-rum iron high: -200
C. nac k»"" -200 (iron Slo~ an' high); note bo", the acrum Iron IfId TlBC are the.s.amt
O. % saturation high: J00'-
£. serum fcmtin is high
<r UP( l[ _riot:
I. PICA fDr de lIId ar- iron deficiency
2. 1-r - old "i .~ IN% Hb
or
A. hered itary persistence Hgb F (\'anant of jl.lhalasscmll)
O. Ibsent ~ and 5 chain synlhcsis with I concomitant increase In t-Igb F (0. and y chain
I)'Tlthesis)
HbA nnd HbA 1 are absent
D. hl8h HbF prevents cli nical symptoms of thAlasst:mlu
E. unifoml distribution ofH bF in all RBCs
f . compatible wllh life
Dirfcrf.ad. t rea tu ru or tbe mic rocyd c anemias: bolded areas I'tp~sent let} ditTertnlial POints
Fe dencltacy ACD a, Il-T hai mlaor Iderobludc (pb polson)
Mev Lo Low Low Low
Sc.rum iron Lo" Low ormal Higb
nBC lJie b 1.0" ' onnal Low
~Saturauon Lo Low onna! If,,,,
Smlm fmilln Low Ifi'" Nonna! If,,,,
RDW " lab onna! onnal ormaJ
RBe counl I.ow Low " IC b Low
Rae FEP Ifi'" Ifi'" Normal 111gb (pb po,son)
Hb clt:C[ro. onnal Nonnal a.- tbJal: normt l 1'10"",,1
il-.hal: lib I
aad f
Marrow Iron A bitDt Nonnal Hlgb
Misccllnlh:ouJ Ferrit in best Hb electro. Coarse blUophilic
gold stn ndnrd slip l,ling ringed
sld eroblasts
FEP - free tl')'tl\roc:)'1e protoporphyrin. MCV -= mean corpuscular \'olume. nBC · tOll1 iron binding
capacity. ROW - red blood «II disuibulion width

'lacl'OC'ylic anemias: k-e Nutrition nOles, see Tables and schematic


". ilH ror nabJorption or Iron. (olate. Bu :
I. duodcaum -ltof1
A. Billroth II (diml stomacb 10 jejunum. duodenum bhnd loop) anociaoed with iron
Mficiency
B. mJabsorplion syndromc:s decrea2 iron ~absorphon ' c.a . cehac disease
2. Jej u. um- folate
A. malabsorption syndromes decrease reabsorption

5
,
Methionine Chi N'-Mdb)'l-flU. (plasma)

) M.. hyllnn.r...... ~ )( (1 uribe. FIG III In only rola le dencle.ncy)

(i in folale Dnd 8 11 defklency) ltomoc),Jlelne Melhyl-Chl FII, (r'plenlJhed by .erlne Ind FIGh,)

DI.",,,,rolile redoet_ (1•• lbllooi by


_",II'.,,............rha)

~ln. ~f5e..t.te.. Olllydrorotale(OIhtb.e.d form) _ ,/ • _,
" I ~ ~kDVe.£.'" . 7J
Lx..: <;:iCrovorum faclor repleni.sbu ---i.~ Nt.!. mdh lene FU. r~~
~~
(~ko~n) Thymidyl.I ••ynlh..... V I ~

IIsed ID
p - . ( nNA
,. 1 ~,:;
~
!!
r _ __ _V_I_ln_n_,I_n_D
_ ,_,_in- '-p_rO-'Ic.,I_o_na_'_e_"_'e_l_a_b_ol_i'_m_ _->.".l'CI\ cyde
(reduced lIynlhul, III huch Ulllilld r.,h,Ce deOceney)

Propionyl eoA -..Meth),lmalon),1 e oA --i.~ Succiny. eoA

t "rille me'h),huftlonlc acid (llu deOcierlcy only)


Methyl-FII. - mdl,ylle.lnhydrnfoIAle
FIGln - fnnuAmhluglululI1lc acid
FU4 ~ lefrah),clrofolole
Chl - cobalamin (011)
Mel'I)'I-C bl - melhylcoba lArnill (0 11)
dUMP - ,Ieo"lynridillc mOllopluuphale
Macrocvtic: A.oemw
A.Demia PathOftnesis Causes CUalcal. Fiaw n.. TreSt'me nt Laboratory Tfiis
Fula te • Defect i. • Alcobolistn- • Mee, .. nat b«r • Glossitis • FnUt Ilcid in • Serum (olaf&-- +
• RO folate-- . J., •
deJicie-ocy DI'IA .yatk.. ;" drinking type: rieh in fola te ,)harmllcnlogic
• Uyp.negm. • Nuclear mat- • 'Poor diet: • missing in goat's doses- correctS" hCSl resl of overall
ented neotm- uratlon ddect-- milk.. . diet poor in vegetables hemIttologic supply or fohne.
pbU. - >5 nuc- Duoleus remains • Pregnanc),naC:latioa- .1' findings in Bu • Serum hqm,oc:ysle.-
lcarl_ large uliliutiOD orrotlu; • only 3--4 deITcie:nc.y but Ine:-. t , · also t in
• Paaeytope- • Nodded in mth supply in liver nOI the lleurolo- Bll, defICiency, .... vessel
0;. proprioulf: • Ones-. block dibvdmrotale giea l disra;e damngc:"and lhronlbosis
mellbolism- no reductase: • melhotteltlte• • trim- • lInne FJ.GIU (form i-
CNS disease elhoprim, • block in1estiogl coniu- Dbglulamic acid)- t
gase ami convcaiQO g[llQl}!&IM!!1" - Serum LD~ t
mates (0 mopgglulamalg:: pheuy. dcslruc:lion ofRBCs
toin, • block jejunal reabsorption
of monoglulamate: • binh control
pills, • alcohol
• - MJlllllbsorpli~n- celiac: disease
. .. t fJ'uraovtr or cc:Us- bigh grade •
malilUlancv; "-2, leukemia
8 11 deficiency • Defett in • Pemlc:Tous anemia (FA) • • PAp.tient .. ..sallow. + • BI1 tnjections • Serum Bu.- J.
DNA .ynthesl. MCC of Bn deficieney.. . autoim- waxy complexion • • elderly • Serum homocyste-
• M above for • Proprionate mtlne destruction ot ~ariet·81 (ells women of Scandinavian 100- 1'

.,
rolare ddicitn- rndaboll.5 m- •
odd chain rauy
acid metabol-
In body/rundus: tno intrinsic fav
lor• • no ac.id
origin
• GloultiJ- • smooth, sore
~ODgUe.· atrophy ofpapiJIa
• Urine m.etby fmafo-
nie:. acid- .1' .• only in
• Pun vegan diet- Bn only in ani- D11 def,icieney.. . hom-
ism: ma l producls • Autoimmune: disease- PA oe.ysteinelmethylmalo--
proprionyl CoA • Terminal ilea' disclJe- " site of MCC Bll deficiency nie acid most sensitive
~ jrllrinsic. faclor-B11complex reab- • Atrophic gastritis of body tests
methylmalonyl sorplion• • Crohn 's disease and fundus- " anliparietnl I LS$ilDi"Chst- • 'cor-
GoA • Fish tapeworm cellllF ant ibodies destroy reclion of B11nlalab·
~ B" • Chronic. panc:reatitis-· cannOI parietal cells, .. achlorhydria; sorption by adding
suc.c"inyl eoA cleave off R faclor from R faclor- predisposes 10 cancer i~(rmsjc:. rac.l.ol' to orally
[rCA cyole) •
all complex, • R faclor from sal· • Malabsorption-loss of administered mdi03.ct-
iva protects BI1 fr.om acid deslrue· stem cells in Gllrac;;1 ive 812; PA,. corree--
tion • Ne.uroJogica.1 d'!ease- • J. tion fo llowing antibia-
• Bacterial overgrowth- • diver- vibrotory sensation and prQp- tics: bncterinl overgr-
tieular disease, • J. perislalsis from rioceplion: posterior column ()wth ... correction fol-
autonomic neuropaLtiy... replace-- disease, • UMN signs: hneral lowing. pancrealic edr-
ment of muscle with collagcn: corticospinal tract disease.. .. acts: chronic panCreD{-
progr:essive systemic. sclerosis dementia itis• • 110 corrcction
, with any of th~ above;
probable lenninal ile:d
discase
B. birth control pill s and a lcohol decreases monoglutamnte reabsorption
3. Bil- lenninal ileum
A~ Crohn's di sease/resection oflermina l ileum decreases reabsorption
B. also d~crease in bile salt reabsorption~ malabsorption
Vitamin BI% (cobalamin) a nd folate:
I. soune! of B u -
A. meat/dairy products
B. not present in vegetables/fruits
2. sources of folale-
A. beer: cannot become folate deficienl by drinking too much beer
B. fruits/vegetables
C. grains
3. (unctions of balh (olale and BI!"'"" DNA synthesis
4. runetlon! of Bu only-
A. propionate me tobolism: propionyl-CoA-+ methylmalonyl-CoA - Bit succ inyl-CoA
n. succinyl eoA used in:
(1) TeA cyc le as 0 substrate fo r gluconeogenesis
(2) heme synthesis
5. funelion of rolale 001),- I-carbon transfers to other intermediates for the synthesis of:
A. amino acids
O. purines
c. pyrimidines
6. Bil melabolbm-
A. BI ~ requires intnnslc factor (IF) for reabsorption In the tenninal ileum: [f is
synthesized in parietal cells located in the body/fundus
B. Bu is initially bound to R factor in saliva: R factor prevents gastric acid destruction of
BII
C. pancreatic enzymes cleave off R factor: this allows Bu 10 bind 10 LF
D. BIl,IF complex is reabsorbed in the tennina1 ileum
E. all is bound to transcobalamin in the plasma: delivered to metabolically active cells or
stored in the liver (6-9 y supply)
7. folale mtlabolls m-
A. folate is in a polyg lutamate form in vegetableslgrnins~
( I) converted into monoglutamates in GI by intestinal conj ugase: enzyme is
inh ibited by phenytoi n-+
(2) monoglutamate is reabsorbed in jejunum: reabsorption bloc ked by alcohol and
birth control pills ~
(3) fo late circ ulates in blood as methyltetrahydrofo lalc
B. 3-4 mths supp ly of folate in liver: dietary deficie ncy common in alcoholics
8. BIl/folate In DNA syntbesis (see st-bematier
A. 8 1: removes meLbyl group from methylterrahydrofolale
(I) methyl transferred to homocysteine: homocysteine converted into methionine
(2) melhylletrahydrofoiate becomes tetrahydrofolate (lliF)
(3) THF conven:ed into Ns,lo_ methylene tetrahydrofolate-t
(4) NS. ICI~methylene THF + thymidylal'e synthetase + deoxyuridine monophosphale
dihydro[ol.,. (DAF)
(5) DHF (oxidized form ofTHF) + deoxythymidine monophosphate (used for DNA
synthesis)-+

6
(6) DHF is converted by dihydrofolale reductase back into THF (reduced fo nn )
9. causes of 8.: de fici ency-
A. pernicio us a nemia (PA):
( I) MCC
(2) autoimmune destructio n of parietal cells
(3) autoantibodies against [F and parietal cells
(4) loss of [F leads to Bn deficiency
(5) ac hlo rhydri a: increases serum gastrin levels
(6) chro nic atrophic gastritis of body/fundus: predisposition to gastric
adenocarcinoma
B. pure vegan diet
C. chronic pancreatitis: cannot cleave off R factor
D. fish tapeworm
E. bacterial overgrowth in small bowel:
(I) bacteria destroy Bll-I.F comp lex
(%) also destroy bi le sa lts leading to malabsorpt ion of ratS
F. tenni nal ileal disease (normal site for B l~- I'F reabsorption): e.g., C rohn's disease
10. ca uses of folale defici ency-
A, alcoholis m:
(1) MCC of folate deficienc),
(%) not deficient in folate in a beer drinking alcoholic
B. poor diet: elderly
c. pregnancyllaClalion: uses up folate
O. disseminated cancer: uses up folate
E. phenytoin: blocks intestina l conjugase
F. birth contTol pills/a lcoho l: block uptake of monoglutamates in jej unum
G. inhibition of dihydrofol8te reductase:
( 1) methotrexate
(2) trimethoprim
11. SIS of Bil/folate defici ency- ,
A. macrocyt'ic anemia: delayed nuclear maturation of hematopoietic cell s
B. glossitis
C. diarrhea :
( I) malabsorption
(2) affects duplicat ion of stem cells in the intestine
12 . SIS uniqu e to Bll defici ency-
A. subacute combined degeneration
B. posterior column demye lination
( 1) lack proprioception
(2) lack vibratory sensatio n
(3) positive Romberg's test
C. Illleml conicospinal tract demyelination
( 1) positive Babinski
(2) upper motor neuron disease
(3) spasticity
D. dementia
13. SIS uniq ue to PA-
A. Bchlorhydria
B. chronic atrophic gastrit is body/fu ndus

7
C. gastric adenocarcinoma
D. autoantibodies
E. correction of Shilling's test with additio n of IF
14. lab findings in Bu/folate defitiency-
A. large nucleated cells with immature nuclear fearures: cells are called megalob lastic
B. panc)1o penia: megaloblastic cells in the bone: marrow are destroyed by macTophages
before they enter the sinusoids
C. hypersegment'ed neutrophils:
(I> >S nuclear lobes
(2) eKcelJent marker for 8 11/ folate deficiency
D. increased plasma homocysteine levels: no methyl group to tran sfer to homocysteine to
produce methionine if either BIl or fololc defi cient
15. lab finding s unique to folate deficiency-
A. increased formiminoglutamic acid (FlGlu): usually metabo lized by THF
n. decreased serum fo late
C. decreased RBe folate : best test for folate deficiency
16. lab findings unique to BI! deficiency-
A. decreased serum B I Z
B. increased urine methyl ma lonic acid : bui ld.up mcthylmalo nyl-CoA proximal to the
block
C. increased propionates (odd chained fllny acids) cause demyelination and CNS find ings
17. WWI.'. lest.-
A. non·radioactive 8 12 is first gi,fen to bind to all available transcobalamin in the
periphtnll blood:
( I) prevents any reabsorbed radioactive B I ~ from binding to transcobalamin
(2) forces it to be: eKcreted in the urine.
8. rad ioactive 8 12 given by mouth followed by 24·h urine for % radioacth·c B I~
reabsorbed: no radioactive Bu in 24·h urine confirms BI2 deficiency.
C. if corrected with addition of IF to ora l radioactive 8 11 : pati ent has pernicious anem ia
D. if corrected after antibio tic therapy: patient has bacterial overgrowth
E. if corrected with addition of pancreatic extract followed by intake oral radioactive B r~ :
patient has chronic pancreatitis
18. IU of folate deficiency- folate in phannacoiogic doses CnD correct 8 12 deficienc), but nOI
the ne-urologic abnonna lities
19. Rx of Bil deficienc),- 1M BI2

8
t11" Differential compariso n of pernic.ious anemia (PA), other causes of Bil deficiency a nd fol ate
deficiencv :
Pernicious anemia OtherBn Folate defi cie.ncy
deficiencies
MCV Increased Increased Increased
Pancytopenia Present Present Present
Bvpent2meoted oeutrophiLs Present Present Present
Mee:aJoblutic bone marrow Present Present Present
Autoandbodles _gainstIF and Present Absent Absent
I oarietal cells
Chronic atrophic gastritis Present Absent Absent
I (bod. aDd rundu.f
Aclllorbvdria Present Absent Absent
I Serum 2Dlna levels Increased Nonna! Normal
Risk of stom acb Increased None None
adenocarcinoma
Plasma homocysteine lncreased Increased lncreased
Uri ne methylmalonic acid Increased l"creoBed Normol
Urine forml.noglutamic. acid Nonnal Nonnal Increased
(FIGLu)
Neurol02lul diselUe Present Prescnt None
Schilling's test Corrected by add· Not corrected Not used
ing intrinsic factor byadd.
ing intrinsic
factor

ormocytic
1. anemias: Sie~T~'~bl~"~""~~;;2~~~~: congenital spherocytosis: do splenectomy
2. autosplenectomy or functional asplenia
3. JoUy-~ indicates absent or dysfunctiooal spleen
4. precipitates hemolysis in G6PD deficiency
S. 1_ WtDfuBlotY-Hc::iitiililii:post.-4iapsou.o(sulfur'" dru&, used in leprosy) ud: ~..ilJ-
G6PD deficiency
6. •• wHllil(lili CiIIs-:li aLwith osteom erUH: Sa/mo"ella species
c:r Summary chart of m echanis ms of hemolysis in tbe bemolytic anemlas:
J. eltrnvascular-
A. macrophage removal and destruction of RBCs:
(I ) abnonnally shaped RBCs (e.g., spherocyte5, sickle cells)
(2) 19G and or C3 coating RBCs (e.g., autoimmune hemolytic anemia)
B. increase in unconjugated bilirubin
2. intravBScular-
A. intravascular dest:nJction ofRBCs
B. RBCs damaged by:
(J) calcified aon ic valve in aomc stenosis (MCC)
(2) fibrin clOll in DIC
(3) platelet thrombi in thrombotic thrombocytopenic purpura and hemolytic uremic
syndrome

9
Normontic anemIas: Non-hemol tic wi th corrected retic ulocvte counl < 1%
Anemia Palhonnesls Ca uses CliolCllI Flndin .. T reatm ent Labora lory T ....
Acute/chronic • Norm l ll1 b1 • G I bleed- · MCe, • upper 01 • r .,III • • UIIC.,C- OP .J. and • Pa.cked ROC • No specific lab test!
blood I.., II t t with lnllil l Me site: duodenal > gaslric ulcer pulse. l' when moving from trans fusion 1£ • General- • order a
acute blood • Lower Gl - • diverticu losis, • supine to upright position symptomatlt: slool guaiac, + dipstick
loSJ- • plasma angiodysplasia, • colon cancer: • Lower left rib tead em- of urine for blood
deficit replaced • >50 years old. t right-sided as- ruplurrd spl...
first and uncov- cancer > left-sided cancer • FMldares- pelvic fractures
crs ROC deficit. • MisuUl neou- • splenic rupt- followed by femoral fractures
• infusion of ure: trauma, + ruptured aortic ane~ are associaled with signifi-
0.9% noons) urysm• • bleeding Meckel's divert- cant internal bleeding
sa line uncovers iculum: child
• • Exacerbation of a agina-
ROC defICit im- anemia reduces blood visco-
mediately sill'. which J. lotal peripheral
t
resislance and cardiac
output and demand for 0 :

Aplastic • Stem cell • Idiopathic MCC • Oleetiing- thrombocylo- • <SO Y'" old- • C 8 C- pancytopenia
a,neml. disease- suppr~ • Genetic.- Fanconi 's syndrome penis consider bone • Done ma rrow-
essioR of Iriline- • Cbemical- benzene: t risk fo r • Fenr- infection assoeill· marrow trons· hypocoll ulnr: lymphoid
• Pancy lope. agc stein ce ll in acute leukemia led with neutropenia plantation cells st ill present
nla marrow • Drugs- • Me known cause, • • Fatigue- anemia • Other IU- ·
chloramphenicol • • phenylbuta- antithymocyte,
zone • antilympho-
• lor.. 600- NANBlHCV eyle globu lin, •
• Radlatioa cyclosporine
• Anemia- • aplast'ic crisis duc 10
hemolytic anemia: • HbSS dise-
ase., . congenilBl spherocytosis,
t often associated with parvovirus
infection
• Pu re RBC apwia- • no bone
marrow RB precursors• • asso-
ciations: • parvovirus ir1Volving
RBC stem cell, t thvmoma
Renal disease • ...t. Eryt.hropoi- • Diabetes mellitus MCC of • Anemia not present untJI • Erythropoie- • Serum BUN/ereati-
• Burr cells elin chronic renal disease creatinine clearance <40 II. nine-- ratio < I S/ I" c.g.,
mUmlo 80/8 ~ 1011
Malignancy .ACDis MC • Lung cancer- MCC of death Anemia is multifadoriaJ • Packed RUe .Oone marrow 81- if
• Leukoeryt- anemia due to cancer in men and women and may be due to more transfu!IIloDlI if metastasis is suspected
h ruhlasUe • Colon cancer- second Me ean- tban one cause sympt'Omatic
smca_r - imma- cer and cancer killer in menl worn-
lure- neutrophil en
precursors and • Other anemias- · metastasis 10
nucleated marrow witl1 replacement ofhe111-
RBCs if marr- atopoietie elements: myelophthi-
ow metaslasis sic anemia.. • marrow suppression
has occurred by chemotherapy, • blood loss:
colon cancer.. . autoimmune hemo-
lytic anemia: chronic.lymphocy-tic
leukemia" • folate deficiency: • t
milization • • poor diet
Normocvtic Anemias: Hemolvtic With Corrected retic ulocvte count > J%
Anemia Pathol!'-ene.sis Ca uses Clinical Findin~s Treatment Laboratory Tests
Congenital • IIcmolytic • AD disease-- Northern European • Triad or anemia, spleno- • Splcncdomy- • Osmotic fragility- •
sp herocytosis ancmia- extra- c;t(lraction mecaty. ja •• dl«- .. unconj - • spherocytes spherocytes begin hemo-
• Spherocytcs- vascu lar macro- • Acquired cause! of spberocytes ugated hyperbilirubinemia, • still in peri ph - lysis at 0.65%: t fragil -
dense RBCs: no phage phagocy- in hemolytic a nemia- • ABO i incidence of black. calc ium eral blood, * ity • • nonnal RilCs be-
central pallor losis and desfr- hCI1101ylic disease o f newborns, • bi lirubinate ga llsloues must give Pneu- gin hemo lysis at 0.50%
uctio n autoi mm une hemo lytic anemia movax and Hib • Total bilirubin- t
• Defect in vaccines unconjugated bilirubin
Silect rin- •
~ Rile mombr-
ane, .. J,. ATPI
I!'lucose
I'arolysma l • FleRlolytic • Abdomi nall)ain • Episodic 11t:~moglohinuria - • Symptomatic • Sugar water lest- •
nocturnal anemia- " intr- • Signs o f iron deliciency- loss noted in firsl AM void initial screen for PNH , •
hemoglobin- avascular hemo- of iron in the urine from hemoglo- • Vessellhrombosis- • due sugar water enhances
uria (PNH) lysis biliuria 10 platelet destruction and complement attachment
• Pancyrope- • Deficiency o r re lease of thromboxane A1• • to RBCs... RBC hema l-
nia decay nccelera- hegatic vein thrombosis: ysis
ting ractor in Budd-Chiari syndrome • Acidified se rum tes'
cell membra- • Acute leukemia (Ilam's test)- confinn-
nes- ' OAF atory lest far PNH
nom1allyaceele- • Low haptoglobin
rates degrada- • Urine bemosiderin -
lion o f c.a mple- present
ment on ce ll
membranes .•
affects all cells
- ·
"' � -�
- -- -
--- --- - - - -- ---.
tli�
''U 1''';
:'-1'''; C
s�I��ii-�" pa:�g'e ..Tlf d·.L ;'.
,,

.
. �
Sickle cell screeD-- ...
,
-- -
em
,wo
• ,

trait/disease •Hemolytic anemia­ •Two mlli... problems- '* chronic hemolytic anemia•• Vaso* positive, .. sodium meta.:
extravascular macrophage occlusive disease: organ damage bisulfite reduces O2 ten-
• Sickle cells destruction of RBCs • Newborns with Hb SS do not sickle-- - highHbF levels in RBCs 5ion-.- induces sickling­
in disease only • Mutation- point mutation prevents sickling. * t in j3-g1ohin chain synthesis asHbF bes1 screening test
• nowell..JoUy with substitution of valine for concentration J. causes sickling within 6-9 mtlls • Hb clectrophoresis- •

bodi.. -� glutamic acid in 6th position • Vasoocdusive dise.ase-.,.. sickled cells block microvascula-ture, - gold standard test: documw
splenic dysrun� of (l-globin chain nonsickled cells stick to endothelium cots amount ofHbS. UbF.
CltQJJ • Factors inducing sickling- • Vasooedusivc erises- • Me clinical manifestation and cause of otherHbs, • sickle disease:
• Target celL� concentration ofHbS in tHbS90-95%, tnoHbA, f

organ damage,. bands and reet(dactylitis): .occurs in 6-9 mth old
• Trait has a RBe >60%: most important. infant, • bone infarcts with painfuJ swelling ofhandslfeet, - acute 5-1(}Q/OHhF• • sickle trail:
normal smear • reduced O2 ten-sion•• high chestsyndrome: • fever. flung infiltrate, tcllest pain, f hypoxemia. •HbS 4045%••HbA 55-
altitude•• low 02 tens-ion in .MC COl) in adults 600/",
renal medulla.-. presence of • RBC aplastic crisis- .. triggered by a parvovirus infection... • Prenatal Dx,- • in normal
otherHbs combined with anemia without reticulocytosis: tno RBes produced. • no marrow genes, MSTIJ endonuclease
HbS: e.g.,HbC. '" volume Rae precursors cleaves gene on I3-globin
depletion: tHbS • Splenic sequestration crisis- .. splenomegaly from sickle cells chain at same point mutation
concentration in RBCs•• filling the cords, • anemia and reticulocytosis: RBO; are produced in site involved in sickle cell
acidosis: rightMshifts 0, marrow trailidisease: t converts a
dissociation curve-+ Ch • Complitations- • aseptic necrosis femoral head: f bone infarction. segment 1.35 kb long into
released to tissuc-)o t amount tnew, reactive bone formation, • X-rays reveal t density. tMRI two fraginents, f I fragment
deoxygenated � sickling. is 1.15 kb long and I frag­
most sensitive lest, .. pigmentedgallstones: due to i extravascular
'"HbF inhibits sickling: high hemolysis,. �lenectomy: tnon-functioning spleen by 2-3 yrs ment is 02 kb long. .. MSTII
02 affinity prevents endonuclease cannot cleave
of age, .splenomegaly is present the first decade before spleen is
deoxygenation this site if a point mutation
autosplencct.omized by infarction•• osteomvelitis: Salmonella Mec
PrevaJence- oil HhSS is present, ., inHbAS. I chr�

(>75%) not Staphylococcus aureus ,. t susceptibilityto infections:
afflicts 1:600 Affican­ omosome bas gene defect
Streptococcus pnellnumiae Me COD in children, • trail ofTers
Americans: Me hemoglob­ and the other one is normal:
protection against falcip-arum malaria," priapism; continual painful
inopathy, '" 8% of African­ • chromosome with defect
erection," micro-hematuria: toccurs in both trait and disease, tlow
Ameri-cans have HbAS (trait) has no cleavage of 1.35 kb
O2 tension in renal medulla induces sickling--+ microinfarctions-.­
Autosomal recessive- segment. tnomlal chromo­
hema-turial dilution/concentration defects/potential for renal papil­

A S !tome splits into J rragmeni


lary necrosis. t African-American with microhematuria should be
A AA AS that is 1 .
screened for sickleHb
S AS 55 groent that is O.2kb long., •
251%. disease (SS) inHb SS. both chromosom­
25% normal (AA) es have the gene defect 2
50% trait (AS) fmgments that are I J5kb
long and no smaller fragm­
ents
G6PD • Hemolytic: • SXR disease • Acute hemolytit episo-- • Avoidance • Adive bemolysis- •
deficienC)' anemia-. mac- • weak varianl oc:curs in aes-. hemoglobinuria, • of drugs Heinz body preparation is
• IIcinz bod- rophage removal African- AmericallS- .. defec- back pain known to used, • enzyme assay may
i"-. clumps ofRBCs, • intm- tive enzyme with.1 half -life • Protcdion against ralcip- precipital"e
hemolysis
be no""al owing to
hemolysis of RBes lacking
of denatured vascular hemoly- only present in older RBe!, • tess num malaria
Hb identified s is from damaged severe hemolysis • Splenect- the enzyme while remaining
with supravital Rae membrane, • Mediterranean variant- • omy is of no RBGs have Ihe enzyme
slain, • best • lack of G6PD: decreased synlhesis and defective benefit • Asymptomatic (non-
marker in acutc t reduction in gl- enzymc present in all ROeS hemolytic) state- an
hemolytic stage Ulathion. (GSH). (young and o ld ), • more severe enzyme assay is used
• -Bite c:ells- t aSH neutral- hemolysis \r.
macrophage ius peroxide • J.nreclion is tbe Me precipit~
removal of • Infedion and ant or bemolytic crisis
port ion of tlle drugs • Ox:idant drugs precipitate
membrane precipUate hemolytic episode5- * lag phase
hemolysis of2-3 d.. antibioti~ : sulfa
~gs O~.
anti . mal a ri a ls :~
-l't~"" nitro drugs, •
!M!iui~ot chloroquine, • anti-
pyretics: acetanil id, not aspirin or
acetaminophen
• Fava beans- precipitate hemol-
I vsis in Mediterranean variant
Autoimmune • Uemo'ytic • :Warm (lgG) type- • SLE: • fever • Avoidance • Direct Coombs ' test- ·
hemolytic ane- Ilne mia- • e.'I(tr- MCC .• drugs: t pen ici llin and • Jaundice- unconjugated of positive in the major-ity of
mia (AUlA) avascula r hemol- methyldopa , t both arc type II type when extravascuh" tra nsfusioDs cases, • detects IgG andior
• Spherocy les- ysis: t mac roph- hyperse nsitivity reaclions hemolysis is present ir possible- C) on surf-ace ofRB Cs
due 10 mac ro- age re moval of • Cold UgM) type • infection: • Hepalo!lplenomegaly in most cases, • .....
Indirect Coombs'-
~ ..... •
phage remova l RDes wilh IgG • Mycoplasma pneumm ,iae wilh • Generalized lymph aden- the trans fu sed oft en positive, • detects
ofRB C memb- andlor C3 coa ling anti-I antibodies, t infectious opaehy- bolh cold and cells are anti bod ies in semm:
rane RBes, twann mononucleosis wilh anli-i anti - wann Iypes destroyed antibody screen
IYpe of AU-lA, • bodies, • C u. .... . drugs: • Quinid- • Warm
intravascular he- ine, • isoniazid, t immunocomp-
l.b .<"12- ey pe--
1I101),sis: tl gM Ie)!; or type III hypersensitivity Jj~.J.L,: k. corticosteroid

:;J::.b
0
0

type of hemo ly- 5


sis, • cold type
, ifTu J J", • Cold type-
.J1!!}
';'

ofA IHA ;1:$ dl alkylating


;, ~ c..--';':'~ d.d.F IV" agents
I!) ))~ : j,J.,,?- • Drug-

d; J;.f."j
~~,..ju - .k.
/J.
ind uced
type-- DC
drug
~-....
• fVy-
~ globulin in
resistant eas-
es- blocks
macr-ophage
receDlors
Microangiopat .I-Iemoly tic • AoMic stenosi~ - Mec • Va rie~ with tbe uRderly- • Varie~ • Serum haptoglobin- •
-hic: hemolytic anemia- • intra- • Prosthetic heart valves ing disease st.ate with .1., • haptoglobin-lib
anemia vascu lar hemoly- .DIC und erlying com pl exes phagocytosed by
• Schislocytes- sis, • ROes dam- • Thrombotic: thrombocytope- dis-ease state macrophages
fragmented aged by: t calci- nic purpura • Urine hemosiderin- •
RBe, urn in stenotic • Flemolytic uremic: syndrome t, • renal tubular cells break
valves, t fihrin • Runn er's anemia in long- Hb down into hemosiderin
clots in Die, distance runners • Iron studies- iron
t platelet plugs in • n ELLP syndrome-. pre- deficien cy may be present
thrombotic Ihra- eclamps ia: • hemolytic anem ia. ow ing to the loss of I-Ib in
mbocytopenic t elevated transaminases, tlow the urine
pllrpuralhemoly- platelets •
tic uremic synd-
rome
C. haptoalobin levels decreased: haptoglobin binds to free Hb in plasma and is removed
""6)i"Ti1aCrophages
D. hemoglobinuria: often leads to iron de,f]ciency
E . hemosiderinurla
Anemia Type of hemolvsis
Congenita l spherocYtosis Extravascular
ConRenita l elliptocytosis Extravascular
SickJe cell anemia/variants Extravascular (m?jo~l. intravascular (minor)
Glucose 6·pbo~phate dehyd rogenase Intravascular (major), extravascular (minor)
deficiency
Autoimmune hemolytic anemia (warm JgG Extravascular
I tvoe)
Autoimmune hemolytic anemia (cold IgM Intravascular
l type)
MicroangLop~tbic anemia Intravascu lar

A 19-year·old African American woman presents with fatigue and exercise intolerance. Sbe has a
hist'Ory of menorrhagia. A esc reveals a mild microcytic anemia, a low normal WBC count, and a
normal platelet count. A corrected reticulocyte count is <2%. Which of the following sets of
laboratory data best represents th e h
I ,emato ' fi nd'lOgs In
ioglc IS a rlent?
. tho
Serum Iron TmC 0/0 Saturation RBCCo unl Ferritin
A. nonnal normal normal high normal
J3. low low low low hiol>
C. low hi.h low low low
D. hi. h low hiol> normal hi.h
E. normal nonnal 'nonnal low [lormal
C
.
A = thalassemia, 8 ""'" ACD, D Iron overload, E - blood loss
r:r Which of the following characterizes pernicious anemia father than 8 12 deficiency from other
causes? •
A. Increased plasma homocysteine levels
B. increased urine methylmalonic acid
C. Decreased vibratory sensal-ion
D. Hypersegmented neutrophils
E. High serum gastrin levels
E (other differences include achlorhydria, autoantibodies against intrins.ic factor and parietal cells,
correction of Schilling's test with intrinsic factor)
ar A 28-year-old man has a motorcycle accident outside the emergency room. PhYSical ex.am in the
emergence room reveals a weak. thready pulse, cold, clammy skin. and a blood pressure of 60/40
mm Hg. An open right femoral fracture is present' as well as tenderness over the left lower ribs.
Which of the following would you expect in this patient if a blood sample was drawn pdor to
insertion of an IV line with 0,9% normal saline? SELECT 3
A. Nonnal hemoglobin and hematocrit
B. Normal effective arterial blood volume
C. Increased central venous hydrostatic pressure
D. Decreased mixed venous oxygen coment
E. Decreased pulmonary capillary wedge pressure
A, D, E

10
qro A 4 year o ld African American child with sic kle cell anemia presents w ith a high fever and nuchal
rigidi ty. Sc leral icterus and hepatosplenomegaly are present. A CSC rcveaJs a moderate ly severe
normocytic anemia and a WBC coum of21 ,OOO c el1 s/~ with left shit. A urinalysi s is normal. A
spinal ta p reveals turbidity, increased protein, and neulrophils. You would expect a gram stain of
C5 F nnd a blood culture to reveal.. .
A. gram pos itive d ip lococci
B. gram negative d iplococci
C. gram positive cocci
D. gram positive rods
E. gram negative coccobaci ll i
A (Streptococcus p"eumo" iae)
,I '1)
wac disorders: see Tables
or Test fo r infectio us mo nonucleosis : heterophile antibody. whic h detects antibodies against horse
RBCs
<if" C lasslfi cu tlOn 0 r1 po vc~· t b c nua :
S ubtype of Polyeyth- Ca usesIDiscussion
emia
Absolute polycythemia : C hronic obstructive pulmo nary disease: I .MeC in tbis ca tegory- low I
Appropriate: stimulus of Pa0 1
tissue hypoxia High altitude residents : 1.pereentnge or oxygen is 21 % bu. the I
atmospherie press ure is decreased- low l)a0 2
Cyanotic coneenilal beart disease: I.rieht 10 len shunt- low PaOl
Hypoventilation syndromes (sleep a pnea , Pickwickian sy ndrom e) :
l .respJralOry a cidosis- low PaO,
Len sbifted oxygen dissociation curve: l.decreased release of oxygen to
tissue- A. CO go isoning : (I) low SaOl . (2) nomlal PaO" 8.
methemoo: lobi nemia: (I) low 5 a0 1. (2) no rmal PaD!. C. increased Hb F
Absolute polycythemi a: Polvc~'themia rubra vera: no rm a l PaO ~SaO . and low ervthropoietiD
Inappropriale- no Renal d iso rd ers: l.increased e rythropoictin- A. nomlal Pa~/5 a 021 B.
hypox ic stimulus no rmal Pa0 2'5aO%~ ( 1) rena l adenocarcinomnlWilm 's tumor. (2) CystS
Endocrine disorders : C usbing' s syndrome- corti sol and 17-ketosteroids
stimulate erythropoiesis
Miscellaneous: l.uterine leiomyomas- compressIo n or ureters, 2.
cerebellar bemangiobla.5toma- part of von Hippel disease : increased
incidence of renal adenoc arc inoma, 3.bcpliloce llula r eurcinoma- ectopic
erythropo ietin production
Rela tive polycythemia: S mo ker's polycythemia: unes plained effects of carbo n monoxide in
due to decreased plasma s moke increases RBC m ass- nicotine decreases plasma vo lume b)
volume, normal RBC vasoconstriction
mass
S tress polycytbemia (Gaisbock's syndro me) : individuals under constant
stress-=-typicallv males with hypenension and o besitv
Volume deplelion : vo lume contraction hemoconcentrales RBCs I

II
Oenie:n Ouanlita'h'c White Blood C ell Di~ ordet5
Leukocyte Disonler Pa thogenesis Causes Clinica l Findlng!l Laboratory .Findings
C haraderistics
Absolute neutro- •t Prod uc.tio n- • Inrcdio ns- • MCC, • e.g., ac ute append· • Va riable • C BC with. 100 WBe difTer-
phllic leukocytosis infection icilis e ntial co uol
• Ahsolu te co unt • J. Ad heslon to • Sterile InnammalJon- • myocnrdial in rare· • Bone ma r row uam- usually
>7000 cellsl uL endolhelium - • tion • • bloodlbile in body ca_vities nol necessary
• Len shift- > I 0% corticosteroids, • • Physiologic- stress • Blood euflure5- if inrection is
band neulrophils or epinephrine, • • Myeloprolife nlUve disease- • polycythemia suspected
any neutrophil you· lithium rubrs vera, • chronic myelogenous leukem ia
nger than a band in • l' Release rrom • Meta bolic disease- . uremia, • ketoac idosis
the peripheral blood marrow post- • l:Iema tologlc disease- acute hemolytic
• Toxic granula- mitotic pool- disease
t
tion- in azuro ph. infec:tion • Drugs tha t decrease adhesion moh!cule
ilic granu les in neu- synlhcsis- • corticosteroids•• lithium••
trophils epinephrine
• Dohle bodies- • Endocrine- - pheochromocytoma, • Cush·
dull gray inc lusio ns ing's syndro me
in neulrophi ls
• M iseellaneous- · smoking, • tumor necrosis
Leukemoid rue- • E18ggerated • lnfcdlonll- • TO, • Whooping cough, • • F req uently con r· • Leukocyte a lkaline phosph·
lion benign le ukoqt.c infeclio us lymphocytosis, • perforated acute used with leukemia alase (LAP) score- • LA P is
• 30,000 to >50.000 response 10 infec- appendic itis present in specific granules: mar·
IcukocylesJu L tioo- common in ker of a mature neutrophil, • neu·
• Readion may children and Iro phils are sta ined for alkaline
involve- • neulro· malignancy: espe- phosphatase and scored (0 to 4)
phils, • eosinoph il s, c ially those with ror the intensity of uptake of the
• lymphocytes increased necrosis sta in in a 100 cell counl, . leuk·
• No blasts present o r superim posed t
emQid reactions have nn LAP
infecti on !2QI£: benign reactive process, •
chronic mxe logenQus leukemia
has a low score: cells are
neoplastic
Leukoerytb.robla· • PeripberalJza- • Mefastasis- reast cancer (rnosl com-moo • Extramedullary • Bone marrow exam- identifies
stie reaction tioo of marrow cause), lung canccr bematopolesiJ- the cause of the rcaction
• immature WBCs elemenu secon- • Fibrosis- • agnogenic myeloid metaplasia,. • massive hepatosple-
in blood- • myelo-
blasts, ~ progranulo-
dary '0--meta-
stasis, • replace-
metastasis with reaclive fibrosis, • leukemia: nomegaly due to mar-
row replacement by
chronic myelogenous leukemia, • polycythe-
eytes, nucleated menl of bone mar· mia rubra vera fibrosis
RBCs row by fibrosis • Abnonnal c:ells- leukemia
• Fracture-" pelvic fracture, + multiple
comminuted rractuccs
• Excess bone- osteopetrosis
Absolute. • J. Production • Dematologlc disorders- - production • Counts <500 cellsl .CDC
neutropenia • t Destruction problem: aplastic anemia, • destruction prob- ulr- risk ror infection • Bone marrow eum: rule out
• absolute count • t Adhesion lems: • B.,Jfolate deficiency• • paroxysmal • Dacleria- usually aplastic anemia versus maturation
1000- 1500 cells! molecule syoth- noctumal hemoglobinuria gram negatives, like defect versus increased margina-
uL- mild esis • Drugs (production proble.m)- • chloram- E. coli tion
• 500- 1000 cell,/ phenicol, - alkylating agents: busulfan worst, - • Fungi- Candido
uL- moderate phenylbutazone: NSAJD, • propylthiouracil: • Viruse5- Herpes
• <500 cells/uIr- anti-thyroid medication simplex
severe • Chemicals (production problem!}- • ben-
zene: also danger of aCule leukemia, • insecti-
cides
.1nrediow disea.ses- • typhoid: neutropenia
is characteristic• • NANBlHCV hepatitis:
aplastic anemia
• i Destruction- " autoimmune neutropenia:
SLE, .. Felty's syndrome: autoimmune neutro-
penia + rheumatoid arthritis
• t Adhesion mol«ule synthesis--
cndolox.ins in gram negative sepsis, • nom}s!
finding in African~ American s
Absolut~ • t Production of • lnreclioD5- • viral: + inreetious mononuclc- • ConditioDJ wUh • CBC- atypica l lymphocytosis
lymphocylOs!.S Iymphocyl.. (ao- osis (1M), +CMV. t infectious lymphocytosis. atypicallymphlr . is > I0% atypical lymphocytes
• Adul ..- >4000 tigen drinn)- t hepal:itis, t HIV, • bacteria; +whooping C)'18 - • infectious • Viral cuhures- CMV (urine
cellsiuL c.g., viral infec'~ cough (lymphocytosis promoting factor) mono is MCC, • best to culture)
• CbHdren- >8000 ions • Enitocri ne-Graves' disease CMV, • toX'oplasmo- • erologic lestJ:- + hepnliti!i, •
ce llsiuL . 1' Itelease from • Oru&,- phenytoin sis, ., phenytoin, • toxoplasmosis, • HIV
• Infants- 11 ,000- lymph nodes (~ • Malignl ncy- acule and chronic lymphocytic viral hepatitis • Done marrow- RIO acutel
17,000 cellsiuL adhesion in iller· leukemia • CoodmonJ with chronic lymphocytic leukemia
• Atypical lympho- eat 'ymphatics)- rna«u_re (nnnreact- versus leukemoid reaction
cytes- antigen stim- e.g., Whooping in) Iym pbocytcs- •
"Ioled lymphocytes cough whooping tough • •
with prominent nuel- infectious Iymphocyt-
eoli and blue cytop- osis: • probably viral1
lasm , . self-limilcd
InredioIQ monlr • Epide.miolngy- • Otber EBV associatlons- • hairy leukopl- • Common findings • CDC: alypieallymphocytosis
DU.deo.j, (1M): • lransmincd by akia on the tongue: glossitis thai precedes indud e- • cxudalive >20%
Itypieal lympho- saliva. blood onset of AIDS, .. nasopharyngeal carcinoma, • tonsi llitis: due to fhe • Serologic tests: *\_itiv.s:.
cylosis transfusion, .. vir- sex-linked Iymphoproli feralivc syndrome• • virus or group A ~l~k!if{e .g..
• Epste.in Oarr vir- us initially repl ic- polyclonal malignant Iymphomll, • primary SlIeptococcus, • pet- Monospol) in > 96% of cascs-
us (EBV) inreclion Ilfes in the oroph- CNS malignant lymphoma: in 8SSOCfalion with echia on the palate. • raIse negatives more common in
• Alypie:allympho- arynx and salivary HIV sudden onsel of a children <2 years old and (he
cytes- .. designaled glands, • BBV • Mononucleosis-like syndrom es- - CMV: rash when given am· elderly, • heterophile antibody is
Downey cells, repre- infects B cells: all MC• • toxoplasmosiJ •• scroconversiofl stage picillin, • posterior an IgM antibody agninsl. sheep
sen t anligenical1y B ce lls have CDu in KJV • • less severe pharyngitis Rnd Iym- cervica l adenopathy, Mes and horse RBCs- basis of
stimulated T cells EBV receptors. • phadenopntby in CMV and toxoplasmosis than • tender hepalospl. the Monospol test
virus remains in"B in infectious Dlono enomegaly: jaundice • [ BV serologies- anti·VCA
cells indefinitely is uncommon, • gen- (vira l capsid ~ nti gen) : IgM besi
eralized painful test: sensitivity 1.00%
IvmDbodcnoDalhv
Absolute Iympho- • J. Production • Drugs- • cort icosteroids: l' adhesion, • anti- • Corticosteroids .CDC
penia • l' Oe~trudion Iymphocyt'e serum effects on WBCs- •
• Adults- <1500 • J. Release from • Autoimmune de!truction- SLE absolute neutrophilic
ceJls! uL lymph .odes,.. t • Immunodeficiency ~YDdromes- • Oi- leukocytosis: J. adhe-
• Cbild ....- <3000 adhesion in effer- George syndrome: pure T cell deficiency due sian molecule synth-
eeJls!uL cnl lymphatics 10 absent lhymus -+ hypoparathyroidism. * esis. - absolute Iym-
• Los! o( Iympho- AIDS, • severe combined immunodeficiency phopenia: t lympho-
cytes • Direct 1055- • protein losing enteropathies cyte adhesion in
associated with lymphatic blockage: +inlest- nodes + Iymphooy\o-
ina I lymphangiectasia, .1' lymphatic pressure toxicity effect, • eos-
from chronic heart failure inopenla: roxie effect
• .Endocrine-- Cush ing's syndrome: effect of on eosinophils
cortisol
Eosinophilia! • Eosinopbilia- • Eosinophilia: • Eosinopbils- • • CDC
eosinopenia due to lhe release • Type I hypersensitivity reactioos- • crystalline material in • Nual smear for eMinophUs-
Definitions: of eosinophil che- asthma., • hay rever, • drug reactions" e.g., their granules: rono allergic rhinitis
• Eosinophilia- mOlactic factor penicillin Charcot-Leyden • Urine stain (or eosinophils-
5700 ceJlS!uL from mast cells! • Parasitic: dlseases- invasive helminthic crystals in the sputum rule out dnlg induced .acute
• Eosinopenia- not- basophil, in type I infections: • not protozoal, pinwonn or adult of aSlhmalics interstitial disease as a cause of
ab le absence of eos~ IgE mediated hyp- ascariasis, +larval phase of ascariasis. strongy- oliguria, skin rash, and azotemia
inophils on a periph- ersensitivity reac- loidiasis, ancylostomiasis, trichinosis • Total eosinophil count- no
eraf smear !.ions • Neopla.sla- • chronic myelogenous leukem- longer a practical. test'
• Eosinopenia- ia. . Hodgkin 's disease
toxic effect on • Myeloproliferative iUse&se!- • polycythe-
eosinophils min rubra vera. • chronic myelogenous leuke-
mia
• Endocrinr,- Addison's disease
• Autoimmune disease- po lyarteritis nodosa
• Pulmonary disease- hypereosinophilic
syndromes
• Eosinone.aja corticosteroids
8uophiUa • Unknown • MyC!loproliferatin dlseases- cxcelleni • MaJt uUs! buo- ·CDC
• >1 J0 celWuL marker for polycythemia rubra vera and pbil> In skin- degr-
chronic myelogenous leukemia anulate after wann
showers with release
of histamine and the
onset of severe pruri·
'us
Moaocytosis • Respo nse to • C bronic infedion- • tuberculosis, • · CDC
• > 800 cellsluL chronic inflam- subacute bacterial endocarditis, • Bone marrow-if monocytic
mation • Chronic Inn.mmation- • autoimmune disc- leukemia or mye l odyspl~ja is
ase: . SLE, +rheumatoid artbritis,. ulcerative suspected
colitis, ~ Crohn's disease
• MaJignancy-
. ., malignant lymphoma, •
carcinoma
MYcloDroliferative Disonlers (MPDsl
..,.. , .
Definition .Parhoeenesis Clinical TreJItment Laboratory Findin2!
rolycythemia rtUUI1 • Adults- • clonal • Slightly more common In • Ph'ebotomy- • Category A criteria:
Vera (PRY) expansion of the men tban women- med ian age phlebotomy alone • AJ- t RBG mass
• Bct >52-;' in an triliocage myeloid ~JO years old Rx of choice, • • Al- oxygen saluration >92%
adult male stem celt associ- ypervoJemla- t in plasma reduce RBe • AJ- splenomegaly
• lid >48-;0 In an aled with a prim- volume is unique to PRY and mass-t iron deli· Category B criteria:
adult femal e MY t in RaC no other c.auses of c ient • Dt- thrombocytosis with a platelet count >4001000
• flcI >65% In 8 mass, plasma vol- polycythemia cellsiuL
newborn ume, and granulo- ~~yperviscOSjty-. t RBC • 82- leukocytosis with a wac count > 12,000
• Thrombncyto- cytes and platelets :\ ass predispose~ to thrombotic cetlsluL in the absence of fever
sis- giant plate lets to a Jesser extent. episodes (MC COD) includ ing: • .83- elevated leukocyte alka line phosphatase score
• Neutrophils .. onl ~ MPD wifh • acUie myocardial .infarction, (> 100)
mature-nota an l' in Rae mass: +stroke, +bowel infarction, • 84- serum 8 11 >900 pglmL: due 1·0 t in neulrop-
l eft~shifted smear RBe mass is not + hepatic vein thrombosis hils, wh ich carry Bl 1 bound 10 lranscobalamin I
- Basophilia! equivalen t with (MCC of Budd-Chiari Scoring system:
eosinophilia- RaCcount ~yndrome) - Al + A2 + A3 present =- PRY
excellent markers flyperuricemla (70-.4)- t A I + A2 and two parameters from category B = PRY
for MPD breakdown of nucleated cells • Note.: • RBC mass Clchromium study) is not the
~nd t in purino metabolism same as 8n t in RBC count or nn f in Her.. although
Histaminemla- • t release they sometimes parallel each olher, • RBC mass is
of histamine rrom basophils expres-sed in RBe mUkg of body weight, whiJe.RBe
and mast cells. • associated count is cells/uL•• J. in plasma volume (volume
with vasodilation (plethoric depletion) t Rae count, bUllhe RBC mass is nonna l
appearing fncc) and pruritus • Plasma volume (radiol!lotope test)-1': on ly MPD
(release reaction of mast cells r
with an in plasma volume
aOer w8nn baths), • peptjc - Done marrow e.um- . al",'ays indicated in any
ulcer disease from histamine MPD,. marrow is hypercellular: all hematopoietic
stimulation of gastric acid elements arc l' except Iymphocyt~s, • iron deficiency
- .Miscellaneous findings- .. is rrcqucntly present, • marrow fibrosis common
splenomegaly (70%), • GI
• Pluma erythropoienn - Iow~ since ROC mass and
bleeding. • acute myelogenous
total oxygen content is 1'; very useful test in disting·
leukemia: rare Icnninal event uishing PV from the other causes of polyeyrhemia
• Enthrocvte ..sedimenlation rate: zero
ChronJI: mye.ro- • Ne.oplaslil: donal up. a- • s«ond Me leukemia bet· • DusoUa. or • Normocytic normoe:bromic aDemi.
geaoUJ (ennuro- sioD of pluripote.ntial stem ween ace 39-60 yn bydrox:yana Is • WBC findi.g,s- • mature and immature
qUe) leukemll ccll- any cell lineage may • Ifepatospienomfglty/ce •• malnSlayora. waCs: myelocyte most abundant cell with
(CML) be involved including RBCs 3 eraliled Iymphadenopathy- • Bone marrow counts between 50,000-200,000 cellslu!., • t
gran ulo-cytcs, megakaryo- splen it. infarcts are common transplantation basophilsleosinophils• • myoloblasrs <10% of
e)'tes, Band T cells • son tlnue collections or - only cutative t'otal cells
• Predispming ractors- • leukemic cells- called ~ Rx • PI.lelet,l- · thrombocytosis (40-50% of
radiation •• benzene 2IIW: (granulocytic sarcoma) • a-Interferon cases) or thrombocytopenia. • only leukemia
• Chromosome abnor· • BI.., <ri,I,- - J yrs, CML • Philadelphia with potential for thromboeytqsis
maUty- • t9;22 transloca· progresses into an accelerated chromosome- • Bone marrow u.m- • hyperceliuJar, • fibr-
lion of abl oncogene from phase (new cytogenelic abn- not lost in Rx. osis CQ mmon~ . <10% myeloblasts
chromosome 9 to chromo- onn.lilies) or a blast crisis except wiLh a · Philadelphia ch~
some 22 with fusion al the with 70% orblasts represenl- interferon pos.ilive bcr rusion
break cluster region (her) to ing my.loblasts and 30%
form a fusion gene -+ cont- lymRhoblasts
inua l tyrosine kinase activity • Lympbold blast cris.. (15
-+ t mitotic activity• • pres- - 30% of eases) leukocytosis
ence of bcr-c-abl fusion gene
is 100% speci'fic for CML
• Chromosome 22 is Phil-
adelpbia cbromosome-
high sensitivity for CML but
not t 00010 s ific
Agnogenic • Prolireration or neopl- • Uncommon In patients <60 • No Spttir'ic Rx • Normocytic normochrom ic anemia with
myeloid asUc stem cell! begins in ye.an or age tear drop cells
md3plasia the marrow- • marrow • Massive splenomegaly- • • W8Cs- • WBC cou nt 10.000- 50,000 ..lis!
(AMM) rep laced by fibrosis. .. neo-. extramedullary hematopoiesis, uL, • leukocrythroblastic smear, • basophilia!
plastic cells relocate in the .. splenic infarcts common: eosinophilia present
• Tear drop splenic sinusoids: cxlntm- frietion rub over infarction sile • Platelets- thrombocytosis in 50%
RBCs- due lo edulla~ b ~mDtOl}Qiesis•• + left-sided pleural effusion as • Done marrow Clam- marrow fibrosis
damage of RBC librosis is Q ben ign rea !t:tiv~ a reaction to injury • LAP More- noml1l1to high LAP score helps
membrane in fibr- fibroblast Qmljferalion ini- • Portal hYf1eritD.5ioD- back- distinguish AMM from CML
otic marrow liaied by megakaryoc:.ytes up of blood. into the portal • Chromosome . tudJes- absent .Philadelphia
vein from the splenic veiD chromosome
• Myelofibrosis in AMM
must be differentiated from
metastatic disease
• Myc.lop.hthisle ane.mia:-
crowding oul of hemalopoielic
eleme.nts Droduces anemia
Essentud throm- • Clonal stem cell dis- • Patients ge.nerally >50 yrs old- - Alkylaling • Normocytic to miuoc:yUe anemia- unless
bocylhemla (El') order with a prefer- leasl common MPD Igents iron deficiency is preselli.
toee for proliferation • Bleeding abnormalltles- • • Plate.let • WBC findlngs- • neutrophilic leukocytosis
of megakaryncytes qualitative platelet defects, • ble- pheresis 10,000--40,000 cellsluL, • basophilia/eosino-
eding is usually 0 1and produces • Compatible philio present
iron deficiency. - distinction bel- with long ure- • Platelets- • count >600,000 eellsluL and
ween reactive thrombocytosis in best prognosis of often in the millions, .. platelets have abnonnal
chronic iron deficiency and EP is theMPDs morp-hology aud abnormal fun ction: prolonged
important bleeding time
• Splenomqaly (80-1.) • Rone marrow nam- dysplastic (abnonnal)
m eo
. r... u k em I..
I laiA cute an deb rallie
MlIyed 0 dl~sp!as
Delinitioo Patbo2:ene:sis CllniC81 Treatment Labora'o" Findil!C.J
Myelodyspl•• dc • CloDal stem u U • Profile- • elderly person with • TraasfWlions- • Normocytic: to macro~yUc anemi.-
syndrom e (MUS) di5order- frequenlfy severe eytopcnias. • severe tlne- iron overload dimorphic Rae populaljon: microcytic
• ste.m cell disease- progresses to acute mia with a constant transfusion and macrocytic RBes
frequently progress- leukemia requirement. · 10-40% progress • Pancytopenia
cs to acute leukem ia: • Chromosome abnor- 10 acule myelogenous leukemia • Leukoerythroblaslic ruction
preleukemin malitie:s in 50%- Sq- (AML) • Peripheral blood myetobl.,u- <5%
and trisomy 8 and may have Autr rods
• Bone marrow elJm- • ringed sidero-
blasts, • t myeloblasls- < 30%,
• Abnornutl chromosome sCudjes-
chromosome sludies should be ordered,
, • since they have some prognostic signif-
icance
Acule Iy mphohl- • Clonal slem cell • MC leuke mia and overaillype • Remi!sion/indlldion • Normocytic anemia
leukemia
DStiC disease of ca nce r in chlldren- • children Rl - • usually • wile OOU."- J0,000- J00,000
(ALl.) < IS yrsold.· mcan age 4 yrs prednisone, vincristine, celisJuL
• Ea rly pre-U- • • Fevtt/ho~ pai n fro m and asparag- inase, • • Plaleleb- Ihrombocywpenia
CACLA antigen leu kemic inOliration in marrow followed by • none marrow cnm- packed with
positive: MC type. • • Genera l17..ed Iymphadenn- consol idation Rx : short lymphob lBSIS
TdT (Iennina l deo;<· palhy doses of the same
ynllclcotidyllrans- • Episla.l iJ/eccbymoses- from drugs • • followet1 by
ferase) positi\'e thrombocytopenia maintenance therapy :
• CALLA antigen . llepalQsplenomrgaly 6-mercaplopurine +
ncga · , ive • Teslicu lar/eNS involve-men I methotrexate.
• rre-B (20 % ) • C NS is a co mmon
• Matun! n (I - 2'Y1O) s ite for residual
.I mmalu~ T blasts- • neuraxis
(15 10/ . ) - tdT pos itive irradiation•• intra-
theca l methotrc"ule
• Bone marrow
Craosplanlation-
improved SUrviVA l
. 60°/. 5 year
!lurvival- • type of
ALL is most important
prognostic factor • •
early pre-B. CALLA
antigen positive ALL
has best nrolmosis
Chronic Iymphn- • Neoplastic disorder of • Me overall leukemia- .0 Me • Rx depend~ on • Nurmocytic anemia 50%
cyHc leukemia -
virgin B cells- • B cell s leukemia in patien ts over 60... degree of anemia and • WOC findings- .. absolute
(eLL) are longo lived but cannol MCC genera li7.ed Iymphndenopo Ihrombocyto-penia Iymphocylos is 15.000-200,000
• smudge ce lls- differentiate into plnsma Dlhy in pEl tient >60 • Prednisone + eells/uLwith smudge ce lls . • absolute
fragile neop lastic B cells, • hypogammaglob- • - 25% asymptomatic al pres- chloramhucil neutropenia
cell s ulinemia is commol! enlat ion • Transformation (41 • Jllateicls- Ihromhocytopenia (40%)
• Generalized, nonpsinful acute leukemia is • none marrow csam- usually
lymphadenop:Uhy rare diffuse infiltmtion
• licpalosptenomegaly • --50% die of • Miscellaneous- • monoclonallgM
• t incid ence of autoimmune infection- related to spike is com mon
hemolytic anemil- warm and hypogammaglolmline-
co ld types mm
.. Autoimmune Ihromhocyto-
.
pellHI, '
.llypogammaglobulinemia
• Second maliEnancies
Adult T cell • Stem cell disease-- • Me leukemia in Japan • Majority die in I • NonnocYlic anemia
leukemia malignan. T eelileuke- • Generalized I)'m phadeno- year dt'!lpite chento- .. WOC finrlings - • WBC counts
mia associated with Ihe pathy therapy 10,000- 50,000 celisiuL
HTL V·I retrovirus • Hepatosplenomegaly • rlntelet!f- Ihrombocytupenia
• Skin infillration- common .. Bone marrow exam- Iymphoblasts
wi th any T cell neop la.~nl s .. Miscellancous- hyperc.,1cemia
• Lytic lesions in bone (lym-
phoblast!'! !'!ecrete oSieoclast
activating faclor)- oOen asso-
ciated with hvnerca lcemia (90%)
.
Hairy cell leukemia • Stem cell disease- B • Me In middle·aged men • 2-c:blorodeoIY- • Normocytic anC!:mia
(B e L) eellieukemia • Splenomegaly Me physical adcnosine- • - 90% • Pancytopenill- characteristic
finding (90"10)- · leukemic cells response rale , • • woe finding.!l - ' malil!.nant B cells
intiltrate red rather than white
pulp • • sp leen is prim ary site for
splenectomy no IOllger
routine Rx.• (1-
with hairv nroiec tions."); ~itivctar-]
tnI&e [gillant M:id Dhosebalase stain ~
proli fera tion ofl he neoplllslic interferon used in (TRAP)~eleclron microscopy reveals
cells some cases characteristic complex lame llae
• Uepalomegaly- 20% • I'latclets- thrombocytopen ia
• L..ymphadenopalhy- ahsenl • Done marrow nam- • always
• i Incidence autoimmune pncked
syndromc!II- • vasculi tis, •
arthrit is
• LyUc bone lesion.!
• j inciden c:e or atypical myco-
baderial inredion (1\1A1)
Acure DDnl,mph. • Clonat stem ceU • Me leukemia between 15-60 • Remi!!lloQ/lnduc:~ • Nonnocytic: anemia
ocytic leukeml ... disease- .. may be years of age tion- daunorubicin • ,VUC finding5- • blasts commonJy
(acute myelogenous a...sociated with:. imd· • Presentatloo- • bleeding: thro.- plus cytambtne present, . <10,000 cetlsluL to
l eukemia.~ AML) inlion, • alkylating mbocytopenia. • fever: infection, • Be!t s urvival with >200,000 cellsiuL
FAD Classifieal.ion agents, • benzene. • im· • bone pain bone marrow tran5p- • Pfa tefetl- thrombocytopenia
• MO- • minimally munodeficiency states. • MO- no Auer rods JaDt • Do ne n.arrow exam- . hyper·
dilTeren tiated AML: • Down syndrome • M 1- rare Auer rods • OveraU loog term cellular. ' packed with blasts >300A.
2- 3% • M2- ' MC AML,' Auer rods .survival ls JO- IS'I.-
• MI - AML with·
out diITerenliation:
20';'
,
,
._,,_lI>ds;>'
preseo~ • 1(8;2 1) lranslocalion majority die of gram
negative infection,
hemorrhage or comb.-
I ~- DJCJ 1(15;17) inations orthe two
• Ml- AML wilh Intruslocation, • abnormal relinoic
maturation: 30-40%
acid metabolism: _-=:-:~~~ I
• I vitlmin CJ!!II.-inI!,~ -_. •
I ~locl!ic::~I~ • M4- • AUer rods uncommon
• M4- acute myelcr • M5- • no Aller rods, • gum
monocytic: 15- 20%. innhration
• M5- acute mono-- • M6- • bizarre, multinucleated
cytic: 10% erythroblast. thaI are PAS
• M6- acute eryth· positive
roleukemia (Oi • M7- • myelofibrosis. • platelet
Guglielmo's: 5% peroxidase slai n positive using
• M7- acute mega· electron microscopy
brvocvtic: 1%
Differential dillgnosis of tbe polycythemia disorders: balded areas represent good differentia l
pomts
Condition RUe mB.!I5 Plasma SaOl Erythropoietin
volum e
PRY Increased Increased Normal Decreased
COPD Increased Nonnal Decreased Increased
Cvanotic CBD lncreased Normal Decreased Increased
Renal adenocarcinoma Increased Normal Normal lncreau d
Volume deolelioD
. Normal Decreased Nannal
.Normal
PRY "'"' polycythemia rubra vera. COPO - chrome obstrucll ve lung disease. CAD ". cyanotic
.
congenital heart disease
(Ig lO~Io"""" ..d'"
1. acute vs chronic-
---7 A. bone marrow exam revealjng >3 0% blast cells is acute leukem ia ~~'----
B. blast count not high in chronic
2. usual profil
A. anemia: usually normocytic
B. thrombocytopenia: CML only leukemia that may bave thrombocytosis
C. high wec count with blast cells: may be a nonnal count, but blast cells will be present
D. generalized lymphadenopathy
E. hepatosplenomegaly
Ii'. bone pain
G. fever
3. ~~~~~~~~~
A. < I S ALe
B. 15- 39 AML: look for Auer rod in mye loblast
C. 40-«> AMI. and CML
D. :> 60CLL
(1) also MCC of generalized lymphadenopathy in the elderly
(2) also MC .v.ralileukemia
Summary of laboratory tests in hematology:
Laboralory Tesl Comments
Osmotit fragility -increased in congenital spherocytosis. Decreased in conditions
with tar..Eer cell (excess RaC membrane).
Sugar water testIBam/s test Sugar water test is the screen and Ham 's test the confirmatory test
for paroxysmal nocturnal bemogJobinuria- absent decay
accelerating factor.
H emoglobin electrophoresis Gold standard test for identifyi ng an increase in nonnal Hb (e.g.,
Hb A) and F in I)-thalassemia) or an increase in an abnormal Hb
I (•.•., Hb S5, Hb SCl.
Enzyme assays Confirmatory ,." in glucose 6-phosphate dehydrogenase
defic iency in Quiescent staIle and pyruvate kinase deficiency.
Serum iroo Decreased in iron deficiency and anemia of chronic inflammation
(ACD). Increased in sideroblastic anemias (e.g.• lead poisoning)
and iroo overload conditions (e.g., hemosiderosis, hemochroma-
losis).

12
~';;6on binding capadr), Correlates with lransferrin levels. increased in iron deficiency.
Decreased in ACO and iron overload disease
Per cen t saturalion o( Decreased in iron deficiency and ACD. lncreased In iron overload
traaJfcrrin states.
~ Serum (undn Decrease in iron deficiency. Increased in ACD and iron overload
diseases.
I ro n s tain Prussian blue stain. Absent iron in iron deficiency. Increased iron
in ACD, sideroblastic anemia (identifies the ringed sideroblast) and
iron overload states.
Fr.e ,~~~rotoporphYri n Increased in iro~:~ficiencYI ACD. and lead poisoning. Decreased
lenls EP in pyridoxine (B6 deficiency.
I Se rum Bu Decreased in Bn deficiency and increased in polycythemia rubra
vera and chronic mvelo~enous leukemia
I Serum (ola leIRBC folate Decreased in folate de'fi ciencY.
Urine ro rmi no~lutamk acid Increased in folsle deficiency.
Urine mett;;imaJonic acid Increased in B" deficiencv.
I Se rum h o m o~J teln e Increased in Bn and fo late deficienc v (MCe).
Seru m lactate dehydrogenase Increased in .ny condition with RBC hemolysis (e.g.,
(LDD) intravascular/extravascu lar hemolytic anemia, ineffective
erythropoiesis in B I~fol8 t e deficiency) or with massive breakdown
of cell," r•.•.. leukem i.l.
U nconjugated bilirubin Increased in extravascular hemolytic anemias (e.g .• congenital
sDherocvtosis. autoimmune bemolytic anemia),
I Serum haptoglobin Decreased in intravascular hemolysis and severe extravascu lar
hemolvsis (fib leaks out of ph ••ocvtosed RBC,\.
Urine bemosideriD Increased in chronic intravascular hemolysis (e.g., microangio-
I Dathic hemolvtic anemia. oaroxvsmal nocturnal h.mo.lobinuri.i.
Sickle cell screen Solubility test and metabisulfite test. Positive tn Hb AS. Hb SS, Hb
SC. Hb Slthal .... mi.
Don8 lb-Laodsteiner a..otibod-----; Bithe.rmal antibody. Positive in oaroxvsmal cold hemoRlobinuria
Direct/lJldl reci Coombs tH I DetecllgG, 19M or C3 on the surface of RBCs (direct Coombs) or
detect IgG or IgM antibodies in serum (e.g., anti-Oj indirect
Coombs). GoJd standard test for autoimm une hemolytic anemias.
Positive in ABO and Rh hemolytic disease of the newborn.
Reticulocyte couol Supravital stain to Identity RNA ftIamenlS in young RBes. Gold
sta ndard tes t to evalua te effectIve erythropoiesiJ. Increased >
3% in hemolytic anemias, ircatment phase of iron deficiency,
BIl/fo late deficiency and in blood 10$5 >7 days old. Increased in
disorders that metastasize to the marrow (e.g., breast metastasis to
bone) or in primary diseases of the marrow that push reticulocytes
out (e.g., acute/chronic leukemia). Decreased <2% in most of the
microcytic anemias (exceptions: lead poisoning, severe a and Jl-.
thalassemia), B'l and fo late deficiency, aplastic anemi a, anemia of
renal disease. blood loss < S-7 days.
Heint. body prepa radon Supraviml stain to identify denatured Hb in RBes secondary to
peroxide damage In glucose 6-phosphat'e dehydrogenase
deficiencv.

13
Leukocytt Illkallne phospba- Stain of neutrophils and score bosed on the intensity of the stain.
tase (LAP) score. Marker of mature. neutroph ils. lncreased in benign neutrophil
disorders but detreased in chronic myel02enOus leukemia.
lmmuDopbenorypiD&of Gold standard test to subclassify the type of cell Involved in aCUle
Iym.bocyl.. Ivrnphoblastic leukemia.
Tartrate resistant add Key stain for identifying neoplastic 8 cells in hairy cell leukemia.
I .bosob..... rro • D ' .Illi.
Philadelphia cbromoso me 1(9;22) located on chromosome 22 in chronic myelogenous
leukemia.
Heterophile a.otibody 16 1 Test to identify the unique heterophile antibody in infectious
mononucleosis. Heterophile antibodies in mono are against horse
RBCs (most sensitive antigen, used in slide test). sbeep RBCs, and
bovine RBCs. 1M anribodies do not react against guinea pig kidney
(GPK). Blood mixed with GPK does not clump, but when horse
RBCs added it does ciumo.
Epstein-Barr body tiler Anti viral capsid antigen-lgM and IgG, anti early antigen, anti
Epstein-Barr nuclear antigen In the evaluation of heterophile
an tibody nejlative infectious mononucleosis.

A febri le 23 year old college coed presents with fatigue and difficulty with swallowing. Physical
exam reveals exudative toosillitis, palatal petechia, cervical lymphadenopathy. and tender
hepatosplenomegaly. A CBC reveals a mild microcytic anemia, lymphocytic leukocytosis with -
20'10 of the lymphocytes exhibiting atypical fearores. and a mild thrombocytopenia. You would
expect this patient to have ..•
A. alowTlBC
B. a normal serum ferritin
C. an elevated total bilirubin
D. heterophile antibodies
E. normal serum AST add ALT titers
o
(I" A 6S-year--old man with an elevated RBC count has a history of gout, frequent headaches. and
pruritus after bathing. He presents with a sudden onset of abdomi nal pain and bloody diarrhea.
Which of the following sets of laboratorv dal'a best reoresent this oatient's he matologic disease?
RBCMass Plasma Volume Sa02 El)'lhropoicll n
A. Increased Normal Decreased increased
B. Increased Normal Normal Increased
C. Increased Increased Normal Low
D. Normal Decreased
.
Normal Normal
.
Sa02 :a oxygen saturauon C A a COPO, cyanotlc CHD, B - ectopic EPO productlon, 0 - relative
polyoythemia (volume depleted)

14
Lympb noduIPlu ma cellslHblioCYleslSpleen

Reaclive lymph adenopathy:


I. cllnieal- 6,,.,1
A.;Jf painful lymph nodc ___ ." J
B, localized in localized disease: e.g.. tonsillitis
C. generalized in system ic: disease: e.g., HIV, infectious mono
2. lymph node relation.s bips-
A. genninal follicle contains B cells: absent in Bruton's agammaglobulinemia: also no
plllSmo cells
8. parafolJiculnr area contains T cells: absent in OiGeorge syndrome
c. sinus contains histocytes
( 1) increased hiniocytes in histiocytosis X
(2) sinus histiocytos is in nodes draining cancer
(3) histiocytes are COl positive
Maligna ncy:
I. cllnieal-
A. pain less, hard lymph nodes
O. loe.alized in localized disease: e.g., axill ary nodes in brenst cancer
C. general ized in:
(I) leukemia
(2) non-Hodgkin·, lymphoma
Follicular 8 celllympbom a:
1. MC maUgaa ntlympbom a in adults
2. exa mple or traasloc:alion or 114;18 leading to Inactivation or apop losis genc: see
Neoplasia notes
Burkih's Iymp boma:
l. MC lymph oma in ehildrt'.n
2. loca ted in a bdominal t.avity-
A. paraortic lymph nodes
B. I'e nni nal ileum
C. "StarT)' sky " Bppearance under low power
3. hig h grade B cell malignancy
A. EBV relationship
B. t8 ; 14 translocat ion of c·myc oncogene
Mycosis fuo goides:
1. CD4 T helper cell malig nancy
2. skin Inn.sion produces nodular lesions and Pautrier's mlcroa bscesse! (malignant T
ce lls)
3. systemic disease
4. Cllled ezary syndrome when PAS positive T cells a re present in blood
Histiocytosis X:
I. mainly cbildhood dinlUe- histiocytes are COl positive
2. typ<>-
A. eosinophil ic granuloma~
( I) benign histiocytes
(2) lytic lesions in bone with pathologic fra ctures

15
tV ~ (}J 3'5f"I'"~ 5 "
B. HW1d~Scbu ll er~Christian disease . . .
(1) malignant (j) ~ ®
(2) triad of exophthalmos, diabetes insi.pidus. lytic lesions in the skull
C. Lenerer-Siwe disease:
(1) malignant: infants and young children
(2) d iffuse eczematous msh
(3) organ involvement
(4) lytic lesions in bones
3. mPiftG- twO year old with eczematous rash and organomegaly and biopsy of skin
revealed COl positive cells
Hodgkin" disease (lID):
1. cUnical-
A. RS ce ll is the neoplastic cell : see question #1 p. 186 afRe.view book fo r picture
(1) can be a B or T cell
(2) must be preseni to Dx Hodgki n's
8. noduJar sclerosing Me type
C. fever, night sweats. weight loss often confused with TB
D. prognos is corresponds with number orRS cells and type of Hodgki n's
E. death by second malignancies due to alkylating agents; e.g., non-Hodgkin's lymphoma
2. summary of the subtypes ofUodgkin 's Disease (HO): nOle how the types move from those
with the. least number to the greatest number of RS ce lls, from best prognosis to poorest
prognosis) and from a young age to an older age

Hodgkin!s disease

Asymptomatic young male with isolated node or


predominant group (cervical or supraclavicu lar). Ha rd to identify RS tells. UsuaUy stage I.
BeSl survival (90% 5 year survival). increased incidence of second malignancy

predominant.
RS variants are called lacunar ceUs.

Middle age. RS cells easy to find. - 50% year

Lymp hocyce years old. year

cell
cr X-ray witb multiple lytic lesions in the hip in a patient with ant-mja and hypercalcemia:
1. multiple myeloma- see table for desc ription of all monoclonal gammopathies
2. s ummary of moaoclooal e:ammopatbies (includes amyloldosis)-
D iso rder Commeois:
Monoclonal gammopatby -60% of aU cases or MG (MGUS is MCC or monoclonal spike).
of undetermined Most are an IgG M-spike. Majority pursue. a stable course. Some
.igaificaace (MGUS) develop multiple mveloma or related disorder.
Multiple myeloma (MM) - 20 % of all cases of MG. Me primary malignancy of bone. More
common in African-Americans than whites (2: I). Rare under 40 years
old. Increased risk with radiation exposure. M-spike in 80-90% and
urine BJ arctein lliohr . in 60-80%. M-snike usually loG.

16
Sheets of malignant plasma cells> I 0% of cells in marrow. Increased
~::;l; microglQbulin QQnend:i; a Qoor suo::ivai (correlates with degree of
plasma cell proliferation). Suspect M1v1 in any midd le aged 10 elderly
patient with unexQ lained amaDio. bolle BRin. gathQlogic fracrure,
recurrent infection, !Jne3"I~ineg hn2ercalcemi~ or renal failure
without hygeaeDliiQn. Skeletal svstem: Ivtic lesions ' ''"unched out" },
bone ~ain, Q3thQIQgi!C fractu[!;:~ .
H.ematologic system : anemia, rouleaux,. increased sedimentation rate,
thrombocytopenia
Urinary system: nephroca lcinosis (from hypercaJcemia) MeC of
acute renal failure. Light chains are toxic to tubl1les and produce a
giant celJ reaction. Prone to primary amyloidosis involving kidneys.
En docrine system: hypercalcemia due to local release of osteoclast
activating factor by pl asma cells.
Inrection Me CO D.
Wnldeostrom's Clinical: monoclona l proliferation of Iyrnphoplasmacyto id cells
macrogiob ulinenlia secreting excessive IgM .
Clinical: hyperviscosirv svndrome (85-95%), hepatosplenomegaly.
lvmohadenooailiv. anemia. B1 orot'ein in - 80%.
Heavy chain diseases (at M protein part of heavy chain devoid of ligh t cha ins localed in serum
I Y. U t".• • l and urine.
Amyloidosis Amyloid is a fibrill ary proteiD that deposits into inlerstitial tissue
resulting in organ dys function .
Properties of am yloid: twisted 13- pleated sheet (l3-fibrilloses). Congo
red positive and demonstrates apple green birefringence (Uke a
Granny-Smith apple) under polarized light Electron microscopy:
l.inear. non-branching fiber with hollow cores.
Derived rrom di(ferent proteins: light chains (association with
MM). serum associated amyloid (SAA. an acute phase reactant in
inflammation). prealbumin, e: amylQjd proteins. and peptide hormones
like ~alcitonin.
Two main types of amyloidosis: pdmary amyloidosis (light chain
derived) and ~~~ [!di!!J( (reactive) amyloidosis (SAA derived.
rheumatoid arthritis, TB. leprosy, renal adenocarcinoma).
Co mmon sites 0'( involvem ent : heart, spleen, tongue, ad'rena!s.. liver
and kidneys.
Cardiovascular system: restrictive cardiomyopathy- localized fann
in the elderly due to prealbumin-derived amyloid
Gastrointestinal sy5tem: macrogloss ia, malabsorption, hepatomegaly
with increased alkaline phosphatase
Musculoskeletal system : carpal runnel syndrome common.
Renal system: renaJ fai lure is Me COD (nephrotic syndrome)
CNS: Alzheimer' s djliea5e: (l3-amyloid protein coded by chromosome
21. Amyloid toxic to neurons. Me COD in Down's syndrome patients
>40.
. Diagnosis: Bx of rectal mucosa, gingiva. omental fat pad, or the organ
involved.
Other types or amyloidosis: medullary carcinoma of thyroid
I (calcitonin derived amyloid)

17
~ summsry_cbart 0 n mpDrtant disorden 0f t be spl ecn :
Disorder Comments
I Systemic. infections! Infectious monoDucleosis: splenomegaly invariably present Ruprures with
innammation trauma (contact sportS) more than spontaneous.
Kala au r: massive splenomegaly in visceral leishmaniasis. Leishmania in
macrophages.
Malaria: MeC of splenomegaly in third world countries.
Auto immune disease: splenomegaly common in SLE and rheumatoid
arthritis. SLE has hyperp lastic arteriolosclerosis (onion skinning of the
I penicillary arteriole).
Neoplastic disorders Myeloproliferative disease: splenomegaly invariably present (mass ive in
agnogenic myeloid metaplasia) , Extramedullary hematopoiesis common
(triJineage hematopoiesis in sinusoids).
Leukemia: splenomegaly invariably present often massive (chronic
lymphocytic leukemia). Red and white pulp involved in most cases. Red pulp
onty in hairy cell leukemia.
Malignant Iympboma: NHL and HD commonty produce splenomegaly.
Metastatic mali~nant lymphomas are Me splenic mali~nancy ,
HypersplenisDI Hypersplenism is an exaggeration of normal spien.ic function. RBCs,
wacs and platelets either singly or in combination are seq uestered and
j

destroyed. Pona l hypertension in cirrhosis is MCC. Felty's svndrome is the


combination of rheumatoid arthritis, sp lenomegaly, and autoimmune
neutropenia.
SickJe cell dlJeasc Sickle cell disease begins with splenomegaly owing to entrapment of RBes in
the cords and sinuses. Spleen non·functional at early age (-2 years old).
Markedly reduced in size (infarction) in late adolescence (called
a utosDlenectomv).
Portal bypertension PH most commoaly secondary to cirrbosis. Increased portal vein pressure
I (PH) I produces congestiv~splenomegaly . Hypersplenism common.
Splenic infarction Most are, pale infarcts second ary to emboli originating in tb e left side of
tbe heart (e.g., thrombus, vegetations). Commonly produce left upper
I quadran~pain and friction rubs (fibrinous exudate on c30su lar surface),
Congenital asplenia Congenital asplenia assoc iated in >80% of CasC5 witb malformations in
Ihe heart.
Splenectomy Splenectomy (or fun ctional u plenia {sickle eell diseaseD predisposes to
Infections (septicemia, peritonitis). Streptococcus pneumofliae followed by
Hemophilus injluenzae are Me infections. HemntoJogic nbnonnalities with
splenectomy: presence of nucleated RBCs, Howell Jolly bodies (nUclear
remnants in the cytoplasm), increased reticulocytes (RBe membrane is nOl
removed by macrophages), target cells (excess membrane cannot be removed),
Heinz bodies (denarured FIb), and an increase in plat'elets (thrombocytosis),

18
rJ ~ ,.-fr''''' . " ~'
Coagula'ion: see schemntie e-+~oo,~

C7 FuClo ... prevooling ' mall v....1ciotting: /


I. htparin- enhances antithrombin lU (ATIJI). which neutralizes most serine prole35t
coagulation factors: XII. XI, IX. X. pTOIhrombin (II). thrombin
2. PGI,-
A. synthes ized by endothelisl cells
B. \'asodihuor
C. inhibits platelet aggregation . J.. \
3. protein C aDd 5- (VI+n,~,1\J 1<.-"'(' -, "")
A. inactivate factors V and VUI
B. enhance fibrinolysis
4. tiss ue phlsminogen actival'Or-
A. release of plas min
B. destroys coagulation fac tors and elms
Faclors ac ting Ittl procoag ulllnlS «(orm daIS) in s mall \'csscllnjury:
1. Ih romboxane Al-
A. synthesized by platelets
B. vasoconstrictor/bronchoconstrictor
C. enhances platelet aggregation
D. cyclooxygenase blocked by aspirin and NSAlDs
2. VOD WUlebran d faclor-
A. VIU :vWF
B. synthesized by endothelial cells and megakaryocytes : platelcts cany some VllI :vWF
C. platelet adhesion factor: platelets have receptors for VLII :vWF
3. extri nsic aad iatriaJ:ie coagulation SYJ:lem
N'ormal enol! with small vessel injury: sequence of eVenlS-
1. vessel injury-.
2. activation of factor vn in the extrinsic coagulation system by tissue thromboplastin and
acti\'ation of factor XU in the 'intrinsic system by exposed collagen evil
can directly aCliv%ue
lX in the intrinsic system}-fo
3. platelets stick to VII1 :vWF in damaged endothelial cells via their receptors platelet
adhesion) -.
4. st'imulu5 for p!tllelet release of ADP from dense bodies causing platelet aggregation and
synthesis ofTXA1-'
5. temporary platelet .,lue with fibrinogen draped over it (fibri nogen receptors on platelets)
stops bleeding (end of the bleeding ti me)-.
6. thrombin generated by coagulation pathway stimulation converts fibrinogen into fibrin and
fonns a stable platelet plug -.
7. plasmin destroys the plug and reestablishes blood now
BI.eding lim. (BT): D<", r1vf" -Int to l'W~W~'/;~rV ~t ~ 1~/:"h I
1. de.tW!JJlatelel abnormalirj p e formation of 'he tern ora la.clel plug
2. cau.ses of a prolonged BT-
A. thrombocytopenia
8. no VIII :vWF for platelet adhesion
patienl on a.spirin: MCC, due to no TXAl for aggregation
renal f.ilure

19
( 1) qualitative platelet di sorder due inhib ition of platel et phospho lipid
(2) reversed with DDAVP (desmopress; n acetate)
(3) reversed with birth contro l pi lls
RjstOC~lj n cofa ctor assa~' : ')
t. best test fo r VITI:v\VF (a/' <t.fbPh C
2. norm a platelets clump w hen ristoce tin is added to a test t'ube
3. pla tt lels without VID:\'\VF o r Ihe receptor fo r VlTI :\''''F- Berna rd·So ulier d isease
(thrombocytopenia and gia nt pla lc lels) do nOI clump
S ummitry of platclet tests :
I. piaieici co unt : nonnal number does not alwllo ys mean they are fun ctional- e.g., patient on
aspi ri n
2. bleeding time- tests platelet functio n
3. ris tocNi o cofactor assay- detects VlU :vWF or its recepto r o n platelets
Prothrombin time (PT):
I. de tccts extrinsic coagulation system factors d ow n to formation of a clot- VII (extr insic
system) -+ X -+ V -+ II (protJlrombin ) -+ 1(fibrin ogen) -+ cl ot
2. iolerlliltionlli normalized ratio-
A. fo r at ients o n \ adacilL
n. standardizes the tcst thro ugho ut the world so all test results are !.he same regard less of
the reagent used
J. PT is bes t lest (or liver sy nlhe li c (unc lion
PT •••••. . • " , PIT
.., ..,
."1·~trillsic system ...
••••
•. '
X1J":
XI ..
IX .,,
..
,,
VIII
·
: V

··
~ u
.'.
\ I
" ):101
.'
...."
" .' . '

Purth'll thro mboplastin lim e (PIT): sec above schematic


I. deicers Intrinsic coagulation system fucl on (XII. Xl, IX. VIU) d own to (ormat ion of tl
clol- Xli -+ Xl -+ IX -+ VIII -+ X -+ V ~ n (prothro mbin) -t I (fibrinogen) -+ clOI
2. used to follow be arin Iherapy a nd (~ct o r defi ciencies
Fibrinolytic sys lem lests :
I. fibrin (ogen) split products- X, Y. D. E fragments aft er plasmin breakdown of fibrinogen
or a fi brin clo t
2. D-dimcrs-
-{ A. measures cross·linked fibrin mono mers in a fi brin c lot
B. cross· links indicate factor xm
activity in fo nning a sla ble elOi
USMLE pktare or aD elderly patieDtI Web wl1h H Dlle purpura: normal age·dependenl
findin g and not a sign of patient abuse

20
Signs lind symptoms of vessel insta bili ty a nd pla telet p roblems:
I. epist8.x is- nosebleeds (MC symptom)
2. bleeding fro m s uperficial scratc bes- no temporary platelet plug
J. easy bruising
4. ~tec h ia lind ecchymoses-
A. these do Dot blanch since they represent bleed ing into subcutan~ou s tissue
B. capilfary hemangiomas and spider angiomas blanch with pressure
S. mucosal bleeding
U MLE .....do:
I. 1111.[ 1" • cblld (GUo..ed by epistasis .Dd petechia
2. Idlopatllk lJIro ...~pniC' purpUnI-
A. MCC oflhrombocytopen ia in children
O. IgG antibody against phlleletS: type [J hypersensitivity
C. no splenomegaly
D. megakaryocytes are present in marrow
E. responds we ll to corticosteroids
Autoi mmune thrombocyto penia in adults:
1. most C'o mm only associated with SLE-
A. IgG antibody against the platelet type II hyperse nsitivity
8. splenomegaly usually present unlike IT? in children
C. Rx:
(1) corticosteroids-+
(2) splenectomy-+
(3) alkylaling agents
(4) IV y·globulin if bleeding cannot be contro lled: blocks Fc receptors for IgG on
mac:ro phages
2. olbe r ca uses of a utoi mmune throm bocytopeni a-
A. drugs:
(1) quinidine (type 0 hypersensitivi ty)
(2) heparin (antibody against heparin attached to platelet membrane)
B. viruses: I-OV (Me hematologic abnonnality)
T hrombolic thrombocytopenic pu r pu ra:
I. sma ll vesse l da mage (unknown plasma factor) with co nsumption of pl atelelS-
A. poss ible causes o f endothe lial cell damage :
(1) infectious diseases
(2) diseases: progressive systemic sclerosis. hypertension
(3) drugs- e.g., mithramycin, birth control pills
(4) anti·phospholipid antibodies
(5) preeclampsia
8. platelct thrombi :
( I) not ole. since coagulation factors are not consumed
(2) only platelelS are consumed
(3) abnonnal VID :vWF in endothel ia l cells may contribute to platelet aggregation
C. vasoconstriction
( I) endothelin (vasoconstrictor) released in damaged endothelial cells
(2) decrease in nitric oxide and PG I2- nonnally. they are vasodilators

21
2. mic roangiopathic hemolytic anemill-
A. RB Cs hit platelel plugs~
B. schistoc),lcs: must be present to secure the Ox
3. fe\!cr
4. C S problems
5. rt' nal failure
6. Rx with plas mapberesis
7. hemolytic uremic syndrome in chi ldren- similar except kidney is worst hit while in TIP
the brain is worst hit
Ot57:H7 serotype of E. ~o li :
l. hem olytic uremic syndrome-
A. renal fa,iiure
B. anemia with schistocytes
C. thrombocytopenia,
O. CNS findings: less common than lTP
2. orgunls m proliferates in undercooked bed or produ ce contaminated with E. coli- e,g,.
unpasteurized app le cider
3. Shigella toxin clln also prod uce RUS
T hrombocytosis: causes-
t. chronic iron d eficiency: common
2. maligna ncy: common
3. spleoectomy
4. lorection5
S. myeloproliferative diseases: e.g., polycythemia vera
Signs and sym ptoms of coagulation di.5o rd en:
I. hlte rebleeding-
A. temporary platelet plug is only mechanical block ofsmall vesse ls
B. bleeding after wisdom tooth remo\'al or from the incision site in any type of surgery
2. bleedin g into joint! (hemarthroses) and d osed space5- on ly severe types like hemophilia
A and 8 with "ery low factor levels
3. mucous membrane bleeding- GJ and GU bleeds
Hemophilia A:
I. SXR
2. I.b-
A. prolonged PTf
B. normal PT
C. lowvm:C
O. nonnal Vm :antigen and vm : v\VF
3. clinical- see previous discussion
4. IU-
A. recombinant factor VITI for severe cases
B. ODA VP (vasopressin) for mild cases: increases synthesis of all fBemr vm molecules
C la.uical V\VO :
I. AD
2. I.b-
A. prolonged bleeding time
8. normal PT

22
C. prolonged PIT
D. lowVlfl:C
E. 10"' VllI: antigen
F. low ristocelin cofactor assay: decreased VIII :vWF
3. Me ~e n ttic toagu lopa thy
4. clinic.a l-
A. menorrhagia
D. mucous membrane bleeding
5. Rx-
A. DDAVP (desmopressin)
O. birth control pills in women with menorrhagia: estrogen increases s),mhesis of 0.11 Vill
molecu les
C. cryoprecipitate:
(1) blood product containing Vlli . fibri nogen. XIII
(2) rarely used
Patient wlt b a ntibody against fac lor vnt:collgula nt (ci rcula ting anticolligul ant, Inh ibitor) and
prolonged PTT: lab findings
1. nonna l PT
2. pro longed PTf
3. after mix-ing 0.5 cc of nonnal plasma with 0.5 cc of patient plasma, the PIT is repeated and
is sti ll prolonged: antibodies inhibit VW :coagulanl in the normal plasma as well
4. a true factor VUl:coagulam deficiency: correction of the PIT after adding nonnal plasma
Antip hospb oJipid synd ro me:
1. defin ition- antibodies directed against phospholipids bound 10 plasma proteins
2. ty pes of antibodies-
A. lupus anticoagulant:
( I) misnomer. actually causes clotting
(2) usually produces a prolonged PTT
(3) PIT not corrected with addition of nonna l plasma
(4) PTT corrected by Jdding phospholipid
B. anticardiolipin antibody:
(1) cause of a biologic ralsr positivr syphilis ser ology
(2) seen in SLE. HJV, other disorders
(3) causes thrombosis
3. clinical syndromes-
A. repeated abortions: placenta l bed clots
D. Strokes
C. thromboembolism
O. hepatic vein thrombosis
r:r Vitam in K defi ciency:
1. causes bleeding-
A. vitamin K r-carboxylates U, VD, IX. X, protein C and S
B. r-earboxy lation allows calcium to bind to these factors in a clot
C. absence of r-carboxylation leaves vitamin K dependent factors non-functional : patient
is anlicoagu lated
2. c:a uses or K defici e nc),-
A. newborns lack bacteria l colonization for vitamin K synthesis

23
(I) require LM injection of vitamin K al binh: no vi tamin K in breast milk
(2) vitamin K levels normally decrease belWeen days 2-5
(3) danger of intracerebral hemorrhage if newborn is not protected . called
hemorrhagic disease of the newborn
B. malabsorption
C. antibiotic R.\: sterilizes bowel: MCC in a hospitalized pal ient
D. \l.arfarin blocks epox.ide reductase (normally k~ps \·jtamin K in functional K 1 sLate)
( I) vitamin K non·functional
(2) rat poison contains warfarin
3. alto holic with cirrhosis-
A. could be problem with liver synthesis of vitamin K: prolonged PT nm corrected "ith
1M vitamin K
D. could be problem with malabsorption of vi tamin K from bile sah defic iency or chronic
pancreatitis: prolonged PT corrected with 1M vitamin K
USMLE qUeitioa of cbild takia& rat poison: J
1. contains wnrfarin- will ca use bleeding
2. R..l with 1M vitamin K if not medically .ll ignlficunl bleeding- use rresh rrozen plasma i f a
medi cally s ignificant bl eed
Vitamin abnormaliUe~ associated witb prolonged PT:
I. vitam in K deficiency
2. vitami n E lox.ie.ity-
A. inhibits synthesis of vitamin K dependent faclors
B. synergistic with warfarin
Patient 00 warfarin:
t. mechaoism-
A. blocks vitamin Ks ability to y-carboxylate factors II. VU. IX. X. protein C and S
8. blocks epox.ide reductase: normally keeps vitamin K in its active K I stare
2. prolon ,ed PT and P1T- PT is bener test for following patiems than PIT
J. bleeding Me compl~atioo -
A. if overanticoagulatcd and seriously bleeding give fresh frolen plasma
B. 1M vitamin K, if bleeding not serious
4. Ifnvto-ty y-arbosylated vitamin K depeDdeal racton m ... dllappear be.rOrf paUea.
10 fully aDllc••,alated (!JSMLE)-
A. reuson why heparin is given along with wllrfnrin
8. VI] and protein C have the shortest half- life (6 hours)
C. prothrombin has the longest half· life of 3-4 d
5. hemorrhagic ski n necrosis-
A. usually a patient who is a heterozygote with protein C deficiency and 50% factor level
B. when put on warfarin, patient will ha ve 0% protein C levels in 6 hours due to shan
half·life of protein C: causes thrombosis of vessels in the skin before the patient IS
fully anticoagulated
Patient 00 bepllrin :
I. prolonged PT and PTT- PIT is tbe bener test to follow patients
2. ovenaolicoagulattd - give prolamine sulfate
3. heparin is Me drug associated wilh thrombocytopenia In the hospital
4. preyen's venous dot formatioo-
A. does not dissolve the clot

24
8. can be used in pregnaney
C. complications:
(1) bleeding
(2) thromOOc),10penia
(3) osteoporosis
"" D1C:
I. palbogenesi5-
A. intravascular consumption of clotting factors
B. consumption of fibrin ogen, V, VUl , prothrombin. platelets
2. eliaical- diffuse oozing of blood from all breaks in the skin plus muco us membranes
3. cause!-
A. endol'oxic shock : Me
8. infections: meningococcemia
C. ran lesnake envenomation
D. amniotic: fluid embolism
4. lab-
A. pro longed PT and PTT
B. low fibrinogen
C. _ . p U t """'.e'....d D-dimeo: best 'ests [or DIC (USMLE)
D. thrombocytopenia
E. schistocytes: RBCs hit fibrin clots
5. Rx-
A. Rx uoderJying disease causing DIC
B. use blood components to keep the patient olive: basically adding more fuel to the fire
C. hep~n : bl o.c~\ thrombin, hence preventing clots and consumption of coagu lation
factors (I, .... ((I.S1..1
He reditary thrombosis sy ndromes :
I. diaical-
A. venous thrombosis and pulmonary emboli at early age
8 . clots in unusual placcs!"e.g., dural sinuses
2. ca U5es-
A. (aelor V Leiden :
(1) MC hereditary thrombosis syndrome
(2) protein C and 5 cannot degrade fac tor V
B. ATlll deficiency:
( 1) no prolongation of PIT after starting heparin
(2) give massive doses of heparin and the PIT wil l eventually prolong
(3) send home on warfari n
(4) binh contro l pills is Me acquired cause
C. protein C and 5 deficiency:
( 1) cannot inactivate V and VlU
(2) stan with heparin and very low doses of warfarin so that hemo rrhagic skin
necrosis does not occur

25
r. of hemostasis abnormalities:
Bleeding time: Platelet count PT PTT Interpretation

I or '
•• .1
,
b. I I von Will
~US
c. I
d . ~ow I
•• I l or I
Cab) Lc!w I Die
(ac) I I I Blood loss

QaHrioDI used daring tbe board review:

Which of the followi ng is more often assoc iated with mild hemoph ilia A than class ical von
Wille brand's disease?
A. Nannal PT
B. Prolonged PTT
C. Low VlO:Ag levels
D. Nonnal bleeding time
E. Treatment with desmopressin acetate
o
A feb rile 65-year-old man with urinary retention secondary to prostatic hyperplasia develops
ARDS complic8t'cd by endotoxic shock. Within 24 hours he has oozing of blood from all puncture
sites. extensive ecchymoses. and mucosal bleed ing. Which of the fo llowing lesls is MOST
US EFUL as an initial screening test?
A. D-dimer assay
B. Prothrombin time
C. Bone marrow aspirate
D. Partialthromboprsstin time
E. Examination of the peripheral blood
A (DIC)
Vitamin K deficiency is MOST LIKELY present in a patient ...
A. with liver disease who is being treated with a broad spectrum ant ibiotic
B. with traveler' S diarrhea who is being treated with cipronoxacio
C. 00 heparin therapy for prevention of a pulmonary embolus
D. with lactase deficiency who currently has watery stools
E. who has taken warfarin 3 hours ago
A

26
fmmuDobema,olocy

G lyc:oproltlnJ:
I. proteins 10 wblcb art anachtd s bOr1, branc:b-cblllatd oligosaccb aridH
1. r.Dcdons iadudr-
A. cell surface antigenic ity: e.g.. blood group antigen on RBes
B. components of muc in: e.g., mucous layer covering the stomach mucosa
C. tumor markers: e.g.• prostate specific I1mjgen
ABO blood groups:
I. H ceDe- codes for a transferase that ausches fucose 10 a glycolipid to fonn H antigen
1. A genr- transferase product anaches N-acctylgalactoS8Jt1lRe to H antigen 10 produce A
IUlligen
J. B genr- transferase product anaches galactose t'o H anligen to form B antigen
4. 0 cue-
A. inacti ve
B. 0 RBes arC' on ly surfaced by Ii antigen
S. AB RBCs- have H ontigen with A or S sugar moiet ies
6. blood aroup 0 -
A. universal donor: refers to transfusion of packed ROCs nat whole blood
B. no antigens on surface and cannot be destroyed
C. must receive 0 blood
O. increased incidence of duodenal ulcers,
£. have 3 antibodies:
(I) anli·AlgM
(1) anli· B IgM
(3) anli·A.B IgO (can cross th. pl.cenla'
7. blood croup AD-
A. universal m:ipient
B. no Ilntibodies to destroy transfused RBes
8. blood croup A- '
A. hIlS anti·S JgM
B. increased incidence of gastric cancer
9. blood KrouP 8- has anti-A IgM
10. ABO ryplD& or mother and Cather (0 see Ir tbe cblld is Iheirl-
A. AS parents cannot have an 0 child
O. 0 parents cannot have an AS , A, or B child

27
<:r A BO bl00:Jd group summa ry:
Type Antibodies Whiles Blacks Asians Native Comments
Americans
0 anti-A IgM. 45% 49% 40010 79% Universa l do nor (no
anti· B IgM. anligens on the surface).
s nti-A,B IgG Anli-A,8 19G is res pon-
(can cross the s ible (or ABO incompa-
placental tibility. Can only receive
0 blood (antibodies
wou ld destroy A. B. or
AB blood). Pred isposi-
tion for duodenal ulcer
disease.
A anti-8lgM 40% 27% 28% 16% Predisposition for gastric
cll n:inoma .
B nnT i-A '"M 11% 20% 27% 4%
AB none present 4% 4% 5% < 1% Univcrslll recipient (no
antibodies),
Rh positive:
1. mea ns the patienl has 0 antigen
2. other Rh a nligens- C. c. E. e, d (does not exist)
Lewis a ntibodies :
I. na turally occurring IgM antibodies
2. DO dioicalsignlnc.ance
3. DO r isk of hemOlytic disease of newborn (RDN)

Dunyantigens:
1. uncommon in Afri can-Ame rica ns
2. s urface reuptor for P. ~i~ax- African-Americans lacking the antigen have protection
against malaria ,
1 aaugeos :
I. anti-I is a cold agglutinin (lgM)-
A. M. p"'!IImoniae infections
B. poss ible cold rype of aUlOimmu ne hemo lytic anemia
2. anti-i (lgM)-
A. infectious mononucleosis
B. possible cold type of autoimmune hemolytic anemia
Padent w ho is going to receiYe a packed RBC transfu s ion :
I. must perform an a ntibody sc.reen on patient serum- indirect Coombs test
2. major erossma teb -
A. purpose is to see if the patient has antibodies that will deStroy donor RBCs
B. patient serum againSl donor RBCs
C. good quality contro l on whether patient had an antibodies reacting against donor RBe
antigens
D. type n hypersensitivity reaction
E. does not prevent patient from:
( 1) developing antibodies against donor RBC antigens

28
(2) patient from gening infe ction: eMV Me. Hev. HIV. rmv
Me ullbody ILU.s. (USMLE)1 ""ti CMV antibody
Mos t co mmon infection trans milled by blood transfusion : CMV, not HCV. ",hich is the l\ICC
of posnransfusion hepatitis
MCC of posHrans fusion hep:ui lis ' HeV
1\1 Infectio n transmitt ed by accidental needle s tick: HB • \\hich has a high viral burden in
blood
HTV risk post. tra os fu sion: 1:676.000 risk per unit
HBV ris k post. tnmsrusion · I :200.000 risk per unit
HCV risk post-tr:msfusion: I :3300 risk per unit
Risk of HI V positivity posHlccidntaJ needle stick:
I. 1:300
2. M C way of becomin g HlV positive in medic:tl perso nnel
3. Rx wllh triple thera py for 6 months + repeat lnv se rology on a r eg ular bnsis
Fn's h frO 'l cn pl"~mll :
I. contain/! a ll coogulalion factors
2. clinical uses- multip le fact or deficiencies:
A. DIC
8. liver disease
C. \'itamin K deficiency
J. bepa titis risk
Packed RBCs:
t. high hematocrit- contains some plasma
2. trans fuse only if potient Is sy mptoma tic and does not res pond to medicJ&1 tbera p~'
3. hepa titis risk
Platelet tranduslon :
t. rain.s pla telet count 5000- 10.000 ceUslJ.1l. per unit
2. Iransfu.Je plate.lrts only if the patient is symptom a tic
3. bepatitis risk
Cryo precipitate:
I. blood produce. containlng-
A. all fac tor Vlllmolecules
D. fibrinoge n
C. facto r xm.
fibronectin
2. hepatitis risk
r:r Febrile tra.o fusioo reaction :
I. pa tient bas a nti·BLA anlibodie.s against RLA a nti gens on donor leukocyte.s
2. causes the release of pyrogens from destroyed donor leukocytes
3. type II h)-persensitivity
4. patients must ha\'e previous exposure to foreign liLA leukocyte antigens in order to develop
antibodies
A. common in pregnanc)'- fetal maternal bleeds
B. spontaneous abortions
C. previous transfusion

29
Allergic transfusion reaction :
1. Me type of reaction
2. patient develops bives against proteins in donor unit
3. type 1 hypersensitivity
HcmolyHc transfusion reaction :
l. units are ABO in compa tible-
A. patient rece ives wrong blood type
(1) e.g. , patient A (has ant i~ B IgM) and donor blood is 8-)0
(2) e.g., anti~B IgM attaches to B posit ive donor ce ll s-)o
(3) activates compl ement system-)o
(4) intr avascular hemo lysis
B. type II hypersensitivity reaction
2. patient has undetected antibodies tbat react against donor RBC antigens-
A. extravascular hemo lys is
B. jaundice, drop in Hb, positive direct Coombs: reaction may be delayed in some cases
C. Iy pe n hypersensiti vity
Hemolytic disease of the newborn (HDN) due to ABO incompatibility:
1. a mother with A or B baby-
A. 0 people normally have anti AB igG, whi ch crosses the placenta :
(,I ) antibody attaches to baby RB C antigens and splenic macrophages destroy RBCs
(2) mother's liver handles unconj ugated bilirubin released by fe t'al macrophages
(3) baby develops a mild anemia: weak antibody compared to nnti·D antibod ies
B. may occur in first pregnan cy
C. no increase in severity with future pregnanc ies
2. lab-
A. spherocytes in cord blood: nOl present in R.h HDN
B. pos itive direct Coombs on cord blood RBCs
C. mil d hemolytic anemia: rarely requires an exchan ge transfusion
3. clinical-
A. MCC of jaundice in first 24 hs: babies liver cannO{ handle excess VCB load
B. protects aga inst Rh sensitization:
( \) 0 patients have anti ·A 19M, anti~B IgM . and an ti · AB ~ l g
(2) anti A or B IgM antibodies immediately destroy fetal RBCs in matern al
c irculation
Rb immune globulin:
1. Ilnti-D from pooled buman donors-
A. passive immunization
B. does not cross the placenta
C. given to pregnant women who do not have anti~D at 28th week
D. no hepatitis risk
2. baby is Rh positive and motber does Dot have anti-D-
A. amo unt of Rh immune globulin to give is based on K leihauer-Betke test :
(1) test is performed on maternal blood
(2) detects amount of fetal~m atemal bleed (fetal RB Cs resistant t'O a lka li and acid)
B. give Rh immune glo bulin w ithin 3 days
C. immune globuli n destroys fetal RBCs in maternal circ ul ation

30
(if" Rh HDN:
I. motber Rh negative a nd b~ by Rh positive
2. first preg nDnc), with Rb + bab)' bas no effect on bab)'
J. mothe r could be ex posed 10 fetal RBCs witb 0 a ntigen durin g delh·try :lnd develo p
anlibodies- purpose ofRh immune globulin is to prevent this from occurring
4. if s ubsequent prel:nanc::ies ban Rb+ babiH-
A. maternal anti-O IgG antibodies cross placenta
n. attach to fe tal RBes ......
C. RBCs extravascularly removed by fetal macrophages in the spleen ....
D. unconjugilled bilirubin (UCS) is end-product of hemolys is (mo ther's liver handles
UCB)-+
E. fetus develops anemia (severe anemia. chance of hean fa ilure and hydrops fetnlis) ....
F. bilirubin pigment can be detected in amniotic flu id (optical wavelength of 450) and
mapped o n a Liley graph to determine severity of hemolysis -+
G at delivery, baby cannOI handle UCB load and is often exchange transfused to prevent
kernicteru s
S. ese hllnge transfusion:
A. removes UCB
8. removes antibodies
C. corrects anemia
Cro"-seclion of braiastem with yellowish discoloration in a child: probably kernicterus from
Rh hemo lytic disease of the newborn ~ 5/ ((IlC~jllj.'fo:d J;,(,'~ft f,'-N

A blood group O. Rh-negative woman is pregnant with her first child. She has a negotive antibody
screen and no previo us administration of Rh immune globu lin . She delivers 3 blood group A. Rh-
positive baby. The baby develops unconjugated hyperbilirubinemi a 8 hours after binh. Which of
the following statemenlS correctly describes this case?
A. She is Rh compatible with her baby
B. She is ABO compatible with ber baby
C_ She is a candidate for Rh immune gJobulin
D. Negative direct Coombs· of baby's cord blood
E. Jaundice is secondary 10 Rh incompatibility with destruction of fe tal ce ll s
c

31
,"<- I\. ~- •• , - ~ --

Cardiovascula r system

Summary of pbysical diagnostic signs in cardiovascular disease: Abbrc\'iations used- SI : first


heart sound, $2: second heart sound, 53: third heart sou nd . 54: fourth heart sound, MY : mitral va lve,
TV : tricuspid va lve. AV : aortic va lve, PY: pulmo nic va lve, TPR: tota l per ipheral resistance
1. valve locations for auscultation-
A. mitral valve (!vfV) at cardiac apex
B. tricuspid valve (TV) along left sterna l border
C. pulmonic valve (PV) in the left 2nd and 3rd intercostal space (JC5)
D. aortic valve (AV )- in the left sternal border o r right 2nd ICS
2. cardiac cycle rclalionships-
A. P wave: atrial depo lari zation
B. PR interva l: atriove ntricular conduction time
C. QRS: ventricular depolarization
D. 54: comes just before the QR5
E. Sl: comes at the same time as th e QRS
F. T wave : ventricular repoiarization, or recovery
G. 52: comes just after the T wave
3. 5. heart sound -
A. closure of the MY and TV during systole
B. MV closes before TV
C. normall y not sp lit
D. corresponds with carot id pulse
E. accentuated in early mitral stenos is when valve is sti ll pliable
4. S2 heart sound-
A. closure of the A V and PV in diastole
B. aonic component (A.:!:) precede s the pulmo nic component (P,)
c. S ~ spl its o n inspiration ow ing to blood entering right heart:
( I) negative intrathoracic pressure from diaphragm moving down
(2) del ayed closure of PY
(3) becomes a single sound o n expiration
D. Al acccntuatcd in essential hyperten sion
E. P! accentuated in pulmonary bypertension
Insp iration A! " - " PI (separates)
Expiration A2 P 2 (s ingle sound)
S. parad oxical split of Sl-
A. P ~ comes before A2 and split occurs on expiration rather than inspi ration
B. due either to deJay in AV c losure or early c losure of PV : e.g., left bundle branch bl ock
which de lavs A V closure)
6. fixed splitting of S!-
A. S2 is split during inspi rati on (nonnal) and expiration (abnormal)
B. due to delayed PV c losure AV closure A Y
C. c.g. , atrial septal defect where blood is always mov ing fro m the left atrium into the
right atrium and PV closes even later than it normall y does
7. physiologic SJ-
A. normal in children and YOWlg adults
B. occurs in early diastole

32
C. due to rapid ventricular filling with vibration or va lve cusp and ventricular waH or
impact of ventricle against chesl wall or sudden limital ion or longirudinal expansion or
LV
8. palhologit J-
A. abnormal aftrr JJO y old
B. due [ '0 blood entering a volume overloaded left or right ventricle
(I) e.g .. volume o\'erloaded left or right venlricle In left heart failure/right hean
fai lur:-
(2) e.g .. AV/MV regurgitat ion or pvrrv regurgitation
C. fiNI cardiBc Jign or congestive heart failure : Increased ventricu lar volume Slrelches
the MV o r TV rin g~ volume overload rro m mitral/tricuspid regurgit'llion
O. left-sided 5) increase s in intensity on expiration
E. right·sided SJ increases on inspiral ion (more blood in right hean 011 insp iration)
9. S~ heart Jouod-
A. coincides with atrial contraction in late diastolic filling and the ~ in thcJup.ula~
vcuoll.\.J!!!lg.
B. du~ to increased resistance to ventricular filling (decreased complinnce) rollowing
vigorous atrial contraction
C. decrcased compliance may be due to:
(1) ventricular hypertrophy (left/right) o r
(2) already volume overloaded ventricle (IetVright)
D. S, is absent in atrial fibrillation
E. left-sided S, increases in intensit) on expiration
F. righl·s ided 5, incfCllSts o n inspiration (morc blood in right heart on inspiration)
10. mid-syslOlic tjeclion dicks in MVnv prolapse-
A. click is due to ballooning or excess valve lissue into the atrium during systole
B. click is usul!lIy followed by a regurgiLation murmur
C. maneuvers that cause click/murmur to come closer to 51 are those that decrease volume
in the left/right ventricle SO systole occurs faster:
(1) slllnd ing up (decreased venous return to the hean)
(2) Va lsalva maneuver '
(3) anxiety (increased heart rate decreases filJ ing or ventricle in diastole)
D. maneuvers thai cause click/murmur to come c loser to S~ arc those thai increase Ihe
volume of blood in the ventricles so systo le takes longer to occur:
(I) lying down (increased venous rel'um to heart)
(1) sustained clenching of fist ( increases afterlond the ventricles ha ve to contract
against )
It. optninc snap 10 MVrrv stenosiJ-
A. occurs MVfTV (Jess common) stenosis
B. due ( 0 no npliable valve which has difficuhy in opening in diastole until atrial
contraction rorces it open
C. ope.lng snap early In diastolt indicates severe disuse: correlates with intense atrial
contraction
O. ope ning snap that occurs later in diastole indicalcs less SC\'cre stenosis: corre lales with
less intense atrial contraction 10 open the valve
12. heart murmur cbaraCleriJlics-
A. occ ur in syslole andlor diastole
B. mechanisms
(1) structural valvular disease

33
(2) anemia
(3) innocent (unrelated to structural or physiologic alterations, e.g., nonnal child)
C. murmurs radiate
( I) AV stenos is radiates into the neck
(2) MV regurgitation radiates into a.xilla
D. graded I to 6:
(I) grade 3 easy to hear
(2) grade 4 to 6 often accompanied by palpable precordial thrill
(3) grade 6 audible without stethoscope
E. murmurlheart sound intensity with respirations :
(1) right-sided murmurs and abnormal heart sounds increase on inspiration
(2) left-sided heart murmurs/abnomlal heart sounds increase on expiralion
13. contin uo us murmurs-
A. occur through systole and diastole
B. cerv ica l venous hum in children is MCC
C. patent ductus arterioslls (PDA): called a machinerY murmur
14. in nocent murm ur-
A. occur in - 50% of normal chi ldren
B. due to turbulent pu lmonary artery blood flow in systole
C. low grade ( I or 2)
D. best heard with patient in supine position
E. decreases wilh sitting/standing
15. ste nosis m urm u rs-
A. problems with opening valves
B. valves opening in systole: AV/PV
C. valves ope ning in diastole : MVnv
D. AVfPV stenosis-
(I) ejection type
(2) crescendo! decrescendo due to blood forcibly moving duough constricted
opening
(3) produce concentric hypertrophy ofleftlright ventricle, respectively
E. MVrrv stenosis- mid~diastolic murmur occurs after opening snap
16. regurgitation (insufficiency) murmu rs-
A. problem with c losing valves: incompetent
B. valves closing in systo le: MVfTV
C. valves closing in diasto le: AVIPV
D. MVrrv regurgitation :
(I) even intensity pansystolic (holosysto li c) munnur due to blood refiuxing into atria
throughout systo le
(2) volume overload of atria and ventricles (hypertrophy/dilatation)
E. AVfPY regurgilation-
(1) high pitched blowing murmur directly 3fter Sl
(2) volume overload of ventricles (hypertrophy/di latation)
17. Valsa lvo ma neu ver-
A. ho lding breath against a closed glottis: creates posi tive intrathoracic pressures
B. physiologic effects:
( 1) decreases venous return of blood to right hean
(2) decreases cardiac output

34
(3) increases hean rate as compensation for drop in cardiac output
C. effect on rnunnurs :
(I) increases intensity of systo lic munnur in idiopathic hypernophic subaortic
stenosis (hypertrophic cardiomyopathy): decreases left \'cntricle vo lume which
accentuates the degree of obstruction to blood now into the aorta
(2) moves systolic cl ick/murmur closer to SI in mitral valve prolapse
(3) dec reases intens ity of ejection munnur of aortic stenosis: less blood ejected
through the stenotic valve
18. lert pa rasternal heave-
A. sign o f right ventricular hypertrophy
B. right venrricle is located under the sternum
C. hypertrophy due to:
(I) pulmonary hypertension (caJled cor pulmonale)
(2) PV stenosis
(3) TV regurgitat ion
19. perica rdial knock-
A. sign of constrictive pericarditis
B. ca rdiac chambers hit thickened (often calcified) pericardium when filling in diastole
C. causes:
(1) TB worldwide
(2) cardiac surgery in this country
20. pe rica rdia! fricti on rub-
A. sign of pericarditis
B. scratchy 3 component sound heard on auscultation over the precordium:
C. due to separation of fibrino us exudate on visceraVparietal pericardial surface during
the cardiac cycle
D. CBuses:
( I) coxS3ckievirus infection (MCC)
(2) transmura l acute myocardial infarction
(3) Dressler's autoimmune pericarditis post infarction
(4) systemic lupus erythematosus (Me manifestation ofSLE )
(5) acute rheumatic fever
21. puillt press ure-
A. differe nce between systolic and diastolic pressure (nonna lly 30 10 40 mm Hg)
8. causes of a narrow pulsc pressure-
( 1) decreased stroke volume:
a. aortic stenosis
b. idiopathic hypertrophic subaor1ic stenosis
c. shock (cardiogenic, hypovolemic)
(2) increased total peripheral resistance (TPR) due to arteriolar vasoconstriction:
hypenension
C. causes of a widened pulse pressure:
(I> increased stroke volume:
a. A V!MY regurgitation
b. hyperthyroidism
(2) decreased TPR due to aneriolar vasodilatation:
a. endotoxemia
b. thiamine deficiency
c. wann shock in metabolic acidosis

35
(3) increased venous return to the right heart from arteriovenous fi stulas (arterial
venous com munications that bypass the microcirculation) :
a. traumatic (MeC)
b. mosaic bone in Paget's disease
(4) decreased TPR due to decreased viscosity of blood: severe anemia
22. a rlerial pulses-
A. nonnal arterial pulse:
( I ) normal upstroke is due to:
u. interaction between the driving force of left ventricular contraClion 3nd
b. compliance (elasticity) of the ve ssel
(2) initial . rapidly ejected stroke vo lume is ca lled anacrolic lim b
(3) pulse amplitude of upstroke modified by:
a. e lastic ity or arterial wall
b. amou nt of stroke volume
c. total periphera l arteriolar resistance (TPR)
d. blood pressure
(4) down stroke is mod ifi ed by:
n. vessel comp li ance (e lasticity)
b. TPR
(5) closure of aortic valve causes a momentary reversal in blood fl ow: produces it
dicrOlic notch in the down stroke
(6) smaller. secondary positive wave in the down stroke is due to: e lastic recoil of
the aorta and aortic valve
.B. elderly patients:
(1) aorta loses elasticity:
8. rigid
b. less compliant/less distensibility
(2) same stroke volume is distributed in an nona with a sma ller rad ius Icad in g to an
increased systolic pressure: poss ibil ity of systolic hyperten sion
C. bounding pulses (Corrigan pu lse, water hammer pu lse):
( I) inc reased moke vo lume:
8. AVIMV regurgitation
b. hyperthyroidism
c. high output card iac failure
(2) decreased TPR : aneriolar vasodi latation (see above)
(3) arter iovenous fistulas: see above
D. pulsus paradoxus:
(1) diminution in amplitude of pulse/blood pressure with inspiration:
B. drop in blood pressure drop> 10 rnm Hg during inspi ration
b. norma lly drops a linle on inspiration as blood filling tbe RV pushes on the
interventricu lar septum and decreases the LV volume
(2) pathophysio logy of pulsus paradoxus :
a. due to impedance of innow of blood into the right hean during
inspiration,.
b. automatically reduces the cardiac output and drops the blood pressure-+
c. neck veins distend on inspiration (called Kussmaul's sign. neck veins
should collapse on inspiration as blood is sucked into the right heart by the
negative intrathoracic pressure created by the diaphragms moving down )

36
(3) causes:
a. pericardial effusion (MeG)
h. decreased lung compliance:
• interstitial edema in severe asthma
• severe pulmonary edema
• interstitial fibrosis
23. Beck's triad- signs of pericardial effusion
A. neck vein distention on inspiration: Kussmaul's sign
B. hypotension: decreased cardiac ompUi
C. muffled heart sounds : fluid in the pericardial cav ity
24. jugular venous pulses (JVPs)-
• v

+-y

A. nonnal NPs
(1) 3 positive waves : a. c. and \'
(2) 2 negative waves: x. and y
8. a wave:
(I) positive wave due to atrial contraction in late diastole
(2) occurs after the P wave in an ECG
(3) disappears in atrial fibrillation
(4) giant a wave:
a. due to restricted filling of right hean
b. e.g., TV stenosis, pulmonary hy pertension
C. c wave:
(1) positive wave due-'to right ventricular contraction in systo le causing bUlging of
the tricuspid va lve into the right atrium
(2) correlates with 51 and upstroke of carotid pulse
D. x wave;
(1) large negative wave occupying most of systole
(2) due to downward displacement of TV valve when blood is ejected out of the
right ventricle into the pulmonary artery
E. v wave:
(I) positive wave due to right atrial filling in systo le when the TV is closed
(2) giant c·v wave: due to TV regurgitation as blood refluxes up into the right
atrium during systole
F. y wave:
(1) negative wave occupying most of diastole
(2) due to opening of TV with rapid fl ow of blood into the right vemricle In
diastole

37
<F 1 .
; in I
i TG PH Comments

2 85 3 8 I.carries diN-derived TG A. saturated fats , B.


su rfaced by apoiipoprotein 8 48• C. falsely elevates total
TG level if not Fasting. D. small amount of CJ-\ in
chy lomicrons does n Ol alter serum C1-I leve ls: no need to
fast
2.physiology- assembled in the small intestine and
I d~li~.red ")the i I circ ulation via tl,e I
VLDL 9 55 17 22 1.earries liver-de rived TC'.--- A. al so li ver·
de.rived CH, B. surfaced by apo lipoprotein 8' 00
2.LDL is derived from VLDL in the periph crlll
blood - capill ary lipoprotein lipase removes TG from
I VLD~~ 'leaves CH in the LDL i
LDL 20 10 45 121 u
I, bad cho lesterol"- A. derives from VL DL, B.
surfaced by apolipoprotein B [00
2. COL attaches to LDL receptors on cells (USMLE)-
A. inhibits HMG CoA reductase: prevents further
synthesis of CH, B. decreases LDL receptor synthesis, C.
inc reases esterificat ion by acyl-CoA acyl transferase
3. fU D dio n ~ O' r CH (USMLE)- ' A. steroid symhesis, B.
vitamin D synthesis, C. bile sa lt synthesis. D. cell

HDL 50 I3 20 125 I."good choJestero l"- apo lipoprotein A accompanies


HDL and parallel s HDL concenlration -'" ~cr.Vit~ LC
2. functions- A. reservoir for apolipoproteins C and E
that "'. subsequently attached lO circulating
chylom icrons and VLDL. B . HDL removes CI-I from
atherosclerot ic plaques (reverse CH transport ), C.
delivers CH lO the liver fo r disposa l
3. sy nthesis increased- A. estrogen, B. exerc ise. C. red
w in~
CH - cho lestero l, HDL "'" hi gh denSIty ltpoprotelns, HMG CoA - 3-hydroxy-3-mcthy lglutaryl coenzyme
A. LDL "'" low density lipoproteins, PH :: phospholi pid, TG :: tri glyceride. VLDL = very low density
lipoproteins
<F Fredrickson's phenotypic/genotypic classification:
1. type 1- deficiency of APO C n or capillary lipoprotei n lipase
2. ty pe D -
A. po lygenic hypercholestero lemia: 85% (M e Iype)
B. fami lial combined hypercho lesterolemia
C. familial hypercholestero lemia
, D. pathogenesis :
1,)/ \W~ "jhf1 \\J (1) absent or defective LD L receptors or
(2) defect ive internalization of LDL complex
Itr _pr E. clin ical fi ndings in familial hypercho lesterolemia :
i.t (1) premarure coronary artery disease (CAD) and myocardial infarcts
J~"l -:.e (2) Acbilles tcndon xanthomas are atho nomonic (75 %)
i,\\\: \;1 tIl. S ~ )(q~-!he14~ "'4S q~ e q l~D SeJ:/J
38
F. laboratory findings:
( I ) ~D~ > 190mgldL
(2) CH >260 mgldL
(3) nonnal TG or increased TG (type ri b if increased)
J. I)' pe 1.11-
A. familial dysbetalipoproteinemia : "remnant disease "
D. pilthogenesis :
( I) absent or defective apo E
(2) chylomic ron and intermediate de ns ity lipoprotein (lDL) remnan ts are nOI cl eared
C. labor-liory:
(I) CH and TG equaJly elevated
(2) identify APQ E isoforms
4. tYI1C IV-
A. familial hypcmiglyceridemia: Me type
B. pathogenesis: decreased catabolism ofVLDL
C. clinical:
(I) increased incidence of atherosclerosis involving coronnry arteries nnd peripheral
vessels
(2) eru ptive xan thomas (papules with TG)
O. laboratory:
(I) hypertriglyccridemia turbid infranatc-+
(2) slightly increased CH
5. type v-
A. familial hypenriglyccridemia with exacerbating fact ors like diabetic ketoacidosis or
alco ho lism
U. pathogenesis: combi nation of type I and type rv (J + TV = V) mechanisms, which
combi ne to equal a type V phenotype
C. clinica l:
(I) common in alcoho lics and diabetic ketoacidosis
(2) hyperchylomicronemia syndrome:
s. abdominal pain
b. pancreatitis
c. eruptive xanthomas
D. laboratory : supranate-+
(I) markedly increased TG with nomlal LDL infranate-+
(2) supranate and infranate presem: see Lab Medicine notes
Lipid depos its:
1. Achilles tendon xaotboma- familial hypercho lesterolemia
2. of xa ntheillsma-
A. yellow plaque on eyelid
B. consider type n hyperlipidemia
3. IIrCU$ seo ilis-
A. rim of white around the o uter pan of the cornea
B. consider increased LDL if a young pat ient or norma l age-related change if older
patient
..$. eruptive xanthomas-
A. yello\.... pnpulor lesions over the body
8. due to increased triglyceride: type rv or type V

39
Most common vessels involved in atherosclerosis in descending orde r:
I. abdominal aorta- no vasa vasorum below orifices of renal arteries
2. coronary artery
3. po pliteal artery
4. descending thoracic aorta
5. inte rn a l ca rotid
(jt" Atherosclerosis:
1. r eact ion to inju ry t heory- factors damaging endotheli al cells
A. c hemicals in c igarette smoke (e.g .. carbon monoxide)
B. LDLloxid ized LDL
C. previo us Chlamydia pnellmoniae infection
D. increased serum homocysteine: fo late deficiency MCC
2. fibro us plaque-
A. pathognomonic lesion of atherosclerosis
B, smooth muscle cells with CH deposits beneath intima
C. macrophages with CH deposits
@'" General complications or atheroscle rosis :
1. aneurysm fo rmation-
A. weakening of vessel wall with outpouching due to arterial pressure
(1) Law of Laplace states that wall stress increases with increased radius
( 2).', all aneurysms must enlarge as wall stress increases
') q Vn$IJ,(L!I'l ~ a6dom inal aortic llQeurysms is the Me type (picture on USMLE)
w.{ Ik{t'ti~ C. abdominal aorta is the MC site for atherosclerosis
I.f";.:.l? r 2. coronary a r tery disease- isc hemic heart disease
- I A. angina: MC manifestation
'''(1)'
SdJ,- C. B. acute myocardia l infarct ion
C. sudden cardiac death:
rr }fvt';+ ~
T (1) death within I hour
f 5"/1', (2) severe coronary artery alherosclerosis
(3) usually no"thrombosis present
D. chron ic ischem ic heart disease
3. peripb era l vascular disease-
A. predominantly lower extremity disease: may hear a bruit over involved vesse l
B. clinical:
(I) pain
(2) pulselessness
(3) paresthesias
(4) pallor
(5) paralysis
(6) gangrene: common in diabetics
(7) intennittent c laudication: pain on wa lking which is reli eved by resting
(8) Leriche syndrome :
a. Bortoil iac disease in male with tmpotence (hypogastric arteries involved)
b. claudication
c. atrophy of calf muscles
d. diminished/absem femoral pulses

40
4. drcle of WIUis disease-
A. cerebrnl infarttion
( 1) usually pale infm!
(2) MC type of stroke
B. cerebral atrophy:
(1) due to laminar necrosis of neu rons in layers J. 5 and 6 of the cerebra l cortex
(2) apoplosis of neurons- "red" neurons on H and E stnins
C. atherosclerotic aneurysms: e.g.. basilar anery
5. \'crtebrobasilar "fiery diJease--
A. transient ischemic attacks:
(1) dizziness
(2) vertigo
B. brain stem infarction •
6. Inlernal carotid Drury disease-
A. transient ischemic ' snacks: due olo embolization of plaq ue material with neurologic
deficits lasting <24 hours
n. atherosclerotic stroke:
( 1) usually occur at bifu rcation of the internal carotid artery
(2) due to pltllelet thrombus overlying atherosclerotic plaque
(3) prevented by aspirin
(4) ticlopidine used in aspirin allergy: problem with neutropenia
(5) control of higb blood pressure Is the single most important fDoctor tbal
prevtGu stro kes
C. embolic stroke:
(1) embolism of atheromatous plaque materiBI. or
(2) emboli from left hean
7. reoala.rtery dlsean-
A. renovasc ul.ar b)'pertcnsioD MCC of secondary hypertens ion
B. elderly men with severe aLherosclerosis and uncontrollBble hypencnsion
C. high renin hypertcns ion
O. epigastric bruit "
8. celiac. s upC!,;or and Inferior mesenteric artery disease-
A. smBIl bowel infarction:
( t) usually thrombosis/embolism invo lving superior mesenteric artery
(2) bowel distention/pain
(3) bloody diarrhea
B. large bowe l:
(1) symptoms occur in splenic flexure area : overlap area of blood supply involving
superior mesenteric/inferior mesenteric artery
(2) mesenteric angina: abdominal pain in splenic flexure area 30 minules afl'er eating
(3) ischemic colitis: splenic flexure pain + bloody diarrhea
(4) ischemic stricrures: healing of infarction by fibrosis leads to large bowel
obstruction from stricture fonnation
<7' Aa •• rys ms of ,b. vase.la. sym m: *'
U/t(q ~,J i5.f11<.;jlJ ",,-I,M r.R Ii.
l. abdominal aortic aneurysm - 4" 4'11~IlY.s,.., ~
A. MC type of aneurysm: .pI<IlI .... USMLE
n. Me in men ovcr the age of 5S
C. pathogenesiS

41
(I) weakening of the wall by atherosclerosis owing to no vasa vasorum below the
renal anery orifices
(2) Law of Laplace Siaies thaI as Lbo diameter increases the \\all stress
progressive ly increases: funher enlargement and rupture is inevitable
(3) aft'en accompanied by a popliteal artery aneurysm: danger of thrombosis
requiring leg amputation
D. clinica l:
(I) maj o rity are asymptomatic
(2) symptomatic:
D. pulsatile mass with mid-abdominal to lower back pain
b. abdominal bruit (50%)
c. rupture Me complication
(3) >f rupture triad: ..
u. abrupt onset of severe bl}.ck pain (most rupmre into. the , left
reuoperitoneum) ~ JQft t""~f";" J bit ,. ()~ ;~ N~~J/Rt q-I- .
b. hypotension
c. pulsariie mass
E. diagnosis:
(I) abdominal ultrasound is gold standard test
(2) size and risk for rupture color the R.x
2. berry aneurysm-
A. young or middle aged adults
B. most commonly located at bifurcations of cerebral vessels: e.g., anterior
communicating anery with anterior cerebral artery
C. pathogenesis:
(I) most are cODgeaitaJ: not usually atherosclerotic: in origin
(2) vessels lack an internal elastic membrane and muscle wall at bifurcations of
cerebral vessels
O. associations: any cause of hypenension predisposes to berT)' aneurysms:
(I) essential hypertension
(2) adult polycystic disease (10-15%)
(3) coarctation of the aona (increased pressure in cerebra l vesse ls)
E. clinica l:
(I) headache
(2) subarachnoid hemorrhage :
•. severe, occ ipital headache

F.
b.
diagnosis -
"worst headache 1 ever had" followed by 8 loss of consciousness

(1) CT scan/MR..I as a screen


-
(2) angiography confinns
3. mycotic aneu rysms-
A. weakening of the vessel wall secondary to an infectious processes:
(1) t .g., septic embolism
(2) infective eodQCarditis
(3) fungaJ vasculitis (A.spergillus. Mllcor. Candida are vessel invaders)
8. clinical
(1) thrombosis
(2) rupture

42
4. syphilhk a oeu rys m-
A. maJes 40-55 ys of age
B. ca rdiovlUcular maoifeSlatioD of te rtia ry syphilis
C. pathogenesis
( I) T. pallidum produces eada Mends oblitt rans (vascu litis with plasma cell
infiilratc) of vasa vasarum of ascending and transverse panions of arch of aorta
leads 10 ischemia of outer adventitial/medial tissue-+
\\eakening of aona.....
scar with reuaction of the underlying endothelial tissue producing "Iree barldng"
of the endothelial surface-t
aneurysmal dilatation of aon a-+
dilatation of aortic valve ring--+
aortic regurgitation-t
death from hean fail ure or rupture
D. clinicnl:
<I) aon ic regurgitation with dilatationlhypertrophy of left ventricle
(2) respiratory difficulties fro m airway encroachment
(3) brass)' cough from left recurrent laryngea l nerve irritation: stretched by dilated
norta
(4) angina: due coronary arteria ostia narrowing by fibrosis
(5) bounding pulses (water hammer pulse)
E. diagnosis
(\ ) aOl1ogruphy
( 2) calcification in the arth of aorta highly predictive of a syphilitic aneurysm
5. d iss«liog ao rtic an eurys m-
A. mean age of 60-65
8. men > women
C. MC catastrophic disorder of aorta
D. MC aneurysm of ascending ,aona
E. pathogenesis:
(\) pa,holog,v:
a. elast ic ti ssue fragme ntntion (95%)
b. mucoid degeneration: cystic medial necrosis
c. occurs in the middle and outer pan of the media
(2) associations:
a. -4 Marfan's syndrome (defect in fi brillin)
b.-=t Ehlers-Danlos syndrome (defect in collagen)
c. ~ pregnancy (increased plasma volume)
d. copper deficiency (cofactor in Iysyl oxidase)
f. coarctation ofaorta (wall Stress)
f. tnluma
F. cause of dissection:
( 1) hypertension app lies a shearing. fo rce to the imimal surface-+
intimal tear usually within IDem of the aonic valve-t
column of blood dissects under anerial pre,s5ure through the areas of weakness
and progresses proximally andlor distally
(2) eventual sites of ruprure:
a. pericardial sac (Me CO D)

43
f/ - .... ~ .""
b. mediastinum
e. peritoneum
d. reentry through another tear 1'0 creDle 3 double barreled aorta
G. types of dissections
(1) type A aneurysm:
a. Me and worst type
b. involves ascending aorta
(2) type B aneurysm: begin belo\\ the subcla\·ian ancr), Ilnd extend distal!)
H. clinical:
( 1) acute onset severe retrasternal chest pain with radiation into !.he back
(2) A V regurgitation: due to dilatation o f aonic valve ring by aneurysm
(3) signs of cardiac tamponade
(4) loss of upper extremity pulse
(5) stroke
I. diagnosis:
( I ) increased aortic diameter on chest ;'Hay (80%)
(2) retrograde arteriography is gold standard lest
(3) ovcraliiong-term surviva l 60%
J. @!Ueat prome oa -USMLE- anterior chest pain. widening of the aonic rOOl on
echocardiogram, death in 3 days by tamponade
OvervieM' o f , 'eno us system :
1. s uperficial veins (e.g., superficial sapbenous vei Ds) drain Into lhe deep " ei ns via
communicating (penetrating) branches.
2. " a lves prenn l blood flow rrom the deep into the J uperndn' venous system- e.xception is
around the ankles, blood flow is from the deep venous system to the sUJXrficial system
Phlebothrombosis and tbrombophlebitis:
t. pble bothrombos is- thrombosis of a vein without inflammation.
2. ,'uous dOH in d esce nding order of rrequency :
A. de '\'Cia in be calr
B. femoral ve in ~
C. popliteal vein
D. iliac vein
3. pred isposi ng fact ors for phlcbothrombosis-
A. damage to the vesse l endothelium (e .g .• inflammation. varicose veins)
B. stasis of b lood fl ow (e.g., bedrest)
C. hypercoagulability (e.g., oral contraceptives).
4. dotting proeess-
A. begins in stasis areas such as the venous sinuses of the calf muscles and in the valve
cusps
B. plate lets fonn the initial clot in the valve cusps
C. developing clot extends beyond the next branching point at which juncture the clot
becomes a venous clot ( red thrombus) consisting of a mixture of RBCs and fibrin
D. venous clot propagates towards the hean. in the dir«tion of blood fl ow: danger of
embolization
S. cJiDica l-
A. swelling. pain, edema distal to the thrombosis
B. develo pment of varicosities and ulceration.

44
6. dee p ve nous thrombosis (DVf) in the lowe r u trem it)'-
A. produces deep venous insufficiency: post-phlebitic syndrome
D. deep saphenous vein thrombosis leads 10 an increased venous pressure and increased
blood fl ow to veins arou nd the ankles which communicate wilh the superficial s) stem
C. veins in the ankles rupture resulting:
( I) s tas is d . t itis (swellin g.. hemorrhage, ulcers)
(2) secondary va ricosi ties in the superficial saphc n us system
7. co m pliclltio DS or venous th rom bosis indud e-
A. thromboembol ism:
(I) potential for a pulmonary embolism with infarction
(2) femor al niD Me site ror embolizatio n
n. thrombophlebitis
C. varicose veins
8. Doppler (d uplex) ultrasonogra phy-
A. best screen ing test for detection of deep venous thrombos is
B. X-ray venography is gold standard lest
9. Ihro mbophlcbitis-
A. pain and tenderness along the course ofa suoerlicia l (nol deep) vein
D. causes:
(I ) MeC is superfici al varicose veins
(2) phlebothrombosis
(3) intravenous catheters
(4) intravenous drug abuse
C. clinical:
(I ) palpab le cord
(2) pain, induration, heat. erythema
10. migra tory t hrombophlebitis- subtype of thrombophlebitis:
A. venous thrombi disappear at one site and reappear at another
D. may be a paraneoplastic sign of underlying pancreatic cancer (Trousseau's sign)
r:r Varicose veias:
I. dtn nitio n-
A. abnonnally distended. lengthened and tortuous veins IlSsociated with :
<I} superficial saphenous veins: Me site
(2) distal esophagus in porta l hypenension
(3) anorecta l region (e.g. , hemorrhoid s)
(4) left testicle (varicocele)
D. superficia l saphenous vein varicosities Me in women
(1) primary varicose veins are due 10
a. valvular incompetence (sentinel va lve)
b. weakened vesse! walls
c. positive fa mily history
d. occ upation of the individual: standing
(2) secondary varicose veins :
a. damage of the valves from previous thrombophlebitis or
b. deep vein thrombos is.

45
Tborueic outl el syndrome:
I. definidoo- abnormal compression of the neurovascular compartment in the neck
2. caUSH-
A. cervical rib
8 . spastic scalenus anticus muscle
C. positional changes in the nec.ld anns
Subclavian 5Ieu l.!l~· ndrome :
1. definition - due to proximal obstruction of lhe first portion of the subclavian ancl') :
2. Cil uses rc\'er5ll1 of blood noVo' in Ibe ,'crlebral arte ry (produces cerebral iscbemi:l) 10
s upply blood 10 tbe arm
Superior vena caval (SVC) syndrome:
I, seco nd a ry to eX1rinsic compression or tbe SVC rrom Il primlilry lung cancer (90%)
2. puffiness and blue to purple disco loration of tb e face, arms Bnd shoulders
J, CNS fincHngs or dizziness, convulsions, and visual disturbances (congested retinal
veins).
4. dislcnded jugular veins
Lympluuic disorders:
I. Iympbutic vessels bne an incomplete basemen I membrane- predisposes [0 infection and
tumor invasion
2. acUle Iymphangitis-
A. inflammation of lymphatics (Ured streak',)
B, Streptococcus p.rogenes MCC: e.g.. cellul itis
J. lymphedema- interstitial collection oflymphatic flu id due to:
A, congenital disease:
(1) Milroy's disease
(2) Turner's syndrome
B. blockage of the lymphatics:
(1) radiation post-mastectomy
(2) peau d'oraqge of breast in inflammatory carcinoma
(3) filariasis
4. chylous drusioD~ -
A. collection of lymphatic nuid in 0 body cavity:
O. contains chylomicrons with TO plus mature lymphocytes
C. causes of chylous effusions in pleural cavity:
(1) MCC is malignant lymphoma
(2) U'auma to thoracic duct
Benign/malignant vascular tumors and tumor-Uke condltlon.s:
I, Sturge-Weber syndrome (SWS) syndromt-
A. pon wine stain in the distribution of the ophthalmic branch of the trigemina l nen'e.
B. CNS vascular abnormalities often located in the leptomeninges on the ipsilateral side.
(I) calcify (look like railroad tracks)
(2) bleed
(3) focu s for seizure activity
C. mental retardation
2. hereditary hemorrh agic telaDgiec::ta~ la (Os ler Weber Rendu disease)-
A. AD disease : MC genetic vascular disorder
B. small aoeurysmaltelangiectnsias 00 the skin/mucous membranes

46
C. e pistaxis
O. GI bleeds: iron deficiency
3. ~ p ider tela ngicctasias-
A. sma ll arteriovenous communications
B. associated with hyperestrini sm:
( 1) pregnancy
(2) cirrhosis
C. compression of the. "body" of the sp ider causes the blood now to the " legs" to
di sappear: petechia do not blanch W-.SM):;E,)
4. ungio myo lipoma-
A. haman oma composed orb lood vessels, muscle and mature adipose tissue
B. Me in kidneys
C. association with tuberous sclerosis
5. bacillary aogiomalosis-
A. infectious disease caused by Bartonella henselne and Bartonella <11I;mallo
B. associ ated wit h AIDS : simulates Kaposi's sarcoma
C. benign capillary proliferation on the skin andlor visceral organs
O. ~ il vu ~lai n s identify them in tissue
E. organism also causes cat scralch disease
6. oapllJUy _ gIo".T
A. benign tumorl'l hamartoma of mature capillary channels
B. "strawberry type"' commonly seen on the face in newborns:
(I) slightly raised, bright red. lobulated vascu lar tumors
(2) 80% totally gone by 8 years
(3) no treatment necessary '(1JSMLE)
7. cave rnous hemaogioma-
A. Me benign tumor of the liver/spleen. and placenta (called c holangioma)
B. may rupture in liver/spleen
C. associat ion with vo n Rippel-Lindau d isease:
( 1) AD di sease
(2) cavernous hemangiomas or the cerebellum (cerebellar heman gioblastoma), brain
stem. eyes
(3) increased incidence of renal adenocarcinoma Ilnd pheOC hromocytoma
8. anglosarcoma-
A. sarcoma derived from vesse l endothelium
B. liver angiosarcomas: assoc iated with exposure to:
(1) vinyl chloride (Me)
(2) arsenic
(3) thorotrasl
C. electron microscopy reveals Weibel·Polade bodies:
( l ) structural hallmark of endothe lial ce lls
(2) contain von Wiliebrand's fac tor
D. histochemical Stains positive for factor VOl
9. Kaposi's sarcoma-
A. malignant tumor arising fro m endothelial cells (some debate on th is origin)
B. immunodeficiency variant develops in patient's taking immunosuppressive drugs (e.g ..
renal transp lant patient)
C. AIDS varian t:
(1) MC cancer in AIDS

47
(2) AlDS-defining lesion
(3) due to Herpesvirus 8
(4) solitary to mUltiple red ·purple lesions thnl progress from a nat lesion (macule) ro
n plaque to a nodule that ulcerates
(5) microscopic:
a. spindle cells (neoplastic element) with in reased mi totic activity
b. hemosiderin deposition
(6) locations:
a. skin (MC location. picture on ,USMLE)
b. mucocutaneous surfaces (oropharynx Me location)
c. visceral locations ( lung. GI tract, lymph nodes)
(7) R:c
A. c hemotherapy
B. intralesiona l a ·interferon (lISMLE)
10. Iymphaogiusa rcoma- complicates chronic lymphedema due to modified radica l
mastectomy
Llirge vessel vasc ulitis :
1. dcnnllion : Ilorta to large/medium sized muscular arteries
2. gianl cell (tempora.l) a.rteritis-
A. >50 years of age
B. women more often involved than men (2: I )
C. mullifocal granulomatous vasculitis with multinucleated giant cells
D. primnrily involves the temporal artery and extracranial branches of the carotid artery.
E. c linica1:
( I ) fever
(2) unUateral headache along C:OU I"W or temporal artery: MC symptom
(3) jaw claudication
(4) temporarylpennanent blindness on ipsilateral s ide
(5) polymyalgia rheumatica:
a. above.. signs and symptoms plus pain and morning stiffness (>30 minutes)
in neck, shou lders. and hip
b. no e levation or serum CK. unlike polymyositis
f. la b:
(1) e levated erythrocyte sedimentation rate (ESR): best screening test
(2) definitive diagnosis: temporal artery biopsy
G. R.J: steroids
3. Takayasu artcrltis (pulseless dlsease)-
A. Asian women <50 yean old
n. granulomatous vasculitis involving aonic arch vessels
C. clinical:
( I) abscnt upper extremity pulse
(2) blood pressure discrepancy between extremities: low in upper and higher in
lower extremities
(3) Raynaud's phenomenon
(4) visua l distUrbances
(5) stroke
D. diagnosis: angiography

48
Medium to small muscular arte ry vasculitis:
I. pol yarteritis nodosa (PAN)-
A. MC in middle aged men
I(A o. necrotizing immune vascu litis of small to medium s ized arteries
( I) immunocomplex vasculitis:
a. t)'pC ill hypersensitivity
b. immunocornplexes may involve HBsAg
(2) ac:,tivation of neulrophilslmonocytes by anti·ncutroph il cytop lasm ic: anti bodies:
p-ANCA type
(3) re lease of enzymes by neutrophils contributes to inflammation
c. p3thology:
( I ) lesions are in different stages o f development: acute o r heal ing stage
(2) foca l vessel involvement leading to aneurysm
(J) neutrophilic/eosinophili c infi ltrate
D. clinical associations:
(I) jlJlYJuJJl&.Jl.mia: 30-40%
(2) hypersensitivity to drugs: e.g., intravenous amphetamines
E. target organs (o rder of decreasing frequency)
(I) kidneys:
a. vasculilislglomerulonephritis
b. infarctio n
(2) coro nary arteries: aneurysms
(3) liver: ca lcification of cystic duct artery
(4) Gl tract: bowel infarctio n
(5) skin : painful nodules represenring vasculitis in subcutaneo us fal
(6) lung: nOI commonly invoh-cd in c lassic PAN
F. clinical
(I) fever
(2) pain in areas of invo lvement
(3) multisystem diseasJ with infarction
G. la b:
( 1) peripheral neulTophiUc leukocytosis/eosinophili a
(2) anemia
(3) positive HBsAg
(4) anti·neu lrophil cytoplasmic antibodies with perinuclear siai ning: p-ANCA
(5) hematuria with RBC casts: glomerulonephriti s
H. diagno5iJ:
(1) wteriography and/or biopsy of pa lpable nodulations in the skin or organ involved
(2) rena l failure MC COD
2. Kawasa ki Diseue-
A. affl icts children <5 years old
B. peak incidence in spring
C. probable immune mechanisms involved
D. transmural innammation of vessels with neutrophils and mild fib rinoid necrosis:
( 1) leads to aneurysms/thrombosis
(2) coronary arlery vasculitis
E. clinical:
(I ) MCC of an aCUle myoca rdial infarction In cbildren

49
(2) erythematous rash of lrunk and extremities with desq uamation al finger tips
(3) mucosal inflammation: cracked lips. oral erythema
(4) erythema, induration. swelling or hands and feet
(5) loca lized lymphadenopathy
F. I. b:
(I) neutrophilic leukOtytos is
(2) thrombocytosis: characteristic finding
(3) high ESR
(~) abnormal ECG ifmyocard ilis presenl
G. diag nosis: echocardiogram screen for coronary aneurysm
H. Rx:
(I) intravenous gamma globu lin
(2) steroids contraindicated: increase risk for coronary aneurys ms
3. Ihromboangiitis obliterans (Buerger's disease)- 80.'1(10; d:~
A. young to middle aged eigarene smoking males
B. unknown prod uct in tobacco smoke: tox ins prod uce direct damage [ 0 endothelium
C. innammatory vasculitis:
(1) in vo lves the whole neurovascu lar compa nment of medi um sized and small
arteries in the extremit ies
(2) thrombus in vessels contains foca l neutrophilic microabscesses: vessels
eventually fibrose
O. clinical:
( 1) claudication in the feet or hands
(2) Raynaud's phenomenon
(3) distal gangrene of the digits often requiring amputat ion (co mmon USl\lLE
piclure)
E. diagnosis: biopsy
F. R:"C: early stages of vasculitis frequently cease on discontinuation of smoking
r:r- Small vessel vasculitis :
I. definilion- ,
A. invo lves arterio les, capill aries and venul es
B. sma ll vessel vasculitis (hypersensitiviry vascul it ides):
(1 ) leukocYlociastic nDutilis: vasculitis usually involves post-capillary venules
(2) innammation is at the same stage in all vesse ls (u nlike PAN):
a. neutrophil ic infiltrate
b. nuclear debris
c. fibrinoid necrosis
(3) majority are type m immuDocomplu diseases
C-I) all present with "palpable purpura": purpura due to platelet abnonnalities or
non-immuno logic weakness of small vesse ls is not palpable
2. Heaocb-& hoalein purpura (HSP)-
A. M C vasculitis in children
B. immune vasculitis mostly occurring In children following an upper respirutory
infection
(1) IgA-C3 immunocomplexes deposited in vessel wslls
(2) JgA nephropathy (Berger's disease) may be part oflhe syndrome complex
C. cli nical:
(I) 81 able urpura commonly lim ited to the lower extrcmitieslbunocks

h",,~ (o"fRt le('wii',J


50
(2) polyarthritis
(3) abdomina l pain: sometimes with melena
(4) renal disease: hematuria due to focal proliferative glomerulonephritis
D. lab:
(I) neutrophi lic leukocytosis
(2) normal platelet coum
(3) increased IgA
E. Rx: st.eroids
J. a Uloimmune d isease: a nd v8Sculiti.!l-
A. immunocomplex vasculitis
B. types of aUioimmune disease:
( I) SLE
(2) rheumatoid arthritis
(3) progressive systemic sclerosis
C. target organs:
( 1) sk in
(2) kidneys
(3) bruin
4. serum s ickne:ss-
A. prototype of systemic immunocomplex disease
B. so luble immunOC<lmplexes due to antigen e xcess
C. pan ic ularly common in the Rx of ran lesnake e nvenomations with horse·based
antitoxins: antibod ies are synthesized aga inst fo reign antigens fo llowed by the
fo rmalion of immunocomplexes with run.her exposure 10 antitoxin
D. clinical:
( I) fever
(2) urticaria
(3) genera lized lymphadenopathy
(4) arthritis
(5) glomerulonephritis
E. serum sickness-like pr6drome in HBV a nd HeV
Miscellaneous vaJc ulitides:
J. Wegener's G r a nuloma tosis (WG)-
A. childh ood to midd le age: mean age of 40
B. necrotizing granulomatous inflammation:
( I) upper/lower respiratory tract
(2) nec rotizing granulomatous vasculitis in kidneys
(3) anti-neutrophil cytoplasmic antibod ies (c-ANCA type): activate neulToph ilsi
monocytes with subseq uent vessel/tissue damage
C. clinical:
( I) s inusitis
(2) sadd le nose defonnity;
.iii . nasal cart ilage destroyed
b. absent nasal hairs
(3) upper a irway collapse
(4) lung disease:
a. recurrent pneumonia
b. large angioceDlric dens ities on x-ray

5I
(5) necrotizing glomerulonephritis: renal failure
D. lab:
( I) neutrophi lic/eosinophilic leukocytosis
(2) anti.neutrophi l cytoplasmic antibodies with diffuse c)l\oplasmic panem (c-
ANCA): specifi c for Wegener's and mBrker of disease activity
E, diagnosis; biopsy
F. Rx
(1) cycJopbospbamjde: danger of hemorrhagic cystitis and transitional cell
£,arcino!!1a
(2) steroids
(3) high monality iflefi untreated
2. lymphomatoid granulomatosis-
A. si milar to Wegener's granulomatosis
B. differences from WG :
( I) absence of upper resp iratory invo lvement
(2) progresses to a malignant lymphoma i1150% cfeases
3. Raynllud 'J pbenomeooD-
A. IIrcerial insufficiency of the digital vessels
(1) primary disease (rare)
(2) most often secondary to o ther diseases
B. pathogenes~ :
( I) association with collagen vascular diseasc:
8. SLE
b. progressive systemic sclerosis (first sign): starts as vasculitis of vessels
and ends with fibrosis
c. CREST syndrome:
• £il1cinosis of finger tips/£enlromere antibody
• Raynaud's phenomenon
• £sophageal motility prob ems )
• '!clerodactyly ( -t., hr'S""
• !elangiectasia
(2) association with cold reacting (lgM) antibodiesfglobulins:
n. cryoglobulinemia: usually elderly patient with rheumatoid arthritis or
patient w ith Hey
b. cold agglutin in diseases
(3) other assoc iations:
A. thromboangiitis ob literans
b. Takayasu's aneritis
c. ergot poisoning (vasoconstriction)
d. thoracic outlet syndrome
C. dinical:
(1) cold temperatures and stress are stimul i that may tri gger the color changes of the
fingers-+ white-+ blue-+ red
(2) ears and nose cyanotic
(3) often relieved by warmth
4. Infectious vll5culitiJ-
A. fung al vasc ulitis; vessel invading fungi :
(1) Candida

S2
(2) .~spergil/us
(3) Mucor species
8. Rocky Mo unt ai n s potted (ever:
(I) rickensial disease caused by Ricketula ricKtlls/at
(2) transmined by the hard tick Dermactmor anderson"
(3) organisms invade vessel endothelium of arteriole- cnules:
("') inflammation and ruprure of weakened \'t~5sel-+ classic petechial leSIons begin
on the soles and Raims ofhand-t s.Qread 10 trunk (cenlripelalspread
(5) classic triad : - - ~ "> ~' ./5 {, 1t,J I"'r.
a. rash 1'tG'I""S" -Ir. .. .((~.. (
b. fever
c. history or lick bite
C. di se mina ted meningococcemia (Neisseria mt ni" gitidis):
(I) sma ll vessel \·a.scu litis associated with cap illary thrombosis and petechial
hemorrhages
(2) sepsis precipitates o le
(3) Waterhouse Friderichsen'$ synd rome: ncule adrenal insufficienc) due to
hemorrhage
D. disse miRillcd gonococcemia (Neisseria gonorrllOeut·)
( 1) sma ll vessel vasculitis located on the hands, wrist. and fcct
(2) septic arthritis: usually knee
(3) C5-C9 deficiencies
E. vira l vasculltu:
( 1) hepatitis Band C: immuDocomplex
(2) rubella
f. Infective endoca rditis : immunocomplex vasculitis:
( I) Roth's spots in retina
(2) Janeway's lesions on hands (pain less)
(3) Osler'. nodes on hands (painful)
(4) glomerulonephritis ,
<r U M I Z _ . o ....lard......:
1. . . . .pprop...... as for . . eIderty WOIIIU wi.1I • IInd.clle loa.IizeCI 10 die Ildf of bu
<lIOiiI occo• .,.aIed by dlll"lue ,........... lad pal.., lad Ia .I""••td ESR
A. temporal arteritis wi th polymyalgia
B. R.x w ith corticosteroids
2. weltll' HIler witll w..klleu fa tile bud, aumb ..... aad a.al p.lIe-
A. thoracic outlet syndrome
B. scalenus anticus muscle spasm or cervical rib compresses the subclavian artery nnd
brachial plex us
3. p6ctDre of . . . . . . . . . . oa face-of. cII.iId- leave it alone
4. calc1ftcadoa of d",taJ veueIF-- diabetes mellitus
S. MCC of . . anedovuou n.~ trauma :
A. usually a knife inj ury
8. may cause high output cardiac fail ure
Chassi fi c.a lion of bype rtenJion:
I. Huo lia l hypenension- 950/.
2. secondary bypertension-
A. renal disease:

53
(I) renovascular disease: MCC secondary h )' pcru~ nsloD
a. elderly male with atherosclerosis
b. middle aged female with libromuscular hypt'rplasia
(2) renal parenchymal di sease
B. adrenal di sease:
(1) Cushing's syndrome
(2) pheochromocytoma
(3) Conn's (primary aldosteronism)
C. parathyroid disease: primary hyperparathyro idi sm
D. thyroid disease:
(1) severe hypothyroidism: diastolic hypenens ion
(2) Grave's disease: systolic hypenension
E. vascular disease: coarctation of the aona
F. eNS disease:
( I ) increased intracranial prcssure
(2) poliomyelitis
G. gynecologic disease : toxemia of prcgnancy: pregnanc), induced hypcnension
H. drugs:
( I) oral conlraceptives (MCC in women in reproductive age): estrogen increases
liver synthesis of angiotensinogen
(2) alcohol
(3) cocaine
Essen tial bypencosioo:
I. systolic blood pressure correlates wiib s troke ..'olumc
2. diastolic blood pressure correlates wilh slate of contraction of tota l peripberal
resistAnce (TP R) anerioles-
A. diastolic blood pressure is the amount of blood in the an'cries while the hean is filling
up in diastole
B. amount of blood in arteries is related 10 Ihe arterioles, which contro l the TPR
C. abbreviated Poiseuille's equation states that tota l peripheral resistance (TPR) -=
Viscosity ofb loOdl(radius oflhe aneriolet
( I) vasodilatation of arterioles has greatest effect On lowering TPR
(2) vasodilation of arterio les decreases TPR and decreases the diastolic blood
pressure
(3) vasoconstriction of anerioles increases TPR and increases diastolic blood
pressure
J. Me type of bypertens ion-
A. high incidence in African-Americans
B. pathogenesis in most cases:
(1) retention of sodium raises the pla.sma volume lead ing to an increase in stroke
volume (increased systolic pressure)
(2) sodium in smooth muscle cells of peripheral resistance vessels (anerioles opens
up calcium channels leading to vasoconstriclion and an increase in diastol ic
pressure
(3) African-Americans have low renin hype n ension: due 10 an increase in plasma
volume
C. pathology associated with hypcnension:
<I) concentric left ventricular hypenrophy:

S4
a. Me complication
h. potential for left heart failure
(2) acute myocardial infarction: M e COD
(3) hyali ne arterio losclerosis:
a. insudation of protein into wa lls of nnerio les by increased luminal pressure
narrow vessel lumen
b. produces small vesse l ischemi a
(4) nephroscleros is in the kidneys:
a. shrunken kidneys of hypertension
b. cortical atrophy due to hyali ne arterioloscleros is
c. eventua l proteinuria and c hronic rena l fa ilure
(5) intrllcran ial bleeds in the putamen area : due to ruprured Charcot· Bouchard
aneurysms of lenticulostriate vessel s
(6) lacunar infarcts:
a. due to hyaline arteriolosc lerosis
b. pure motor or sensory strokes
4. dinical- accentuated A2
Reno,\'ascular bypertension :
l.
A.
~
pathogenes is:
-
Me secondurv cause of bypertension-
( 1) atherollclerosi! ofrenai a rtery orifice In male
(2) fibromlUcular- hyperplasia of renal artery in a woma n:
a. hyperplasia of smooth muscle in arteries narrow lumen
b. "beaded" appearance of arteries noted with arteriogram
B. clinical:
(1) severe hypertension
(2) epigastric bruit
c. laboratory:
(1) higb renin hypertension -
I,
"L 1),,1It ",,-'W-t
tle~fc.y ,<0,.
,.,..r
(2) involved kidney h<4. increased renal vein plasma ren in activity
(3) uninvolved kid ney has suppressed renal vein plasma renin activity
(4) caplo pri l marked ly increases baseline plasma renin activity:
a. used as a screening test a long with a renal scan
b. renal scan shows decreased uptake of affected kidney. decreased size or
affected kidney, decreased excretion in affected kidney
Plasma renin activity (PRA) in dirfercnt conditionll:
I. lIodium de pletion -
A. increased PRA
B. volume depletion stimulates renin re lease
2. uprig ht pOllture-
A. increased PRA
B. decreases venous return to bean-+ decreases card iac OU1PU1-+ stimulation or ren in
re lease
C. recumbent posture does the opposite: inc reases venous retum-+ increases card iac
outp ut-+ decreases ren in release
3. catecbolaminell-
A. increased PRA
B. directly stimulate renin release

55
C. ~-b l ockers do the opposite
4. captopril (ACE iobibitor)-
A. increased PRA
B. excellem screen fo r renovascular hypertension:
( 1) see exaggerated levels post-stimulation owing 10 block of angiotensin U and loss
of negalh'e feedback on renin
(2) potential for renal failure if bilateral renal anery stenosis is presem (USl\tLE ).
a. angiotensin 0 is primary modulator of inU'arenal blood fl ow in both
kidneys
b. ACE inhi bitor reduces AT II level s and causes renal failure
S. Luix-
A. increased PRA
B. stimulation of RAA system from volume depletion
6. aldosterone blocker (spironolactone)-
A. inc reased PRA
B. vo lume depletion from sod ium loss
C. loss of negative feedback of aldosterone on renin
7. renovasc ular hype rtensioo- increased PRA
8. malignant hypc rteDsioo- increased PRA
9. young hypertcnsives-
A. increased PRA
B. sympathetic nervous system overactivity
10. Afric.an-American hypertea.sives-
A. decreased PRA
8. increased blood volume from sodium retention
C. reason why diuretics are first choice in Rx
II . sod ium overloadJprimary aJdostf: ronls m-
A. decreased PRA
8. increased blood volume from sodium retention
12. elderly hypertensives-
A. decreased PRA I

8. volume overload suppresses renin


Co ncenlric hypertrophy of left ven tricle: due to increased oftcrload-
1. c~~ e ntial hypertension MCC
2. aortic s ten osis
Hypertrophy a nd dilatation of left ventricle: due to vo lume overload of the vemricles-
I. aortic or mitral valve regurgitation
2. lefllo riGbt shu nlS with increased relurn io Idl beart
J. aortic ""Ive riog diiatatioD-
A. dissection
8 . aortitis
4. mitral valve ri ng dilatation- left hean failu re
Left beart failure (LHF):
1. mecbanis ms of beart failure-
A. diminished ventricular contraction:
(1) ischemia MCC
(2) replacement by fibrou s tissue

56
(3) myocarditis
B. non-compliant (restric ted filling):
( 1) amyloidosis
(2) iron overload
(3) glycogen in Pompc's disease
C. increased workload:
(I) increased afterload
(2) increased preload (volume overloaded)
O. blood builds up behind the failed left heart:
increase in left ventricular volume/pressure-+
increase in left atrial pressu re-+
increase in pulmonary venous pressu re-+
hydrostatic pressure overrides pulmonary capillary oncotic pressurt4 pulmonary
edema
2. clinical findin gs-
A. symptoms outnumber signs
B. decreased cardiac output: forward failure
C. dyspnea: stimulation of J receptors in pu lmonary interstitium
D. pu lmonary edema: due to increased pUlmonary venous hydrostatic pressure
E. left ventricular dilatation:
( 1) volume overload
(2) Frank·Starling mechanisms al work
F. left-sided S3 heart sound: volume overloaded left ventricle
C. left·sided S.. heart sound : decreased compliance
H. mitral regurgiullion:
( 1) stret'ching of miU'a1 valve ring
(2) pnnsyslolic munnur al apcx with radiation in to Ilxilla (increases with expirat ion)
I. pa roxys mal nocturnal d ys p~ ea andlor pillow orthopnea 8t night:
( 1) increased venous retum at night due to lack of gravity effect when lying down
cannot be handled by the left hean--+ blood backs up into the lungs
(2) st.anding up or placing pillows under the head decreases venous retum to the
hean
J. cough:
(1) sputum rusty colored from alveolar macrophagcs phagocytosing RBes
(2) "heart fai lure" cells
J. systolic dys function type or LHF-
A. due to decreased contractility:
(1) ejection fraction (EF) <0.40
(2) EF :: Stroke volume/left ventricular end-diastolic volume
(3) normal EF m 80/120 - 0.66
n. causes: ischemic injury MCC
C. R.x:
(1) use of inotropic agents (e.g.• digitalis)
(2) decrease afterload with vasodilators (e.g.. ACE inhibitors)
4. dillStolic dysfunction 'Ype or LHF-
A. due to de<:reased compliance of left ventricle:
( I) places increased load on the left atrium
(2) pulmonary edema common

57
(3) EF >0.40
B. causes:
(1) left ventricular hypertrophy
(2) restrictive cardiomyopathy
(3) hypertrophic cardiomyopathy
C. Rx : increase preload by slowing heart rate (e.g.. calcium channel blockers. J}-blockers)
S. USMLE sceoario!-
A. chest x-ray appearance in heart failure:
( I) prominent congeslion of blood in the upper lobes
(2) perihilar congestion: "bat-wing configuration
(3) Kerley lines: represent fluid in the interlobular septa
(4) patchy interstitial and alveolar infiltrates
(5) pleural effusion
B. microscopic appearance of lung in heart fai lure :
(1) pulmonary edema: transudate
(2) alveo lar macrophages wi th hemoside rin ("heart ra ilure" cells)
Righ i hea rt fail ure (RHF):
I. mecbanisms simila r 10 left bean failure (L HF)-
A. LHF isMCC
B. right ventricular infarction
2. blood builds up bebind tbe failed right burt-
A. increase in right ventricu lar vo lume/pressure ...
B. increase in right atrial pressure ...
C. increase in jugular venous pressure...
D. hydrostatic pressure overrides pulmonary capillary oncotic pressuf'e-io
[. dependent pining edema + ascites
J. clinica l findings -
A. signs oumumber symptoms
8 . decreased cardiac output: backwa rd railure
C. volume overloaq, of right ventricle
O. rigtH.sided Sl heart sound: vo lume overload
E. right·sided S4 heart sound : decreased comp liance
F. tricuspid regurgitation:
( 1) stretching o f tricuspid va lve ring
(2) pansystolic mumlur at left parasterna l border (increases with inspiration)
G. congestive hepatomegaly:
(1) nutmeg liver
(2) increased LDH, isoenzyme
(3) increased serum transami nases
H. increased hydrostatic pressure in venous s),stem:
(I) ascites
(2) dependent pining edema
(3) jugular neck vein distention
4. Rx:
A. decrease preload and afterJoad
B. ACE inhibitor decreases preload (blocks aldosterone) and afterload (blocks
angiotens in II): spironolactone added owi ng to eve ntua l increase in aldosterone
C. diuretics decrease preload

58
D. restrict sa lt and waIer (decreases preload)
High output failure:
I. mechllnisms-
A. increase stroke volume:
(I) hyperthyroidism
(2) increase blood volume
B. decrease blood viscosity: severe anemia
C. vasodihlle peripheral resistance arterioles (decrease total peripheral resistance):
(I) increases venous return to the heart
(2) causes of vasodilatation
a. thiamine deficiency
b. endotoxic shock in early phases
c. metabolic acidosis
D. arteriovenous fistula:
(I) direct commun ication of arterial wilh venous system bypasses the
microcircu la.ti on
(2) increases venous retum to the heart
(3) causes of AV fistu las:
a. surgically produced in radial artery fo r renal dialysis
b. trauma :
• knife wound usual cawe
• pressing afferent vessel causes slowing of heart rale (Branhrun's
sign)
• hear bruit over the soft tissue mass
c. Paget's disease of bone: aneriovenous fistulas develop in soft bone
2. USMLE eornladoD of blab oatpat raullre with P'olaeaiUe'l eqaadon-
A. abbreviated equation: total peripheral resistance (TPR) = viscosity of blood/(rndius of
the aneriolet
8. vasodi latation has greatest effect on lowering TPR : see above causes
C. decreasing viscosity. de~ reases TPR: see anemia
V MLEquffilo.o (Iai--....,.weII):
1. CID02 qaadO. OD wbat Is ktwrr In ambUlnl artery blood of. aoraW fetlll venus
mat'..... tirtiiiaJ blood- answer is POI is lower in the umbilical art'e n'
A. chorionic vi llus in the placenta is the primary site of 0 2/CO l exchange for the fetu s
B. nonnal values in pregnancy:
(I ) nonnal hematocrit (Hct) is -35%
(2) O2 saturation (Sa01) is -98%
(3) merial P02 is -105 mm Hg
C. fetal Hb has a high affinity for O 2: idea l for gllS exchange in chorionic: villi of the
placenta
D. fetallaboratol)' parameters:
( I) Het of -55%:
a. higher than mothers
b. high Hct also increases viscosity
(2) increased percentage of HbF (70% at birth )
(3) left-shifted ODC from HbF
E. umbilical vei n has an SaO}-80% and POl ]0-35 mm Hg:
( I) it has the highest amount of oxygen in the fetal circulation

59
(2) it carries ox-ygenaled blood from the chorionic villus to the fetal li ver
(3) -50% of the umbilical vein blood (SaO, of - 80%) mixes with hepatic sinusoid
blood (SaO, of26%), which contains ponal vein blood
II. SaOl after mixing of umbilical ve in blood with sinusoidal blood is -67~.
b. sinusoids are drained by the hepat ic \·tin. which empties into the inferior
vena cava (rvC)
(4) remaining umbilical vein blood enters ductus venosus: drains directly into the
rve
(5) rvc SaOl before entering the fetal right atrium is -67%; most IVC-derived
blood is directly s huoted tbrough a patent roramen o\'a le into the left a trium
(SaO, -60%)
(6) most SVC-derived blood enters right ventricle '
a. fetal pu lmo nary arteries are h)'penrophied from chronic vasoconstriction
secondary to a low fetal P02
b. most blood entering the pulmo nary aneries is shunted into the patent
ductus aneriosis and from there into the descending 1l0 n B (riSht to left
shunt): placenta-derived prostaglnndin (PGE t ) keeps pOlent ductus open
e. descending nona SOOl is -58% and the PO: is 20-23 mm Hg
(7) blood in the aona flows towards the placentD via two umbilical arteries. which
have a-
a. lower POI (20-23 mm Hg) than maternal art'crial blood (PO~ -1 05 mm
Hg)
b. higher ~ affinity than maternal arterial blood. owing to the presence of
more HbF in the fetal RBCs, which left-shifts the ODC
c:. grearer O 2 capacity than maternal onerial blood (HbF has a higher affinity
for 0 1 than adult HbA)
Conge nita l heart di.s eas~ (CHD) and whae oxygeo sa turaliollJ wo uld be in tach type: see
schematics of all the CHDs
I. stC!p up o r oxygen is compatible wilb a lefl to rig ht shunl- oxygenated blood (SaOl 95%)
from the left hean enters unoxygenated blood (SaO, 75%) with a step up of O~ to - 80% in
the right heart '
2. stc! p down or oxygea is compatible witb a rigbt to len s bunt- unoxygenated blood ( aOl
75%) from the right heart enters oxygenated blood (SaOl 95%) with a step down of O! to
- 80% in the left heart
3. Elsc nmenger' ~ syndromC!-
A. excess blood in the right hean from a left to right shunt causes volume ove rl oad, Which
produces:
(1) pulmonary hypenension
(2) right ventricular hype-nro phy
8. reversal of shunt occurs when right ventricular hypertrophy overrides left ventricle
pressures: patient develops cyanosis (called cyanosis tardive)
4. "eatrlcular septal defect (VSD)-
A. Me coogenit.1 beart disease:
( I> usually spontaneously comclS itself
(2) defect in the membranous septum
(3) associated with en du chat (partial deletion of chromosome 5) and lnsomy 18
8. initially a left to right shunt through a defect in the membranous septum:
( I> step up 0[01 to 80% in the right ventricle Ilnd pUlmonary artery

60
A B

I
I
I
RA RA
I
I \
I
I
I
I RVH
LVH.
dilatation

• aher PH
-
•• shunt "r51
Righi.
Io-Ielt
"'~I
alter PH
~
~
~
c Machinery Differential
D
PostdUCtal
cyanosis
~ cOnstriction

•• Pulmonary
hypertension
(PH)
POSlSienolic

~
• I Blood pressure
distally

~
~

••
RA

••


•I RVH alief
PH "---
Rib notChing ~ Inlarcosl8.l
art• .., -
IFI GU RE 10-1. Scbemadca of cooa:eDltai burt dbea.e. (A) represents a ventrlculu seplal defect. (B) an atrial scptal def~t. (C) a
patent ductUJ arteriosus (PD"). (D) an adult coarctation of the aona. (J:) tetralogy 01 Fallo!. Illd (f) transposition of the great vessels.
• Ao. IOrta: ASO, .trW septal defect: AV, aort ic valve: DA, ductus arteriosus: lVe, Inler lor vena cava: LA left 4lnum; LV, lefl ventricle;
LVH. lett v~lrtcular hypertrophy; MY, mitral valve: PA. pulmonary arter y. PDA. patent ductus arterio.us: pv, pul monic valve: RA.
' right atrium: RV, right Vf: nttlcle: RVH. righ t ventricular hypertrophy; svc. ,uperlo r vena cava; lV. tricuspid vatve: VSD. ventrlcullU
, sePlal defect.
E F L.eft-Io-rigtlt
$hunt thrcugl'l
POA

SubP1Jlmonlc !
fley to cyanosis
versus acyal'lC5ls

Aorta overrides PUlmonary


artery
empties LV

,--'-,AA
Aona
empties '-_,-....
AV '

Alght-to-Ie"
shunt
through VSO
(2) volume overload of right hean may lead to Eisenmen ger's syndro me
S. a tria l se pta l defect (ASD)-
A. M C . dull CRn
8. assoc iation with fetal ak obol syndro me
C. initia lly 3 left 10 right shunt th ro ugh a patent foramen Dvale
( I) step up o f ~ to - 80% in the right atri um. right ventricle. and pulmonary anery
(2) volume o verload of right hean may lend 1'0 Eisenmenger's syndro me
D. fixed spl in ing ofS!
6. pat ent ductus art eriosus (PDA)-
A. assoc iation with co ngenital rubella
B. initially a left to right shunt from the aon a to the pulmo nary anery through the ductus
arteriosus:
<I) step up of 0 1 to - 80% in the pu lmonary anery
(2) left atrium has 95% 5a02
(3) vo lume overload of right hean may lead to Eisenme nger's synd rome:
a. pulmonary artery e mpties unoxygcns lcd blood illlo the aorta (step down in
5,0,)
b. unoxygcnated blood e nters aorta distal to the subclavi an artery
c. patient is pi nk in the upper extremities and cyanotic in the lower
extremities
d. called differential cya no!lis
7. tetralogy of FaUot-
A. MC cyanotic congenitaJ beart disease
8. tetraJogy includes:
( I) overriding aorta; least common anoma ly
(2) VSD
(3) .in.fGvalvu lar pu lmonic stenosis
(4) right ventricu lar hyperuophy
C. cyanosi s depends o n the degree of pUlmo nary valve stenosis:
( I) acyanotic al birth if the stenos is is nOI signifi can t, since most of the
unoxygenated blood enters the pulmonary anery for o xygenatio n in the lungs
(2) cyanos is is present 81 bi rth if the stenosis is sign ifi cant, s ince the majority of
blood is shunted from right venuicle into the le ft ventric le and out the ao rta
D. ~.....c:e Dr •• ABD iii! PDA .... eardlopro'octtve (USMLE):
(I) left 10 right shunt of ASD adds oxygenated blood into RA and steps up righI-
sided Sa~
(2) left to right shunt of PDA takes unoxygenated blood in the aorta and dumps the
blood into the pu lmonary artery for oxygenati on in the lungs
8. complete tran' position of tbe grea t vessels-
A. assoc iation w ith maternal d iabetes
8. usually a PDA, ASD, and VSO are present
C. pathophys io logy:
(I) venous blood normally empties into the right atrium via the vena cava (SaO~
75%)
(2) o xygenated blood (SaO] 95%) nonnally enters the left atria via the pulmonary
vein
(3) the aorta is transposed and empties the right ventri cle
(4) the pulmo na ry aMery is transposed and empties the left ventricle
(5) oxygen sarurations are as fo llows :

61
1&. step up of SaOl to - 80% in right atrium as o"ygenaled blood from the left
atrium empties into the right atrium through the A 0
b. right ventricle \Vith SaO~ -80 is emptied by the aorta (patien! has cyanosis)
c. some righl ventricle blood is shunted right to left through a VSD into the
left venlTicle (step down of Sa~ to 80%)
d. left ventricle is emptied by the pulmonal') anel) for oX) genatlon in the
lungs
e. o nly \'essel with SaO: of95 % is the pulmonlll) vein
r. o nly chamber wim SaO: of95-;. is left atrium
Pos lduc ta l coa r cta tion :
1. co nstriclio n o r llo rta is disla t Co tbe ligame nt um a r1 erios unt
2. clin iu l-
A. prox imal to the aortic constriction:
(I> systo lic munnur between the shoulder blades
(2) inc reased blood pressure/muscuhllure in upper extremities
(3) di latllti on of proximal aorta (subject to n dissection)
(4) dil atati on of aort ic va lve ring wit h Dort ic regurgi tati on
(5) increased incidence of congenita l bicuspid aortic va lve
(6) increased intracrani al blood Oow: c ircle of Willi s vessels prone to bell')'
aneurysm fonnation and rupture
B. dislaito the aortic constriction:
( I) decreased blood pressurelmuscuhuure of lower extremity
(2) leg c1audicafion from ischemia
(3) decreased renal blood flow leads to diastolic hypertension: actl\'ation of
renin/angiotensin/aldosterone system
C. collateral circulation:
( I> intercostal arteries (rib notching)
(2) superficial epigastric artery anastomoses with interna l mammary art'cry
O. T urn er's syndrome: preductal coarctation that is sympto matic at birth
a- USMLE _ r l.... ~
I. Ie Ka......er' • • yadrolDe (situs invers u!, immulil e cilia synd rome). tberr is DO
traalpoOdOD : the heart is 1'013 11)' normal but is located on the right side
2. chlUllber with tbe bl&bett POl I•• PDA...J left atrium or left vc ntricle or norta
3. p.....o ... ry artery SaO] iD • PDA.,.. stcp up of 5nOJ to - 80%
cr Moj or coro nllry artery risk factors (National Choles lcroi Education Punel 1993):
1. age Is tbe m ost important overall risk rllctor-
A. m.l. ~4S yold
B. woman ~55 yold
2. ra mily bistory of premature coro nary artery diseDst (CAD) or stroke
3. t urre nl cigare tte s mo king. enhan ces a tberosclerosis- chemicals in smoke damage
endothelial cells
... bype rtenslon ",d th blood pressure ~ 140/90 mm Hg- CAD is Me COO in hypertension
5. HD L <35 mgld L-
A. enhances atherosclerosis
B. HD L is tbe major risk ractor ro r wo mn in th eir r eprodu t't1\'e yean
C. LOL is the maj or risk factor for males regardless of age

62
6. diabetes mellit'us-
A. enhances atherosclerosis
B. CAD is MC COD in diabetes
1. LDL ;>.160 mg/d.L-
A. enhances atherosclerosis
B. oxidized LDL poses an even greater risk for CAD than native LDL
C. vitamin E neutralizes ox idized LDL
8. HDL ;,60 mg/dL-
A. negative risk factor for CAD
8. subtract I risk factor from the above major risk fa ctors
Faclors responsibl e for myocardial 0 1 demand :
t. hea rt rate (HR)-
A. as HR increases, contractility increases:
(I ) more action potential per unit time
(2) more O 2 consumption
B. reducing HR. reduces O2 consumption
C. HR > 180 bpm decreases the length of diasto le, which decreases fillin g of the coronal)'
arteries and left ventricle
2. ventricular wall pressure-
A. an increase in wall pressure increases O 2 consumption
B. wall pressure inc reases when:
( I) ventricles are hypemoph ied or
(2) when the radius of the ventricle is decreased owing to noncompliam muscle
(most often the result of ischemia)
Factors res ponsible for myocardial O 2 supply :
I. coronary a rtery blood now-
A. most important fac tor
B. co ronary arteries fiJI in diastole
C. clinica l correlations:
(1) decreased cardiac OUtpUi in aortic stenosis causes less fillin g or coronary arteries
and the potential for angina
(2) aortitis in syphilitic aneurysm leads to narrowing ofeoronsl)' ostia and angina
(3) inc reasing heart rate decreases time for filli ng of coronary arteries
(4) subendocardium gets the least amount of oxygen fro m the coronary arteries
2. left anterior desce ndin g (LAD) coronary a rtery-
A. supp lies entire anterior portion of left. ventricle:
(1) danger of rupture in a transmural myocardial infarction
(2) prone to mural thrombosis in anterior myocardial infarctions
B. supplies anterior 2f3 rds of interventricular septum:
(1) danger of permanent bundle branch blocks in anterior myocardial infarctions
(2) permanent pacemaker is often required
J. rigbl coroDary artery (RCA)-
A. supplies the enlire posterior and inferior part of the left ventricle: responsib le for
epigastric pain in a myocardial infarction
B. supplies the entire right vemricle: right ventricular infarction
C. supplies posteromedial papil lary muscle: danger of papillary muscle rupture and mitral
regu rgitation in myocardial infarctions
D. supplies most of the blood to the arterioventricular node: danger of sinus bradycardia

63
USMJ...E'-pietare of coroaary vessel: descriplion-
1. allleroKlerotia: slit lib:spaca
2. dyscroplaic cakilicatio., bhl~blotcb.es
J. Dbrofany plaq.., dil'«lly be.eath e.dotbellum
4. platelet darombus: red-.us bkJclciDg the lamen
T~' pes of isc hemic bea rt disease:
I. anginll ptctons- MC type
2. IIC ute: myocardial iofarctioo- MC COD in the United Statts
3. s udden tardlac d eath
-'. chronic ischemic heart disease:
Angina pectoris "onoolS:
I. exerlional (cltlSsical) angi na-
A. Me variant
n. severe fixed coronary artery ath erosclerosis
C. sudden onset of exercise-induced substernal chest pain Insting 1- 15 min: relieved by
resting and/or nitroglycerin
D. Dx:
( I) exerc ise ECG (stress lest) reveals ST depression: indicates subendocardial
ischemia
(2) confirmed with aneriography: intracoronary artery ultrasound recemly used
2. PriDt.z.metal's aogiDa-
A. secondary to coronary artery \'8sospasm: ? vasoconstrictive effectS of thromboxane A2
released from small platelet thrombi overlying non-occ1usive atherosclerotic plaques
8 . chest pain at rest
C. Ox:
( I) stress ECG reveals ST elevation
(2) represents transmufl~1 ischemia
J. unsta ble (crescendo) angina-
A. severe. fixed , multivesse l atherosclerotic di sease
B. eccentric stenosis of vesse ls with disrupt'cd platq ucs wilh thrombosis present:
( I ) nonocclus ive thrombus
(2) re lease vasoconstrictive compounds like TXAl
C. freq uent bouts of cheSl pain at rest
D. Ox:
(1) stress ECG is unsafe
(2) - 20% progress to an acute myocard ial in fa rct ion in 3 m
4. s urgical procedures fo r Rx of coronary a rtery disease-
A. percutaneous translum inal coronary angioplasty (PTCA)- M e procedure
B. coronary artery by-pass graft (CABG)
C aun: of thrombosis after aogioplasty: locataro dissection
VesselJ used ia tbe:. CABG procedure:
1. internal mammary artery-
A. best graft patency
B. 90% patency rate after 10 years
2. sllpbenous veins-
A. 40-50% patency rate aftcr 10 years
B. "art'erialization" of lhe vesse ls. fibrosis. and occ lusion

64
Sudden cardiac death:
l. death within 1 b
2. Stvere a therosclerotic CAD
J. u.s ually no occlusive "Hul thrombU5 ~
4. die of ventricular arrh)1bmia
S. associatio n ""ilb smoking and noo·Q wave infarctions
C hronic bchemic beart disease: repeated infarcts with replacement of cardiac tissue by fibrous
tissue leading 10 systo lic dysfunction
Gross (G) and Microscopic (M) changes in an acute myocardia l infarction (Al\U):
I. 0-4 hs- no GIM change
2. 4- 12 h, -
A. no G change
O. M shows coagulation necros is after 6 hs
3. 12- 24 h, -
A. G shows early pallor
B. M shows more advanced coagulation necrosis
4. 1- 3 d-
A. G - definite pallor
B. M - neutrophil ic infiltmte + advanced coagulatio n necrosis
5. 3- 7 d-
A. period of maximal softness
O. lime for ruprures
C. macrophages move in to remove dead tissue
6. 7- 10 d-
A. G is the same as 3-7 d
B. M shows collagen deposition
AMI complicalioDs :
I. arrhythmias-
A. Me compl ication
B. ventricular arrhythmias
C. MCCOO
2. hellrt (allure- usually in fi rst 24 h
3. rupturc-
A. Me on the J rd 10 7th day
B. usua lly anterior wall rupture with tampo nade: thrombos is of the left anterior
descending (LAD) coronary
C. posteromedial papillary muscle rupture:
( I) right coronary anery (RCA) thrombosis
(2) acute onset of mitral regurgitation and heart fai lure
D. ventricular septum rupture:
( I) LA,.[) thrombosis
(2) left [0 right shunt
4. mural thrombus- danger of embolization
S. pcricarditis-
A, first week in U1Ulsmurol infarction :
( 1) fri ction rub
(2) fibrinous pericarditis

65
B. Dress ler's syndrome:
( I) autoimmune pericarditis 6-8 wks later
(2) systemic signs/symptoms: fevcr,joint pains, friction rub
6. \lenlricula r aneurys m-
A. late compl ication
B. precordial bulge during systole
C. hean failu re is Me COD
7. p rogaosis- depends on ejection fraction
U5MLE plctan...or ltdIiiI ..~ 1IIIlarct1oa: blotchy fibrosis unlike sheets of pale staining
tissue as in a pale infarction
La b abnorma lities io au acute myocardia l infa rction:
1. inc re05e in CK -MB-
A. begins in 4-8 hs
B. CK-MB peaks in 24 hs
C. d isappears in 1.5-3 d
D. reappearance ofCK·MB >3 d is a r~infarc t io n
2. Increase in troponin-I -
A. begi ns in 2-6 hs
8. peaks in 15-24 hs
C. gone in 7 d
D. will likely replace LDH isoenzymes
E. cannot identify reinfarction
3. LOB III fli p-
A. begins to appear in 10 hs:
( I) LOH, is nonnally> LOH ,
(2) LDHI has highest concentration in cardiac muscle
(3) AMl causes greater release of LOH 1 than LDHl leading to an LDHII1 flip
B. peaks in 2-3 d
C. gone in 7 d
O. mainly used to identify AMI after 3 d
4. increase in AST-
A. begins 6- 12 hs
B. peaks 1- 2 d
C. gone in 5-9 d
Sequ ence of microscopic findings and their co rrelation with cha nges on ECG in an acute
myoca rdial lofarcllon:
I. peaked T waves- corre lates ..... ith area of ischemia in thc myocardial tissue at the periphery
of the infarct
2. ST segment elen tion-
A. corre lates with injured mvocardial tissue around area of necros is
B. many of these cells die during reperfusion :
(I) superoxide free radica ls
(2) called reperfusion injun'
3.
4.
. .
Jl'mm etric T wave iD\'enioD- correlates with area of ischemia
Q wa\le- area of necrosis

66
r:r Rheumatic fever:
1. patboge nesis-
A. cross-reactivity (mimicry) of antigens in M prote ins of group A SlJCptococcus are
simi lar to those to the patients hean and other tissues
O. blood culrures are negative: it is an immunologic reaction and not a septicemia
C. group A streptococcal infection usually begins a pharyngitis
2. Ascb , . is tbe pathognomonic lesion of rheumatic fe\'tr- area of fibrinoid necrosis
and reactive histioc),les in myocardial tissue
3. .Jonc' lI criteria for acute rheuma i (RF)-
A. 01 arthritis-
(I) MC initio l presentation
(2) no pennanentjoint damage
(3) differential diagnosis :
a. juvenile rheumatoid arthritis
b. Henoch-Schonlei n purpura
B. Sw:~
(1) pericarditi s
(2) myocardi tis: MC COD in acute RF
(3) endocarditis:
a. mitral va lve vegetations: develop along the line of closure of the va lve
b. mitral regurgitation (not stenosis)
C. Sydcabam's ~bo~
( late manifestation
(2) self Hmjted
D. subcutaneous nodules-
(I) fibrinoid nec,:o;;s in nodules
(2) occur on extensor surfaces
(3) histologica lly the same as in rheumatoid arthritis
E. ~ tbem..!.,!lla!J:inatu-
(1) circular ring 0 el');tema around nonnal skin
(2) vegetations on margin of the valve-+ erythema marginatum
F. I0 criteria-
(1) clinical findings:
a. arthralgias
b. fever
(2) lab: elevated ESR or C-reactive protein
(3) ECG: prolonged PR interval (first degree block)
G. s~ evidence of antece~ s.!!egt4?S~caIIDf~i~s­
(I) positive throat cu ture for group A streptococcus
(2) positivc latex agglutination test
(3) elevated or risi ng anti-streptolysin 0 (ASO) titer
r:r MilralslenosiJ:
I. MCC 15 chrooic rbeumatic beart disease
2. palhophysiology-
A. volume overload in the left atrium and lungs
8. decreased cardia.c output
3. physical Dx-
A. accentuated SI due to c losure of a non-pliable valve:

67
A .P2 B
Pulmonary
hypertension
I cardlac (PH)
output
Lett alnal
l'Iyperuophy + Incompetent
d ilatatIOn MV

LA pushes on
esophagus
RA
NoLVH
unless MV
regurgitation
also present

RVH
alter PH
develops
Stenotic MV opfIrung
.. Opening snap
.. Diastolic rumble

c o Hyperdynamic
circulation
.......ater·hammef
pulse"
dllata!lon
LVH ...
dltatallon
Concentnc LVH
(pressure overload) Austin FIll'll
murmur
.. Blood hit!
RA anlerior leanel

Stenotic AV valVe
.. SystoHc ejecTion murmur
.. I AmpUlude 01 PM!
.. I Pulse AV Incompetent
.. ~ Pulse pressure .. H!gh-pilched C1astol!c
.. AnginaIsync:ope with blowing murmur
exercise

FI GU RE 10- 3. SchemJIth:a o r acquiJ"t:d v:a.lvular dbeuee or tbe hearl. (A) represents mUral stenosis, (B) mitral reguri\t,lllon, (q
/lonk "enosI5, and (0) aortic regurgitation, (See Flgure II}.Z lor US! 01 abbreviations.)
B. opening snap (OS) occurs when the non. pliable "D ive gives way under increased left
amal pressure
C. OS followed by a mid.d iaslolic rumbling rnunnur heard best at apex with the patient in
the lateral decubitus position: increases with expirlltion
D. accentuated P: indicates pu lmonary hypenension (PH) from chronic pulmonary venous
congestion
E. right venlriculllT hYIXnrophy secondary to PH
4. elinica l-
A. dyspnea and hemoptysis secondary to pulmonory congestion
B. aeriat fibrillation secondary co left atrial dilnll!.lionlhypenrophy:
( I) danger of systemic embolization
C. dysphagia for solids:
( 1) due to left atriol enlargement
(1) left atrium is the most posteriorly localed chamber and com presses the
esophagus: requires transesophngenl ultrasound to eva luate its s ize
D. hoarseness from irritation of the rccurren tlaryn scnl nl:rve (Ortner's syndrome)
Mitral ,'. lve prolapse (MVP):
I. Me ,·. h'ullir disease in the young population
2. patbogenesi!-
A. vnlve lenflets (anterior and/or posterior (MC) undergo myxomatous degeneration
(tl8MLE):
(I) increased glycosaminoglycans (dennalD.n sulfate)
(1) causes redundant (voluminous) valve leaflets
(3) tricuspid ''l.lve involved in 20-30%
B. leafleu proJapR into me left atrium during systole like a parachute
C. AD inheritance panern in some cases
D. association with Marfan'slEh lers Danlos syndromes
3. I's ..... ('t1SMIZ)-o
A. mid· syslolic ejectioD click occurs when valve proJapscs into left atrium and IS
suddenly resu-ained by gae chordae tendineae
B. click rollowed by a mid or late systolic munnur; due to MY regurgitation
C. click/murmur closer (0 SI when there is less left ventricular volume: e.g..•
(1) stand ing up
(2) anxiety (DSMLE): tachycardialeads to less diastolic fi ll ing
(3) Volsalva maneuver; increased pos itive intmthoracic pressure decreases fillin g of
right heart
O. c Uc k/m urmur move closer to S: when left ventricu lar vo lu me is increased:
(1) lying down/sq uaning: more venous blood returns to right heart
(%) sustained hand grip exercise: positive intrathoracic pressure decreases c.ardi3c
output
(3) passivc Icg lifti ng in the supine position
.t. cll nical-
A. usually asymplomaric, taU. thin women
B. symptoms include:
(1) palpital ions (MC)
(1) ches t pain
(3) fal igue
C. rupture of the chordae produces acute milnll insufficiency

68
D. sudden cardiac death from ventricular tac hycardia may occu r: seen \0 Marfan's
syndro me (1JSMLE)
5. diagnosis- echocardiography
6. Rx of sym ptomatic: cases-
A. calcium channel blockers o r ~-bl oc kers
S. negative inotropic agents reduce the force of contraction
Mit ra l regurgi tation :
I. C!a usts-
A. I\1VP Me struC!tural cause
B. left heart failu re: stretching of MV ring
C. chronic rheumatic heart disease
D. infective endocarditis
E. rupture of the papillary muscle: usually right coronary artery thrombosis
2. pl1 lh opbysio logy-
A. reflux of blood into the left atrium during systole
B. vo lume overload in the left ventricle and left atri um: leads to hypenrophy/dilatnlion or
both chambers
3. physica l Dx-
A. pansystolic munnur w ith rndialion into the axilla
O. heard best at the apex
C. increases on expiration
D. S] heart sound: volume overload
E. S4 heart sound: decreased compliance
4. clinical- vol ume overload handled well until latc in the disease when pulmonary symptoms
begin to surface, like dyspnea and cough
t7" Aortic stenosis:
1. CIl USes-
A. MCC is a calcified congenita l bicus pid valve
B. chronic rheumatic hcart diseasc
C. degenerat ive"changes associated with old age
2. pa th opbysiology-
A. reduction in the valve orifice: area offers increased resistance to the ejection of bl ood
during systo le: decreased stroke vo lume/cardiac output
B. poststenotic dilatation of aortn owing to stream of blood hitting the wall under
increased pressure
C. pressure overload in the left ventricle lends to concentric left ventricular hypenrophy
3. pbysical Dx-
A. delayed/diminished aneria l pulse wave :
<I> decreased pu lse amplitude
(2) narrow pulse pressure
B. ejection murmur during systo le:
(1) best heard in right sternal border with radiation into carotid
(2) also heard along the sternal border/apel( in the elderly
(3) increases intensit), on expiration
4. cli nical-
A. MC valvulur lesion associated 'W ith a n gina :
(1) decreased cardiac output leads to less filling of coronary arteries during diastole
(2) hypertrophied heart receives Icss blood

69
B. MC va lvular lesion associated with syncope with exe rcise: due to reduced cardi ac
output and less blood available to the brain
C. MC CIl Use or microaogiopathic hemolytic an emia with schistocytes
Aon ic regurgi ta tion:
I. causcs-
A. MCC is long-stllnding essential bypen ens ion
B. medial disease in the: aortic root
C. chronic rheumatic heart disease
D. infective e ndocarditis: Me left-sided valve invo lved in IV drug abusers
E. dilatation of aortic valve ring:
( I) syph ili tic aortic aneurysm
(2) di ssecting aortic aneurysm
(3) coarctation of the aorta
F. ankylosing spondylitis: ao rt itis
2. path ophysiology-
A. vo lume overload in the left ventricle fro m an incompetent valve that leaks blood back
into the left ventricle during diastole: left ventricular dilatation/hypertrophy
B. increased stroke volume: due to Frank-S tarli ng mechan isms
C. increased pulse pressure: hyperdynami c circ ulation (see below)
3. phys ical Dx-
A. Corrigan's pu lse: water hammer pu lse or bo und ing arteria l pu lse
B. high-pitched blowing early diastolic munn ur heard immediately after S2:
(1) heard best a long left sternal border
(2) increases in intensity with expiration
C. Austin Flint murmur:
( 1) regurgitant stream fro m A V va lve hits anterior leanet of miua l valve causing it
to vibrate iUld produce 8 diastolic murmur
(2) sign of significant aortic regurgitation
4. clinical- volume overload tolerated well until late in the co urse of the di sease when angi na
and hean fail ure occur ..
T ri cuspid ste nosis:
I. ca use- usually associated with mitral stenosis post-chronic rh eumatic heart disease
2. pilthophysiology- volume overload in right atrium and systemic venous congestion
3. pbysica l Dx-
A. cannon 11 wave in the j ugular venous pu lse: atrial con traction against a st'enotic v31ve
with reflux of blood back into the j ugular vein
B. diastolic munnur heard best along left sternal border:
(1) opening snap not commonly heard
(2) munnur intens ity increases on inspiration
4. clinical- s igns of systemic venous congesti on
A. neck vein distention
B. congestive hepatomegaly
C. dependent pitl'ing edema
T ric uspid regurgitation:
1. ca uses-
A. stretching of the tricuspid valve ring in righl hean fail ure: MCC
8. chronic rhe umatic hean disease

70
C. carcinoid hean disease
D. infective endocardi tis in TV drug abuse
2. pathophysiology-
A. blood regurgitates inlo the right atrium and the j ugular veno us system
B. volume overloads right ve ntric le and atrium : h),pertrophy/dil:uation
J. pbys ical Dx-
A. giaDt c-v jugular venous pul se \"'ave : blood regurgitates into the atrium during systole
S, pansysto lic murmur along left S1'emal border: intens ity increases with inspiration
C. pulsating liver
D. righ t sided Sl a nd S,,: increase in intensity o n inspiration
or Pulmonic stenosis-
1. causes-
A. co ngenita l
B, assoc iation with carc inoid syndrome
2. pa thophysio logy- pressure overload of right ventricle
J. physical Dx-
A. systolic ejection munnur heard best in 2nd left intercostal space
B. right ve ntricular hype rtrophy
C. cannon a wave
Pulmonic rcgurgilation-
I. caules-
A. usually a functional munnur induced from congestive heart failure and heart dilatation
B, pulmonary hypenension
2, patbophysiology-
A, vo lume overload of right ventricle
B, right sided Sj and Sa: increase in intensity on inspiration
3. pbysical Dx-
A. early diastolic murmur after ~ : increases in intensity on inspiration
B. called Grabam Steell murmur if associated with pulmonary bypenension
Carcinoid heart disease--
1. carcinoid syndrome is a triad 0(-
A. cutaneous flu shing: serotonin effect
8. diarrhea: serotonin effect
C. va lvular disease
2. patbopbysiology-
A. metastatic carcinoid rumo r in the liver from a primary cancer in the tennina] ileum
allows serotonin access 10 the venous system
B. serotonin is fib rogenic causing tricuspid regurgitatio n and pUlmonic stenosis
lureetin endocarditis:
I. etiology-
A. StreptococcllS lIiridans MCC followed by Swphylococcus Qureus
B. Staphylococc.us aureus MCC in intrave nous drug abuse
C. Streptococcus bovis endocarditis/sepsis is associated with colon cancer
2. patbogcnesis-
A. septicemia seeds a nonnal o r previously damaged va lve
8. common in cyanotic congenital hean disease

71
J. prubology-
A. friable vegetations can embolize
B. mitral valve MC valve involved foll owed by aonic valve
C. tricuspid va lve followed by 80nic va lve most often involved in intravenous drug abuse
... elinical-
A. immunocomplex vascu litis:
(I) Roth ' s spot in the retina. splinter hemorrhages
(2) 0 ler' s nodes (painful) on hands/feel
(3) Janeway lesions hand and feet (painless)
(.. ) glomerulonephritis with hematuria with RBC casts
B. fever
C. splenomegaly in morc subacute cases
S. lahorillory- blood cuhures positive in majority of casts
6. U MLE Jcc nario- described a murmur of tricuspid regurgitat ion in an intravenous drug
abuser: infective endocarditis involving TV
Libman Sncks endocarditis:
I. associated with S'LE
2. InvolveJ mitra l v"lve
3. steri le "egetations witb fibrinoid necrosis
4. vegetations located aU over tbe ,'ah'e unlike rbeumatic revu, whose "t'gttatiO DS arc on
the margin or the nlve
5. usuaJly not clinically significant
Mara,ntic vegttations:
I. parantoplastic syndrome-
2. sterile vegetadons in mucin producing tumon or coloolpancrellJ
3. milral vah'c most onen in\'olved
-.~ :
I. couacidt\'iruJ is MCC
2. U ML£ lIIowed. pictare of Iympllloqrtes ... Dlyoeardt.J duae 08 aa e.domyoardJ.a1
Ih '
Congestive cardiomyopathy:
l. causes-
A. idiopathic in most cases
B. previous myocnrdltis: coxsackie MCC
C. drugs: doxorubicin. tricyclics
D. postpartum: commonly used on USMLE
E. alcohol : direct to:'lic effect or thiamine deficiency
F. hypothyroidism
2. cllDkal-
A. large, heM with all chambers dilated
8. left and right hean failure
Hyper1rophic cardiomyopathy:
1. patbogtoesi5-
A. AD in some eases
B. asymmetrically hypertrophied interventricular septum
C. obstruction to blood now is due to the anterior leaflet of mitral valve being drawn
against the interventricular seprum by Vemuri effect

72
ICardlac output
• Anllrlor MV
leallit obstructs
blood flow +
• AsymmetrIC seplum

FIGURE 10-5. &hematic: of Idloplublc hype rtrophIc subaor tic:


alenoai.s.. (Su FIgure 1~2 for list of abbrevIation,.)
O. nbernmt myofi bers and conduction system in the septum: cond uction di sturbances
responsible fo r sudden death
2. clinieal-
A. M CC or s uddea deatb in young peopl e a nd a tbl etes
B. synco pe with exercise and angina with exerci~
C. munnur i nten si~' decreases and clinical improvement OCcurs \\ ilh anything thaI
increases left \'entricular volume:
( I ) incrc3Sed venous relum to hean (squan ing. lyi ng down)
(2) decrease cardiac contractility and heart nHe 1'0 increase filling (p·blocker)
D. munnur intensiry increases and clinical cond ition worsens with less left ventricular
blood volume or the use of inotropic drugs:
( I) Val .. lv. ,
(2) cardioc inotropic agents like digital is , I
(3) venodilators (amyl nitrile, nitroglycerin)
Rest rictive ca rdiomyo pathy:
1. non ~co mpli a nt hea rt- cannot fi ll properly
2. ca uses-
A. glycogen: Pompc ' s glycogenosis
B. iron : iron overload disorders
C. amylo id: senile amyloidos is (amyloid derived from prcalbumin)
O. endocardial fibroelastosis in child: thick endocardial tissue
CUdIK_:
I. M C prima ry aduh heart tumor- ben ign mesenchymal tumor
2. siu-
A. usuaJly left atrium
D. ltJIMLEI used right atrium on one exam
3. clin iu l-
A. emboliu
B. fever
C. syncope
Ca rdi ac rba bdomyo ma:
1. Me prima r y bea rt tum or jg children
2. part or tuberous scil!rosis complex
Pe r ica rd ia! erfU5ion:
1. cauus-
A. viral pericardilis due to coxsackie virus M C C
B. SLE: Me cDrdiac disease in SLE
C. trauma
2. p. lbopbysiology-
A. all pressures in all chambers are increased but the cardiac output IS decreased
8. chambers cannot fill up with blood during diastole
C. blood renuxes back inlo the jugular vein on inspiration
3. clinicl l- ~:
A. neck vein distention with inspiration (Kussmaul sign)
B. Pmumed hC:Dn sounds
C. hypotension: associated with pulsus parndoxus (drop in blood prwure on inspiralion)
4. Ox- echocardiogram l 8 tl .,). /i. ... ~ ~'vrf~~~

73
s. R.s- pericardiocentcsis
Co nStrlclive pericarditis:
I. uuses-
A. TB MCC worldwide
8. previous hean surgery is fCC in United States
2. pathopbyslology-
A. heart CiUlnot completely fill
O. pericardial knock when \'cntricles hit thickcned prricardium
ECCq _ _ OD~:
I. ..... QT ialen'al- hypercalcemia
2. proloaaed QT latuval'" hypocalcemia
3. U wav~ hypokalemia
4. peake:d T wan- hyperkalemia
S. ST depnuloa- subendocardial ischemia
6. ST.llevatIon- transmural ischemia/infarct ion
7. prolODCed PR IDlerval- .
A. first degree heart block
8 1 common findin g in patients on digitalis
8. IIIriaI DbrWalioD
9. valriC1llar p",_ru", bealailKbyunllo
10. aaterior .yocardlalllllarctloa- new Q waves in anterior leads I through IV
It. ialerior wall KIlle lIiyocardlallalarcdo... new Q waves in n. IlI. and aVF
12. WoUr-I'DrId-'Wlllte-
A. short PR interval with a nonnal P wave
B. de lta wave on upstroke ofR wave
C. aberrant bundle by-passes A V node

cr A 72-year-old man presents with a sudden onset of left nank pain. In the emergency room. the
patient is hypotensive. A"'pulsatile mass is palpated in the abdomen. Which of the follOWing is
MO T responsible for the pathogenes is of this patient's cond ition?
A. Atherosclerosis
B. A defect in fib rillin
A defect in co llagen
D. Long-standing hypertension
E. immune destruction of elastic tissue
A (ruptured abdominal aonic aneurysm)
cr A 42 year old man with a history of cardiac death in his fam ily presents with n sudden onset of
severe. retrostem al chest pain with radi:l.lion of the pain inlo his back. His left pulse is absent. A
high pitched diastolic blowing munnur that increases with expiration is heard immediately after
~.There is widen ing of the aon.ic knob on a chest x-ra) The mechanism for this patient's
condition is MOST CLOSELY related to ...
A. atherosclerosis
B. a defect in fibrillin
C. a defect in collagen
D. an infectious process
E. an acule myocardial infarction

74
B (dissecting aonic aneurysm)
<r A 26.yellr·old woman presents with a history of chest palpitations panicularly when anxious.
Physical exam reveals I mid-systolic ejection click followed by a munnur. The click and murmur
move closer to 51 when the patient is standing and closer to Sl when the lying down. The
mechanism for this patient's valvular disorder is MOST CLOSELY related 10•..
A. a defect in fibrillin
B. an infectious process
C. immunologic damage
D. myxomatous degeneration
E. R defect in collagen synthesis
o (mitral valve prolapse)
A 28·year--old patient has a family history of sudden cardiac death at a young age. The patien! has a
systolic ejection murmur that decreases in intensity when the patient is lying down and increases in
intensity when slanding up. An echocardiogram reveals abnormal movement of the anterior mitral
valve leanel against an llSymmetrica lly thickened interventricular septum. The patient MOST
LIKELY has ...
A. mitral valve prolapse
B, hypcnrophic cardiomyopathy
C. a congenital bicuspid aonie valve
D. 4 cardiac myxoma of the left atrium
E. infective endocarditis involving the mitral valve
B (hypenrophic cardiomyopathy)
<r A 6s·year--old man on the 5th day of hospitalization for an acute amerior myocardial infarction has
recurrence of chest pain and an inc rease in both CK~MB and troponin ~l.The patient MOST
UKELY has ...
A. papillary muscle dysfunction
B. a right ventricular infarct
C. a ventricular aneurysm
D. a myocardial rupture
E. reinfarction
E
r:r Which of lhe following is present in BOTH left and right heart failu re?
A. SJ heart sound
B. Pill ow orthopnea
C, Pulmonary edema
D. Neck vein distention
E. Dependent pitting edema
A (choice B in LHF. choice C in LHF. choice D in RHF, choice E in RHF)

75
You wou ld ex.pect a patient with an atri al septal defect to have which of the following oxygen
saturation SaO,) findill!s obtained bv cardiac catheterization?
Normal Patient PalieDt Patient Patient Patient
SaO~ % A B C D E
Right atrium 75 75 80 75 75 80
Right ventricle 75 80 80 75 75 80
Pulmonary 75 80 80 80 75 80
arten'
Pulmonary vein 95 95 95 95 95 95
Left ventricle 95 95 95 95 80 80
Aorta 95 95 95 95 80 80
SaO! ::: oxygen saturation
B (A::: VSO. B ::: ASO, C = POA, 0 = Tetralogy, E =transposition)
r:r A 30 year o ld man with a viral myocarditis who deve lops hypotension. neck vein distention , a drop
in blood pressure on inspi ralion, and mumed heart sounds most likely has , . .
A. hypertrophic cardiomyopathy
B. constrictive pericarditis
C. hypovolemic shock
D. a pericardial effusion
E. a dissecting aort ic aneurysm
D (Beck's triad or pericardial effusion)
Items 9-11
A. Aortic stenosis
B. Aortic regurgitation
C. Mitral stenosis
D. Mitral regurgitation
E. Tricuspid regurgitation
A 70 year old man with diminished pulses and a history of angina and syncope with exercise has an
ejection type murmur radiating into the carotid aneries A
A 58 yell! old man with left heart failure has an S3 and S4 heart sound and a pansyslolic munnur
located al the apex that increases with expiration D
r:r A 29 yenr old intravenous drug abuser has fever, a giant C~V wave. and a pansystolic murmur along
the left: sternal border that increases with inspiration E
r:r Which of the following types of hepatitis is associated with a vasculitis characterized by p~ANCA
antibodies?
A. Hepatitis A
B. Hepatitis B
C. Hepatitis C
D. Hepatitis 0
E, Hepatitis E
B (polyaneritis nodosa)

76
C7 A 7 year old boy presents with a low·grade fever. arthralgias. colicl.:y abdominal pain. and a
palpable purpuric rash limited to the lower extremities. Laboratory studies reveal a guaiac-positive
stool. a urinalysis with red blood cell (RBe) casts, hematuria. and mild proteinuria, and a CBC
with a nonnal Hb, Hct, and platelet count. Which of the following is the most likely diagnosi s?
A. Idiopathic thrombocytopenia purpura (ITP)
8. Systemic lupus erythematosus (SLE)
C. Poststrcptococcal glomerulonephritis
D. Rocky Mountain spotted fever
E. Hcnoch-Schonlein vasculitis
E

77
Respiratory system

Arterial blood gas (ABC) interpretatio n:


1. review Fluids and Hemodynamics and Acid-Base
2. r espiratory acidosis :
A. pH < 7.35
O. PCO,> 45
c. compensation is metabal ic alkalosis:
(1) HCOl ~o mEq/L is aellie respiratory acidosis
(2) HCOl >30 mEq/L is chronic resp irato ry acidosi s
3. respiratory a lkalosis:
A. pH > 7.45
B. PCO, <33
C. metabolic acidos is is compensati on:
(1) HeOl ~ t8 mEq/L is acute respi ratory alkalos is
(2) HeOl < 12 mEqIL is chronic respi ratory alka losis
4. metabolic acidos is:
A. pH <7.35
B. HeO, <22
C. resp iratorY alkalosis is compensation: peo2 decreased
5. meta bolic alkalosis:
A. pH >7.45
B. HCO, >28
C. respiratorY acidosis is co mpensarion: PCD, increased
6. mixed disorders:
A. if the pH is nonnal (7.35-7.45). it must be a mixed disorder since there is no full
compensation
B. if pH is very low, it must be 2 acidoses: e.g., cardiorespiratory arrest with respiratory
acidosis + metabolic acidosis
C. if pH is very high, it must by 2 a lkaloses : e.g.. respiratory alka losis and metabolic
alka los is)
D. example: salicylate intoxication Me mixed ABG asked on USMLE:
( \ ) normal pH
(2) PCO, <22
a. respiratory a lka losis
b. saJicylates overstimulate the CNS respiratory center
(3) HCO, <22
a. metabolic ac idos is
b. salicylate is an acid and salicylates damage mitochondria leading to tissue
hypoxia and lactic acidosis
Pulmonary functi o n tests in restrictive a nd obstructive lung disease: see schematic
1. restrictive lung disease-
A. problem in gening a ir inlo the lungs : decreased compliance
S. elasticity is increased: o nce air is in the lungs, it comes Oul rapidly on expirat ion
C. a ll volumes and capac ities are eq ually decreased
D. forced expiratory vo lume in I second (FEV l~ ) is decreased :

78
Seconds
a 1 2 3 4

RV FRC
6
t t
FIGURE 11-1. Schematic of the normal lung volumes and Cil-
pacities and the forced expiratory volume 1 second (FEVt.,.J
and forced vital capacity (FVC) I.n a normal penon (A) , a person
with restrictive lu.ng disease (8), and a person with obstructive
lung disease (C). me (functional residual capacity) represents
the volume of gas that remains in the lung at rest at the end of a
normal respiration and Is the most reproducible part of a puJm~
nary function test, since It does not require patient effort. Person
A (nonnal Individual) has an FEV,_ 01 4 L and an FVC of 5 L,
with FEVu.JFVC ratio = 0.80. Person B with restrictive lung
disease has a "mlniaturlfed" curve of person A. Note that the
FEVt - : and FVC are the same (3 L) owing to the lncrease In
elasticity In the lungs. hence FEVb.JFVC ratio::; 1.0. Person C
with the obstructive pattern Is having dlificulty in expelllng air
from the lungs owing to decreased lung elasticity. FEV t ....: = I L,
FVC ~ 3 L, and FEV b..fFVC ,.tio ~ 0.33. Both people B and C
have _reduced values for FEVtocc and FVe when compared with
the normal person; however. person B with restrictive lung dis-
ease has values between those of the normal Individual and
person C With obstruction. Person B has decreased compliance
In the lungs. hence less air enters the lungs. but owing to the
Increased elasticity, the air that Is In the lungs is expelled faster.
Person C has Increased compliance (alr easily enters the lungs) ;
however, reduced elasticity causes the lungs to trap aIr on expira·
tion, hence FEVb~ and FVe are much lower than In person B and
the FEV t.,JFVC ratio Is reduced. nc, total lung capacity; TV,
tidal volume; ve, vital capacity.
(1) FEV IJ« is how much air a person can expel from the lungs in I second after a
ma.ximal inspiration
(2) normal FEY he: is 4 liters
(3) usually <3 liters in restrictive disease
E. forced vita l capacity (FYC) is decreased :
( l ) FVC is how much air the patien! can expel from the lungs after a maximal
inspiration
(2) normal is 5 liters
(3) ofte n the same va lue as FEV 1m due to increased elasticity of the lungs
F. rat io ofFEV1 ,c/ FVC is increased :
(1) normal is 4/5 "" 0.80
(2) in resu ictivc disease, il is usually increased (3 /3 "" 100%)
2. obs tructive lung d isease-
A. problem with gening air out of tile lungs rather than into the lungs:
(1) compliance increased
(2) elasticity decreased
B. residual volume is increased. since all the air cannot get OUI of the lun gs:
(1) this automalica lly increases the total lun g capacity
(2) other volumes and capacities afe decreased
C. forced expiratory volume in I second (FEV Inc) is dec reased :
(I) no rmal FEV I ",. is 4 liters
(2) usua lly <2 liters in obmuctive disease
D. forced vital capacity (FVC) is decreased :
( 1) normal is 5 liters
(2) usually <2 liters, due to decreased elasticity of the lungs
E. ratio of F EVlw'FYC is decreased:
( 1) normal is 4/5 = 0.80
(2) in obstructive disease, il is usually decreased (1 /3 = 0.33)
.
3 summan' 0 r i monarv runcb on tests ID 0 bstructlvc a n d r estrictIVe une Id'Isease
' pu
Par a me ter O bstructivc Rest'r ieth-'e I
TOia l lung capac ity (TLC)
.
Increased Decreased I
Residual volume (R V) Increased Dec reased I
Tida l volume (TV) Decreased Decreased
Vital capacity (VC) Decreased Decreased
FEV l « Decreased Decreased
FVC Decreased Decreased
FEV'R.lFVC Dec reased Nonna l to in creased
DLeo Decreased Decreased
FEV luc - forced expiratory volume I second, Fye - forced Vital capaC ity, Oleo - diffuSion capacity
with carbon mo noxide

"'" Ca uses oran increased a lveo lar-ttrteria l gra d ie nt (A-a gr ndient):


1. A-a- difference between the alveolar PA0 2 and anerial Pa02 in the lungs
2. ca uses or i,nc reased g r ad ient-
A. ventilation without perfus ion:
(1) e.g., atelectasis
(2) produces intrapu lmonary shun!ing
(3) giving 100% 0 , does not increase the Pa02

79
B. perfus ion without ventilation:
(1) e.g., puLmonary embolus
(2) increased dead space
(3) giving 100% O 2 does increase POl since nomall)' ventilated lung can make up
the difference
C. diffusion abnonnalities: e.g., interstitial fibrosis
D. right to left shunts in the heart: e.g., cyano tic congen ital hean disease
J. rormulo used to caltuhue A-a gradient-
A. I'AO, - % oxygen (713) - PaCO,! 0.8:
B. using norma l values- PAOl "" 0.2 1 (7 13) ·40/0.8 . 100 mm Hg
4. caUJ6 orbypoumia " 'ilh a Dormat A-a gradinl-
A. depression of the respiratory center in the medulla:
(1) e.g .• blU'bilurates
(2) CNS injury
B. obstruction of upper airway:
(1) e.g., cafe coronary
(2) epi glon ilis
(3) croup
C. chest bellows dysfunctio n: c.g., paralyzed diaphragm
ani polyps!
1. aUergic-
A. MC type
8. occurs in adults
2. aspirin-re.lated-
A. clllled triad asthma:
(I ) take aspirin
(%) develop asthma
(3) nasal polyps
B. usually occurs in patients with chronic pain syndro mes
C. non-immune mechanism:,
( I) aspirin blocks cycloo1C.ygenase and leaves the lipoxygenast pathway open
(2) LTC-, D-. E. are increased., which cause bronchoconstriction
3. cystic fibrosis- any c hild with nasal polyps and a history of repeated respiratory infections
and diarrhea should have a sweat (est
Laryngeal nrdooma:
I. eausC5-
A. smoking: MCC
B. alcohol
C. asbestos
2. sile-
A. supraglottic area is MC locatio n
B. squamous cell carcinoma
3. cUnieal-
A. hoarseness

80
cr Alelecta5i,,:
I. MCC of revu 24 hs arter s urgery
2. palbophysiology-
A. collapse of alveoli due to mucus blocking terminal bronchioles
B. distal resorption of air through the pores of Kohn
l. dinical- signs o f consolidation:
A. increased tactile m:mirus (lAJA.hc ,.L i<d-";'. *~i\'
B. decreased percussion
C. elevated diaphragm (diaphragm moves up when alveol i collapse)
D. chest lag on inspiration
cr urraclanl :
l. drugs Ihal increase " uriaetanl synthesis-
A. glueoeortieoids- women who must deliver prematurely are ghlen glucoconicoids to
increase surfactant synthesis in the baby
n. thyroxine
2. eorn!lollo n-
A. Laplace's law states that coll apsing pressure or alveoli P - 2 T (surface tension)/r
(radi us of alveo li)
B. s urfacta nl deereases s urface tension and keeps the alveoli open on expiration
C. surfactant :
(I) lecithin (phosphatidylchol ine)
(2) synthesized by type II pneumocytes
(l) stored in lamellar bodies
Respiralory distnss syndrome:
1. palbogcacsis-
A. deerused surfactant in rCIUJ due 10:
( 1) baby is.premature
(2) poor glycemic control in mother with diabetes
•• hyperglycemia in mother~
b. hyperglycemia in fetus-t
c. increase in fetal release of insulin~
d. insu lin inhibits surfactant synthesis)
(3) c;sw:can section
•. no stress on baby-+
b. no increase in conisol
2. palhophysiology-
A. widespread atelectasis-+
8. musive iDtrapulmonary 5 buDtiD g~
C. hyaline membranes composed of protein from vesselleakllge (diffusion defect)
l. cliniCAl-
A. respiratory difficulty shonly after binh
8. cyanosis
4. complic.atjoDJ-
A. supe.roxide free radical damage:
(1) retrolental fibroplasia
(l) bronchopulmonary dysplasia
8. necrotizing enterocolitis

81
C. intraventricular hemorrhage
O. patent ductus arteriosis with machinery murmur: due to persistent hypoxemia
5. JU olRDS-
A. positive end. expiratory pressure PEEP)-keeps airways from co llapsing on expiration
B. deliver)' of surfactan t via PEEP therapy
C. oxygen
Adult reS I)ira to ~T dis l.ress syndrome (ARDS):
I. (,Il uses-
A. endotoxic s hoc k MCC
B. gastric aspiration
C. trauma
D. pneumonia
E. smoke inhalation
2. pa th ophysiology-
A. non.card iogenic pulmonary edema
D. neutrophil-relnted injury with destruction of'ty pe II pncumocytes (loss of surfactant)
and damage to pulmonary capi llaries ("leaky capil lary syndro me"
C. massive intrapu lmonur)' shunling from loss of surfactant is the most important
abnormal ity
D. hyaline membranes from protein leaking from capi llaries
3. sepa rate from cardiognic pulmonary edem a by pulmonary ca pillary wedge press ure
(measure of left ventricula r end-diastolic press ure)-
A. low in PCWP in ARDS
8. increased PCWP in card iogenic shock
4. higb morta lity
SpoD t.a neous pneumothorax :
1. causes-
A. id iopathic:
( I) MCC
(2) tall, thin males ,
(3) .!l!plUt<..IlLs.ubpkll,al bullae in a x of lung
B. iatrogenic : e.g., needle puncture of lung after performing II subc lav ian vein Slick
C. Marfans syndrome
2. pa thogenesis- portion of the lung collapses: pleura l cav ity pressure is the same as the
atmospheric pressure
3. clinical-
A. sudden onset of pleuritic type of chest pain
B. tympan itic percuss ion note
C. absent breath sounds
D. diaphragm elevated
E. tracbea de" iate! to the side of th e colla pse
4. l'1JSMLE ~ sudden onset of pleuri tic chest pain in a scuba d iver with collapsed lung,
elevation of the d iaphragm, and shi ft of Il11chea to the side of the collapse: spontaneous
pneumothora.x
T ension pneum othorax :
I. cu use-
A. penetrating trauma to the lungs

82
B. tension pneumatocysts:
(1) occur in patients with S. alll'eUS pneumonia (e.g.. cystic fibrosis)
(2) intrapleural blebs occur thai may rupture
2. pa lh ophysiology-
A. tear in the pleura allows air into the pleural cavity bUI prevents its exit
B. in creased intrapleural press ure sbifu tbe medias linal Sl rU CI ures to tbe oppos ite
side: compromises blood flow int'O the heart and OUI of the heart
C. diaphragm is depressed on the affected side
D. breath sounds absent on affected side
E. tympanitic percussion notc
3. Rt- insert needle into the pleura l cavity to relieve pressure
Typica l VI atypi ca l pneumooia:
I. ty pk.l-
A. sudden onset of symptoms
B. high fever
C. productive cough: usually pos itive gram stain for bnctcrin
D. signs of conso lidation on physical exam and x-ray
E. StreplococCW; pneumolliae MeC
F. chest x-ray is first step in management
2. atypical-
A. insidious onset
B. low grade fever
C. non to mildly productive cough
D. no signs of consolidation (interstitial pneumonia)
E. Mycoplasma pneumonioe MCC followed by Chlamydia pnelimoniQe (TWAR agent)
Community-acquired \ '5 nosocomial pneum onia:
t. commun ity a cqui~d - MC due toSrreplococcus p"eumo"iae
2. nosocomial-
A. develops while in the hospital
B. organisms in descending order:
(I) E. coli
(2) P. aeruginosa (Me if a respi rator is invo lved)
(3) S. allreus
Differential for rusty colored sputu m:
1. Streptococcus pneumonioe pneumonia
2. chronic congestive beart failure- heart failure cells
3. mit ra l stenosis
4. Goodpas ture's synd ro me
Summ arY ta bles of infectious d isease
Microbial Epidem iology Clin ical
Piltboeen I
Minol,irus MCC (25-30%) of common cold. Main Increased mucus secretions. sneezing
reservo ir are school children. Direct hand to and coughing. URI Me infection in
hand transfer of infected materiaVrespiratory clinical practice.
droplet infection. -100 serotypes (vaccine
unlikely).
J

83
Respiratory Mec of interstitia.) pneumonia (20%) and Ox with direct immunonuorncent
syncYlia/ virus bronchiolitis with wheezing (50%) in infants. techniques or ELISA test on
(RSI? Late falVwinter. H and [0 hand transfer of nasopharyngeal swabs. Rx: ribavirin
infected materiaVrespiratory droplet infection. for very severe infections.
lnfluen=avirus Significant cause of monality especially Mild cold to bronchitis to severe
among those over 55 years of age who have pneumonias (with exudate). Pneum·
underlying renal, cardiac or lung problems. onia allen has a superimposed
Type A virus produces pandemics and bacterial pneumonia (Staphylococcus
epidemics (most severe form). Type B aureus). Vaccine is effective in
produces epidemics. Type C is involved in preventing Du in 10 to 90% of healthy
sporadic cases. Hemagglutinins bind the virus young people. 1n older people. it is
to cell receptors in the nasal passages. only 50% effective in preventing flu
Neuraminidase dissolves mucus and but 85% effective in preventing
facilitates the release of vira l particles from death. Reye syndrome may occur in
the infected cell. Local epidemics resuh from children . R.x: amantad ine (inhibits
minor changes in the antigeniciry or the viral uncoating or transcription of
organisms, called antigenic drifts (point vi ral RNA).
mutations). Pandemics are due [0 Jlntigenjc
lb.i&. which involve mutations in
hemaJlJllutinin (need new vacci~~).
Rubeola Symptoms of rubeola begin to appear after the Fever. cough, conjunctivitis, and
(regulor 7- 14 d incubation period is finished . coryza (excessive nasal mucus
mewles) production) initially occur. K2R.I..ik
}IKU1 in the mouth precede the onset
of the maculopapular rash. Pneum·
onia is the Me coo. Wanhin·
Finkeldy multinucleated Riant cells.
I Chlamydia Ornithosis, or psittacosis, is a zoonosis (a Primarily resu lts in interstitial
! pslrraci disease contracted frp m animalS). lnhalation pneumonitis. Rx: erythromycin.
of C. pS;lIocl from p ~inacine birds (parrots,
I I Darakeets. Di.eons, and turkeys).
I~C~h~l~a-m-~~I~a----~D~m~p~l~el~~i~nfi~~~ti~on~~~""~'ili~o~ut~-an----.-v7iM--~Rx~:-,-etr~.-C-YC~Ii-ne-,--------------~
pneumonlae intermediate. 5% of community acquired
I (11VAR) atypical pneumonias. SeroeDjdemiologic
1 ~~~~__~I~ r."I~••t~i'lowmlS~shwi·~~w
~wwi
· lh~IC~of(o~nwv~a~
~ n'N.vdis.~.,se~,
~ __~~____~~______~~__~
I Chlamydia Newborn pneumonia. -10 to 20% of Presents with 51accaro cough. can·
fraclromatls newborns that pass through an infected birth juncti... it'is, tachypnea, bilateral lnspir·
I canal develop pneumonia. 8tory crackles, scattered expiratory
I wheezes. and hyperinnation (trap air).
t Afebrile. Eosinophilia. Rx: erylhro-
mycin.
t ~C~OJC;ii;'el~la~~--hOkD~lyUri~c~k.~nsYJ!i!lUitrainrusmiiii]j~n~ed~WI~·¥ili~o!iiui:1:!.~v!ie<:;£i!IO![r.tS~u~dfcd~e~nCO~Qs.se;,;coJriih~igh~~fi:;ev;'e;;r,~b~ead;a;i.;;c~h~e,l
~ bumeri; (Q Inhalation. Contracted by dairy fanne rs, chest pain, myalgias. Interstitial
• f~lIe1') veterinarians associated with the birthing pneumonia. Other problems: granu1o-
process of infected sheep, cattle and goats and matous hepatitis (50%), infective
J hand ling of milk in these animals. USMLE: endocarditis. Rx: doxycycline.
IL-________~I~
Dle r~so~Q~s~h~o~v.~I~in~.s~
he~e~
Dd~u~nn.~. ___________L______________________- - "

!
84
Mycoplasma MCC of primary atypical pneumonia. I Nonproductive cough. Upper respira-
pneumoniae 15- 20% of pneumonias in adolescents. 50% lOry tract symptoms (pharyngitis.
of pneunlonias in military recruits. earache) precede pneumonic manifest-
lncubation period 1- 2 wks ations. Interstitial pneumonia. Low
grade fever. Complications: bullous
myringitis (hemorrhagic vesicles on
the membranes), erythema mulriforme
(target-like leSions), and Stevens
Johnson syndrome (involves skin and
mucus membranes in a disseminated
manner), cold autoimmune hemolytic
anemia due to anti J. Lab: increa.sed
cold agglutinin titers. Rx: erythr~
mvcin or tetracvcline.
Microbial Gram Stain Comments
Pathogen
Streptococcus Gram + lancel-shaped Mec of community acqu ired tyPjcal pneumonia
I pneumo,tlae diolococcus I (bronchooneumoniB or lobar oneumonia). Rx: erYthromycin.
Staphylococcus Gram + coccus Pneumonia commonly foIlQ''''s influenza infections (Me
(IlIre,u bacterial pathogen), Major pathogen in cystic fibrosis.
Common cause of nosocomial pneumonia. Hemorrhagic
puJmonary edema, abscess formation, and tension
pneumatocyslS (intrapleural blebs), which may rupture and
I produce oneumotborax. y,.l lnw I'!nlor,.d c.nutum.
Hemophilus Gram-rod Common cause of pneumonja in cystic fibrosis and COPO.
injluetrzoe Rx: TMP/SMX. Acute epiglottis in children. Decreased
incidence due to immunization. Cause of inspiratQry stridor.
Thumbprint sil!;tl on lateral x-ray of neck. Rx: cefuroxime
Pseudomonas Gram - thin rod . Water loving bacteria transmitted by respirators. Common
aerugitto.ra cause of nosocomial pneumonia and MeC of pneumonia in
cystic fibrosis. Me pneumonia in ICU/CCU (due to
respirators). Blood vessel invader (hemorrhagic infarctions).
Green colored sputum (pyocyanin). Rx: antipseudomonalj3-
lactamase susceptib.le oenjoiUins (~.IZ .• mezlocill in)'
Klebsiella Gram - fat rod with Pneumonia commonly associated with alcoholics and Me
pneumonlae capsule pneumonia in nursing homes. Blood-tinged. mucoid sputum.
Tends to involve the upper Jobes and cavjtales like
reactivation lB. Lobar consolidation and abscess formation
are common. Rx: third generation cephalosporin
Nocardia Sirict aerobe. Gram + Granulomatous microabscesses in the lungs in
Qsreroidu filnmentous bacteria. immunocompromised patients. Rx: TMP/SMX
Panially acid fast
Actinomyces Anaerobe. Gram + Draining sinuses in the jaw. chest cavity, and abdomen.
Israeli filamentous bacteria. Sulfur granUles contain bacteria. R.x: ampicillin or penicillin
G.
Legionella Gram - rod (need IF Water loving bacteria (water coolers). Pneumonia with dry
pneumophila staiD or Dieterle silver cough. malaise, flu·Hke symptoms, bloody sputum, and
stain sttikinR. fever. Other flndintts: anhrule.ias renal and CNS

85
findings. Macrophage rather than a neutrophil response in
tissue. Can produce hyponatremia secondnry to hypo-
reninemie hypoaldosteroojsm from interslitial nephritis. Rx:
erythromycin or tetracycline + rifumpin.
Mycoboclerium Strict aerobe. Acid- Droplet infection. Primary IB: upper pan l o~r lobe, lower
tuberculosis fast pan of upper lobe. Ghan complex. Usually resolves.
Reactivation IS: upper lobe, cavitary lesion. Kidney Me
exuapulmonary site.
Candido Not dimomhjc. Sud- Vessel invasion produces hemorrhagic infarcts. Rx:
olbicans ding yeastS and pseudo- amphotericin S or fluconazole
hypha. Lung disease
contracted from infect-
ions of indwelling cath-
eters.
Coccidioides Djmorphic. Spberu les Flu-like symptoms and erythema oodosum (painful nodules
Immilis with endospores in tis- on lower legs). Pneumonia may be localized (egg shell
sues. Inhaling arthro- cavity in lower lobes). "coin lesions". miliary spread in the
spores while living or lungs andlor throughoUi the body. African-Americans3
passing through the Mexicans, and Filipinos have severe infections. Lab:
Southwest or San Joa· culture, direct visualization of the spheru les with
quin vaHey in Califor- endospares, skin test (usefu l), and serologic tests (useful).
nia ("valley feve r"). Rx: fluconazole

CryPIOCOCCUS Not dimornbjt. Sud- Primary lung disease (40%). Produces a granulomatous
neojormolls ding yeast with narrow reaction. if immunity is intact but no in'flammatory ~action
based buds. Found in ifimmunocompromise<i. Lab: culture., direct visualization of
pigeon eXcreta (around the organ isms and by serologic tests. Rx; amphotericin B or
buildings, outside of- fluconazole
fice windows, under •
bridges). Me fungal
opportunistic infection.
HislopifUmo Dimorphic. Me syste- Simulates TB lung disease. Donnant phase like TB (can
capsulolum mic fungal infection. reactivate). Produces coin lesions, conso lidation, miliary
Me in Midwest Inhal- spread. cavitation. Old infection sites characteristically
ation of spores. Assoc- contain c.alcified granulomas. Mec of sclerOSing
iation wilh ~ lUrm mediastinitis (fibrous tissue In mediastinum with
(Starlings), cave em: encroachment on all mediastinal structures). Lab: culture,
lorers (spelunkers), direct visualization in tissue (organisms in macrophages),
abandoned warehous- skin tests (useful), and serologic tests (useful). Rx:
n. yeaSt fonus in mac- ilraconazole (moderate disease), amphotericin B (severe
roPhaacs. disease)

~o
~ 00 8
00

86
BlastomYCl!s Dimorphic. Yeasts lnyolves skin (skin has a verrucoid appearance resembHng
dermatitidis have b[oad based buds. squamous can:: inoma) andlor .lung. Male dominance. Lab:
Primarily along the culture, direot visualization of the yeast forms in tissue. R.x:
Southeast coast and itraconazo le
into Midwest. AJong
inland waterwa.ys with
beaver darns. Inhalat·
ion.
Aspergillus Not dimol]hic. Fru iting Aspcrgilloma, refers to a fungys ball (visible on :Nny) of
fumigatus .QQQy and narrow·angl· matted hyphae and fruitin g bod ies that deve lops in a
~ brall~h il]g sel2tate preexisting cavity in the lung (e.g., old TB site). Cause of
hl!l1hae. massive hem!:H2lY:ti5:. Allergic Q[Q(lcbol2ulmonm disense
involves both type J and type III hypersensitivity reactions.
IgE levels increased. Vessel invader with hemorrhagic
infarctions and a necrotizing bronchopneumonia. Common
sinus infection in AlDS. Lab: culture, direct visualization.
Rx: amphotericin B or itraconazole
Absidi.. ~ot diwQmbiSl, Wide· Clinical settings: diabetes, immunosuppressed patients.
Mucor. angled h:mhae withoy! Vessel invader and produces hemorrhagic infa rcts in the
Rhizopus septa. lung. Invades tbe frQntal Jobes in diabetic keroncido 5i~. Lab:
culture, direct visualization. Rx.: amphotericin B
Pneumocyslis Reclassified as fungus. Opponunjsli c infection. Me initial AIDS-defining infection.
carinil CySts attach to type I Lungs are dry and consolidated. Patients present with low-
pneurnocytes. Poorly grade fever, dyspnea and tachypnea. Bronchoalveolar
visualized with gram lavage and lung Bx identify organisms. Chesl x·ray: diffuse
stains but stajn well alveolar and interstitial infiltrates. Rx: TMP/SMX. Given
with silver and Qiemsa prophylactically when CD4 counts <200 ceUslJ,LL.
stains. Foamy alveo lar
infillTBte. Occ\lrs when
CD" T b!i:l~r count
<200 celisiuL. >
Parasites with Ascaris, Strongy l oides~ hookworm.
larval phase in
fu,-,g~
cr ._.~WliJi:iiSpIEiI~d~:
1. mliiiii Iiiii:ijiII<-ft6....1s podeIItJ-USOe:lited wttIlL._Ur.U11htOJ~
rpjij
A. pneumonia
B. bronchiectasis
C. sepsis
D. tension pneumothorax from ruptured pneumatocysts
2. ot:paeumoatalD ~ flliroslJ- P. aerugillosa
3. v e_ ~O{er 10 lirizoaa: itb _" Jra ory Infemoa+- coccidio idomycosis (nol
histoplasmosis, too dry a climate and mainly in Ohio and Midwest)
4. fl'iiIge.poID.er In N. '" Clty:Mili .... pliiifOi'jIlar.dlo
A. most likely histoplasmosis (starling dung) or cryptococcus (pigeons)
B. may ask what to Rx the patient with: amphoteric in
5. (!!J!lraroo:Jllji)JJem usoclat&l Wltbfll!!i!arno R!e<llDS aUU\l...OH WlDdiJ.l" illl or aad
cryptococcus

87
6. dilHles wbere StreplOC:OC:cUJ paeumoa.iae is MCC-
A. community acquired pneumonia
B. meningitis in adults > 18
C. otilis media
D. spontaneous peritonitis in children with ascites
E. ~psis in childre n with HbSS d isea.R
F. sinusitis
7. dlieuetl "lIere PSt!lIIiolftDnas ae'''IiJrosa is MCCI-
A. ICU pneumonia (respirators)
n. COD in bum patien ts
C. COD in cystic fibrosis
D. cellul itis/osteomyelitis in puncture wounds of foot in patient's with rubber footwear
E. ma lignant external otili s in diabetes
f. ec thyma gangrenosum
G. hot tube folliculiti s
8. di.Hatet wbere BC!mophUus IDOueDZIIe ".MCC- nc ute epiglottit is ( d~c rease d due to Hib
immun ization)
9, Me !JDs-denDiDg IDfeedOD!... Pneumocyslis cari" U pneumonia
10. eldetty mIlA, wlio lives .1 bome wlda It.ia wife. develops pDe.moDia- Slreprococcils
p nellmonia
II. ellat J·r.y'Widt ricIIt middle lobe p.eamo .... (obsc ures right ma rgin of the hea rt)-
A. probably related 10 obstruction by a bronchogenic carcino ma
8 . could also be aspiration with the patient lying down on the right side
11. dell"le le'aat wfth mttllto cough, 'igas of hype.....ratioa. roDjUDCtivt~ Chlamydia
trocMmalis pneumo nia
Lung a b5ces!l~ :
I. ca USe5-
A. aspiration o f oropharyngeal male rial Is MCC: mixed acrobe/anaerobe infection
8. lobar pneumonia
C. hematogenous spread
2. lIi rlnuid level on x-ray
Lung locations wilh aspi ration :
I. standing/s il'ting- poslerobasal segmeDl right lower lobe
2. lyi ng down on back- s uperior segment rigbt lowe r lobe (MC site for abscess)
3. lyi ng on right side-
A. right midd le lobe
B. posterior segmenl of right upper lobe
4. lyi ng o n len sid e- lingula
PulmoDaryem bolus :
I. sou rce- femoral vein
2. pathopbys lology-
A. perfusion defect: increases dead space
B. majoriry of peripheral emboli do nOI infarct the lun gs
C. produce mild hypoxem ia
3. dinical setting-
A. postpartum
B. pos t~opero.li ve

88
4. clio lcal-
A. sudden onset of dyspnea and tachypnea: Me symptom and sign. respectively
B. feve r
C. pleuritic chest pain
S. lab-
A. perfusion scan firs t step in work-up
B. respiratory alkalosis
C. mild hypoxemia
D. increased A-a gradient
E. pulmonary angiogram gold standard for Ox.
6. 118MLE Kea~ gross photo of a large saddle embolus in a patient on prolonged bed rest;
usually die of acute right heart strain
Pulmonary hyperlension:
I. causes-
A. chronic hypoxemia:
(1) hypoxemia vasoconstricts pulmonary vessels and vasod iJates peripheral vesse ls
(2) high altitude residents
(3) chronic lung disease
B. loss of pulmonary vascu lature : e.g.,
(I ) COPD
(2) restrictive lung diseases
C. left to right shunts with eventual volume overload of right hean
D. mitral stenosis with backup of blood into pulmonary veins
2. patbology-
A. atherosclerosis of pulmonary arteries
B. smooth muscle hypertrophy of pulmonary vessels
C. angiomatoid lesions
l. clinieal- ,
A. accentuated P2
8. cor pulmonale:
(1) pu lmonary hypertension (PH) leads 10 righl ventricular hypertrophy
(2) definiti on applies to primary PH of pulmono.f)' Ilrlel)' or PH due to lung disease
(3) does not apply to PH and RVH of cardiac origin of primary origin
a. e.g., mitral stenosis
b. left to right shunts
C. primary PH occurs mainly in youne women: progressive dyspnea, chesl pa in.
syncopal epi~es
D. pruning o f pulmonary arteries noted on x-ray
Immotlle cilia syndrome (Kartagener's syndrome):
1. abse_DI dynein arm in cilia
2. dinical-
A. situs inversus:
(1) vessels and chamber in the hean are Slill nonnal (USMIZ)
(2) not a complete transposition
B. infenility in males/females
C. bronchiectasis
D. sinus infections

89
Restrictive lun g diseues:
I. decreased compliance and increaJed elasticit)' du e to Interstidal fibrosis
2. tau es-
A. pneumoconioses MeC: dust borne diseases
B. sarcoidosis
C. hypersensitivity lung diseases
O. drugs: see Environmental pathology notes
J. coa l worker's pneumoconio5is-
A. exposure to coal dust
O. "black lung" disease
C. increased Incidence af TS but not cancer
D. Caplan syndrome: rheumatoid nodules in lungs + coal worker's pneumoconiosis
4. !'I ilicosi!'l-
A. exposure to s ilica dust: e.g., sandblaster
B. nodular, fibro tic masses in the lungs: filled with silicA crystals
C. increased risk for TB but not cancer
O. association with Cap lan's syndrome
S. as bes losls-
A. exposure: to asbestos:
( I ) pipe· finer in shipyard
(2) roofer for over 20 ys
8. no risk for TB
C. smoker + asbestos exposure predisposes to primary lung cancer > mesothelioma
D. non-smoker + asbestos exposure predisposes to primlll)' lung cancer > mesothelioma
E. asbestos body (ferruginous body) looks like a dumbbell (fiber covered by iron)
6. bypersensitivi ty pneumonilis-
A. farmer'S lung:
( I) inhalation of thermophilic actinomycetes
(2) see Immunopatho logy notes
B. silo filler 's : inhalation of nitrogen dioxide fumes
C. byss inosis:
(1) "Monday morning blues"
(2) patient works in a te:<tile factory and has contaci with cOllon. linen. hemp
7. lIumman Rich lung-
A. ho neycomb lung
B. end-stage of a lveo litis syndromes: interstitia l pneumon itis syndromes- e.g., usual
internitial pneumonitis
8. sarcoidos ls-
A. ..&DllJuJomatous disease of unknown etiology
B. primarily targets the~ noncasealing granulomas in hilar nodes and parenchyma
C. uveitis with blurry vision ~A4 )
D. salivary gland enla'l!,men\:.d<.t<h ~ j , S
E. MCC of non-infectiolr~ngTanulomatous hepatitis
F. nodu les in skin contain granulomas
G. la b:
(1) increase in angiotensin converting enzyme
(2) anergy to common antigens
(3) low CD. T he lper cell counts- used up in produci ng granulomas

90
(4) hypercalcemia
O bstructive lun g dinuc:
I. types-
A. chronic bronchitis:
( I ) MC Iype
(2) clinical Dx.- productive cough > 3 mths for 2 consecutive yrs
B. emphysema
C. bronchial asthma
D. bronchiectasis
2. .s umma n ' cbart CO mpa nD}! Cb rO Rlc • b ro ne h"IUS w .ll h empthvsemu-
Parametl!r Empb\'sema Cb ronic Bro nchitis
Onset of dyspnea Progressive. constant. severe Imerminent and often exacerbates
with infection
Sputum production Scanl Increased and purulent
Appearance " Pink puffer" (not cyanotic), thin, "B lue bloater" (cyanotic due 1O
weight loss respiratory acidosis). obese
AP diameter increased (hyperinflation) Less hyperin flati on than emphysema
Breath sounds Diminished owin~ to hyperinflation Wheezes and sibilant rhonchi
PaD, Mild hvpoxemia at rest- Moderate to severe hYPOxemia
PaCO, Normal 10 low (respiratory Increased owing 1O respiratory
alkalosis. reason for "pink puffer") acidosis- trap C02 behind tenninal
bronchioles filled with mucous
Totallunfl capacitY Markedly inc reased Nonnal to slighlly incrt3SCd
Residual volume Markedly increased Mildlv increased
I Cor pulmonale Infrequent untillal'e in the disease Commonl y present
Vent"illltionlperfu- Matched losses of ventilation (resp- Major mismatch owi ng to primary
sion iratory un it) Ilnd perfusion (loss of involvement of the tenninal bronch·
capillary bed) iole (prox..imal lo the respiratory uni!.
hence more units are affected)
J. types or I! mpb yse~-
A. emphysema involves ~n ions:7fthe ~iratQO' l!njl '
(I) respiratory bronchiole '
(2) alveolar duct
(3) alveo,:Ii::-:=:-;:::==,
B... ccnlrilobula r emphysema :
( I) ~sm.2ke rs
(2) destruction of elastic tissue suppon in the respiratory bronchiole
(3) EEper lo~ involved
C. *" panllcin ar emphysema :
(1) a.-I antitrypsin (AAT) deficiency
D. primary AR disease
b. a~quired in smakw;: chemicals in smoke inactivllie AA T
(2) involves the enlire res irarorv unit (respiratory bronchiole. alveolar du;:t, and
alveoli)
(3) lower lobe disease
D. IlMLE .....rIo:
( I) identify x-ray of a patient with emphysema
(2) look for increased AP diameter and ~I?!!~~~

91
4. bronchiectasis-
A. cystic nbrosis is the MCC in the- United States: TB is the MCC in third world
countries
B. pathogenesis:
( 1) o bstru ~io n and~
(2) ~d broDcbi extend to the lUll!!, periphe'l'
C. clinical: cough up cupfuls of foul smelling sputum
5. bronchial asthma-
A. MC chronic respiratory disease in children
B. episodic, hyperreactive, reversible, small airway disease that primarily targets the
temlinal bronchioles
C. causes:
(I) exposure to allergens
•. MeC
b. IgE-mediated type I hypersensitivity
(2) non-immunologic
a. aspirinfNSAID sensi tivity
b. co ld temperature
c. exercise
d. environmental pollutants

D. clinical:
•• smoke

(I) epi sodic wheezing


(2) nocturnal cough
(3) increased AP di~ter due to air trapping
E. lab findings: l:" . . .
·d l't )
(1) respiratory a!!.-a!Q~js may progress inlo respiratory acidosis if bronchospasm
"fsnol relieved
(2) hypoxemia
(3) decreased FEV fltt values
(4) eosinophilia ~
(5) positive skin tests for allergens
Rx:
(1) albuterol medihaler for mild disease
(2) corticosteroid medihaler for moderate to severe disease
Lung cancer:
1. cause!-
A. see neoplasia
B. decreasing inc idence in men/increasing incidence in women
C. 2nd MC cancer in men and women
D. MCC of death due to cancer in men and women
2. MC primary cancers in descending order-
A. adenocarcinoma
B. squamous carcinoma: ectopically secrete PTH-like peptide (hypercalcemia)
C. small cell carcinoma: ectopically secrete ACTH (eel'opic Cushings) and ADH (SiADl:I)
3. MC cancers of lung- metastasis:
A. breast MCC
B. renal adenocarcinoma

92
C. choriocarcinoma
D. colorectal cancers
... lung sites-
A. centrally located:
( I) squamous cell
(2) ,mall cell
8. peripherally located: adenocarcinoma
S. s moking rehllions bips-
A. squamous and small cell cancers: strongest relationship
n. adenocarc inoma:
(I ) Me primary lung cancer in smoke rs and non-smokers
(2) bronchioloalveolo.T carci noma has no smoking relationship
6. eliniclll-
A. cougb Me symptom
o. weight loss
C. hemoptysis : someti mes massive
D. Pancoast tumor (superior sulcus tumor):
(1 ) squamous cancer at lung npc:( invo lving brachial plexus and superior cervical

1.
E.
F.
ganglion (Homer's syndrome)

clubbing
sites ror melUtasis outside bilar lymph Dodu-
r/
(2) ~~e includes ips ilatcrallid Illg. miosjs. anhvdr sis
superior vcna caval syndrome

• •

B.
ad",n.I, MCC ,ite
liver
~"' ~l;;:..,-y ~
-
-Ir~-
f3<.
C. brain -t~::, ho.v;JP,?/*
O. bone: lytic metastases .. (It;" 'K1,~d
8. prognosit- !Sk.'ao "'j
A. best for squamous
8. worst for small ce ll: usua lly non-opc::rable
9. bronchial carclnoid-
A. low grade ma lignant APUD tumor
8 . cl in ical:
(I ) hemoptysis
(2) local metastasis
(3) carci noid syndrome without need to metastasize
r:r lJSMLE 4 . . . . .boat • roofer for l! yean ........oker for 10 yean: I
I. &rt!alest riJk is for primary IUDg cancer
2. primary lung cancer is more com mOD tban a mesothelioma, ,,'hetber a s mok ~ r R.r ~
3. mH otbelioma-
A. no smoking relationship
8. asbestos relationship in most cases
C. highly malignant tumor originaling from the pleura
D. takes 20-30 yrs to develop
<r CIWIiCIIq lo. .d II ....... ia tile _k .... uterlo ••bat III • palin. lllvoIvttl III .....d-o.
collision (USMLE): ruptured esophagus (Boerhaave's syndrome)-
1. Hamman 's sign in tbe chesl due to s ubcuta neous emphysema from air dissecting into
che tiu ue

93
2. other scena rios could be an a lcoholic w ho is recthing o r n bulimic w ho is vomiting
Solita ry coin les ions:
1. ca uses-
A. MCC is gran ulo ma tous disease; e.g .. TB. hist'op lasmosi s
B. most are benign in patients <35 years o ld
C. most are malignant in patients >50 years o ld
O. calcifications and lack of growth are benign reatures
1. bronchial ba ma rto ma -
A. solimry coin lesion
B. loca lized overgrowth or cani lage: nol a neoplasm
C. " popcorn type" or configuration on x-ray
Mcdia"Unum :
1. a nterio r ruediustinum Me involved with disease-
A. thymoma Me tumor fo llowed by nodular scleros ing Hodgkin's disease
B. neurob lastoma in ch ildren, gang lioneuroma in adu lts Are Me overaH mediastinnl
tumo rs: located in posterior mediastinum
2. thymus ond myastbenia gravis-
A. J!!y.!l1ic byp~[J)lasiQ is Me fi nding in thymus; ·enninal fo llicles composed of B ce ll s
that s nthcsize: antibodies against acerylcholine receptors
B. thymoma is less common finding: pure RBe ap lasia sometimes noted with thymomas
C. thymectomy is sometimes used in R.lo: of myasthen ia gravis
3. middle mediastioum- pericardial cyst M e disorder
C'ir Pleural nuid :
I. la b fiodings (hat diSiinguish a transudate from ex udate in pleural nuid -
A. PF pro tei n/serum prolein ratio >0.5 is exudate
B. PF LDHlserum LOB 0.6 is exudate
C. PF LOH twcrthirds the upper limit of nonnal of the serum LDH is exudate
2. PF uudates-
A. pneumonia MeC
B. pu lmonary infarctio n: hemorrhagic exudate
C. cance r: hemorrhagic exudate
3. PF transudates- congestive heart fai lure MeC
4. PF fiodings in TB- e:<.udate with a lymphocyte dominalll ce ll count
5. PF with a mylase-
A. acute panc reatitis: left-sided pleural effusion
B. Bocrhl18vc ' s syndrome

QaestioDl ..... duria, lb. board revl... :

In which of the fo llowing cl in ical scenarios invo lving patients with lu ng disease wo uld you expect
pulmonary function studies to exhibit decreased com pliance. increased elasticity, and an increased
FEV InJFVC rario?
A. 6 year o ld child with recurrent respiratory in rections and sleatorrhea
B. 28 year old non·smoking male with bilateral lower lobe emphysema
C. 56 year old smoker with productive cough, dyspnea, and cyanosis
D. 10 year o ld girl with bro nchial asthma requiring systemic steroids
E. 39 year o ld dyspneic African A merican with bilateral hUar nodes

94
E (sarcoidos is. restrictive lung disease)
CiI'" A 45-year old woman 24 hours post-cholecystecl0my devel ops fever and dyspnea. Physical exam
revellis decreased percussion, increased tactile fremilus. and decreased breath sounds in the right
lower lobe. The diaphragm is ele,'ated and there is inspiratory lag on the right side. The patient
MOST LIKELY h.....
A. atelectasis
B. B lung abscess
c. bronchopneumonia
D. a pulmonary infarction
E. :I spontaneous pneumothora.lo:
A
CiI'" An afebri le 23-year-old man develops a sudden onset of left-sided, stabbing chest pain with
dyspnea . Physical exam of the left chest revea ls hyperresonance to percuss ion, deviation of the
lrac hea to the left. elevation of the diaphragm. decreased tac tile frernirus. and decreased breath
sounds. The MOST LlKEL Y diagnosis is ...
A. pleural effusion
B. bronchopneumoni3
C. tens ion pneumothora.«t
D. n pulmonary infarction
E. spontaneous pneumothorax
E
r:8" A newborn child develops dyspnea., tachypnea, intercosta l muscle retractions. and cyanosis 4 hours
after binh. The mother developed gestational diabetes mell itus and was in poor glycemic control
throu8hout the pregnancy. A chest x-ray revea ls a "ground glass" appearance in both lungs. The
primary mechanism for this patient's respiratory problem is ...
A. aspiration of amniotic fluid
B. group B Slrl!ptococcus pneumonia
C. decreased production of surfactant
D. Chlamydia trachomaris pneumonia
E. heart failure from congenital heart disease
C (RDS)
Which of the fo llowing describes 8 pneumon ia due to MycoplllSf1/n pneumoniae rather than
Streptococcus pneumoniae?
A. High fever
B. Insidious onset
C. Productive cough
D. lncreased tactile fremitus
E. Neutrophilic leukocytosis
B (all other choices are those ofrypica l pneumonia)
CiI'" A 58-year-old smoker presents with weight loss and cough. Physical eum reveals a mild lid lag on
the left and a pinpoint pupil, scanered sibilant rhonchi throughout all lung fields that clear with
coughing, and an increased anteroposterior diameter. Based on these findinl!$. you suspect the
patient has ...
A. a Pancoast rumor
B. a thoracic outlet syndrome
C. the superior vena caval syndrome
D. obstructive lung disease without pri mary cancer
E. obstructive lung disease with metastatic cancer from another primary site

95
A (Homer's syndrome also present)
<Jf" A 6S year old man wilh urinary retention secondary to prostatic hyperplasia. develops spiking
fever. and tachypnea. Physical exam reveals intercostal musc le retractions and bilateral inspiratory
crackles. A chest x~ray exhibits bilateraJ interstitial and alveolar infi ltrates. ABGs demonstrate
severe hypoxem ia. You expect the blood cu hure reveals ...
A. gram positive diplococci
B. grum negative diplococ.ci
C. gram positive cocc i
D. gram negative rods
E. grnm positive rods
o (Gram negative sepsis due to E. coli [gram negat'ive rod1 into an AROS)
<Jf" Inspirotory stridor is commonly associated with ...
A. a respirOlory syncytia l virus infection
B. 3 parainfluenza vi rus infection
C. aspirin· induced asthma
D. rhi nov irus infections
E. choanal atresia
8 (croup or laryngotracheobronchitis due to parainfluenza virus, obstruction is in the trachea, "steeple"
sign on latera l x·ray of neck)
r.r Chlamydia trachoma!is and the respiratory syncytial virus are BOT H commonly associat~d with .. .
A. an interslitinllype of pneumonia
B. laryngotr8chcobronchilis (croup)
C. the respiratory distress syndrome
D. typical community-acquired pneumonia
E. hosp i U\l ~ acquired (nosocomial) pneumonia
A (RSV MeC of pneumonia Ilnd bronchiolitis in chilrlrf!n)
<Jf" Which of Ihe foll owing is more often associated with Klebsiella pneltmoniae than P5eudomoIJa5
aerugi" osa?
A. Upper lobe cavitarion
B. G ree n ~co l ored sputum
C. Assoc iation with cystic fibrosis
D. Association with respirators
E. Productive cough
A (choices B, C. 0 are fea tures of P. aeruginosa, both have producti ve cough fchoice ED

"'" In a 30 year old man who lives in Tennessee. you would expect a calcified so litary coin lesion in
the lung to represent.. .
A. n foreign body
8 . an old granuloma
C. metastatic cllncer
D. a primllty lung cancer
E. 3 bronchial hamartoma
B (histoplasmosis)

96
1\ 55-ycar-o ld non-smoking coa l \\orker has anhri lis and nod ulnr lesions III the lungs His PPO sl..in
le:,1 b. negll li vc. You suspecllhe patien t has . ..
A. system ic lupus ery thematosus
B. Caplan's syndrome
C mctast.1tic lung disease
D pnmary lung cancer
l: miliar) tuberculosis
13
In a 6:! year o ld man who ha) been a roorer for :!5 years and a :,m" l..c:r f(lr 10 ) ears. \\ h,,:h oi Ihe
fo llO\\ 111£ ca ncc:rs would he be most likel) prone 10 dc\'clo pmg7
A 1)lcuralml!sOI hclio ma
B. PrimaJ) lun g cancer
C Laryngea l carcinoma
D Oral c:m cer
E. Pancrcntic cancer
A fa:.bc:slO$ exposure. same answ\.': r even if hc was not a smoker}

97
Gastrointestina l

(jf"' Common infections in the ora l c:lvihr


Orpullism Disease C linical Com ments
Co~sack i evirus A. Herpangina Children between 3- 10 yr. Fever, pharyngitis, c.ef\!jcal
lymphadenopathy. Multip le vesicles andlor ulcers
surro unded by erythema located on soft pa.late and
pharym!:ea l ti ssue.
Coxsackie.vi rus A Hand-foot-mouth Young children . Fever and ves ic les located in mouth
disease and dista l extremities.
lierpes simplex G ingivostomat itis Children <5 years o ld . Prinllu'Y disease produces
type I systemic s ign s of fever and cervica l adenopathy. Painful
vesicles usuall. develop on the lips. gingiva,
oro pharyngeal mucosa . Remains dormant In sensory
ganglia . Recurrent herpes simplex ( herpes labia lis) is
nonsyslemi c and invo lves the vermilion border of lips .
Reactivated bv stress, s unli~ht , menses.
Epstein·Barr virus Pharyng itis lnrectious mononucleosis is com monly associated with
(EBV) an exudative pharyngiti s/tonsi llit is and painful cervica l
adenopathy. Palatal petechia are present .
Hairy leuko plakia Bilateral white, ha iry excrescences all laleral border of
to ngue . Pre-ArnS defining lesion . Virus ind uces
squamous hy perplasia o f the ep ithelium (not a
nrecursor to cancer) . R.x: acvclovir
Streptococcus Pharyngitis - 20- 35% of cases o f exudative pharyngiti s/tonsi llitis.
p),ogenes Ito nsi llitis Majority (-50%) vintl induced (adenovirus, EBV,
coxsac,kievirus ). Lab Ox: direct antigen detection or
culture (go ld standard). It,,: oenicillin G.
Candida albicalls Oral cand idias is Thrush (pseud omembranous candidiasis) may occur in
, neonates (while passing through the birth canal ) or
immunocompro m ised patients (common 111 AIDS).
Pseudomembrane covers the ora l mucosa. When wiped
off, it leaves a bleeding mucosal surface . Gram stain
revea ls budding yeasts and pseudo hypha . Pre-AIDS
definine.lesion .
Lcukoplakic lesions In mouth or genital area : Bx to RIO squam ous dysp lasia/cancer
Smokeless tobacco: verruco id squamous cancer in the mo uth
Sq uamous cllncer in mouth:
1. caUSC5 -
A. smoking MCC
B, alcohol
C. smoking/a lcoho l are synerg istic
2. sitcs-
A. lateral border or tongue Me si te
B. lower lip: note- upper lip is a basa l cell carcinoma
C. under tongue

98
Oral pigmentation :
I. Pcurz-Jeghers syndrom e
2. Addi50o's disease
3. Pb pobooing in adult
Gum hyperplasia:
I. phcnytoin
2. pregnancl'
J. sc un')'
4. acute monocytic leukemia
Deviatio n of uvula to tbe opposite side in a patient witb c'x udalive tonsillitis : peritonsillar
nbscess
Snlh'ary gland5:
1. mumps MC infection
2. mixed tumor MC neoplasm- parotid Me locution
Dysphagin for solid, not liq uids: sign o rmechlln ical obstruction-
I. st rictures
2. Plummer· Vinson esophageal web associllted with Iroo deficiency
3. esopbageal cuneer
4. lIarrea's _ ....... willi aJcuatio. oad strict... (USMLE)
Dy5pbugill for 50lids aod liquids:
1. peristalsis problem
2. calQCS-
A. acbalasia MCC
B. progressive systemjc sclerosis/CREST syndrome
C. polymyositis: upper esophagus in striated muscle
D. myasthenia gravis: upper esophagus in strialed musc le
OdynopbagiD:
1. IJDinful swallowing
2. MCC Is esopbagitis .,
TE fis tula :
1. proximol e!opbagus ends blindly a nd distill esophagus arises from Ib e trllcben (air in
the stomach)
2. polyhydrymnios in molher
Po lybydnlmnlos:
1. TE fi s tula
2. anencephaly
3. duodenal atresia
Patient wilh dyspbagia and a fetid odor to his breath :
1. Ztnktr' s diverticulum
2. MC diverticulum in the esophagus
3. pulsion diverticulum througb an area ofweakne5S In cricophnryngeus muscle
4. (o llecl!J food (bad breath)/innamed
MOlor dborden of esopbagus:
I. Rchalasia is MC motor disorder or esopbagus-

99
A. ra ilure or r elaxatio n of LES s pb iocte r due to absent myenteric ganglion cells:
ganglion ce lls nonna lly contain \'asoinleStina l peptide. which re laxes the LES
B. ape ristalsis and dil atation of proximal esophagus
C. clinica l:
( t ) regurgitation of und igested food at nighl
(2) bird's beak appearance with barium study
2. compa rison o r acha lasia wirh Hirschsp run g's disease-
A. in Hirschsprung's. both submucosal (Meissner's) Ilnd myenteric plexus (Auerbach's)
are mi ss ing
8. in Hirschsprung's proximal bowel is dilated but does hove perista lsis
3, compa rison o r acha lasia with PSS/C REST ~ y ndro m e-
A. PSS/CREST have a rela.''(ed LES
8. both have absences of proximal esophagea l motility
C. both I1re best diagnosed with manometry
<if" GE lID:
I. rela xed LES
2. acid Inju ry leads to ulce nuion or distal esophagus mucosil
3. Barrett 's esophagus
A. ulceration and stricture
D. dista l adenocarc inoma Me cancer in esophagus
4. CE Rn M eC of nocturna l cough
Eso ph agilis in AIDS:
J, Candido MeC
2. olber caUSH- Herpes and CMV
3. odynopb agia
Lye n rict'Ures:
I. alkalis produce liqudactive necrosis wilh stricture rorm ation
2. add produces coagulalion nec rosis
Vessels in esophagea l va ri ces:
1. len gasl ric (bra nch off the po rtal vein) a nd azygo us \'ein
2. du e to po rta l hype rtcnl io n fr om cirrb osis o r liver
J. ruplure is M e complication a Dd COD in ci rrb osis
4. cnd osco py is fi rsl slep in ma nagem enl
!\IIldlory Weiss:
I. tu. r or distal eso ph agus
2. CQuses-
A. retChing in alcohol ics
8 . bulim ia
r:Jr Boe rbaave's:
1. ruplure o r d is tal eso pb ag us r
2. ca ulCS-
A. endoscopy MeC
B. retching in alcoholic
C. bulimi o
J. Hamma n's mediastinal c runch -
A. complicalion of Boerhaave's
B, air dissecting subcutaneously (subcutaneous emphysema)

100
MC primary ca ncer of eso pb agus:
I. adenocarci noma of distal esophagus: com plication of Barren's esophagus
2. squamous c:a ocer MC io third wo rld co untries or if loca ted ill mid-eso phagus
U MLE: X.ray of baby w1tb loops of bowella Iel'I ple..a l tority:
1. diaphragmalic hernia
2. po lero latcral defect in diaphragm
Hemutemesls:
1. vomi lin g of blood
2. duodenal ulcer> gas lric ulce r > esophageal varices
Congeni la l pyloric sre nosiJ:
I. vo mitin g 2-4 wk.. afeer binh of non-bile s tained nuid
2. mass palpa ted in e pigastrium
J. multifactoria l inheritance
4. comparison ""ilh duodenal alrcsia-
A. Inner disease has a Down syndrome relationship
O. luuer disease has vomiting of bile stai ned nuid at birth: atresia distal to nmpulla of
Voter
latte r disease has doub le bubble sign on x·ray: air in stomnch and prox imal duodenum
O. laner disease has polyhydramnios in mother
ar NSAIO ulcers/erosions:
1. usuaUy s upe rficia l erosive ganriLis
2. inh ibit rCEI wbic: h produces mucosal barrier-
A. PGE t normally:
(1) increases blood now to mucosa
(2) increas~ secrtlion of mucous
(3) increases secretion ofbicarbono.te into mucous
B. misoprostol. a PGE I analogI is cytoprotective
C hronic a trophic gas triliJ:
I. type A - ,
A. body and fundus
B. associated with a utoimmune destrucrion of parietal ce lb in pernicious IlRemia
C. aCh lorhydria :
(1) danger of adenocarcinoma
(2) problems with digestion
(3) high gastrin levels
2. 'Yp.D-
A. pylorus/antrum
B. assoc iated with H. pylori
C. intestinal metaplasia predisposes to adenocarcinoma
J. M.eoelrier', disease--
A. giant rugal hyperplasia with increased mucus secretion
B. hypoproteinem ia
H. pylo,/:
l. gra m oegalive rod
2. urease producer- urea convened to ammonia damages mucosal barrier
3. det« ted in Bx sprcim en

101
4. serology excellen t bu t nOI usefu l in defecting recurrent d isease
5. complicatio ns-
A. gastric adenocarcinoma
B. type B atroph ic gastritis
C. low-grade malignant lymphoma
6. Rx-
A. mClronidazole
B. bismuth subsal icylate
C. tetracycl ine
Com pa rison of gast ric with duodena l u lcer'
Characteristics Gastric Ulcer Duodenal Ulcer
Percentage of 25% 75%
PUD
Epidem io lo{!Y Ma lefFemale ratio II I MalefFemale ratio 2/ 1. AD panem in some
Pathogenesis Defective mucosal barrier duc to- Pathogenesis- t. H. pylori associalion
I. H. pylori (>75% of cases), 2. >90%: decrease bicarbonatc In mucous
mucosa l ischemia (reduced PG£I), barrier, 2. increased acid production. 3.
J . bile reflux. 4. COPO, 5. renal blood group 0 relationship, 4. MEN I
failure. relationsh ip associated with Zollinger-
Blood group A re lationship. No Ellison syndrome componcnt
MEN I or II relationship . Smoking
delays healing bUi not a cause of
PUD.
Location Single ulcer on the lesser curvature Single ulcer on the anterior ponion of the
of Lhe antrum . Same locati o n for first part of the duodenum (most common)
cancer. foll owed by si ngle ulcer o n posterior portion
(dam!.cr of perforation into the pancreas).
Malignant Gastric ulcers do not predispose to No malignant potential. Not biopsicd .
potential cancer. Cancer can be ulcerative
and confused with gastllic ulcer.
CannOI tell malignancy by the size
of an ulcer. Must Bx 10 RIO
cancer.
Complications Bleed and/of perforate (both less Bleed (Me compl ication). perro rate (air
common than duodenal ulcers). under diaphragm radiates to left shoulder),
Rastric oullet obstructi on, oancreatitis.
Clinical Burning epigastric pain soon after Burning epigastric pllin 1-3 hours after
eating. Pain increases with food eating. frequently relieved by antacids or
(afraid 10 eat and lose weight), food (do not lose weight). Pain wakes potien!
Relieved by antacids. at night.
ZE synd rome :
1. malignant is let cell tumor secreting gastri n
2. ulcers in usual place b u t can be mu ltiple or in unusua l p laces: association with MEN I
3. labora tory-
A. increased baslll and maximal acid output
B. increased gastrin
C. increase in gastrin with IV secretin test

102
4. IU-
A. proton blockers can decrease gastrin
B. surgery
5. ot her taU5H or hypergastrincmia-
A. Hl or proton blockers
B. chronic atrophic gastritis of body and fundus
C. renal failure
D. gtlStric distention
Treatment ror recurrent u1ccn: vagotomy
Heterotopic rest! : benign pancreatic tissue in wall ofslomach
Pluie n' " 'fth Idt J UpraChlvicular nod e:
1. Virchow's nodt- associated with metastatic stomach canctr or ony cancer arising in the
abdominal cavity
2. right s upraclavicula r node drains the lung and upper neck
StomAch (/.lneer:
I. decreasi ng in Incidence in Uniled State,
2. Me cancer 10 .Japan- re lated to smoked foods
3. ca useJ-
A. If. pylori causally linked
B. smoking
C. nilrosamines
D. adenomalous polyps
E. achlorhydria
4. linitis phutica-
A. leather bonle stomach
8. does not peristalse when barium is present
S. KrukenMrg lumors- hematogenous spread to both ovaries
Ext ranodal lymphoma :
1. stomllc b MC extranQda l site- Peyer's patches second Me site
2. mOl' an high grade immunobwtic Iympbomas
3. H. pylor/assoclated with low grade Iympbomas
Leiomyoma:
J. MC bcnle n tum or in G)'rad
2. Me location Is s tomach
3. bleed
Malabsorption:
I. pa 1hoeencsis orrat malabsorpdoa-
A. pancreatic: disease: no lipase
B. small bowel disease: no absorptive surface
C. bi le salt deficiency: no emulsificatio n and micelle fonn:uion
( I) liver disease
(2) bile obstruction
(3) cholestyramine
(4) bacterial overgrowth
(S) terminal ileal disease

103
2. Inb Dx-
A. stool for fat best screening test
B. D-xy lose test evaluates small bowel ability to reabsorb: no increase In bloodJurine
indicates small bowel disease
C. calcifications in pancreas presumptive evidence of chronic pancreatiti s
J. Ilib aitera rioDs-
A. hypoalbuminemia
B. fat soluble vitamin (A. 0 , E. K) deficiencies
C. combined anemias: iron, folate, Bil
D. hypocalcemia from vitamin 0 deficiency: tetany and secondary hyperparathyroidism
E. pro longed PT from vitam in K deficiency
4 ma.\l
I bso rpllon d .Jo
' r d en
Disease Epid emioloe:vlPatho2enesis Gross a nd Micro Clinica llDial!Dosis
~Iiac Begins in infancy with expo- Atrophy of villi in Failure to thrive with abdominal
disease sure to wheat gluten. Anti· the duodenum distention and dilUThca in child-
bodies d evelop against glia- (iron deficiency) ren o Adults have weight loss,
din ext ract in gluten (best and jejunum (fol- diarrhea, malnutrition . Risk of
screemng tcst)- im mune are de fi ciency). smail bowel T cell lymphoma.
destruction of mucosal ce ll s. Sp lenic atrophy. Gl uten free diet
Associations- HLA 88, -Or3 gold standard R.~ . Lab: abnormal
Bnd -oQ2 Bnd dennatitis D-xylose. anti-gliadin antibodies
hcrpetifonnis (vesicular skin (best test). anti-endomysia I anti-
disease). bodies.
Whipple's Systemic disease of middle Blunting of vill i in Present with fever, diarrhea. joint
disease aged males. Infection by the jejunum and pains. emaciation, pecu liar ~
Trophermyma whippleii. ileum . Heavy inf- bro\\ITl skin l!i&,!!!entatiQn, gen-
Cannot culture. Only seen iltrate of foamy era li zed lymphadenopathy. Lab:
with electron microscope. In macrophages in abnonnal O-xylose. R-, with
AIDS, MAl produces a lamina propria TMP/SMX
Whipples-like syndrome . (contain the org-
, anisms) which
blocks lymphati c
uDtake of fat.
r:r Diarrh ea:
I. ca uses-
A. invuive
(I) e.g., Shige llos is
(2) low volume diarrhea
(3) positive stool for fecal leukocytes
8. sec relory:
(1) cAMP stimu latio n by toxins
(2) e.g. , cholera
a. glucose added to oraJly administered hypotonic salt solutions in order to
ensure sodium reabsorption
b. no mucosa l inflammation
(3) traveler's diarrhea due to enteroloxigenic E. coli
(4) high \'olume diarrhea with osmolality similar to that of plasma
(5) negative 5tool for feca l leukocytes

104
C. osmotic:
(1) lactase deficiency: alias brush border enzyme deficiency. alias disaccharidase
deficiency
8. co lon anaerobes degrade InClose to fatty acids (acid pH stool) and H l gas)
b. breath test fo r Hl best screen
(%) certain laxatives
(3) high volume diarrheCl; with osmo lality lo\ver than that of plasma
(4) negative stool for fecal leukocytes
(j"
Microbial Pathogens Associated with ]).tarrbea
Pathoecn Specific Patbo2cn Comments
Virus Rotavirus (Reovirus) MCC of childhood diarrhea. Winter months. FecalooOral route.
Infects small intestine mucosa and damages brush border
enzymes and transport systems producing a watery, non-
bloody diarrhea. RotazYllle test on stool estabtishes Ox.
Norwalk (picorn avirus) Common cause of both adult and childhood gastroenteritis
I (vomitinlU'diarrhea). Fecal·oraJ transm ission.
Cytomega lovirus Common cause of diarrhea and biliary tnci disense in
AIDS.
Bacteria Food poisoning from S. aureus type (gram + coccus) occurs in 1·6 hs (self.
preformed toxin limited). Culture food. B. cereus (gram positive rods noted in
SlaphylococCtJS aureus, stool) associated with fried rice/tacos (self-limited). Adult C.
Bacillus cereus. CJostri- botulinum (gram positive rod) has heat labile exotoxin
dium botulinum (adull) (neurotoxin). Organisms proliferate in canned/preserved
food. Toxin attaches [0 synaptic vesicles of cholinergic
nerves. blocks the rel ease of acetylcholine, and causes
I Datalysis.
Food poisoning after Clost,idia: gram positive rod. C botulinum in infants from
colonization in bowel eating spores in boney. Colonization rust in bowe l llnd then
C. perfrlngens. C. botul- toxin production. Salmonella: gram negative rod and MCC
inum (illfant), Sabnon- of food poisoning in United States. C perj',ingens accounts
ella enteritidis
, for 20% of cases of food poisoninp:.
Clostridium diJficile Gram positive rod associated with pseudomembranous
colitis. Administration of antibiotics (ampici llin #1, Cleocin)
results in the overgrowth of toxin producing C difficile in the
colon. Fever and watery diarrhea. Enterotoxin-induced
pseudomembrane covering the colon (creamy to greenish flat
plaques). Pseudomembranes also noted in shigellosis,
enterohemorrbagic E. coli, ischemic bowel disease. Toxi n
assay of stool best test. R.'t: metronidazole (Flagyl)
Shigella species Gram negative rods. S. sonnel is the M CC of shigellOSis
followed by S. flexneri. S. dysenteriae produces severe
disease. Oral-feca l route transmission. Produce endotoxin.
No animal reservoir. No chronic carrier state. Targets the
ileum and colon. Mucosal ulceration and pseudomembranous
inflammation. Diarrhea contains blood/mucus. Positive fecal
smear for leukocytes. Association with HUS syndrome and
ElLA 827 positive ankyJo!ling spondylitis. Usually
produces a sel f-limited diarrhea. Rx: cipronoxacin

105
Salmonella species Gram negative rods. Pathogeni c Salmonella species include
S. O'phi, choleraesuis, and ryphimllr;um (MCC of
enterocolitis, sclf· lim ited). Animal reservoirs (pou]try.
turtles, lizards). Oral· feca l tranSmiSSion. Typhoid fever
(enteric feve r) caused by S. O'phi (human reservoir). First
week- invades Peyer's patches (longitu dinal ulcers) and
I
produces a se pt'icemia (best culture medium for isolation ).
Second week diarrhea (pos itive stool cu lture) and classic
triad of bradycardia, abso lute neutropenia, hepato·
splenomega ly. Compl ications: osteomyelitis (sickl e disease ).
jaundice, renal fa ilure, chronic carrier state (persistence of
the organism I year post infec tion, Me In ga llbladder).
Posirive feca l smear (mononuclear ce lls). Ceftriaxone is R.~
of choice fo r ryphoid and ampi cillin fo r the ch ronic carrier
statc. Cholecystectomy may be necessary.
Mycobac terium Organi sms swallowed from a pn mary foc us in the lung. I
luberC fdosis In vade Peyer's patches in term inal ileum. Infections spreads
via the circumferentially oriented lym phatics leading to
stricture fonnalion/obstruction. M bO\lis MeC in third world
countries (unpasteurized milk).
Campylobac/er jejuni Gram H nagellated . curved. Vibrio·l ike rod. Me im'asi\'c
bacterial enterocoli tis in United States. Contracted by
eating contaminated food (poultry) or drinking contaminated
milk or water, Invade jejunum and colonic mucosa, Produce
crypt abscesses/ulcers re sembling ulcerative colit is.
Blood/mucus prese nt in stool. Positive fec al leukocyte smear.
Usually self-l imited . Rx: erythromycin or cipmfl oxacin in
severe cases
Escherichia coli Gram negative rod. Produces toxin ind uced and invasive
diarrheas.
,
Enteropathogenic s train : non·tox in type of mild diarrhea in
infants and young children, Mucosa not inflamed.
Enterotoxigenic strains: some stra ins produce heat labile
(LT) loxin stimulatin g cycl ic Al'vlP (secretory diarrhea
s imilar to cho lera, normal mucosa). Other strain s prod uce a
heat stable (ST) toxin stimulates guanylate cyclase prod ucing
a secretory diarrhea (MeC of trave ler 's diarrhea) . Rx with
TMP/SMX.
Enter oinvllsive strain : invade large intestine producing
mucosal nec rosislintlammation.
Enterohemorrhagic strain: 0157: H7 serotype 10 raw
hamburgers. May produce hemolyt ic uremic syndrome and
hemorrhagic or pseudomembranous colitis.

106
Vibrio cllo/erae Curved gram negati ve rods. Enterotoxin stimulates adenylate
cyc lase In smal l bowel. Severe secretory diarrhea (" rice-
water sloo ls"). Drinking contaminated water/eating cOll1am-
inuted seafood, especially crustacea (Gulf coast ). Volume
depletion, no nnal gap metabo lic 3l:idosis (lose bicarbonate),
hypokalemia. R.., is flu id replacement. Gl ucose enhances
sod ium uptake in ora l supplements . r"lo rtalif), - 40% if left
untreated.
Parasitic Entameba hislO/YlicCl See table in the Hcpstobiliarv section
OyplOsporidium Contracted by ingesl lng oocysts (ac id fas t positive). Brush
parvum (sporo=Qan) border of intestinal epithelium. Diarrhea/abdominal pain.
Biliarv uact disease In AIDS . Me protoloal pathogen In
AIDS diarrhea along with microsporidia species.
Contaminates city waler suppl ies producing a self- limi ted
diarrhea in immune competent hosts (outbreak in Milwaukee.
Wiscon sin) . Detected w ith the string test Rx : paromomycin
is Rx in l-UV positive patients.
Microsporidia species Primarily occur in l-UV positive patients. Diarrhea and I
wasting synd rome. Rx wi th albendazole.
Giardia lamblia )\1 ~ grotozoal cause of diarrhea in United Slates. Contracted
by ingesting cysts (mountain springs. etc .). Attac h to small
intestine mucosa (may involve bil iary tract) . Produces acute
and chronic diarrhea with malabsorption. Rx: metronidazo le.
Amebiasis:
l_ Oask s ha ped ulcers in the cecu m
2. afe brile, bloody diarrhea
3. trophozoites phagocytose RBCs
Traveler to Mexico who 1 week later develops n watery diarrhea with muc us ao d blood and
co licky bowe l movements: amebiasis
Abdominal a bscess: ..
t. Me BaCleToides fragilis
A. B. fragilis below the diaphragm
n. B. melanogenicus above the diaphragm)
2. Me due to II perforated appendicilis
AIDS pa tien t diarrbea-?what orga nis ms can be identified with the string l est (Enl cro-Tcst):
I. Gia rd ia
2. Strongyloid es
3. Cryptosporidium
fFarmer .Do bii wife are brought to the ER by their I on because they a re too weak to walk or
drive and their vision is bl u~ry aDd eum reveals ptosis, facial weakDess, DooreatUn dilated.
pupils. dry m ucous mem branes, and DormaJDTRt-? diagnosis :
I. C. botulinum food poisoning
2. R,;: with botulism a ntitoxin
3. d anger of respiratory para lysis
Fecal smear for leukocytes : best screen for invasive diarrheas
Bowel obstruction :
l. ca uses-

107
:\.. IldhesioD5 from pre\'io us surgery MCC : if no hislOry of previous surgery. pick
indirect inguinal hernia
B. duodenal atresia :
(1) assoc iation with Down syndrome
(2) vomiting or bile stained nuid after birth
(3) double bubble s ign- air in stomach and proximal duodenum
(4) po lyhydramnios in mother
C. Elirschsprung:
(I) absent ganglion cells in rectum
(2) empty rectal vault on rec tal exam
(3) common in Down's syndrome
(4) bowe l proximal to defect is dilated and does have peristalsis and ganglion ce lls
O. indirect inguinnl hernia: 2nd MCC of bowel obstruction
( I ) Me type
(2) superficial epigastric artery med inl 10 finge r in canal (artery is lateral bo rder of
triangle of Hessclbach)
(3) ~: Cross or lilfaretell bowel i. weigbt litter:
a. incarcerated sma ll bowel in an indirect inguinal hern ia sac
b. see picture on page 3 10 of review text
E. direcl inguinal hernia:
( I) lhrough posterio r wa ll ofrriangJe
(2) anery lateral to finger
F. umbilical hernia :
(I) common in ch ildre n
(2) common in pregnancy and ascites
G. intussusception:
(I ) lenninal ileum intussusc:epts into cecum
(2) child with colicky abdominal pain a,n d bloody diarrhea
H. vo lvulus:
(I ) Me due to sigmoid ca jon rwisting around mesemery
(2) obstruction and infafclion
I. gallstone ileus:
(1) e lderly woman with chronic gaJlbladder disease
(2) fistu la between gallbladder and small bowe l
(3) stone passes into small bowel and obstructs at ileocecal valve
(4) a ir in biliary tree
J. meconium ileus: impacted meconium in cystic: fibrosis in newborns
K. USMLE: X-ray wItIi Iocaldll!lted bowol:
(I ) ileus (lack of peristalsis) of a small po rt ion of bowel is called a sentine/loop
(2) due to bowel close to an area of in fl ammation
(3) Dcute pancrealitis- sentinel loop near duodenum or transverse colon
(4) retroceca l appendicitis- sentinel loop of cecum
2. cliDiclll- . . lld ;fIJ or t!'<", DJf,oJ~J.<i.e iWL'~/ lUll / or jlf'''; -rite if,/~VII ......
A. col ICky pam -:, r ' r' fr'
B. di stended abdomen with no rebound tenderness
C. constipation
0 , obstipllti on: no gas d<Jt,J~ ~~
E. . dilated small bowe l with airlfluid levels in step- ladder appearance (O N \-~~ y)

108
Sigmoid colon most common site ror:
I. CIlncer in Gl tract
2. divertic ular disease
3. po lyps- exceptio n: polyps of Peu12·Jcghers syndrome. which are M e in the small bowel
Hemluocbaia:
I. loss or brigbt r ed blood pcr rfft'um
2. di'\'trticul~ MCC rollo"'ed by angiodyspluia

Vessel d isorders:
I. s mall bowel inrarclion-
A. thro mbosis/embolism of the superior mesentcric artery with a transmuraJ small bowel
infarction
(I) embolism is most onen associated wi th hean disease and atrial fibrillati on
(2) thrombosis most often associated with atherosclerotic plaque at orifice
B. c linical :
( I ) sudden o nset of diffuse abdominal pain. bowel distention, and bloody diarrhea in
e lderly patient
(2) no rebound tenderness un less perito nitis is prescm
(3) absent bowe l sounds (ileus)
(4) "thumbprint sign" no ted on barium enema: due to edema in bowel wall
(5) amy lase of bowel origin is o ften incrtased
(6) bowel di!ilention, air/ fluid levels
(1) profound neutrophilic leukocytosis
2. ischem ic colitis-
A. severe pain in splenic nn un after eating
8 . overlap area between superior and inferior mesenleric arteries
C. pntient losses weight for fea r of eating
O. bloody diarrhea from localized infarction
E repair with fibrosis leads to ischemic strictures
3. a nglodyspllu ia-
A. dilated vascular-channe ls in submucoS3 of cecum
( I ) increased wall stress related 10 increased diameter of cecum increases tension on
blood vessels causing dilatation
(1) second MCC o fhematochezia
(3) M e in clderly patients
B. ossociated with:
(1) von Willebrand's disease
(2) aortic stenosis
Dive rticula r di5eaJe:
I. . mall bowel divert ieula-
A. uncommon
B. site for bacterial overgrowth:
(I) Bu deficiency
(2) bile salt deficiency and malabsorption
2. Meckel's diverticulum :
A. vitelline duct remnant: 2 feet from ileoceca l valve
B. true diverticulum
C. MCC of iron deficiency in a newborn and young child (GJ bleed)
. ' 'I
.J ,c..-t;'j
109
3. sigmoid diverticula~
A. diverticula are ju.xtaposed to a blood vessel... which is the area of weakness: explains
why diverticula are the MeC of hematochezia
B. diverticulitis Me complication:
(1) impacted fecalith serves as nidus for infection/inflammation
(2) clinical ly described as a "left-sided" appendidti.s
(3) danger of perforation and peritonitis
C. diverticula are MeC of fistulas in GI tract: colovesical fist~l a Me fistul,a
t:ir Comparison of ulceratin Colitis (UC) an d Crobn 's disease (CD): ( r:i.;ow ~/1ts,.
Characteristic IDcerative Colitis Crohn 's Di5ease
E.xtent of Mucosal and submucosal Transmural
disease
Location Primarily targets the rectum, May Tenninal ileum alone (30%). ileum/colon
extend up into left colon in cant· (50% ), colon alone (20%). AnaJ involvement
inuous fashion (no skip lesions) or 75%- fissures, ftstulas, abscesses. Invo lves
involve, entire eo,lon. Does not' other areas of Gl tract (mouth to anus).
involve other areas of 01 lTBCt.
Gross fearu res lnflammalOry polyps- pseudo- Fat creeps around the serosa. Thick bowel wall
polyps representing areas of and narrow lumen. Skip areas (important fea-
inflamed residual mucosa. Friable, ture). Strictures, fistulas (very diagnostic) in
red mucosa bleeds easily when areas other than an us. Deep linear ulcers, cob-
touched. Ulccrationlhemorrhage. blestone pan-em, aphthoid ulcers (small mucosal
Colon shortened and mucosa flat ulcerS, origin of linear ulcers~ early sign).
in quiescent chronic disease. No
skip areas.
Microscopic Active disease: mucosal inflamma- Ven' diagnostic feaiures: noncasealing granul-
fearu.res tion with crypt abscesses (neutrop- omas (60%). transmural inflammation with
hil,). Dy'plasia/cancer may be subserosallymphocytic infiltration.
present Chronic disease: oeutro-
phil' replaced by Iym~hocytes/
plasma cells. Dysplasia/cancer
may be present.
Complications More common than CD: 1. tmde More common than DC: 1. fistula fo rmation~ 2.
megacolon- hypotonic/distended obstruction, 3. disease in other areas of GI tract,
bowel, 2. sclerosing pericholan- 4. calcium oxalate renal stones
gitis, 3. }{LA 827 + ankylosing
spondylitis/uveitis, 4. pyoderma
gangrenosum, S. adenoc:areinoma-
- 10%, greatest risks are pancolitis,
early ooset of UC, and duration of
disease> I 0 years
Clinical Left sided abdominal cramping Right lower quadrant colicky pain (obstruction
(nol obstruction), diarrhea \\ith in area of tenninal ileum) with diarrhea.
blood/mucus, rectal bleeding. Bleeding if colon involvement.
tcnesmus (painful straining at
stool).
Radiograpb Lead pipe appearance in chronic "String" siW in the tcrminal ileum from luminal
disease. narrowin ~ by inflammation. Fistulas.

110
Youns woman with iotermittenf bouts of diarrhea and coostipation associated with
cramping right and left lower quadrant paln:
I. irritable bowel synd rome- intrin sic motility defect in bowel
2. nuible sigmoidoscopy is negath't
Melanosis coli: black colored colon due to laxative abuse
Polyps:
I. sigmoid colon MC si te
2. hypcrpla!tic polyps Me type- hamarto mas
J. j U\'fnil e: polyps-
A. Me polyp in children
B. o nly located in rectum
C. hamartomas
4. PJ polyp. -
A. Me in small intestine
B. hamarto mas : very little risk for adenocarcinoma
C. mucosal pigmentation
5. Ildenomntous polyps-
A. precursors for cancer:
<I) villous adenoma worst type: sessile polyp
(2) tubular adenoma M e type (looks like a strawberry o n n stick)
B. risk for cancer greatest:
( I) polyp >2 em
(2) has increased villous component
Polyposis syndrom es:
I. familial polyposis Me type-
A. AD disease involving:
( 1) inactivation of APe suppressor gene on chromosome 5
(2) activation of ras oncogene
(3) inactivation, of p53 suppressor gene
B. 100% penetrance:
(1) all will develop cancer by age 40
(2) screen with flexible sigmoidoscopy in lhose affected beginning age 10- 12 every
1-2 ys
(3) genetic testing t'o confinn and to test first degree famil y members
(4) congenital hypertrophy of retinal pigment is Me extraintcstinal manifestation
2. Gardner's synd rome-
A. variant of familia l polyposis
O. AD inheritance
C. benign osteomas in jaw
D. desmoid rumors in abdominal sheath
J. TurcOI's syndrome-
A. on ly AR polyposis
8. brain tumors
Carcinoid lumorlsyndrome:
t. tip of appendix Me overa ll site-
A. APUO tumor with neurosecretory granules
B. S I 00 amigen positive

III
C. yellow color
z. aU carcinoid tumors are- maligDIIDt- ""\
A. size detennine:s metaStatic potential f.?d..
C "" )
B. appendiceal carcinoids rarely metastasize
l. lenniDal ileum 1\1 'te for I'umor tbat metastasizes to live to produce the carcinoid
sy ndrome-
A. serotonin is fibrogenic and may cause bowe l obstruction
B. pona l vein drains serotonin to liver where it is metabolized and excreted in urine 35 5-
H1AA
4. ca rcinoid s),Ddrome-
A. !!!etastasis to; the liver from a primary in the small intestine
(I) metastatic nodules drain serotonin into hepatic vein tributaries: access to
systemic circulation
(2) nushing and di arrhea Me symptoms: due to serolon in
(3) card iac valvular lesions:
a. tricuspid regurgitation
b. pUlmon ic stenos is
B. measure 5-HIAA (metabolite of se rolonin) in urine: tryptophan often depleted from
increased synthesis of serotonin --- --
c. bronchial carci noids do nOI have to metastasi7.e to produce carcinoid syndrome
r:r Colon cancer:
I. risk(aclon-
A. age
8. tubu lar adenoma >2 cm
C. fami lial polyposis (100% penetrance)
D. villous adenoma (vi llus component increases the cancer risk)
E. 10..... fiber diet: increased exposure of mucosa to lithocho lic acid
F. smoking
G. Lynch syndrome (fami ly cancer syndrome):
(I) AD dis....
(2) type I limited to co1on
(3) type U is fami ly cancer syndrome
•. colon cancer
b. brcasllendometriallcervical cancers
c. gastric cancer
2. screening-
A. stoo l guaiac yearly aft er 50 years old with flexib le sigmoidoscopy every 3-5 years,
aspirin has a protective effect
8. second Me cancer and cancer killer in men and women
3. sitf'S-
A. rectosigmoid 60%
8. cecum/ascending colon 25%
4. clinica l-
A. left side obstructs: change in bowe l habits: annular configurat ion
8. right side bleeds:
( I) polypoid
(2) found later than left sided type

112
5. Aslu-Coltln , tae.ine sntem for co orecta e:a nCCf-
Stae.c C ha r acteristics Pro211osis I
Tumor limited to the mucosa. Negative lymph 100% 5 year survival.
A

BI
nodl!s.
Tumor in vo lves the submucosa and in vades imo - 60% 5 yenr survival.
I
but no t thro ugh the muscularis propria. No lymph
node involvement. I
B2 Tumor penetrates through the muscularis pro pria 55% 5 year survival. ,
and extends inlo the serosa. No lymph node
involvement .
C I and C2 Same as 81 !lnd 82, respectively. except lymph
nodes are involved.
C I: 30% 5 year survival. C2: 20%
5 vear surviva l.
I
D Tumor with any level o f invasion with or without <5% 5 yea r surviva l.
lymph node involvement but distant metastas is is
I presen!. I
6. CEA- used to fQ ll ow~ up for pote ntial recurrences
Acute uppcnd icitis:
t. Me du e to viral infection (adenovirus) lo...ch ildr£.n- lymphoid hyperp lasia in appendix
causes ischemic changes
2. Me due to an obstructin g recalitb in adul ts
3. cli nica l-
A. pai n mi grates from umbi licus to RLQ
B. fever
C. rebound tenderness at McBurney's point
O. Rovsing's sign: pajn in RLQ wben palpating LLQ
E. psoas sign: pain when extending right th igh
F. perforation Me compUcation
Abd o minal PaiD:
1. order for eva luatin g a bdomen-
A. inspection ;
B. auscultation
C. percussion
D. palpalion.
2. causes of paln -
A. hollow viscera:
(I) distenti on of the visc us with fl uid or air
(2) [orccfu I muscle contractions
(3) inflammation
(4) ischemia
8. solid viscera:
(I) srretching of capsule
(2) in nammation of capsule
C. viscera l peritoneum and greater omentum are nol sensitive 10 pain
O. mesentery, parietal peritoneum and peritoneum covering poSlerior abdomen are
sensitive to pain
3. types of paio-
A. viscernl
B. parietal

113
C. referred
4. "iscera l pain-
A. mediated by small unmve linated afferem C fibers
(1) poorly localiac pain to the midline of the abdomen
(2) p'iin usually \l U. slow in onset, and of long duration
(3) mid-epigastric visceral pain:
a. duodenum
b. hepatobiliary system
c. pancreas
(4) periumbilical (m id-abdominal ) pain:
Q. j ejunum/ileum
b. cecum/appendix
(5) mid-lower abdomen:
a. colon
b. internal reproductive organs
S. parietal pain-
A. mediated by both.£.ind myelinattd..A Q..nerve fibers
B. ...E!inJnor acute sha a d better localized
C. causes of parietal pain involve irritation of parietal peritoneal surface by:
(\) pus
(2) bile
(3) urine
(4) cyst fluid
(5) GI sec retions
D. example of appendicitis:
(1) pain begins in the periumbil ica l area when appendix becomes inflamed
(2) shifts to RLQ (McBurney's point) when parietal perironeum becomes inflamed
6. referred pain-
A. pain at a distant site owing to shared cenual pathways for afferent neurons from
different sites
B. right scapular pain in acute cholecystitis owing to afferents from the gallbladder
entering the spinal co rd from T6-T9
C. left shoulder pain with perforated duodenal ulcer and air under diaphragm irritating C4
7. pain o n!let-
A. explosive within seconds: e.g., perforated ulcer
B. rapidly progressive within 1-2 hours: e.g .. strangu lated bowel
C. gradual over several hours: e.g., acute cholecystitis
8. eharacler of paln-
A. colicky pain:
(1) pain interspersed with pain free intervals of minutes to hours
(2) sign of obstruction in a viscus that has peristalsis
a. small bowel obstruction
b. tenninal ileal disease in Crohn' s disease
B. steady pain
( I) dull and aching
(2) e.g .. ulcer pain
C. stabbing pain
(I) bowel infarction

114
(2) acute pancreatitis
9. obSlip.lioa-
A. absence of passage of both stool and natus
8. characteristic of mechanical bowel obstruclion
10. rtbouad teaderness- peritoneal in:flamma rion
II. ioucased bo"'cl sounds (byperperistabis)-
A. diarrhea
8. early obstruction
12. absent bowe.! souads (Ucus)-
A. Intestinal obsh11ction due to inhibition of bowel motility
( I ) adynamic or paralylic ileus
(2) peritonitis
B. mechanical obstruction (dynam ic ileus): adhesions

Q ...do.. woed darla. til. board review:


Items 1-5
A. Vibrio cho lera
B. Bacillus cereus
C. Shigella so"" et
D. Salmonella fJ'phi
E. Campylobocler j~junl
F. Ytrsinio tnierocolilicQ
G. Staphy/ococclLJ aI/nus
H. Enlerotox.igenic £. coli
An afebrile 22 year old man and severa l other members of his family developed seven \'omiling
withom diarrhea - 1- 6 hours after eating pawo sa lad at a picnic. They all recovered uneventfully
12- 24 hours laler.
Answer; G
(#- A 23 year old man deveJ9ped explosive. watery diarrhea \Vith blood, leukoc.ytes, and mucus - 3
dllYs Ilner eating chicken that was improperly cooked. Comma-shaped organisms are noted in the
fecal smear of stool along with RBCs and leukoc)1es.
Answer. E
A febrile 10 year old child presents with severe right lower quadrant pain that is interpreted by the
attending physic ian as acute ap~ndicitj s . AI laparotomy, the surgeon notes thaI th e appendix is
nonna!. However, the mesenteric lymph nodes arc markedly enlllrged and have focal Ilrell5 of
microabscess (onnation on cut section.
Answer: F
r:r A 29 year old man develops watery di31Thea lind volume depletion shortly after visiting the Gulf
Coast states
Answer: A
r:r On a trip 10 lnd ia. a man develops a high fe\'er llS50cilucd with brudycardia., absolute neutropenia.
and splenomegaly. A blood culture is positive for a gram negative organ ism.
Answer. 0

l iS
Items 6-7
A. Yersin io enrerocoli,ico
B. Slophylococcus aureUJ
C. Enterotoxigenic £. co/(
D. Cryp,osporidium parvum
E. Mycobaclerium Q\·lum·;ntracellulare
Qr A 28 ycar o ld Olan with AIDS presents with chronic . recurrent, profuse. nonbloody. watery
diarrhea. An Entero-Test (string test) reveals oocysts that are partillll) acid-fast positive.
Ans"Cer. 0

'" A 25 year o ld medica l student during Spring break in Tijuana. Mexico develo ps fever. vomiting.
abdominal cramps, and water), diarrhea -14 hours after eating a few tacos purchased from a street
vendor. He recovers uneventfully in 48- 72 hours.
Answer: C

" Odynophagia in a HJV -positive 28 year o ld man with while plaque· like material o n his tongue and
bucca l mucosn MOST LIKELY has an AIDS-defining lesion caused by...
A. Epstein-Barr virus
B. Cat/dido alblcolls
C. Kaposi 's sarcoma
D. cytomegalovirus
E. Ht.rpes simplex
B
r:r An afebrile 25-year-old medical student presents with intennittent complaints of left and right
lower quadl1lnt abdomina l pain and distentioD associated with altemBting periods of mucoid
diarrhea and constipation. He states that stOOling relieves the pain. A flexible sigmoidoscopy and
slool guaiac exam are both normal. The patient MOST UKEL Y hu ...
A. an intrinsic bowel motility disorder
B. innammatory bowel disease
C. chronic appendicitis
D. melanosis coli
E. celiac disease
A (irritab le bowe l syndrome)
<r Which of the fo llo wing correctly describes a gastric rather than a duodenal ulcer?
A. Association with Helicobocler pylori
B. Highest incidence of perfomtion
C. 5mB II risk for adenocarcinoma
D. Pain awakens the patient at night
E. AssociBtion with Zollinger-Ellison syndrome
C
r:r The MOST COMMON location for divenicula, polyps. and cancer in the gastro intestinal tracl i5
the...
A. ascending eolon
B. sigmoid colon
C. esophagus
D. stomach
E. rectum
B

116
r:r Which of the following characterizes ulcerative colitis rather than Crohn 's disease?
A. Discontinuous spread
B. Toxic megacolon
C. Fistula formation
D. Perianal dise~
E. Obstruction
B
cr A 38-year-old Asian woman has a long history of explosive diarrhea and abdominal distention after
eating d,ajry products. The pathogenesis oflhis patient 's diarrhea is MOST CLOSELY relaled to...
A. antigliadin antibodies
B. activation of cyclic AMP
C. intraluminal osmotically active so lutes
D. mucosal injury with increased permeability
E. loss of the absorptive surface of the small bowel
C (lactase deficiency)
rr A 62-year old man smoker presents with weight loss, a dragging scnsalion in his right upper
quodrant. and crampy left lower quadrant abdominal pain. He has alternating bouts of constipation
and diarrhea. In addition. he states that blood coats and is m.i xed in with his stoo ls. He sometimes
has pain with defecation. There is mild hepat'o megaly. The rectal exam reveals non-thrombosed
external hemorrhoids and a sentinel tag in the poSierior midline. The stool is guaiac positive.. A
complete blood ce ll count (eBC) reveals a mild microcytic anemia. The MOST LlKEL Y cause for
this patient 'S condition is ...
A. angiodysplasia
B. an onal fi ssure
C. diverticulitis
D. hemorrhoids
E. colon cancer
E (a lso evidence of liver metastasis)

117
Hepatobiliary/Pancrcas

Lh'e r function tes ts (LITs): Ih'er necrosis indicators


I. enzymes-
A. aJan inc transam inase: ALT
B. Aspart:ltc transaminase : AST
C. Lactate dehydrogenase: LDH
2. AST (formerly SCOT)- primarily a mitocho ndrial enzyme
3. ALT (fo rm erly SGPT)-
A. located in the cytosol
B. more specific for liver d isease than AST
4. transamioases increased in bepa loC)~e inju '1/ne~ ro~is .
A. ~e: ~ "lllsTkJ 1~ ALT
(1) AST > ALT, since alcoho l is a mitochondria l poison
(2) enzyme levels rarely >200-3 00 U/L.
B. viral hepatitis:
(I) ALT > AST
(2) AL T last enzyme to return to normal
(3) levels> 1000 UIL in aCUIe hepatitis
5. LDH-
A. increased in cell necros is
B. non-spec ific marker of primary or metastatic tumor in the liver
Liver function tests (LFTs): cholestasis indicators
l. enzymcs-
A. alkaline phosphatase (AP)
B. y-glutamyl transferase : GGT
C. Sf-Nuc leotidase
2. ~is refers 10 intrabe aric or extrabe atic obstruction of bile Oo\\,-
A. ~and Q.,G! increase secondary to syn thesis (not cell damage)
B. GGT synthesis increaseS: when drugs enhance the c oc hrome P4S0 s stem
(I) a lcohol
(2) phenobarbital
(3) rirampin I-;'<!F> Y/<>I~
C. AP is presen t in:
(1) liver
(2) bone
(3) placenl8
D. GGT primarily located in the liver in smooth endop lasmic reticulum
3. obstructive jaundice-
A. AP and GGT are markedly increased (iii)
B. transaminases mildly increased (i)
4. a lcoholic liver disease-
A. if AST>AL T
B. ~ GGT disproporti onate ly increased (iii) due [0 induced synthe sis by alcoho l
5. focal benig n liver disease (granulomas)-
A. bilirubin, LD H, transaminases normal
B. AP and GGT sl ightly increased (t)

118
6. rocal metastatic liver disease- as above except LDH also increased (1')
7. AP or live r venus bone or otber tissue orlgin-
A, both AP and GOT increased if AP is of liver origin
8, on ly AP increased if it is nOlliver in origin
U\'er rune lion lests (LFTs): excretory indicators . see schema tic
I. tOlal bilirubin (TO) with rractionation inlo conju gated (C 8 ) a od unco nj ugnll!d
bHlrubiQ (UCBl.
A. CBrra x 100 '" CB frac tionation
8. see schematiC of bilirubin metabol ism
2. CB<20%-
A. primary increase in VCB
(I) extravascular hemolysis
(2) problem with uptake or conj ugation
B. ~ B 20- 500/.: mixed CS and UC S: hepatitis
C. CB > 0%: primaril y CB: obstructivejllund ice
J. urine bilirubin lind urobilinogen rclationship-
Uri ne bilirubin Urine urobilinogen Int erpretation
Normal : absent trace
CD <!il'!o: absent ttf extravascu lar he molytic anemia
CB 20-50%: tf tt vira l hepati tis
CB > 50%: ttf absent obstructive jaundice
Liver runction teSlJ (LITs): jnde); or se\'erify or liver disease
I. severity iodu indicators-
A. serum albumin
8. prothrombin time (PT): best indicaror
2. bypoalbuminemia and prolonged PT-
A. indicate scve~ functional impairment of the Ih'er
B. in ciJThosis, the prolonged PT does nOl correct with intramuscular vilamin K
J. t~osa min a.Jts dKrease and PT is rurther prolooged- indicates fulminant hepatic
necrosis
Liver runction lests (LITs): immune indices
I. serum protei n electrop boresis in chronic liver disell.5e-
A, polyc:lonaJ gammopathy due to an increase in IgO
8. alcoholic c irrhos is has an increase in ISG and IgA prod ucing a O-v bridge
C. in primary biliary cirrhosis (PBC) IgM is elevllted.
2. a Ufoan tlbodies-
A. increase in anti-mitochondrial antibody in PBC
8. increase in anti-smooth muscle antibody in au toimmune hepatitis
Li ve r runction tUll: (LITs): (umor markers
I. alpb a r.,opro,ei n (A FP)-
A. hepuloblastoma in c hi ldren
B. hepatocellular carcinoma in adults
2. a.-I antitrypsin (AA1')- hepatoce llular carcinoma
Ana tomy:
J. triadJ co ntain portal veiD, bepatic artery, bile ducts-
2. hepatic: artery and portal vein dump blood into sinusoids which drain into the hepatic \'em
back to the right hean

119
I

Macropbage Destruction RBe ....... globin Jlplil off.-.. amino acid pool

Heme: (protoporphyrin + iron)


~ heme oxygeoase


Protoporphyrin + iron

Biliverdin + carbon monoxide (CO )


.. biliverdin reciuC:l.a5e
I Peripheral Blood Unbound bilirobin (lipid sol uble)
T
Bilirubin + albumin (uDconjugated bilirubio; VeB)
[ Liver I uptake of UeB by carrier proteins
.. VeB then binds to ligaodins
UCB
.. UDP-glucuronyl traosferases (conjugating enzym es)
Conjugated bilirubin (CD, DO aCCHS to blood from bere on in)
.. canalicular traDSport system (rate limiting step)
CD in common bile duct
t stored/concentrated in gallbladder (cbolecyslokin)
CD in terminal ileum + colon
I bacterial ndUCtiOD by P-c1ucuroniduc
• (also present in brc1Ut milk)

+
UCB
further red uction by bacteria

Urobilinogen (colorless) oxidi.ud to urobilin (color of stool)

E nterobepatic circu lation

Liver (90%) Urioe (100/., urobilin gives color to urine)


CD - conjugated bilirubin UCB - uDconjugated bilirubin

Normal Bilirubin Metabolism


3. zone around the central vein is most susceptibl e to hypoxia and alcohol damage
Summary of acq uired/genetic causes of jaundice:
t. un conjugnted cueD) hypcrbilirubiuemja witb C B <20-/ 0-
A. in c rcased production:
( I) hereditary hemo lytic anemias with extravascular hemolytic anemias
8. congenital spherocytosis
b. sickle cell disease
(2) acqu ired hemol)'tic anemias
B. ABO hemolytic disease of newborn
b. Rh hemolytic disease of newborn
c. autoimmune hemolytic anemia
(3) ineffective erythropoiesis (RBe destruction in the bone marrow)
a. severe Jl-thalassemia
b. pern icious anemia/folate de fi ciency
B. impoired uptake by bepatocYleS: acutc/chronic liver disease
C. impnired conjugation by hepa'iocytes
(1) G ilhert! disease
11. ? autosomal dominant
b. second MeC of jaundice (hepatitis MCC)
c. increased production ( mild hemolysis in 50%), decreased uptake,
decreased conj ugalion
d. jaundice exacerbated by fasting
e. liver histologically nonnal
f. fast the patient and nOle a doubling of bilirubin over the baseline
g. no Rx
(2) Crigler Najjar syndrome: genetic: disease
3. type I has total absenc~ of UDPG transferase activity: incompati ble with
life
b. type n has reduced UDPG transferase activity (activity increased with
phe.pobarbital)
c. liver histologically nonnal
d. Ox with enzyme assay
(3) physiologic jaundice of newbo rn
11. peaks on th ird day
b. immature uptake and conjugati on
(4) breast milk jaundice
a. occurs 3 days-3 wks
b. fany acids and honnones in breast milk inhibit conjugation
c. increased bilirubin reductase in breast milk causes more UC B to be
reabsorbed in the bowel
2. mixed ty pe of jaundice with CB 26-50%-
A. viral hepatitis: early phase of obstruction with light colored stools
B. alcoholic hepatitis
3. obstructh'e type of jaundice with CB >50%-
A. Oubio-Jobnson syodrome:
( I) AR
(2) impaired cana licular transport system for CB
(3) black pigment in hepatocytes

120
(4) non-visual i7.at ion of ga ll bladder with oral cho lecystogram dye
(5) abnomlal clearance srudies
(6) benign di sease
B. ROlo r 's syndrome
(I) AR
(2) pathogenesis same as Dubin-Johnson
(3) nomtal liver
Col ) normal visua lization of gallbladder
(5) abnormal clearance stud ies
C. intrahepa tic cboles tasis
<I> drug induced: see table
(2) primary bil iary cirrhosis
(3) in trahepatic biliary atresia in children
D. extrahepatic choleswis
( I) extrahepatic biliary atresia in children
(2) stone in common bile duct MCC in adu lts
(3) carc inoma of head of pancreas
(4) primary sc lerosing pericholangitis: associated with ul cerRtive coliti s
<lr V'oraI h i
e PIIIIS: sec sc lematlcs a nd ts bles
FlAV F!BV HCV HDV HEV
T)'pe of virus RNA : picom a- DNA : hepadna- RNA : Oa\·i- Incomplete RNA RNA : caki-
vi rus virus Viru S virus (req uires virus
HBsM).
Prevalence 40% 35% 20% S% < 1%
Transmission Fecal-oral. Parenteral , close Same as HBV.
Same as HBV. Same as HAV
food, waler, contact with an CQjn[ettiQD (same
anal intereour- infected patient. needle has both
se. 10010 of in fe- Present in blood, viruses). Supcrin-
clions in day semen, saJiva. [celioD (exposed
care ccnters.
- 10 HDV at laler
dalc).
incubation 2-6 wks 2-6 mlhs 5-7 wks Same as HaV 2-9 wks
Clinical Traveler'S hcpn- Seru m sickness MCC of post- Cytolytic to hepa- Poor prog-
associations lil is (80%): drug picture (5-10%): transfusion he- tocytes (rapid del- nosis in pre-
addicts (20%). polyarthritis, urt- patilis (1 /3300 erioration with gnant women
homosexuals icari a. nephritis, chance). Com- supe rinfect ions). (20% mon a 1-
(35%), inmates vascu litis (PAN). mon in alcoh- Drug add icts and ity).
in jail V irus is not cyto- olic liver dise- homosexuals.
lytic but CD. ey- ase (25 -60%).
totoxic T cells
ki II hepatocytes
Chronic carr- None Yes: ) 00/. in Yes : 40-60% Yes: 10-40% None
ier statelehr- adults, 90% in
onic hepatitis newborns left
untreated
Massive necr- Rare MCC of fulmi- Rare Common Common in
osis nant hepatic nee- pregnont wo-
rosis men.

121
r_...,';--:-",2;:-_-i3=--_'i4,-_;:5_-;0c-:: " 12 24
• "Serologu: gap
a--- - -, Jaundice
Symptoms
1-- - - - -. ~,..~ti-HBc IgG • • • • • • • • • • • •• • • • •
• • • • • •
• •• •• •

" - \ '-- 19M - -- anti-HSs


: H SsAg',

• -- -
• - 'r+-HBeAg
-• - HSV DNA
-•
'---=';'--'..:,-~:,.I--",~;-...u4:---;;5'-- -'.:0-1· fl -:'.!::~---:2'"'4-
Mon[tls after exposure
RAY HBY HCY HDY HEY
Lab diagnosis Anti-HA Y-IsM: See schematic Anti·HeV: in- Antl-HOY-lsM or Anti-HEV-
active disease. dieates infeet- IgG: indicates in- IsM or IgO:
Antl-HAY-IgG: ion, not protet- fection, not prot- 19M indicates
inactive disease, tive. EUSA ective. current inree-
protective. immunoassay tion. IgG in-
testS positive dicates reeDV-
in 2--6 weeks. cry and prore-
Recombinant ction.
immunoblol
assay (RIBA)
is confinnatory
test aJong with
polymerase
chain rcaction
I (PCR) testin•.
ActivelPas- Immune serum Vaccine (active), ? Immune ser- 1-mV vaccine pro- None
sive immuniz- globulin (pas- Immune globulin urn globulin. tec15 against HDV
atian sive). Vacci ne (passive). and hepatocellular
(Bctive). carcinoma due to
HBY
HOV serology (aclS:
1. a nti-DBs protKtive
2. BBeAg aDd HBV DNA infective
3. aaU-Hlk-fcM present in serologic:: gap wbe.a: all otber antigens are CODe and a nti-HBs is
nOI yel presenl
4. viec.inalioD protec.ts a,ai n5l-
A. HaY
B. HOY
C. hepatocellular carcinoma from HBV postnecrotic. cirrhosis
S. recovery (rom BBV- anti·HBs and anti-HBc-lgG
6. vaccinated- only anti~ HB s
7. chronic BBV-
A. Ha,Ag >6 mths
B. inrective carrier if have HBV-DNA or HBeAg: correlates with chronic active hepatitis
with piecemeal necrosis and fibrosis
C. "healthy carrier" if lacking HBV.DNAlHBeAg: corre lates with chronic persist'ent
hepatitis and porial triaditis
8. asJocialion of HBV with membranous glomerulonephrlt15 and polya Mentis nodosa

~ ~ ADri-HlI<-IgG

:1<8P
' -~
acute : if
I" 1>6 mths
I from HaY

: ohase of acute HaY

122
Intravenous drug abuser in prison-?type of hcp:uitis: HBV
County jail wilh outbreak of hepatitis:
I. SA V mosl likely
2. if intraven ous drug ab users, then HBV most likely
Hel)Jtlilis most commonly chro nic: HCV
Hel)3titis Il5S0cialed with urticaria, fever. arthl1llgios, und tbe nephrotic sy ndrome:
1. "BY
2. se rum sickness type of disease- type III immunocomplex mechanism
J. vasculitis associated with polyarteritis nodosa
Immunizations given at birth to a baby whose mother is pos itive for H:BsAg:
1. Hep B vaccine (active immunization)
2. t[BIG (passive immunization)
r:ir Most common iofection {rom accidental needle stick :
1. HBY
2. greatest viral burden in blood of a ll hepatitis "iruses
r::r Summ ary of infectious diseases:
I Disease Comments
! Rickettsia Pathoge nesis: Coxiella burnelii. Only ricket1sia transmitted without a "eclOr.
Q/e\'er Inhalation, Contracted by people associated \... ilh the birthing process or fecal
material of infected sheep, cattle and goats (handling of milk) or exposure to
pregnant cats. ClinicalJPatbology: interstitial pneumonia. granu lomatous hepatitis
(80%. anicteric). endocardi tis. R.1( : doxycycli ne
Bacteria Pathogenesis: cholangitis from concurrent biliary infection and obstruction, the
Acute ascending latter either by a stone or a stricture from surgery (60% of cases), Mixed bacterial
cholangitis nora (Escherichia coli MC). CIiDicalJPathology: Charcot's triad (70%) with fe ver.
jaundice and RUQ
, pain. MCC of mUltip le Ii"er abscesses.

U",:r abscl!sses Pathogenesis: biliary tract disense MeC (acute ascending cholangitis). Escherichia
coli MC organism, ClinicalJPatbology: fever, jaundice (30%), hepalOmegaly,
Usually multiple and located in the right li ver,
GranulomatOus Pathogenesis: TB Me infectious cause (sarcoid Me non-infect ious cause). Other
hepatitis bacterial diseases include atypica l Mycobacteria (M avium·imracellulare, MCC in
AIDS), leprosy, brucellos is, syphilis (see be lOW), Other non-infectious causes
include sarcoidosis (MC), drugs, primary biliary cirrhosis,
Syphi/i,f Pathogen esis: Treponema pallidll",. ClinicalfPotbology: in congenital infec tion
there is diffuse fibrosis wi thoUi nodularity, In tertiary syphilis there is a
granulomatous hepatitis, extensive scarring and gummas resu lt ing in a conrrllcted
liver called heDar lobalUm_
Fungus Pathogenesis: disseminated disease associated with histoplasmosis, coccidioido-
mycosis is MCC. Granulomatous hepatitis (caseous or non-caseatin.e necrosis),

I ,·
.>
Parasitic Pathogenesis: Emameba hislolyliea. Ingestion of food/water containing cysts. Anal
Amebiasis intercourse. Cyst waHs are resistant to acid destruction. Excyst in the alkaline
environment of small intestine with formation of infective irophozoites.
CliolcallPatbology: trophozoites favor cecum And right co lon . They invade crypts
and burrow Into the lamina propria with cytolytic enzymes ("histolytica") to
produce "flask shaped ulcers". Bowel inflammation produces diarrhea containing
blood and mucous. Afebrile (very characteristic). Trophozoites pe.netrate vessels
that drain into the ponsJ vein and into the right lobe of liver to form a single
abscess (noted in 90-; ' of patients who die of amebiasis). Abscess contains digested
liver tissue resembling "anchovv easle". Complications: extension through
diaphragm into the righl lung cavity and lung, bematogenous dissemination
throughoul the body (o.g., brain). Lab: identify trophozoite, (usually diarrheal
stools) and/or cysts (usually more solid stool). Trophozoites have a small, centrally
placed karyosome (nucleolus) and eoohrophagos;ytosis (important distinction from
other protozoal diseases). CySts contain up to 4 nuclei and have blunt chromatoid
bodies. Rx: metronidazole.
, EchinococcosiJ Palhoge.nesis: Echinococcus granulOSiS or mllill/oculoris (cestode (Iapeworm)). A
(Sheep herder 's sheep dog eats I"fecied sheep meat conta ining the larva. Larva develop into adult'
disease) worms in the dog (definitive host). Man l inteancdiate host) ingests eggs from
.Bo>f.... infected dogs and the eggs develop into larvae but not adults. Larvae penetrate the
duodenal waH, transmigrate across the peritoneum and penetrate the liver (75%) to
"9-r~ produce single or multiples cysts. Clinical! Patbology: in the liver, larva develop
into hydatid cysts containing scolices with hooklets represeming future heads of the
aduh' tapeworm. Cyst walls have an outer layer (frequently calcified) and an inner
genninaJ layer. Germinal layer develops vesicles (brood capsules) containing
scolices that are referred to as "hydatid sand" . Rupture ofmls often produces fatal
A!l,pb~IAlElie sh!l£~. Lab: eosinophilia. Serologic tests are available. Rx:
81beadAZoie and surgery are used for therapy of the CYSts if the patient is
sympromatic or they are excessively large.
Schislosomiasis Pathogenesis: SchiSlosomo mansonl (primarily). Trematodes (nal worms).
(Bilharziasis) in fection acquired by penetration of larvae from jnfect'e d snDiI, into the patient skin
with subsequent entry into lymphatics and distribution to subcutaneous tissue
("swimmer's itcb") and mesenteric veins. S. mgn.fon; favors intrahepntilE mmal
YrinI, Larvae develop into adult worms that mate and deposit eggs, to which the
host develops a flbroinflammatory response ("pipe stem cjrrhosis") in the vessel
wall . Complications: portal hypertension with hepatosplenomegaly, ascites and
esophageal varices. Lab: ~m brYona~d !;&&s with g abnm hmm!l ~pin~ are noted in
stool or in a liver biopsy. Eosinophilia. Rx: praziquamel.
Clonorchiasis Patbo&en~sis: Clonorchis sinensis (Chinese liver fluke) . Contracted by ingesting
encysted metacercaria larvae in fish. Metacercaria enter the ampulla of Vater and
ascend to the small bile ducts and gallbladder where they develop into adults. Eggs
are passed into the stool. Adults cause the bile duct epithelium to become
hyperplastic and fibrotic leading to cbolMgjocmjnoma. C linicallP.l.tbology:
hepatomegaly, jaundice, diarrhea. Lab: embryonated eggs are present in stOOl or
duodenaJ aspirates. Eosinophilia. Rx: praziquantel.

124
SMlZ ICeDmOl: hydatid cysts in li ver in sheepherder (Basque. Greek)
Conges th'e hepatomegaly:
I. MCC is right heart railure- nutmeg liver
2. transaminases elevated
Prehepatic obstruction to blood now:
1. portal vein thrombosis
2. ascites/varices
3. no hepatomegaly
POSlbepatic obstruction to blood now :
1. Budd-Chiari syndrome- .hepatic vein throm bosis
2. causes:
A. po lycythemia rubrn vera MeC
B. binh control pills
3. cl inical:
A. liver painful and congesred
B. ascites
C. portal hypertension
r::if'" Aleoholliver disease:
t. types-
A. ratty change Me type: reversible
8. alcoholic hepatitis: reversible
(I) fever
(2) neutrophi l infiltralion
(3) neutrophilic leukocytosis
(4) ascites
(5) Mallory bodies
C. cirrhosis: irreversib le
2. pllthogeoesis-
A. acetaldehyde-protein complex:
B. ratty change
C. stimulates collagen synthes is by ho ce lls: nonnallYl Ito cells store retinoic acid
D. peri ven ular fibros is around central vein very characteristic
E. immuno logic damage to hepatocytes
3. sec table of lab abnormalities
Autoimmune hepatitis :
1. progressive type of chronic active hepatitis
2. commonly seen in women
3. se rum antinuclear antibody commonly positive
4. autoantibodies against smooth muscle
5. Rx with corticosteroids
Obstructive jaundice:
1. intrahepatic and extrahepatic types- stone in common bile dUCl MCC
2. bile contains cholesterol and bile salts/acids
J. clinical correlations-
A. rat so luble vitamin defici encies from red uction in bile salts leadin g to mal absorption
B. hype rcho lesterolemia rrom backup orbile contain ing CH

125
BOX 1~2 . Laboratory As pects of Alco holic L iver Disease.

Alcohol metabolism:
alcohol dehydrogenase (cytosol) aldehyde dehydrogenase (mitocbondria)
Aicobol-----.acetaldehyde + NADH + I-r .. acetate + ADH + J-r ~ acetyl eoA

The increase in NADH, acetate (simply fatty acid), and acetyl eoA are responsible for many of the
metabolic and histological changes associated with alcoholism. lncreased NADH alters the redox
potential in favor of NADH over NAD"'. which reverses many biochemical reactions in the body. An
increase in NADH over NAD~ leads to an increase in the production of lactic acid, very low density
lipoprotein (VLOL) and fJ-hydroxybutyric acid.
Lactic Acidosis

r
Pyruvate
NADH ~ NAD-
.. Lactate . . Metabolic acidosis
Lactate dehydrogenase

Glucose~ Futln&: bypoglycemiJI


Note how increascd production of laclic acid produces metabolic acidosis (increased anion gap type).
Since pyruvate Is changed into lactate. there is less pyruvate available for gluconeogenesis, hence
patients commonly have fasting hypoglycemia.
Ine .....ed VLDL (Bypenrlglyetridemia)
NADH ~ NAD-
1,3 Bisphosphoglycerate - + glyceraldebyde 3-PO, ---i~~ Oihydroxyacetone PO,

:!~
1 acetate
Glycerol 3-PO, ... TG (VLDL)
Nore how the increase in NADH reverses the reaction between l ,3.bisphosphoglycerate and
glyceraldehyde 3·pbosphate in the gl~olytie cycle in favor of glyceraJdchyde )·phosphatc. which is
converted into dihydroxyacetone phosphate (DHAP) and from DHAP into glycerol 3· pbosphate, the
carbohydrate backbone of triglyceride (TG). Acetate, a simple fatty acid, may be used in the process of
synthesizing TO as well. An increase in VLDL in the liver results in a fany liver, while in the peripheral
blood, the increase in VLDL leads to hypertriglyceridemia (type rv hyperlipoproteinemia).
Ketoacidosis (p.Hydroxybutyric Acid)
Acetyl CoA - + Acetoacetyl CoA ----+ HMG CoA ---i~~.'~ri

~hydioxybutyric acid (ketoacidosis)


Acetyl CoA, the end-product of alcohol metabolism, may be converted by the liver into ketone bodies
(acetone, acetoacetate, ~hydroKYbuty·rate) . The increase in NADH drives the reaction in favor of J3-
hydroxybutyrlte, hence producing ketoacidosis (increased anion gap type). Since the nitroprusside
reaction utilized in the laboratory to detect ketone bodies is only sensitive for acetone and acetoacetic
acid, the ketoacidosis of alcoholics remains undetected. Hencel alcoholics hove two [)"peS or increased
anion gap metabolic acidosis, mainly lactic and J3-hydroxybutyric acidosis. Since organic acids utilize the
same secretion site in the proximal rubules of the kidneys, the competition of uric acid with these
additional acids often results in hyperuricemia and gouL
C. generali zed pruritus from bile salt deposition: R.x with choiestyram ine
D. CB >SO%
E. clay colored stools: lack of urobilin pigment
F. urine contains CB and no urobil inogen
G. AP and GGT pri marily elevated
Prima ry biliary cirrb os is :
I. rema le do min llot
2. lI utoim mun e, gran ulomalOus deSlruClio n or bi le ducts in tri ads
3. clioical-
A. early presentation w ith..,erurirl!s due to bi le salt deposition ill skin
B. increased AP and GGT
C. no jaundice until late in diseag when most triad ducts are destroyed
D. increased anti-mitoc ho ndrial antibodies and IgM
E. association with SjOgren syndrome and renal tubular ac idosis
Secondary bi linry cirrbosis: comm only noted in cysti c fibros is
Primary scle rosing peric hola ngitis:
I. complication or ulcera live colit is
2. cliuical-
A. jaundice
B. danger of cholangiocarcinoma
C. ERC P gold standard for diagnos is: see beading effect of bi le duct
C hcmlca
' I .0 dd r ug- 'In d uee d I'Iver d 'Isease :
Mo rpbologic C hcmicaVDrug
Pattern
Acute hepatitis I. isoniazid- 10-20% liver damage, toxic metabolite acetylhydrazine
2. salicylate!
3. balotbane- sympto ms after I week, fevcr precedes jaundice, metabolites fanned
from P4 50 system
4. melby ldo.,pa- positive Coombs test
S. acelami nop be n- FR damage, R.." with N-acetylcystcine , MeC of drug-induced
fu lminant hepatitis
Zonal necrosis 1. zone I a ro und triads- yellow phosphorous poisoning. ferroLls sulfate poisoning
2. zone ill around central vein- carbon tetrach loride poisoning (CCI l FR), ace-
taminoohen. Amanita mLlshroom DOi so n in~ .
Intrahepatic I. non-in n a mmatory type- oral contraceptives (estrogen responsible, in terfe res
cholestasis with intrahepatic bile excretion), anabolic steroids.
2. inflammatory type- erythromycin estolatc, amoxici llin-c lavu lanic acid.
chlorpromazine, thiazides
Fatty change I. single droplet (nucleus peripheral ly displaccd)- ethanol . con icosteroids,
amiodarone (looks like alcoholic hepatitis including Mallory bodies and progression
to cirrhos is)
2. microvesicular (droplets witho ut nudeus displacement)- tetracycl ine, valproic
acid
I
Fibrosis I . m ethotrexate
2. hvpervitaminosis A- eatinl! bear meat. isotreti noin Rx for acne

126
Mo rpho log ic C h(! mi ca1lD r ug I
Pa n e r n
Vascular les ions 1. Budd·C bia ri !ly nd ro me- ora l contraceptives I
2. pclios is he pa ri, : ora l contraceptives. anabolic steroid I
J . a ne.iosa rco ma - vinvl chloride. arsenic , Thorotrnst
Tumors o r IUmor· 1. benign tumo rs (hepatic o r liver ce ll adeno ma)- o ral con.!"acep~ives. anabolic I
like conditions $ I . . _ tendency (or ruptu re and intraperitoneal hemorrhage (US M.L E) - I
2. ma ligna nt tum o rs- hepatocellular carcinoma and oral contraceptives/anabolic
I st'cro ids
Granulomatous 1. a llopurinol
hepatitis 2. hydra lazi ne,
3. sulfonam id es
i
-USMLE teeaarlo of a w'lablliller wllo bas iDtraperitoaeal be:morrhaee:
). ass um e rh e ~' a re ta king a nabolic steroids leadin g to Ih'er cell a d enoma k{
2. s po nta neu us rupture of Ih'er cell ade no ma
USML[ ~een.rio o( a pregnant woman with intrapniloae.1 hemorrhage: effect of estrogen on
produc ing li ver ce ll adeno ma with tendency to ruplUre
Birth eoatrol pUlt (estroge n compo nent) a.ad uabolie .teroicb effect 00 Uur:
I. in t ra hepat ic cholestas is
2. liver cell a den o ma - te nd ency 10 rupture du r ing pregnan cy
J. bepa tocellula r ca rcino ma
USFtU..£ IHurio wttb aD ••blett wbo prese... witll jaa_dice, DO previous bealtb problems,
DO blatory of liver d1aeue. aad DO previous trauf",IoU j H rum fraIlIam iDues are sligbtly
elevated aad seram allutliJIe phOlpbalaJe is markedly elevatfd : assume patient is taking
anabo lic steroids (cho lcstatic jaund ice)
Hemochromal o, ;" - It. 1'1"51 ~,",uJ (dM'f,',,- D, YQ9? iN (). S, 7J-
I. AR d isease ,
2. unrest r icted reabsorptio n of iron from s mall bowel
J. cli nical-
:\. liver target o rgan: ci rrhosis with 30% chance of hepatocellular carcinoma
8 . pancreas:
( 1) malabsorption
(2) d iabetes me llitus
C. skin hypcrp igmented: "bronze_d.iabetes"
D. henn with restrictive cardiomyopathy
E. iron overload fi nd ings:
( 1) increased se rum iron
(2) decreased TIBC
(3) increased percent saturation
("') increased se rum fe rritin
F. R.'I( with phlebotomy

127
~ Wl bon 's disease:
I. AA _\
2. defeci In copper secretion into bile I ~ ~ •~ {J, IlD:rI"' S~Y
3. clinical- f)c. r- (Ill dl
A. chronic: liver disease-+ . /
(1) low ceruloplasmin fevels with subsequent increase in free copper in blood ....
(2) Kavser·fleiscbcr ring in eye
~f..( -/,,('8 . (3) len1icular . de eneratiOn (chorea... rigidity)
low total <opper due 10 low ceruloplasmin (6.~ f1 ~ "W'~ I""'~)
.lis c. high serum/urine copper
o. Rx with esnicillamine
cr AA T ddiciency:
l. AR disease
2. decrean d liver ,y othes;" of AA T -to panacinar emph ysema
3. defective seerelioD of AA T by hepatocytes-+ cbronic active he patitiS/cirrhosisl
bepal ocelluhar cancer in child- AAT is PA S pos iti ve in hepalocytes
4. normlll alleles MM
S. a bn orma l a lleles MZ or ZZ
Lh'tr d L~eaJe in pregna ncy:
1. \'Ira l hepatitis Me liver disease
2. benign Intrahepatic cbolestasi.5-
A. estrogen re lated
8. nOI dnogerous to mother or baby
3. acute ratty Ih'er of pregnincy- fatal unless baby delivered
... pre-ec.1a mpsla-
A. HELlP syndrome
(I) H - hemolytic anemia
(2) EL - elevated rransaminases
(3) LP - low platelets
8. liver cell necros1s around triads
Li ver dbcue In cbildren:
I. neon.ci l cbolestasiJ-
A. neona181 hepatitis MCC
B. bi I iary atresia:
(1) extrahepatic biliary atresia Me type
(2) bile duct proli feration in uiads (no proli fe ration In intrahepatic atresia)
(3) rad ioactive dye cannot get into the smtlll intestine
(4) jaundice in fi rst week of life
C. metabolic diseases:
( I) e.g .• galactosemia
(2) tyrosinemia
2. Reye syndrome-
A. infection associations are chickenpox and influenza
8. microvcsicular fatty change in liver with increased ammonia and transam inases
C. cerebral edema with encephalopathy
D. salicylates damage mitochondria in liver: disrupts urea cycle and metaboli sm of
ammonia

128
17' Cirrhos is:
I. ty pcs -
A. micronodular/macronodular/mixed
8. regenerative nodules lack nonnal liver architecture
C. inrrasinusoidal hypertension due to regenerative nodu les : leads to portal hypertension
2. portal hypcrtensioD-
A. esophageal varices
B. caput medusae
C. ascites
D. hemorrhoids
3. heputorenal.syndrome- acute renal failure without any gross or mic roscopic changes
4. hepatic encephalopnthy-
A. due to bu ild up of ammonia and fal se neurotransmitters like octopamine and y-
aminobenzoic acid
B. coma, asterixis, mental status abnormalities
5. hyperestrinism-
A. due to decreased metabo lism of estrogen and 17-ketostero ids (aromatized into
estrogen)
B. cl in ical:
( 1) gynecomastia
~ ,. 2) female secondary sex characteristics )
;.t. (!J>olfIP ,rJ Y (3) spider angiomas ( a/..k<.;(JI'-V(I!N'-'-~ h·~.,..r.
rJatJ\
•.I I~~ /'10'
· (4) palmar erythema
6. Dupu ytrcn 's contraclures in fingers- fibro matos is in volv ing tendon sheaths COntraCL'i
fingers
7. oscites-
A. pathogcnesis :
( 1) increased hydrostatic pressure in portal vein
(2) decreased oncotic pressure
(3) s~oDdary a~nism
a. decreased metaboli sm of aldosterone
b. activation or reni n-angiotensin-aldosterone system from decreased arterial
b lood volume
B. danger of spontaneous peritonilis from E. coli
C. use a ldosrecom' bJ o<ili.er rather than loop diuretics:
(1) produces a metabol ic ac idosis which increases the excretion ofN~' in stool
(2) use of loop diuretics produces a metabolic alkalosis causing greater bowe l
production of diffusible NHl : danger of precipitating hepatic encephalopathy
Cave rnous hemangiomas: Me benign tumor in liver
Hepatic adenoma (liver cell adenoma):
I. associated with estrogens/anabolic steroids
2. tendency to rupture and produce intraperitoneal hemorrhage
Uepatocellu la rca rcinom a : ( o.{1AIJ1(> ~rJ ,rJd Ill(''J~0<1N.j o/-ct((kA.:.s./-..))
1. causes-
A. HBV MCC in Uni ted States: MC cancer in Far East due to ch ronic HEV liver disease
+ exposure to afla toxins (molds in food )
B. HeV

129
C. hemochromatos is
D. alcoholic cirrhosis
2. patboJogy-
A. develops in a background of cirrhosis
B. blood vessel invader
C. neoplastic hepatocytes secrete bile
3. c1iniul-
A. weight loss
B. abdominal pain due to rapid increase in bloody ascites
C. increase in AL T is characteri stic
D. increase in AFP and AA T as tumor markers
E. ectopic secretion of erythropoietin produces secondary polycythemia
F. ectopic secretion of insulin· like fact or produces hypoglycemia
r:r Metastasis is MC cancer or liver: lung cancer is Me primary si te
qr Gallbladder adenocarcinoma :
I. Me primary cancer or biliary tree
2. causes-
A. gallstones MeC
B. porcelain gallb ladder: dystrophic calcificat ion of gal lbladder
Gallstones:
1. pathogenesi5-
A. too much cholesterol (obes ity)
8. too little bile sal ts/ac ids in bile (cirrhosis)
typcs-
A. m.ng are cholesterol stones
8. 51ack calcium bilirubinate stones suggest extravascular hemolytic anemia
C. most stones do not calcify
J. Dx- ultrasound is gold standard sc reening teSt
4. compUcatlon5-
A. obstructive jal1ndice in common bile duci
8. acute cholecystitis/pancreatitis
C. gallbladder cancer
Ac ute cholecY5titis:
1. impacted stone in cystic duct
2. E. coli Me pathogen
3. AIDS patient with acute cbolecystids : cryptosporidium and/or CMV are MCC
Chronic cholecystim:
1. 5tones are invariable
2. chemical inflammation
Cystic fibrosis:
1. AR disease--
A. defect on chromosome 7
B. 3 nucleotide deletion which codes for phenylalanine leads (0 defective CF transport
regulator for chloride ions:
(1) decreased cr reabsorption in sweat glands (basis of sweal test)

130
(2) increased Na' reabsorption and decreased cr secretion imo tenninal bronchioles
(infect ion) and pancreatic ducts (malabsorption: causes obstruction by thick
mucus
2. complicalio ns-
A. respirnlory infections/failure:
(1) M CCOD
(2) P. oeruginosu MC pathogen
(3) MCC of bronchiectasis
B. mal absorption:
(I> pancreatic exocrine deficienc),
(2) diabetes mellitus
C. secondary biliary cirrhosis
D. total infeni liry in maleslllJMJ ·fJ:
( I ) atresia of vas deferens
(2) women have thick cervical muc us but can have children
E. meconium ile us in newborn
UsMO: ROtiii'IO, lIIo.ber .......la. dal!d W • ii1ty w •• wb•• ldUed:
1. tyljlc nbro."
2. do SWtllt tes...
Acute paacreatitis:
I. caUSe5-
A. alcoholis m
(1) M CC
(2) causes thickening of pancreatic secretions leading to obstruction
B. stone in common bile duct or accessory dUClS in pancreas is a close second
C. enzymatic fat necrosis
2. clioical-
A. fever
B. pain in epigastrium with radiation into back
3. lab-
A. increased serum/urine amy lase: cleared quickly from serum in few days and is present
in urine
8. serum lipase is more specific than amylB.\ie
4. complicatlons-
A. pseudocyst MC complication:
(1) persistence of elevated amylase beyond one week
(2) mass in abdomen
B. abscess: high mortality un less drained
C. left·sided pleural effusion: contains amylase
D. ARDS with hypoxemia
E. hemorrhagic pancreatitis:
(1) Gray-Turner sign in nank
(2) Cu llen's sign around umbilicus
F. hypocalcemia from enzymatic fat necros is a bad sign
S. CT best test ror pancreatic disease

131
C hronic pancreatitis:
1. alcoholism M.CC
2. clinical-
A. malabsorptio n
n. severe pam
C. diabetes mellitus
O. predispos ition to pancreatic cancer
E. calcifications usually notes o n x-rays
r:r Pancreatic canccr:
l. causes-
A. smoking Mec
8. chronic pancreatitis
2. clinical-
A. painless jaundice: head of pancreas MC location
B. palpable ga 1Jbladdcr: Courvoisier sign
C. clay co lo red stoo ls
D. we ight loss
E. superficial migratory thrombophlebitis : Trousseau's sign
F. metastasis to umbilicus : Sister Mal)' Joseph sign
3. lab-CAI9-9
Summary c ha ri of laboratory abnormalities associated with select disorders of the linr:
arrows represent the degree of magnitude. CBrra % ,.. conj ugated bil irubin fraction of total
bi lirubin . AST = aspartate transaminase. ALT =- a lanine transaminase. AP =- a lkali ne phosphatase.
EHA IE extravascular hemo lytic anemia, GGT == gamma gl utamyltransferase. LOH =- lactate
dehydrogenase. N =- normal.

Disease CBffB AST ALT AP GGT LOR Urin e Urine


% Bilirubin Urobilinol!cn
Vita l
beaatiti! 20-50% tft tHt rt tt rt tt tt
Alcoholic
hepatitis 20· 50% tt t t tH tt tt t
(obstructive
eomoonen t)
C hoiesta lic
live r disease >5 0% rt rt tttt Htt tt Ht Absent
EHA
<20% tt N N N tt Absem m
RBC RBC (i ncreased fro m
bilirubin lond)

Gran ulomas N N N ti ti N Absent N I


MelascUL5 N N N tt tt tit Absent N
Note: AST IS present III RBCs

132
Questions used duriDg [be board review:

A febri le 12 year o ld boy wi th a viral in fection lapses into co ma. Physical exam reveal s
papilledema and hepatomega ly. T he serum ammonia and transaminases are e levated Dnd the PT is
prolonged. The patient MOST LlKEL Y has ...
A. viral hepatiti s
B. Reyc's syndrome
C. salicylate intoxication
D. acetaminophen toxicity
E. a -I antitrypsin deficiency
B
<:r Whi ch of the fo llowing sero logic data best represenlS a patient who has recovered from hepatitiS
B'
HBsAg HBeAg Anti-HBc-lgM Anti-HBc-lgG Aoti-HBs

A.
nCQative nellative Dositivc nee.ati ve nCf.!3tive
B.
nositi ve I nos itive DOsitive neEative ne~atj\'e

C. negative n e~ative ne~tive positive posit ive


D. negative nC2ative ne2ative ne~ative Dositive
C (A= sel'Olog lc gap, B - acute or chrOniC HBY) 0 - vaccmated)
An afebrile 42 year o ld migrant worker from Mexico presents with bloody diarrhea and right upper
quadrant pain . The patient MOST LIKELY has ...
A. amebiasis
B. ec hinococcosis
C. acute cho lecystitis
D. ascending cho langitis
E. metastatic colon cancer
A
<7" A 48-year-old alcoho lic with cirrhosis and chronic pancreatiti s has steatorrhea and a pro longed PT.
The PT does not correct to norm'al after giving an intramuscular injection of vitamin K. You
concl ude that the patient MOST UK£L Y has ...
A. vitamin K deficiency
B. a c irculating anticoagu lant
C. an iso lated factor deficiency
D. inadequate synthesis of coagulation facto rs
E. vitamin K deficiency secondary to malabsorpti on
o
(if"
A 45-year-old man presents wi th increased skin pigmental'ion, steatorrhea, and diabetes me llitus.
The mechanism MOST UK£L Y responsible for this constellation of findings is ...
A. amyloidosis
B. alcoho lic ci rrhos is
C. a defect in iron mClaholism
D. a lphn ]-nntitrypsin deficiency
E. a defect in copper metabolism
c

133
or In which of the followina diseases would you expect a conJupted bilirubin >50% of the tOliI
bilirubin?
A. GIiben'sl)'Ddrome
B. CIuooic vital hepatitis
C. CriaJer-Najjar I)'Ddrome
D. _ in the eonll.... bile cIutt
E. ExIra_lor hemolytic ....... ia
o (choice A <20%, cboice B 2t}.5O%, choice C <20%. cboice E <20%)
or Ao afebrile 62 yaT old .... with a bistOf)' of alcobolism aod chronic panc-ms preIC1Its with
... labt lou, a slow OIIICI of painIcss jaundice, and a normocytic .......ia. Pllysical ..... rcvcals •
palpable pllbloddcr and • liabHoio<od stooL The potieat MOST UKELY bu...
A. hepotoccllulor carcinoma
B. carcinoma of the pllbladdcr
C. primary sci_sinS cbolangiti.
D. • rtone In the common bile duct
E. carcinoma of the head ofpancreu
E

134
Kidn ey, lower urinary tract, male reproductive:

Syndromes in renal disease: ON = 210meru onept ntI s


Sv ndrome C harac teristics E xamples
Nephritic 1. glomerular diseases arc 1. type IV diffuse proliferative GN in SLE
characterized by- ol iguria, 2. post-streptococca l GN
hematuria, RBC casts, 3. rapidl y progressive crescentic GN- e.g. , Good-
periorbital puffiness (ret"ention pasture's syndrome
of salt), and mild to moderate 4. 19A GN (Berger's disease)- Me G
proteinuria. 5. Aloort 's svndrome- hereditarY nerve deafness I
Nephrolic I. glo merular diseases nre 1. minima l change disease - MCC in children
characterized by- >3.5 gm/dL 2. focal segmental glomeru losclerosis- MC type in
of protein/24 hours, ova l rat AIDS, IV drug abusers
bodies, fatty casts with Maltese 3. membranous GN - MC type in adults, HB V
crosses, generalized pitting assoc iation
ede ma, ascites 4. membranoprol ifera ti ve GN - ~pe I associated
with HeV
5. diabetic glomeru losc lerosi s- begins with
microa lbuminuria
6. amYloidosis
Rapidly progres- I. formation or crescents I. end -stage di sease for many diseases- Good-
sive crescentic (parietal cell s) in Bowman 's pasture 's disease, Wegener's granulomatosis, post-
GN space in fectious GN
2. rapid loss of renal func tion
with hematuria and protein uria.
3. worst prognos is of all types
orGN.
Ac ute renal 1. abrupt reduction in GFR 1. ischemic ATN- MCC is prerenal azotemia
fa ilure 2. oli guria (sometimes poly. 2. nephrotoxic ATN- MCC is aminoglycoside
uria)
, followed by radiocontrast dye used in IVP
3. pro po rt ionate increase on
BUN/creatinine (ratio < 15/ 1)
4. tubu la r dysfunction- loss of
concentration, FENa" > 1, renal
tubular casts
Chronic renal 1. reduction in GFR >3-6 mth s 1. chronic GN
fai lure (C RF) 2. waxy and broad casts 2. chronic pye lonephritis
3. protei nuria 3. end-stage diabetic nephropathy- MCC ofCRF
4. metabo lic acidosis
5. hypocalcemia- vitamin 0
deficiency
6. hyperphosphatemia- cannot
excrete
7. total loss of concentrat ionl
ditution

135
Syndrome Charac teristics Examples I
Renal lUbu lar
1. type 1- defect in prmonfK - 1. normal anion gap metaboli c acidosis
acidosis (RTA) pump in collecting tubule 2. urine pH usua lly >5.5
2. type 11- lower threshold for 3. all have hypokalemia except for type IV
rec laiming bicarbonate In
pro1(imul tubule
3. type rv- destruction of jux-
t'a glomerular apparatus, hypo-
reninemic hypoaJdosteronism.
Me due to diabetes
Urinary traci 1. ascending infectio n from an I. acute and chronic pyeloneph ritis.
infections incompetent vesicoureteral
va lve.
2. feve r, flank pain. lower
urin!!r}')ract siEns, wac casts.
Hypertension I. mean systolic pressure > 140 I. essent ial hypertensioll - Me type
mm Hg and diastolic pressure 2. secondary hypencns ion- renovascular MC.
>90 mm Hg on 3 separate atherosc lerosis in men, fibromuscular hyperplasia in
occas ions women
3. nephroseierosis- type of renal disease seen in all
~ types of hypertension
Rena l stones I. decreased water intake I. ca lci um stoncs- MC type, radiodense. calcium
2. reduced urine citrate- nann- oxalate MC type. calci um phosphate stones In
ally binds excess calcium ch ildren
3. hypercalciuria- MC metab- 2. magnesium ammonium phosphate-- struvitc
olic abnormality in urine, ex- stones, due to urease producers
cess reabsorpt'ion of calcium in 3. uric acid- urate nephropathy from chemother-
GI uaCt apy, rad iolucent stones
4. alterations y, urine pH fa vor-
ing stone precipitation- e.g.,
alkl1line pH with Proteus
infections (urease producers)

136
Chemical dipstick reactions in urinalysis:
Tes' Comments
pH J . pure vegans have an alka line pH- citrate in fruits is converted into bicarbonate
2. meat eaters have an acid pH- increased excretion of organ ic acids
3. urine pH is manipulated to prevent stone recurrence- uric acid is insoluble in an acid
pH bUI solub le in an alkaline pH
4. urine pH is manipulated in Rx of drug overdoses- alkalinize the urine in salicylate
intoxication to increase excretion
5. ammonium smell in an alkaline urine indicates a Proleu.J infection- urease producer
that convens urea into ammonia
6. ascorbic acid is the best agent for acidifying the urine
7. carbonic anhydrase inhibitor alkalinizes urine- blocks reclamation of bicarbonate
8. acidotic states have an acid urine pH- increased secretion of NaH1PO" (titratable
acidity) and Nl-LCI. exception is renal tubular acidosis where the pH is >5.5
9. alkalotic slates generally have an alkaline urine pH- exception of advanced metabolic
alkalosis from vomiting, where excess reclamation of bicarbonate due to volume
depletion causes a paradoxical aciduria
.Protein 1. proteinuria is the first metabolic abnormality in renal disease
2. all positive dipsticks for protein are checked with sulfosalicylic acid (SSA)
l. SSA is equal ly sensitive to bolh albumin and globulins
4. if only albumin is present, the dipstick and SSA result wi ll paraUel each othcr- e,g.,
both +I reactions
5. if albumin and globu lins (e,g., Bence Jones light chains in mUltiple myeloma) are
present, the dipstick may be only trace or +1, while the SSA is +4- order a urine
electrophoresis 10 RIO light chains.
6. microalbuminuria (<30 mg/day) is the first indication of diabetic nephropnthy-
microalbuminuria dipsticks are more sensitive (1.5-8 mg/dL) than standard dipsticks,
ACE inhibitors prevent progression of the disease
Glucose 1. glucosuria is diabetes mellitus until proven otherwise
2. non-OM g!ucosuria-1lregnanc), (renal threshold for gJucose is lower than normal).
benign glucosuria (low renal threshold). blood glucose is nonnal
. 3. Clinitest lests for reducing substances in urine- glucose, galactose, fructose, lactose,
, pentoses- does nOl detect sucrose (not a reducing sugar)
4. urine Clinitest is used as a screen for children <2 years of age 10 RIO one of the
I
inborn errors of metabolism- e.~ ., }'!;alactosemia, hereditary fructo se intolerance
Ketones I. Il-hydroxybutyrate ketoacidosis (alcoho(jcs) will have a negative nitroprusside
I reaction ("masked ketosis")
2. ketones are norma l in pregnancy
I
3. pathologic ketonuria- fasting/starVation, ketogenic diets, diabetic ketoacidosis,
I isoprQ~ alcohol ooisonina. von Gierke's Rivco2enosis
I Bilirubin 1. ooly water-soluble conjugated bilirubin (Ce) enters urine- bilirubinuria is always a
pathologic finding.
,
I
2. bilirubinuria is present in hepatitis or obstructive jaundice
3. bilirubinuria is not present in conditions with an excess in unconjugated bilirubin
I I (UCB)- e.R.• ex....vascular hemolytic anemia

I
! 137
Test Co mnlcnts
Urobi linogen 1. increased urobilinogen indicates either eKuavnscu lnr hemolysis or hepatitis
2. false positive reactions with porphyrins- e.g., ncu le intenninent porphyria
3. absence of urine uro bilin o~en indicates obstructive iaundice- also absent in the stool
Blood 1. dipstick is positive for either Hb and/or myoglobin
2. RIO myoglob inuria with serum creati ne ki nase
3. hematuria secondary to a lesion anywhere from the glomerulus down 1.0 Ihe urethral
meatus
4. always order a sickle cell screen when African-Americans have unexplained
microhematuria- renal dama~e in both trait and disease
Nirrite 1. detects nitrite produced by nitrate reducing bacteria- sens iti vity 30o/a, specificity I
90%)
2. £. coli is a niuat'c reducer and the Me urinary DDthon,e n.
Leukocyte 1. leukocyte esterase react ion has a sensitivity/specifici ty of 80% for a UTI
I
esterase 2. dipsticks for nitrite/esterase are a good urine sc reen for infection and whether to
culture urine or nOl
c:Jr Classic urine casts:
I. RB C casts-
A. nephritic syndrome
B. t .g., post-streptococca l GN
2. WBC casts-
A. acute pyelonephrids
B. acule interstitial nephritis due to drugs
C. excl udes a lower UTI
3. renal tubular cans- acute tubular necrosis
4. wuy casts- chronic renal failu re
S. broad casts- ch ronic renal failure
6. hyaline casts-
A. ghost-like cBStsl non-refractile, smooth borders
B. sign of proteinuria
C. no cl in ical significance in tbe absence of proteinuria
7. fatty casts wltb Maltese cr osses- nephrotic syndrome
Urine crystals:
I. caleium oxalate- consider
A. ca lci um oxalate stone
B. Crohn 's disease

2.
C.
cystin e-
A. hexagonal crystal
0
ethylene glyco l poisoning

B. indicates cystinuria
Renal dysplasia :
1. MC cystic disease in cbildren
2. no Inheritance panern
3. a bnormal developmen t of ODe or both kidneys
4. prescnt as unilateral flank mass in most cases

138
Juvenile polyel'stle kidney disease:
1. AR discasc
2. bilateral discase
3. oligohydramnios in mother: Poners facies due to cramped quaners
4. eysl$ In other organs
5. incompatible with life
Ad ult polycystic kidney disean (APKD):
I. AD disease with bypertfnsion
2. association with CNS bern' Ancurvs ms-
A. due to hypenension _ . (0
8. subarachnoid hemorrhage due to ruptured aneurysm
C. intracerebra l bleed with hypenension also possible I
J. C)'SIl n OI prHent at birtb-
A. deve lop in early teens
B. screen with renal ultrasound
Renul agenesis : oligohydramnios
POller facies:
I. child with cystic diseuse of kidneys
2. 10\00' u t tun;
J. parrot beak nose
4. hypoph15ia of lungs due to oligob)'dramnios
Relencion cysts:
1. MC cyst in adu lts
2. acq uired cySIl derived from Ob51ructiOD of tubules
3. cysts also acq uired witb bemodialysis
Glo merulon epbritis: ( ff?)
1. majority a re immuDocomplex (IC), ty~ m bypersenJhlvlt)· disorders-
2. nomeoctluure-
A. if it ends in -ilis. it is type lU: exec lion is Good allUre'S svndrume. which IS ~
B. diJlY.se if ~II alomerulj abnormal
C. ~I if only a few glomeruli arc abnQnnaL
D. Erolifcralh~ e if> J 00 nuclei
E. mcmbnmou if membranes thick
F. mcmbran o~Li~if membrane ' k and h)~pcr.t.t.J.MQ.t
3. immunonuorescence (IF)-
A. always linear if anti-basement membrane antibodies are prcsent : e.g.. Goodpa.~ure's
syndrome
B. always lumpy bumpy (granular) if Ies are present: e.g .. post-streptococcal ON
C. can tell what is deposited in glomeruli: e.g., (gA deposits in IgA GN
4. e.leelron microscopy (EM)-
A. detects electron dense Ie deposits
8 . delects fu sion of podocytes (indicates nephrotic syndrome) and other 511Uctural
abnonnalities
S. GBM made by visceral epithelial cells (ba,'c podocyuJ and slit ports)-
A. strong negative charge due to heparan sulfiue: repels negatively charged proteins like
albumin
B. negative charge is lost in minimal change disease

139
.... G lomerular diseases commonly presenting as a nephrllic synd rome:
1. po.sI ~5 Ireploc~cal GN-
A. mic.roscopic: diffuse proliferative panern with neutrophil infiltration
B. pathogenesis:
( I) poSt group A-streptococcal i nf~l ion in II child (usually skin infection) wlI.h
nephritic strajns of streptococci
(2) Ie GN with planted bacterial antigens
(3) activation of ahcmale complement pathway
C. granular iF panern
D. EM with subepithelial deposits
E. clinical:
( I) smok)' colored urine (hemoglobin converted into hematin with acid pH) 1-4 wks
following group A Streptococcal infection: skin or pharyngeal
(2) children fare better (95% recover) than adu lts (60% recover)
(3) high ASO titers. anti-DNAase B titers, low CJ
2. systemic lupus erytbemal05us-
A. microscopic ofrype IV is diffuse proliferative ON
(1) ".wire loop ing" of capillaries (subendothelial deposi ts)
(2) neutrophilic infiltration with fibrinoid necrosis
(3) hematoxylin bodies (altered DNA)
B. pathogenesis:
( I) DNA=;!nti-DNA IC.
(2) act'ivarion of classical pathway
C_ gr:tJ1ular IF
O. EM with subendothelial deposits
E. clinical:
~ -
(I) anti-dsDNA correlates with renal d isease 't'
(2) decreased C3 correlates with disease activity
(3) common COD in SLE
l. Alport 's sy ndrome-- ~
A. microscopic: foam cells in glomeruli
B. puthogenesis:
( I) sex-linked dominant
(2) common in Mannons
(3) abnonnality of a5-chain type IV collagen
(4) lack of a3-chain (Goodpasture antigen)
(S) not an IC disease
C. no IF pattern
D. EM has no deposits
E. clinical:
( 1) disease is worse in males than fema les
(2) nerve deafness
4. leA glomerulooepbritis (Berger's dinase)-
A. pathogenesis:
(1) increased c irculating levels of polymeric (gA and Ie with tgA
(2) activates alternate pathway
(3) similar find ings in Henoch-SchOnlein purpura
D. granular IF : only way to Ox is to document !gA

140
C. EM with deposits in mesangium
D. clinical:
( I) MC glomerulonepbritis
(2) MC in children/young adults
(3) ch ildren present with gross hematuria 8 fcw days after an URI
(4) adults present with asymptomalic microscopic hematuria and proteinuria
(5) somt: cases become nt:phrotic
(6) chronic in 30-50%
(7) nonnal CJ
5. 7f rapidly prograsiVC! crescentic GN (RPGN)-
A. microscopic; glomeruli compressed by crescent derived from parietal epithelium
O. pathogenesis:
( I) - 'to Good pasrure's disease
(2) -"'systemic diseases like SlE, polyanerilis. Wegener' s granu lomatosis
(3) ~ drugs ~\
(4) infectious disease: e.g .. infectious endocarditis
(5) antibody types:
a. ant i-GBM (Goodpasture. rype IJ reaction)
b. anti-neutrophil cytoplasmic ant'ibodies (Wegener's)
c. ICs in some cases
C_ Goodpasture's disease:
( I) anli-GBM : IgG antibody directed against non-collagen doma in of aJ type TV
collagen in GBM and pUlmonary capillaries
(2) linear IF
(3) EM with no depoSits
(4) clinical:
a. RPGN progresses to acute renaJ failu re in a few weeks to months
b. begins with hemoprysis and ends in renal fai lure
c. usually young adu lt males in 90% of cases
(5) plasmapheresis useful in removing antibodies
,,, Glomerular diHa.ses commonly wociated witb e broti ome: :>
3_~ ," J 'f {'I
l 1. minimal cbange disease (lipoid nepbrosls, nil dlsease)-
A. microscopic nonnal except for positive fat stains in the glomerulus and tubu les
B. pathogenesis:
(I) T cells produce lymphokine that destroys negative charge barrier (polyanion
loss)
(2) selective proteinuria (albumin)
C. negative IF
D. EM shows fusion of the podocyt.Cs: no deposits
E. clin ical:
(1) MCC of nephrotic syodrome ill chiJdren
(2) MC in bQ)5 be"'.. n 6- 8 v. old usu. lly after URI _
(3) normal blood pressUR
(4) associations:
•. atopic history
b. nodular sclerosing Hodgkin' s disease
(5) respond dramatically to conicosleroids

141
2. me mbranous GN-
A. microscopic with diffuse thickening of membranes but no ce ll proliferation
B. epimembrWlous spikes with a "hair o n end nppto.rWlcc" noted with special stains:
correspond with subepithelial deposits of IgG and C3
C. pathogenesis:
( I ) in·situ Ie deposition: resembles Heymans GN 10 rats with antibody directed
against epithelial antigen
(2) majority are idiopathic
(3) drugs
a. captopril
b. NSAlDs
(4) infec tions
II. maJaria
b. hepatitis B
(5) malignancy
Q. colon
b. mal ignant lymphoma
o. granular fF
E. EM with subepithe lial deposi ts: fu sion of podocytes
F. c linical:
(1 ) MCC ofocpbrotic syndrome in adults
(2) -40-5(}O/O progress 10 end·stage disease
(3) nonnal e3
l. focal segmental glomerulosclerosis (FSG)-
A. pathogenesis:
(1 ) pan of minimal change disease spectrum in adults
(2) No Ie deposits
(3) assoc iations
a. HJV
b. renal transplant palients
c. i ntra~e n o u s heroin abuse
B. EM with fus ion of podocytes: no electron dense deposil's
C. cli nica l:
( I) poor prognosis
(2) recurrence after renal transplantation.
(3) nonnal e3 .
4. type t and n membraaoprolifcrative GN-
A. microscopic has ~ trjE.~f in type I > type II: ingrowth of mesangium between
endothelial cell and GBM
B. pathogenesis:
( I) type I is an Ie disw e: associations with hepatitis C, C oglobulins. neo lasms.
II
twr f,....(11
J Il<ll< n has e3 "<pM;';, CaolO'
a. autoantibody aga inst alternate pathway C) convertase (C3bBb)
w~ b. causes sustained activation of C)
c. intramembranous deposits are C3
e. granular IF in type J and n
o. EM:
( \ ) type I has subendothel ial deposits.
(2) type II has intramembranous deposits: called dense depos it disease

142
E. clinica l:
(1) type I more common than type D
(1) always order HCV se rologies in type IIVLPGN
(3) poor prognosis (50% develop chronic renal failure in 10 years).
(4) low C3 in both types
(5) C3 nephritic faclor in type 11
5. Kimmelstiel Wilso n disease (diabetic glomeruloscierosis)-
:~J A microscop ic:
..t'"'o'\i '- .
\ r~ ( I) focal areas of glomeru loscle rosis ("Christmas balls ") due to protein
I I accumu lation
,I ~~L.b'" (2) afferent and efferent hyaline arteriolosclerosis
)10'
~ k' byaliDe .. Ie.ioloscle.osis or arre.'Dtierre.ent •• tc~iolcs

8. pathogenesis:
(I) non-cnzymatic glycosy lalion of GBM and tubule basement membranes:
increases vesse l penneability to proteins
(2) glomeru lar hypertrophy fro m cytokine release from leukocytes
(3) osmotic damage due 10 toxic effect ofsorbilo\ on endothelial cells and GBM
(4) hyperfihrat'ion damage to mesangium : increased GFR early in disease due to
efferent an criole hyaline an eriolosclerosis
C. neg:ltive iF
D. Elvt with fusion of podocytes
E. clinical:
( I) Me disease iD\lolving the glomerulus
(2) more common in type I than II OM
(3) JUjcroaibumjnum is first sign of nephropathy (begins after -I 0 years)
captopril slows onset of nephropathy by decreasing angiotens in IJ : re leases
AtCwI,"
_'"'
_ ( 4_)
pressure on glomeru lus
(5) hypenension occurs when proteinuria is present
(6) retinopathy parallels nephropathy
<n MeC of end-stage renal disease in Unhed States
(8) other renal problems with OM
a. renal papillary necrosis
b. acute and chronic pyelonephritis
6. amyloidosis-
A. microscopic:
(1) thickening ofG BM and mesangium
(1) positive Congo red stain and apple green birefringence
B. pathogenes is: deposit ion of amyloid derived from light chains in multiple myeloma or
serum associated amyloid in reactive amyloidosis
C. negative [f
D. EM demonstrates amyloid fibrils
E. clinica l:
(1) kidneys are most commonly affected in systemic amyloidosis in 80% of cases
(1) renal failure is Me COD in systemic a myloidosis

143
7. pregnanc), induced bypertension (preeclampsla/edampsia)-
A. microscopic shows diffusely swollen endothelial cells ("endothclios is")
B. pathogenesis:
(1) abnonnal placentation results in decrease in perfusion
(2) vasoconstriction from angiotensi n U and endothelin o verrides vasodilation of
placental ,'esse Is
(3) damage to endothelial cells in placenta and kidneys
C. cl inica l:
(I ) hypenension. proteinuria, pitting edema (convulsions IS called eclampsia)
usually in the third trimester
(2) mo lar pregnancy if the above occurs in first trimester
BUN/Creatinine ratio in work-up of oliguria:
1. parmA I ratio is1!lL!.
2. prcrenlll azotemia-
A. due to '!;:creas.£...in c8!:,d.l!9, QUIIDtt:
((21» heart railure [(J~ Yf~'f>.{\ .); ('.0.)
hypovo lemia ' .J
8. r8tio > 15/ 1:,
(1) clearance of c reatin ine . in urine qecre8$c..$ when glome~lar filtn;nion fate
decreases L. iJ/c. .10,.;/+ ~ C'{ftlJt..,;JI~(.· ... ~'';dJ ~s1", · c.JdTI ,.('t.~.fo ,, (}..
(2) BUN increases d isproportionate ly due to reabsorptio n in the prox.imal tubule
(3) e .g.• BUN 80 mgldL. creatinine 2 mgldL. ratio =4011
J. acute tubular oecrosis-
A. ratio remains <\ 5/ 1:
B. both creatinine and BUN are equally affected when there is tubular dysfunction
C. e .g., BUN 80 mgfdL, c reatinine 8 mgldL, ratio "" 1all
4. posfrenal azotemia-
A. due to obstruction of urine fl ow behind the k..idn eys
B. initia lly, the ratio is >1511 but may become <1511 if obstruction is not relieved and
tubular damage occurs
Iscbemic ATN:
I. prerenal azotemia MCC
2. aJfeets multiple pam ortbe ncphron -
A. basement membrane disrupted
B. pigmented renaJ tubular casts
C. thick ascending limb in medulla is mas!' sensitive to ischem ia
Nephrotoxic ATN:
I. amlnoglycosides and JVP dyes MCC
2. damages tbe proximal tubule-
A. basement membrane intact
B. better prognosiS than ischemic ATN

144
Laboralon' d ifferentiation 0 r oliguria:
Disorder FEN. UNa UOs m Ur inalysis
mE.IL mOsm~
Prerenal azolem ia <I <4 0 >500 Few hval ine casts.
Acule glomeruloneph· <I <40 >500 RaC casts. hematuria.
rit is
Acute IUbular necrosis >1 >40 <350 Renal tubular caslS. renal tubular cells.
Postrenal aUHcmin >1 >40 <350 FewWBCs rena l tubular cells. No casts.
. , . . . ..
I. FENa "" [Unne Na x Plasma creatmme] [Plasma Na x Unne creatinine] x 100
"T"

2. tubular funct ion inUlct:


A. FENa < I: loss of tubular funcli on leads to problems with sodium reabsorption
B. UN. <40
C. UOsm >5 00 (kidney is concentrating)
3. tubul ar dysfuncrion:
A. FEN. >1
B. UNa >40
C. UOsm <350 (kidney has lost it's concentrat ing abil ity) indicate lubular dysfunct ion
C hronic renal flli lure (CRF):
l. causcs in desccndlng order-
A. diabet ic nephropa.thy
8. hypertension
C. glomerulonephritis
(1) rapidly progress ive crescentic GN
(2) foca l nodular glomeru losc lerosis
2. pathopbysiology ofCRF-
A. loS! of concntration is first abnormality
8.

E.
F.
=
loss o f concentration and dilution: fLXed specific gravity of 1.010 indicating lack of
concentration and dilution

~: ~~:/:~~atfo:e < I :(~(;1> tllf.~


nonnocyt ic anemia due to loss of erythropoietin
vitamin 0 deficiency:
(I) loss of I a-hydroxylase enzyme and no second hydroxylation
(2) low calcium with normal to high phosphorous
(3) secondary hyperparathyroidism
G. renal osteodystrophy:
(1) osteoporosis (bone is a buffer for acidosis)
(2) oSleomnlac ia from vitamin 0 deficiency
(3) oste itis fibrosa cysticn from hyperparathyroidism and increased osteocillSlic
activity
H. increased bleeding time (platelet dysfun ction)
I. hemorrhagic pericarditis
J. non.cardiogenic pulmonary edema
Acute pyelooep brltis (uppu u rinary tract infection) in fem ales:
I. mecbanism -
A. ascend ing infection
B. vesicourereral reflu.",

145
C. E. coli Me pathogen
2. cliniclll-
A. fever
8. flank pain
C. WBC casts
D. lower urinary tract signs
E. danger of sepsis
C hronic pyelonephritis:
I. c.aU5eS -
A. MC secondary to vesicoureteral reflux
8. obstruction
2. U s haped cortical scan with undulyiog calyceal blunting
Acute drug·induced intentitial nephritis:
I. drug!l-
A. methici ll in is prototype drug
B. sulfonamides
C. NSAlDs
D. diuretics
2. mechonism- ~ IV cellular immunity
J. clinical-
A. abrupt onset of oliguria
8. fe ver
C. azotem ia
D. rash
E. eosinophilia
F. urinalysis:
(I) mild proteinuria
(2) hematuria
(3) WBC casts
(4) eosinopbiluria
4. IU- withdraw drug and do not use again
Analgesic nephropathy:
I. combination or aspirin + acctaminophen-
A. aspirin inhibits synthesis of renal prostag landins (vasodi lator) leaving angiotensin n
vasoconstrictive effects unopposed
8. acetaminophen FRs damage tubules in medulla
C. cumulative damage due to drugs
D. danger of renal papillary necrosis
2. otber causes or nnal papillary nccrosis-
A. diabetes mellitus
8. acute pyelonephritis
C. sickle cell trait/disease
J. s·ray fia.ding- fVP shows ring defect where papillae used to be
Urale aepbropalby:
I. eaURS-
A. Rx or disseminated neoplasms (e.g.• leukemia, lymphoma) : excess release of
purines and synthesis of uric acid. whic h blocks tubular lumens

146
B. lead poisoning:
<I) increases reabsorption of uric acid
(1) produces an interscitial nephritis
2. IU-
A~ allopurinol
8. alkalinization of urine
Myeloma kidney: mechanisms inc lude
1. Bence Jones prolein precipitales in tubule lumen- produces a forei gn body giant cell reaction
2. nephrocalcinosis due to hypercalcemia causes tubular dysfun ction
J. light chains toxic to tubular epithelium
Pb poiso nlne:
1. Iud produea proximaJ tubule damage-
A. proximal renal tubular ac idosis
O. possible Fanconi syndrome
( 1) proximal RTA
(1) Ilminoac:iduria
(3) uricosuria
(4) hypoglycemia
(3) hypophosphatemia
2. produe" Inlersthial nepbrilis-
A. increase in uric acid reabsorption
8 . urate nepllropathy
C. hypc:nmsion
D. gout
ephroscierosis:
.. kfdney of hypeMfD,loa-
A. conical surface cobblestoned: due to hyaline arteriolosclerosis leading to tubular
atrophy and glomerular sclerosis
8. kidneys small:
( I) proteinuria
(1) hematuria
2. cilnkal-
A. renal failure
B. potential for malignant hypertension
Malignant b)'perteosion:
t. develops In background of "sential byperteoJion and benl,n nepbroscierosiJ
2. grols/micro-
A. "net binen" kidney
B. nec:rocizing l11eT1olitis with fibrinoid necrosis in glomerular capillaries causes flea
binen appearance
C. onion skinning of arterioles (hyperplastic l11eriolosclerosis)
J. ellolul-
A. extremely high blood pressure
8 . cerebral edema with papilledema
C. progress to acu te renal failure
D. potential for intracerebral bleed
4. 1l9F P · w(U8Mll .

147
Most common si te to evaluate in a cbild witb bypcrttD!lion:
I. kid ntys-
A. Wilm 's tumor
B. cystic kidney disease
2. adre na l mtdu lla- neuroblastoma
Hyperplasia of J C a ppara tus witb increased blood pre ure:
I. renal artery ste nosis
2. usually due to athe rosclerosis of renal artrry in r ld erly man
l. fibromuscular hype rplasia of rnal artery in you ng womn
4. high renin bypertl'.Dsion
Kidneys with irregular wbitt pa tches on the co rtica l s urface: probable pale infarcts from
embol iUlion from the left heart
DiITust: cortical nec rosis :
I. pale Inrarction limited to renal co rtex
2. sce n In pre·cclampsill
Hydronephrosis:
I. MC due to a renal .stone
2. dUated ureter and renal pe"-i, with compress ion a trophy or renal cortC';(
3. MC COD in cervical cancer
" Ren lal sto ne :
I. see renal sYDdrom ts for pathogenesis at beginDing or thi section
2. calci um oxalate MC stone-
A. hypercalciuria Me metabolic abnonnality
B. hypercalciuria due to excess reabsorption of calcium from gut
3. clinical-
A. colicky flank pain with radiation into ipsilateral groin
B. hematuria
C. spiral CT is best initial screening test
D. routine x-ray often identifies calcium st'o nes
4. IU-
A. increase water intake
B. hydrochlorothiazide increases ca lcium reabsorption
Angio myollpo ma:
l. hDmartoma of kidneys
2. most commonly 8S!otiated with tuberous scleros is
Reo. 1 adenocarcinoma:
1. c:.auus-
A. smoking MCC
8. von Hippel.Lindau disease
2. gross/micro-
A. hemorrhagic mass >3 em: size det'ennines maJignam potential
B. derives from proximal renal tubule: clear cells
C. tendency for renal vein invasion

148
3. clinical-
A. triad of:
(\ ) nank mass
(2) hematuria : Me
(3) pain
B. rClopic secretion:
( 1) erythropoietin: secondary polycythemia
(2) parathonnone like peptide: hypercalcemia with low PTH
C. metastasis
( \ ) lungs; MC site
a. "cannon ball "
b. hemo rrhagic
(1) bone (lytic)
(3) skin
WUm', tumor:
I. common childhood cancer
2. derh'cs from mesonephric mcsodcrm-
A. some cases are AD
( I) relatio nship with chromosome 11
(2) aniridia and hemihypertrophy in genetic type
B. abortive glomeruli
C. primitive cells
O. rhabdomyoblasts noted
3. clinlca l-
A. hypertension due 10 renin secretion
.B. unilateral palpable mass
C. hematuria
D. pain
U~~ or~: faulty c losure of urethral folds
,
l/$MLI:: pallo _ _ or epllpadlu:
1. defect in geaital tubercle
2. aJsociated witb eUlropby of bladder
USMJ::E:_ ........ fro [Dj~___ : persistent umbil ica l (vitelli ne) sinus
Urine draining from umbilicus In 4 day old: persistent urachal sinus
Acute eyslitis:
I. E. co/l MCC
2. ascending Infection
3. afebrile patient with incrtaRd frequency and d ysuria
4. urinalysis-
A. positive dipstick for nitritesl1eukocyte esterase
B. pyuria
C. bacteria
D. hematuria
S. IU-
A. TMPISMX
8. ampicillin
6. hemorrhagic cystitis often due to adenovirus

149
Sltrlle pyuria (Wacs in tbe urine) and negative cuhure (sha ndard cult.ure) aJter 24 hours :
I. a lways tbink renal TO
2. could also be Chlamydia trac.homatis- non'specific urethritis
Complications or cyclopbospbamide:
I. hemorrbagic cystitis
2. traD ilio nal ca.n:inoma or the bladder
Di5cast:S where E. coli is MCC:
I. uppernower UTI
2. J(,p is in hospitalized patienu
J. s pontaneous peritonitis in adu lts wilh ascites
.... acute appendicilis in adulu
S. acute cholecystitis in adults
6. Irln'elcr's diarrbca- emerotoxigenic stroin
7. hemolytic uremic synd rome- enterohemorrhngic stroln with 0 157:1-17 serotype
Trllnsilional ccll carcinoma or bladder:
I. CQuses-
A. s moking MCC
8 . anil ine dyes
C. phenacetin
D. cyclophosphamide
E. benzidine
1. dlnical-
A. pajnless hematuria
B. usually papillary tumors
C. tendency for ~c:urrence and mullifoc:ality
Bladder cancer assoc:ialed with parasilic d bell.!ie:
I. squamous cancer or bladder
2. Schistosoma ht!matobium- egg hl15 a nipple at the end
Epididymitis: ..
I. <JS y. old-
A. GC
B. Chlamydia trachomorls
2. >35 ys old-
A. E. coli
8. Pseudomonas oeruginosa
Varicocele:
I. lerr slde-
A. "bag ofwonns "
8. spermatic vein comes off the left renal vein
2. nr')' common cawe oriofertility

Gross of tonion or tbt testicle:


1. testicle bas a bluisb black tolol",
2. abstnl cl"emuterie rdln
J. testicle dnlwn up into tbe inguinal canal
.... surgery Impenltive

150
r:r Orchitis:
t. mumps-
A. infeniliry uncommon
B. most are unilateral
C. o lder child or adult
..3.,syp hilis
HJV
Hydrocele:
I. scrotal mass tbat tra nsilluminates
2. persis tent tunica vaginalis
Prostatic hyperplasia :
I. develops in transitional zone around tbe uretbra- prostate cancer is in peripheral zone
(outside) and detected by rectal exam
2. dlbydroteslos terone and estrogen mediated
3. clinical-
A. dribbling
B. urinary retention is more likely benign than malignant
C. PSA does nOI distinguish hy perplasia fro m cnncer
D. PSA is nO[ increased after a rec ta l exam
Prostllie cancer:
I. age Is greatest risk factor
A. increased risk with first-degree relative
B. smoking
C. Me cancer in men
D. second MC cancer killer in men
2. dibydrotestosteronc-mediated
3. dinical-
A. detected by rectal exam
B. perineural spread and extension into semina l \lesicles is common
C. screening lest is recla l exam + PSA
D. confirmatory [est is transrectal ultrasound with Bx
E. osteoblastic metastasis with elevated alkaline phosphatase
<r ~sM~~''~iI~a~.i'Iy~ IIiliIi Willi loW baOlL paln:
I. 'tid • recti.t H.l1II to RIO proit.te.-eaMer. before a"y other test
2. if.lIaey.:atate.---tJaat tbe Ie""" alkaline pbosptiatase.iI elevated. iti! ·osteobUtatie-mehlsf1I"is
r:r Testicular cancers:
I. painless mass that does not transilluminate
2. se minoma MC testicular cancer-
A. cryptorch id relationship : risk extends to nonnaltesticle as well
B. most radiosensitive
C. para-aortic lymph node metastasis
3. cboriocar-cinoma most malignant-
A. increased p-hCG from syncytiotrophoblast
B. may produce gynecomastia
C. lung metastasis
4. yolk sac tumor Me In thildren-
A. endodennal sinu s tumor

15 1
B. Schiller DU\lal bodies
C. Incrused AFP
5. maligaaol tympboma Me in c.lderly m.1IO (mefuillic)
6. Itraloc::arciaoma Me assoc.iated witb increue 10 .FP aad ~ b CG: embryonal carctnonu
+ teratoma
Peals squamous caDcer: causes include
I. 8o"'cn', disease
2. erythroplasia ofQueyrat
J. lack of circumcision
Pure semlairerous tubule dysruactioo:
I. PO .perm
2. laern.sed FSEI - loss of inhibin synthesized in Senoll ce ll
J. Donaal LH-
A. acts on Leydig cells
8. norma l testosterone
rurt' Leydig cell dysfunction :
I. no s pcrm - testosterone also necessary for spennatogenesis
2. norma l FSR
J. higb LH- low testosterone
octum " penile erections: presence RIO any org1U1ic cause fo r impotence

r:r All 82 year old man presents with lo",er back pain and complaints of problems with ,"oiding urine.
Th~e is plinl tenderness over the lower lumbar venebra. His bladder is percussed at the le\'eJ of
the umbilicus. Whicb of the following tests or procedures is indicated as the first step in the
management or this patient?
A. Radionuclide bone scan
B. Prostate specific antigen
C. Digital recta l examination
O. Serum aJkaline phosphatase
E. TransreclDl ultrasound with biopsy
C (metlt5UHic prostate cancer)
or A 28-yeo.r-o ld man with a history of remova l ofa right cryptorchid testis o.s B child is noted to have
a po.inless en largement of the remaining testicle in the left scrota l sac. The testicle is enlarged and
docs not transilluminate. The patient most likely hu I ...
A. hydrocele
8. seminoma
C. varicocele
D yolk sac tumor
E. c.horiocarcinoma
B

152
A 66 year old man with a 45 pack year history of smoking presents with hematuria. fever, and a
palpable mass in the left lower quadrant. A chest x·ray reveals multiple nodular masses in the
lungs. These findings most strongly suggest which of the followin g diagnoses?
A. Miliary TB in\'olving the kidneys
B. Renal cell carcinoma with lung metaStasis
C. Primary lung cancer with metastasis to the kidney
D. Choriocartinoma with metastasis to lungs and kidneys
E. Acute pyelonephritis with metastatic abscesses in the lung
B
a- A febrile 23 year old woman presents with an ncute onset of right flank pain. supropubic
discomfort, dysuria, and increased frequency of urination. The urinary sediment examination
reveals clumps of leukoc)1es, wee casts. occasional RBCs, and numerous motile bacteria. The
mechanism orthis patient'S urinary condition is most closely related 10 •••
A. It renal stone
B. ascending infection
C. immunocomplcx disease
D. drug· induced interstitia l nephritis
E. hemAtogenous spread o f infection to the kidneys
B (acute pyelonephritis)
<7 A 25 year old ma le presents with a history of hemoptysis and malaise. His blood pressure is
140190 mm HG. He has periorbital edema and smoky colored urine. Pettinent laboratory data
include a serum urea nitrogen of 40 mgldL (7-18 mgldL) and a creatinine of 4.0 mgldL (0.6-1 .2
mgldL). rinalysis sho\ys 2+ protein. 3+ blood.. RBCs too numerous to count, and RBe casts. A
chest radiograph demonstrates opacities in both lung fields. The mechanism of this patient'S lung
and renal disease: is most closely related 10 a ...
A. IYpt I hypersensitivity reaction
B. type. II hypersensitivity reaction
C. type: IU hypersensitivity reaction
D. type IV hypersensitivity reaction
E. type n and III hyperStnsiti\'ityoreaction
B (GoodpaSiure)
Items 6-7
A. IgA glomeru loneph ritis
B. Membranous glomerulonephri tis
C. Foca l segmental glomeru losc lerosis
D. Rapidly progressive crescentic glomeru lonephriti s
E. Type I membranopro liferati\,e glomeru lonephritis type I
r:r A 14 year old man with D 80 pack history of cigarette smoking presents with hemoptysis and
shortness of breath. Radiographs of the chest demonstrate a left hilar mass. He also has generalized
anasarca with 4+ proteinuria, hypen::holeslerolemia, and fany casu in the urine.
Answer. B (usociation of lung cancer with membranous GN)
cr A 34 year old maJe who is a known drug seeker and heroin addict comes to the emergency
department in an agitated state. He is restrained and you note multiple needle tracks on both arms.
He also has severe pining edema of the lower extremities. Urina lysis is positive for protein and
shows fatty casts on urine sediment.
Answer: C

153
qr A 10 yenr-old-ndo lesccnt boy presents w ith a unila teral. te nder subareolar mass in the left breast
Physical exam is otherwise unremarkable. Which of the following would you recommend for this
p:l llent'l
A Serum gonadotropins
B. Chromosome analysis
C. Smlm p-hCG
o Surgical consult
E. No lreatmeOl
E (pubertal g)necomastia)

15-'
Female reproductive, breast:

r:r Sex ually Ira nsmll1ed diseases:


I. Berpe.s\·jrus ty pe 2-
A. genital herpes Me due to herpesvirus. type 2 (HSV 2): scxu311y transmitted
B. primary infection:
(1) syslemic signs and symptoms
(2) painful vesides that ulcerate
(3) vesicles located on penis
(4) lesions on labia. vulva. cervix: erosive cervicitis with discharge
(5) proclitis: anal intercourse
(6) vesicles heal within 3 wks and may recur every 4-6 wks
(7) most infect ious phase for congenital infections
C. recurrenl herpes without systemic signs or symptoms
D. Tunck preparation
( I ) scrapings at base of vesicles
(2) muhinucleated squamous ce lls with intranuclear inclusions
E. R,,,
( 1) acyc lovir decreases the frequency of recurrences
(2) no cure
F. pregnancy:
( I> viml shedding may occur without visible lesions
(2) presence of viral sheddi ng. babies must be delivered by Cesarean section
1. buma n papiUomninlJ (RPV)-
A. types 6 and 11 associated with condyloma acuminata:
(1) venereal warts
(2) located in moist areas in the anogeniLaI region and on cervix
(3) fem~like appearance
(4) confused with condyloma larum in secondary syphilis
(5) kolloc:ytic cbaDce-in squamous epithelium: wrinkled pyknotic nuclei surrounded
by. halo
(6) Rx: lopica l podophyllin
B. HPVtypeJ 16 (most common), 18 and3!
C. olher HPV squamous cancer associations:
(1) cervical
(2) vu lvar
(3) vagina l
(4) anal squamous ce.11 carcinoma (homosexua l ma les)
3. Clrlamydia trachomatls-
A. MCC of sexually transmitted disease (STD) in males/fema les: Chlamydia and GC
infections frequen tly coexist
8. males:
(1) nonspecific urethritis (NSU)
a. incubation period is 2-3 weeks post.sexual exposure
b. dysuria
c. thin wateT)' exudate
d. numerous neutrOphils without bacteria on gram stain
t. sterile pyuria

155
C. R.'IC :
• single dose erythromyc in
• doxycycline
(2) epididymitis in men <35 yellrs o f age
C. Chlamydial urethritis in women:
( I) acute uretbral syndrome
(2) muco purulent cervicitis:
A. source for conjunctivitis in newborns
b. MCC of ophthalmia neonatorum
c. erythromycin eye drops have reduced this complication: usually occurs
after 1 wk
d. cervical Pap smear: Chlamydia produce red inclus ions in endocervical
cells
(3) 50% ofPID due to Chlamydia trachomalls
4. lymphogranu loma veoereum (LGV)-
A. due to Chlamydia trachomalis subspecies
B. tiny papules in the gen ital region associated:
(1) reactive lymphadenitis with granulomatous microabscesses and multiple draining
sinuses
(2) fibrous stricture formation :
a. localized lymphedema of scrotum/vulva
b. women may have rectal stric tures
C. R."t.: doxycycline
S. Neisseria gono"hoeoe (GC)-
A. gram negative diplococcus
B. women more likely to be asymptomatic carriers than men
( 1) men more likely to transm it to women after each sexua l exposure
(2) drug resistance:
a. chromosomal mUlations
b. f;l-Iac lamase production (plnsmid medislcd)
C. clinical presentation in men:
( I) GC urethritis Me pre.sentation
(2) creamy, purulenl penile exudale w ithin 2- 5 d of exposure
(3) stricture
(4) epididymitis
(5) proctitis (homosexua ls)
(6) prostatitis
(7) sterility
D. Rx in men:
(I> Rx with ceftria.'lCone
(2) spectinomycin for Rx failures
E. clinical presentation in women:
<I) GC primarily infects glandular epithelium
a. estrogen protects squamous epithelium from bacterial adherence
b. prepubescent female lacking estrogen rna)' have vulvovaginitis
(2) urethritis (dysuria)
(3) cervicitis:
a. MCsile
b. cervical discharge

156
(4) Bartholin gland abscess
(5) PID:
a. during or shortly after menses
b. lower abdominal pain
c. adnexal tenderness on movement of tile cervix (chandelier sign )
d. Fever
e. elevated neutrophil count
F. perihepatitis results in Filz· Hugh~ C urii s syndrome :
• adhesions
• RUQ pain
g. R." with ceftriaxone (for GC) and doxycycline (Chlamydia)
(6) complications of GC:
a. sterility
b. ectopic pregnancy from tuba l scarring (MCC of ectopic)
c. disseminated gonococccmia;
• dermatitis
• tenosynovitis
• septic arthritis
(7) birth control pills protect against GC but nOI Chlamydia
f. other GC in fcclion s:
(I ) pharyngitis (result of fe llatio)
(2) proctitis (annl intercourse): recurrent disease un less patient with GC proctiti s is
treated (t,lSMLE)
(3) ophthalmia neonatorum : bilateral conjunctivit is in first week '(JL8MLE)
6. Hemophilus ducreyi-
A. chancroid caused by the gram negative rod : gram stain shows a classic "school of fish"
orientation of the bacteria
B. painful genital and perianal ulcers: suppurative inguinal nodes
C. R."<: ceftriaxone or azithromycin or eryth romycin
7. Ca~vmmalobacler;um (Don9vQIl IS bacillus) grallulom alis-
A. granuloma inguina le is caused by a gram negative coccobac ill us:
B. organism is encapsu lated : phagocytized by macropbages (Donovan bodies)
C. creeping, raised, sore that hea ls by scarring
D. no lymphadenopathy
E R.... : doxycycline
8. Gardlll!rella vaginalis-
A. bacterial vaginosis is caused by gram negative rod Gardnerella vagillalis
B. often associated with other anaerobes like Mohi/llllCUS and some Bacleroides species
C. does not produce an inflammatory ex.udate (no neUi rophils or erythema)
D. common ly noted when vagina l pH between 5-5.5: reduction in Lac/obaeflli
E. Gardnerella adheres to surface of squamous cel ls: characteristic " clu e cells"
F. potass ium hydroxide added to discharge produces a fishy, amine· like smell
G. Rx:
( I) R.x patient (not the panner) with metTonidazole
(2) avo id merronidazole in fi rst tri mester

157
9. syphilis-
A. caused by the spirochete Trepon ema pallidflm:
(1) primarily contracted by sexual Contllct
(2) organism produces a vascu litis called ~ndarteriti s ob literans:
II. localized ischemic necrosis
b. heavy plasma cell infiltrate
B. primary syphilis:
( I) so litary painless, indurated chancre
(2) 3-4 weeks aft:er exposure
(3) shaft of penis
(4) labia
(5) chanc re persists for \-5 wks: spontaneous ly resolves
C. secondaf)' syphilis:
(I) occurs 6 wks to 6 mths later
(2) most contagious stage of syphilis
(3) diffuse macu lopapu lar rash: commonly involves palms and so les
(4) generali zed lymphadenopathy
(5) condyloma latum :
8. raised white les ions
b. located around moist areas of anogenital region
(6) pericholangitis
(7) meningitis
(8) nephrotic syndrome
(9) lesions heal in 4-12 wks
( 10) djsease may enter latent phase
O. latent syphilis:
(1) usually asympto matic
(2) positive serology
(3) considered non infective when duration >4 ys:
a. exception is pregnancy
b. disease can be transmitted to fetus
(4) -one·th ird of patients left untreated with syphilis will enter tertiary phase
E. lertiury syphilis:
( 1) eNS disease (,MC): see eNS nOles
(2) locally destructive disease due to gummas
(3) cardiovascular disease:
8. aortic arch aneurysm
b. see cardiovascular nores
F. congenitalsypbilis; sec genetics notes
G. dark field microscopy:
(1) considered gold standard for primary and secondary syphi lis
(2) direct fluorescent tests just as accurate
U. nonspecific serologic 'os .. ,
(1) reagin an('ibodies react with cardiolipin antigens fTom beef hem: basis for the
RPR (Rapid Plasma Reagin) and VORL (Venereal Disease Research Laboratory)
tests
(2) sens itivity 75% in primary syphilis, 99% in secondary syphilis, 70% in latent
syphilis, - 50% in tertiary syphi lis:
a. fal se positives in SLE due to anticardiol ipin antibodies
b. false negatives in secondary syphilis due to antibody excess (prozone
phenomenon): must dilute serum unt il all agglutination react ion occ urs
(zone of equivalence)
(3) RPRlVDRL become nonreactive after I yt in primary. 2 yrs in secondary, 2-5
yrs in latent syphi lis
(4) VDRL test ofchaice on CSF in diagnosi ng neurosyphilis
1. specific lests for syphilis:
(1) Fluorescent treponeme antibody-absorprion test (FTA-ASS :
P. treponemai l'est with highest sensitivity/specificity for Dx of syphilis in all
stages
b. remains positive after R.'"(
c. positive FTA-ABS either represents active or inactive syphilis
(2) microhemagglutination-Treponema pallidum (MHA-TP): similar in sensit iviry!
specificity to FTA-ASS
J. R..t:
(1) syphilis is only bacterial STD that has not changed its an tibiotic susceplihiliry:
(2) exqui sitely sensitive to penicil lin
(3) Jarisch-Herxheimer reaction (not a penicillin allergy) commonly occurs with in
n few hs of Rx of secondary syphi lis:
B. fever
b. headache
c. intensification of rash
LO. Candida alhicans-
A. Candida vagi nitis:
(1) causes;
a. uncommonly transmirt'ed sexual ly
b. diabetes mellitus
c. systemic antibiotics
d. pregnancy
(2) intensely pruritic vaginitis with a "conage cheese" discharge
(3) KOH preparation -revea ls budding yeasts with pseudohypha
(4) Rx: fluconazole single dose
II. Trichomonas vagillalis-
A. flagellated protozoan
(1) locates in the urethra (males and females), prostate. seminal vesicles, vagina
(2) pear shaped
B. vaginitis:
(1) intense pruritu s
(2) strawberry colored mucosa
(3) leukorrhea (discharge)
(4) urethritis
(5) tumbling motility or trophozoites in a hanging drop prep of discharge
(6) Rx:
a. patient/partner Rx. with metronidazole
b. metronidazole contraind icated in first (rimester
Qr Condom: best protection against syphilis and HIV~)

159
Vulvar leukoplakia :
1. possi ble precurso r for squa mous cancer or P2gcr's disease
2. d o Ox
Licben scl erosis elalrophicus:
I. atrophy of epidermis of vulva
2. pll rcb menl like a ppearance of s kin
3. no risk for squam ous carcinoma
Squnmous hype rplas ia of ,'ulva:
1. precu r'Jo r for squamous cancer
2. leukoplllkic lesion
Sq uamous cancer of vuh's :
l. Me cllncer of vulva
2. labia majora Me localioo
3. HPV relationship
Vulvar Pugel's disuse :
I. lotraepithelial adenocarcinoma
2. confused with mnlignllDt mela noma
Gartner's duct cyst:
I. remnant of wolman duct
2. cyst on lateral wall of vagina
Young gi rl with grape- like masses protruding from the v"gina:
I. embryonal rbabdomyosa rcoma-
2. Me sarcoma in childr
3. nut rellued to DES
Clear cell adenocarcinoma of vagina:
l. associated witb DES exposure during preg nancy
2. vaginal adenosis is tbe precunor-
A. located 8t junction of MOilerian epithelium with that of urogenital sinus
>
D. DES inhibits MUllerian differentiation :
( 1) tubes, uterus. cervix., upper one-third of vagina are all affected
(2) cervical stenos is and incompetence lends to repeated abonions
3. receDt VSMLE lCea.rio: woman with multiple spontaneous abon ions
A. showed A picture of endometrial biopsy: probably showed clear ce ll s
B. probable clear cell adenocarcinoma- not gonorrhea. endomelriDI cancer, or HPV
Cervical Pap !imear:
l. mus t be endocervical cells (go ld standard)- indicates proper sampling of the endocervical
canal where dysplasia occ urs
2. s uperficial squamOU!i cells- indicate adequate estrogen
J. intermediate squa mous cclls- indicate adequate progesterone
4. pllrabasal cel1s- honnonally unstimulated squamous cells
S. norm al non-pregnant adult woman-
A. 70% superficial
O. )0% intennediate
6. pregnllncy/ prepubertal- 100% intennediates from progesterone effect
7. elderly womQn nol on eslrogen- atrophic smear with predominantly parabasals
8. woman on uno pposed estrogcn- 100% superficial

160
Cervical polyp:
1. non- neoplas d c
2. ha ngs out of cervical os
3. postcoital bleedi ng
Nllbotbilln cysts: blocked endocervical glands with mucous retention tU ~

Koilocyt osls:
I. UP" effect in squamous cells
2. pyknolic nu cle us s urround ed by a clea r ba lo
Ce n !ica l intnepitbelia l neoplasia (ell',):
l. rela ted to I:lP V
2. em 1- mild dysplasia that involves lower one/third
J. em IJ- moderate dysplasia that involves lower [wo thirds
4. CrN U I- SC\fcre dysplasia to carcinoma in situ that is full thickness dysplasia
5. requires 10 yrs to move from C IN I to CIN In
6. requ ires an oth e r 10 yrs ( 0 move from CIN m to in \'llsive coll cer
Cervical squllm ous cuncer:
1. c.a USC!I-
A, HPY 16, 18: early onset of sexual activity with mul tiple partners increases exposure
risk
B. smoking
C. birth control pills
D. immunosuppression
2. de\'elops In t ra ns form a tion zone
J. death due to r eaal fa il ure from ext ension into the ret ropenlonf um and blockage of
urelen
4. average age -4S ys old
Gynecology canccn by d ecreasing incidence: endometrial > ovarian :> cervica l
Gynecology ca ncen by decreasing mortality: ovarian > cervica l > endometrial
,
Gy necology C8.0cers by a ge: cervical 45, endometria l 55, ovarian 65
Seq uence to mena rc he:
I. breast budd lng-+
2. growth spu r l-t
3. pubic hair-+
4. axillary hllir
5. me narche (mea n aee 12.8 ys)
Proliferativc pbue:
L H l rogc n media ted proliferation of glllnds
2. estrogen su rge 24- 36 bs pnor to ovulation has positive feed back of L H a nd negath'c
re~ b .ck on rSH-+
A. LH surgc-+
B. ovulation
Ovulation:
I. usually on day 14
2. indica Ion of ovula tion-
A. increase in temperalure due {'o progesterone

161
8. subnuclear \'acuoles in cells
C, feming of cervical mucus
D. presence of secretory endometrium on day 21 confinns ovulation has occ urred
E. mittelschmerz may occur: blood from ruptured follicle causes peritoneal irritation
3. birth control piUs-
A. mixture of estrOgen + progestins
8. baseline levels of estroge n prevent midcycle estrogen surge preventing LH surge
C. progeslins arrest proliferalivc phase and lend to gland atrophy
D, progestins inhibit LH
E. cervical mucus is hostile to sperm
F. alters fall opian tube motility
G. progeslins produce lipid abnormalities
Secr etof)' pbll!'le:
I. progeslerone- medlaled-
A. increase in gland tortuosity and secretion
D. development of pseudodecidua
C. most fixed stage of cycle
2. Arilol5-Stella (exagge rated secretory phlolsc) indicates pregnancy
Menses:
l. initiated by drop off in estroge.n and progestero ne
2. plasmin pre\'cnl! menstrual blood from clolting- excessive clots ind icates menorrhagia
Impla nta tion on day 21 :
1. 3 days in the tube
2. 2 day, in uterus
Functions of FSH:
I. prepare follide or month
2. Incre.ase aromatase syntbesis in granulosa celt.
3. synthesis of LH receptors
FUDctions of LH :
1. proUrerative ph ase-
A. increase synthesis of 17-ketosteroids (OHEA and androstenedione) in theca intema
8. DHEA converted into androstenedione
C. androstenedione converted into testosterone
D. testosterone enters granu losa cel ls and is aromalized into estradiol
2. LH surge- induces ovulation
3. sec retory pbase- theca intema primarily synthes izes 17-hydroxyprogesterone
Estrogens:
l. estradiol-
A. main estrogen in non-pregnant woman
8. derived from aromal'ization of testosterone in granulosa cells of developing follicle
2. estro ne- weak cstrogen in menopause derived from adipose cell arornatization of
androstenedione from adrenal
3. estriol-
A. end-product of estradiol
B. primary estrogen of pregnancy

162
Androgens Ip women :
l. aodrostenedlone-
A. SO'/. ovarian
8. 50% adrenal conex
2. DHEA-
A. So-.I. adrena l
8. 20% ovol')'
J. DHEA- uUate
A. 95"_ :1drenal eonex
8. best test (or adrenal origin orhiBUlism/virilization
4. ltstoste.roae-
A. 50% (rom peripheral conversion of androstenedione into testosterone (o~idort:ductase
re3ction)
B. 25°;' in ovaries
C. 25% in adrenal COrtex
Su hormone bindinG Ilobulio (alias androgen blndinl globulin):
1. hindlul proleln ror tescosterone and estrogen
2. blgber amnlty ror tcstolteroae
3. higb leveb of SElBG decrease rree testosteront le\'eb
4. syntbesized in liver 10 women aod liver aDd Sertoll cells In men
S. Htl"OCto and chyroid bormone increases srotbesis of SHBG: reason (or gynecomastia in
men with hypenhyroidism
6. aadroafDJ aad insulin decrease syothesis ofSHBG
7. obesity aod bypotbyroidism bave decreased SBBG: reason for hiButism in these
condit ions
Menopause:
1. DlHO ace 51 yrs
2. elevated IODadot'r opiJu- FSH best marker
3. dinkaJ -
A. secondal')' amenorrhea-
O. hot nushes
C. decreased vagina l secret ions
D. nighl sweats
4. Ib- estrogen
Menstrual dysrunction :
1. ml!Dorrbasla-
A. loss >80 mL per period
B. MeC o( iron deficiency in women
2. mel:rorrbaeJa- interval bleeding
3. olieome.uorrhu-
A. infrequent bleeding
B. polycyslie ovarian syndrome
4. dys muorrbta-
A. painfu l bleeding
B. primary t)-pe due to prostaglandin Flo
C. Clrboprost tromttbamint' is anaJogue of PGF2a that increases uterine conU'llCtiJil)
(I) abortifacient

163
(2) contro l postpartUm bleeding
D. secondary rype Me due to endometriosis
DydunctioDal uteri ne bleedin g:
I. bleedin g unrelated to an a natomic cause- hormonal imbalance
2. anoyulalory bleeding-
A. MeC of bleeding after menarche
B. estrogen increases mucosal thickness and then tissue sloughs
C. inadequate amounts of progesterone
D. Rx with progesterone or birth control pills
l. ovula tory typH-
A. inadequate luteal phase: poorly deve loped secretory endometrium
B. irregular shedding of endometrium: persistent luteal phase with mixed
proliferative/secretory glands in menstrual effluen t
Primary amenorrhea:
I. definillons-
A. no men ses by 14 ys of agc in an adolescent with poor secondary sex characteristics
B. no mt:n ses by 16 ys of age in an adolescent with good secondo.ry sex characteristics
2. no blet:dlng pos t adm iuistratio n of progesterunc- . - fRt'1~~e c (,.//PIUJ! ir
A. indicau:s that me uterus is not estrogen primed. or _ ,.f' .b/~'~ ~'"
8. end-organ defect prohibiting exit of the menstrual blood !'oys-k.irr()fl !ri'tt.
3. hypoth alamic/pituitary ddecl- (j;A.,/-1 ~~
IL decreased FSHlLH: ;ojdirAj) -k
8 hypogonadotropic hypogonadism "E" ~
C: causes: C;), ,"e". IVCI'i'~ I
( I ) anorexia nervosa or weight loss syndrome
(2) pituitary tumor
(3) K.ollman syndrome:
a. absent GnRH
b. anosmia and color blindness
(4) const itutional delay
4. ovarian defect In synthesis of estrogen-
A. increased FSHlLH:
8. hYJ)C'rgonadot"ropic hypogonadi sm
C. probable Turner'S syndrome
(1) Me genetic cause of primary amenorrhea
(2) poor secondary sex characteristics,
S. cnd-organ defect-
A. nonnal FSWLH
B. imperforate hymen
C. RokitnnsJ..'Y·Kuster-Hauser: no vagina
c:r Secondary amenorrhea :
1. ddinition- no menses for 3 mths
2. prrgnancy MCC
3. bypogoaadotropic bypogonad ism-
A. prolactinoma
8. anorexia nervosalweight loss syndrome
C. Stress

164
D. hypopituitari sm
4. hypergonadolropic bypogoDlldillm-
A. surgical removal of ovaries
B. autoimmune disease
S. cnd-organ di.5ease-
A. Asherman syndrome
B. removal of stratum basalis owing 10 repeafed dilatation and curettage
Endometritis:
I. plasma cells prescnt
2. causcs-
A. group B streptococcus (SrrepfOCOCClIS agalactia/!) in pregnanc),
B. Neisser ia gonorrhoeae
C. rUD with Actinomyces israeli
Endometrial polyp:
1. no malignant potential
2. menorrhagia
End ometrial hyperplas ia:
I. precursor for endometrial cancer
2. simple and complex types
3. due to unopposed estrogen-
A. taking estrogen without progesterone
8. obesity
C. polycystic ovarinn syndrome
Adenomyosis:
I. glands and st.roma in myometrium-
A. not endometriosis
B. coarse, th ickened myometrial tissue
2. c1inical-
A. menorrhag ia
8. pelvic pain
C. dysmeno rrhea
Endometriosis: &k. I( f',4;.N+ l.f
I. glands and stroma outside confines of tbe uterus- 1.,(,,' /• ./I «<q+;N
A. Me due to reverse menses
B. metaplasia
C. hcmatogenousl1ymphohematogenous spread
- . y.,SI -1/]
D. only occurs in reproductive life ~J)Jv l'\ e+~:~-:.i
2. clinical-
A. MeC of secondary dysmenorrhea
B. ovaries most often invo lved (chocolate cysts)
C. induratio n in pouch of Douglas
D. dyspareuni a
E. intestinal obstruction
3. luparo5cope best for Ox and Rx
r:r Endometrial carcinoma:
1. due to unopposed estrogen-
A. w oman on unopposed estrogen

165
B. early menarche/late menopause
C. nulliparity
O. polycystic ovarian syndrome
E. obesity
F. !-Lx of breast cancer
2. ble.eding to postmenopausal woman
3. MC gynecologic cancer witb best prognos is: spreads down into endoc,ervix and QU! into
the wall of uterus
Leiomyoma:
1. MC benign tumor in womeo-
A. sometimes caUed "fibroids"
B. Me tumor ofG I tract: Me in stomach
2. clinical-
A. cause of menorrhagia
B. obstructive delivery
Leiomyosarcoma: Me uterine sarcoma
Ectopic pregnancy:
l. MCC is previous PID
2. rupture Me COD in early pregnancy
3. (l--hCG best iniriallest-
A. vaginal ultrasound to check for amniotic sac
B. unc loned blood in pouch of Douglas
4. sudden onset of lower abdominal pain
Polycysdc ovarian syndrome:
I. patbogen ~is-
A. increased LH stimulates ovary to produce testosterone and 17-ketosteroids (androgens
leading to hirsutism)-t>
B. increased adipose aromari2es androgens to estrogens (endometrial
hyperplasialcancer}-io
,
C. estrogen inhibits FSH and enhances LH release-+
D. continued cyc le ofLH slimutation-t>
E. lack ofFSH causes atresia of follicles and large ovaries with subcortica l cyslS
2. cllnical-
A. obes ity
B. hirsutism
C. irregular menses
D. infertil ity
E. endometrial carcinoma
3. lab--
A. . LWFSH ratio > 3/ 1
B. inc reased 17-ketosteroids
C. increased free testosterone
D. increased weak estrogens
4. IU-
A. ~ birth control pills suppress LH
B. ~ clomiphene if patient wants to become pregnant
CiI'" Follicular cyst : Me overa ll ovarian mass

166
r:,- Ovarian tumors:
I. mOJt o\'anan tumon a re be nign In women <-IS ys old- serous cystadenoma Me type
2. moSI are malignant in women > 45 )'5 old-
A. serous cystade nocarc inoma Me type
B. should nOl bt' able to fee l an o\'ary in postmenop<lusn l women (presumptive c\'idencc
of cancer)
3. risk faclor'S -
A. increased ovu lation: binh control pills decrease incidence o f ovarian cancer
B. hereditary ovarian cancer syndromes: c.g.• Lynch syndrome
C. Turner's syndrome: dysgenninoma
~. most malignant fumors spread by sceding- feci induration in pouch of Douglas
" Su rface dcrind ovarian tumors:
I. bcnign-
A. serous cystadenoma with cysts lined by cilia: f 5ltIt1O(fJct A,,/. t"S
(I ) Me overall benign tumor
(2) M e bilateral benign tumor
B. mucinous cys tadenoma: cys ts lined by mucous secreting ce lls
C Brenner's tumor; Walthard 's rests
2. malignanl- classification scheme abo lists borderline types
A. serous cystadenocarcinoma:
( I) MC primary cancer of ova ry
(2) MC bilateral ovarian (umor
(3) Me ovarian cancu wilh psammoma bodies
B. mucJaow cystadenocarcinoma:
( I ) largest ovarian tumor
(2) most often associated with pseudomYJoma perltonei
C. endotnetrioid carcinoma:
(1) grentesl llSSOCiation with endometriosis
(2) greatest 1lS5ociation with endometrial carcinoma
Ge rm celli umors of ovary: '
1. cystic leratoma Me ~~}.L-
A. undergoes torsion in adolescent
B. x-ray shows ca lcifications
C. struma ovarii is II teratoma wi th a component of fu nctional thyroid tissue
2, dysgerminoma MC malignanl tumor- association with Turne r syndrome
3. yolk sac (uma r -
A. Me rumor in girl
B. increased AFP
ex cord , t'TOma) tumors:
1. fibroma Me rype-
A.~ benign tumor
B. MOii.,....",m. (USMLt:).
(1) -" fibroma
(2}-+ asc itcs
(3)-- right sided pleuraJ effusion

\67
2. gra nulosa cell tumor- (ct.".,-'•."' :> ~ (Q'Y'Jq ;, ~ )
A. feminizing tumor
B. low grade malignancy
C. Call Exner bodies
3. tbecomas-
A. benign estrogen se<: reting tumors
B. yellow color
4. Sertoli Leydig cell tumor-
A. benign masculin izing tumor
B. alias androblastoma or arrhenoblastoma
5. Leydig cell t'umor- ( fl1' Jc.. o;" "TP.::fos+el!.vn",,)
A. benign masculinizing rumor
8 . alias hilar cell tumOr
C. contains crystals or Reinke
6. gonadoblastoma-
A. germ ce ll and sex cord stromal tumor combined
B. ca lcifies
C. assoc iation with Turner' s
<if" ~... laJDor(QSMLE): I
1. hematogenous s pread of gastric cancer 10 both ovaries
2. signet ring cells
r::r C horionic villus/umbilical co rd :
1. lined by trophoblastic tissue-
A. syncytiotrophoblast:
(I) syniliesizes hCG
(2) LH analogue
(3) keeps corpus luteum functional for 8- 10 wks in normal pregnancy
(4) synthesizes human placental lactogen: growth hormone of pregnancy
8. cytotTophoblast
2. villus contains mys:olJlatous appearing st roma with blood vessels
3. two umbiUcal aneries and ODe vein (oxygenated blood)- single anery has an increased
incidence of congenital anomalies
r:r Placenta abnormalities:
t. placenta previa-
A. implants over cervical as
B. painless bleeding
2. abruptio placenta-
A. retroplacental clot
B. painful bleed ing
C. association with smoking/cocainelhypenension
J. plat-uta a~creta- direct implanfation into muscle without intervening decidua
4. eolargoo placeatas-
A. diabetes mellitus
B. Rh hemolytic disease of newborn
C. syphilis

168
5. twin placenla -
A. monochorio nic: are assodated with identical twins whether monoarnniotic or
dillmniolic
( I ) SiImeIe twins (USMLE) in monochorionic monoamniotic placenU!
(2) fetal to fetal transfusion in monochorionic mono or diamniotic
(3) tangle in umbilical cord only in monochorionic monoamniotic
B. dlcborionic can be either fraternal or identical twins
6. bleeding 8- 10 wk gestalion-
A. corresponds with drop in progesterone due to involution of corpus luteum of pregnancy
maintained by l3-hCG
B. not enough progesterone being synthesized by placenta
C borioamnlonilis: usually Streptococcus ago/DC/iae
Preeclam psllllcclampsio:
I. UJually occun In third trimester- thin k hydatidifonn mo le ifin first trimester
2. pathogenesls-
A. ~ nnal plncenultion causing mechan ical or func tional obstruction of sp iral aneries
h:nding to decreased perfusion in placenta:
( I ) .,. increase in angiotensin II. a vasoconstrictor
(2) ,J. decrease in prostaglandin Eh a vasod il ator
(3) t increase in TXA21 a vasoconstrictor
(4) '" decrease in nitTic oxide~ a vasodilator
B. placellta exhibits premature aging
3. eUniul-
A. preeclampsia:
(1) "'nypenension
(2) ,f'proteinuria
(3) pitting edema
B. eclampsia:
( 1) seizures
(2) R.... with magnesiura sulfate
C. renal disease: swollen endothelial cells in glomerulus
D. liver disease: see Hepatobiliary notes
4. lob-
A. increase in serum BUN
B. increase in uric acid
C. increase in serum creati nine
D. decrease in glomerular filtration rate
E. increase in se rum transarninases
F. schistocytes/anemialthrombocytopenia.: consider HELLP syndrome
Gestatio nal neoplasms:
I. IU dl*1fOnI _Ie (USMLE pImore)-'
A. looks like a bunch of grapes
B. complete mole:
(I) 46 XX (90010)
a. both cbromosomes of paternal origin (androgenesis)
b. egg has no chromosomes or the chromosomes are inactive

169
c. egg is fe rtil ized by one baploid X sperm that dupliclltes itself after
fe rt ilization
(2) 10% of eggs witho ut chromosomes are rertilized b)' 2 haploid spenn that are
e ither X o r Y , resu lting in either a 46 XX or 46 XY genotype
(3) most often associattd. with cboriocarcinoma
(4) preeclampsia in first trimester
•• uterus too large for gestational age
b. increased tl-hCG
C. pa rtial mole:
( I) lriploid (69 XXV)
a. hap loid egg with 23 X may be fertilized by I or 2 spenns
b. diploid 46 XY spenn + 23 X - 69 XXY
c. o ne haploid 23 X spenn + o ne haploid 23 Y spenn + I 23 X egg -= 69
XXV
(2) embryo present
(3) does not transform into chorioca rcinomll
2. invlIslvc mole- benign but does metllstas ize
J. gestationully derived choriocurcinoma-
A. syncytiotropho bl ast and cytotropho blnst
B. no chorionic vill i
C. metastasizes to lungs
D. responds dramatica lly to methotrexate
r::r Amniotic nuid:
1. composl lion-
A. retal urine
8. alka line pH
C. high salt content causes femi ng when allowed to dry on a slide
O. swallowed and recycled by the fetus
E. poly hydramnios:
( I) 1'0 0 much fluid
(2) TE fi,lUl. '
(3) duodenal .<=ia
F. o ligohydramnios :
( I) 100 link fl uid
(2) retal re na l a bnonnalities
2. AFP-
A. high AFP:
(I) open neu ml tube de fects
(2) re lated to fo late defic iency
(3) should be on folate before pregnancy
8. low AF P: Down syndrome
3. lecilhinlspbiagomyelin ratio-
A. lecithin is synthesized by lamellar bodies in type II pneumocytes in reml lungs during
34lh- 36lh week:
( I) lecithin is phosphatidy1choline
(2) decreases aJ\'eolar surface tension
B. >21 1 in amniotic fl uid indicates adequate pulmonary surfactant
C. surfactant increased by cOr1isol and thyroxine

170
D. surfactant inhibited by insulin: see respiratory for RDS discuss ion
cr Uri ne estriol:
l. derived from felal adrenal. placenla, malernalliver
2. low levels Indicllle fetal. placental or maternal problem

Breas t
Brea~it masses:
1. fibrocystic cbange-
A. Me breast mass <50 years
B. cysts with apocrine metap lasia (red epithelial cells)
C. duel hyperplasia:
( I) estrogen mediated
( 2) ntypicn l ductal hyperplasia o nly risk fac lor for cancer
D. fibros is
E. sclerosing adenosis in term inal du ct lobules: small risk for cancer
2. bloody nipple disc harge-
A. <50 ys old : benign intraductal papi lloma in lacti ferous duel
B. >50 ys old : consider malignancy
J. lumor <3S ys old-
A. fibroadenoma:
(1) free ly movable mass
(2) stroma is neoplaslic component
(3) stroma compresses ducts fa nning sl it like spaces
(4) estrogen-sensit ive
4. breast mass >50 ys old- infiltrating ductal cancer
BrcasllesioDs by loeation:
l. nipple/a reola comple.l-
A. Paget's disease of breast :
(1) rash-like lesion
(2) indicates und e rl y in ~ breast cancer infiltrating nipple ep ithelium
B. breast abscess:
(I) usually assoc iated with lactation
(2) Staphylococcus aureus
2. hlctiferous ductlsinus-
A. intraductal papilloma in lactife rous duel
B. galactocele
C. plasma cell mastitis
J. major dUCl3-
A. fibrocystic change
B. ductal cancer
4. lerminal ducl- tubular carc inoma
S. lerminallobule--
A. sclerosing adenosis
B. lobular carcinoma:
( I) bilaterality : does not have to be the same type of cancer in other breast
(2) slow progression into invasion
(3) "Indian fil ing" of cancer cells throughout the stroma

171
(4) not palpable
6. stroma-
A. fib roadenoma
B. cystosarcoma phyllodes:
( 1) malignant, bulky. fungating tumor
(2) rarely metastas izes
Breast cancer risk:
1. >50 ys old- most im portant risk
2. famil y bislory-
A. first generation relatives most importan t
(1) overrides age as most important risk factor
(2) mother andlor sister
B. yoWfg woman with a fi!!!lj1YHi< should gOl gon<.!ic counseling to RIO jJRnr18RCA2
connecUgwtYSMI 'D :
(1) mammography shou ld begin 10 yrs earlier than age of affected re lat ive having
the cancer
(2) e.g .• mother had cancer at age 45, mammography in daughter shou ld sIan at age
35
3. Rx of contralateral breast c:anc:er- overrides age as major risk fac tor
4. unopposed estrogen-
A. early menarcheflate menopause
B. null iparity
S. Hx of endometrial cancer
6. radialioD exposure
Mammography:
I. purpose is to detect DOD-paipabit masses
2. screening should begin at age 40 ys-
A. every other year until 49 ys
B. yearly from 50 ys on
Breas t cancer types: .I

1. intraductal carciuoma- comedocarcinoma with central dUCl necrosis


2. infiltrating ductal carcinoma-
A. simple type: MC overall type
8. medullary carcinoma:
(\) bulky
(2) younger age bracket
(3) pus hing borders
(4) large malignant ce lls with a lymphocytic infiltrate
(5) better prognos is than infiltrating ductal cancers
c. colloid (mucinous) carcinoma:
(\) elderly
(2) tumo r nests in lakes of mucus d
(3) good prognosis ... fin _ . 111_. -i\ _ A./H.tJrflt.c;NO'..... r.I
.J+,../I11.<='/
D. Paget's disease: (~~,...., W" \ ') . '.jqJ6 -li~5~t of",;;;)?' QJ?~1
( I) nipple rash witfi underlyin! cancer invading epldennis causmg the rash' Sk·/J
(2) poor prognosis
E. inflammatory carcinoma: clinica l tumor
(I) worst breast cancer

172
(2) peau du orange appearance due 10 plugging of subepidenna l lymphatics
3. lobular carcinomo-
A. M e term inal lobu le cancer
B. _bilatcrul
4. stromal tumon-
A. cystosarcoma phyllodes
B. sarcomas: angiosarcoma Me type
Clinical presentation of breast cancer:
I. presentatioo-
A. MC presentation is a painless mass in uppi:!r outer quadrant
B. skin/n ipple retraction
C. peau du orange: innammatory type
2. inve"igarive 1001, -
A.
, _I
mammography -.=:, tf et\ ~
Ia+ \
P.
A rJ r,I/ ,JJ~ /Il<l!>"c,
,~
B. fine needle asp iration: moSt often used for sampling pl! lpjlb lc mass
Prognostic tools: *" I'''' '-7 "r i v M'~JGo<vf-
l. estrogen receptor assay (ERA) and progesterone receptor aSSAY (PRA)-
A. if pos it iY~. indicates that ablati ve R.x. is indicated:
(1 ) mmox.ifen is an anti-estrogcn that blocks estrogen rect:.RlOrs
(2)"" protects against coronary anery disease and osteoporosis
(3}!It. can produce endometrial cancer
8. less likely to be pos itive in women in reproductive period of li fe: down· regulation of
receptors from estrogen
C. morc likely to be positive in posbtlenopausa l women:
(1) up·regulation of estrogen receptors
(2) better remission rate
2. S pbase fraclioD- determines number of neo pl astic ce lls in proliferating in the cell cycle:
less the 5 phase fracrion, the bener the prognos is
J. diploid (even number of cbromosomes) vs aneuploid (odd number of chromosomes)-
diploid is bener than aneuploid
4. erbB: oncogene- poor prognosis if present
5. grading of cancer-
A. nuclear chromatin panern
O. necros is
C1r Rx of breast cancer:
L modified radical mastectomy-
A. all breast tissue including nipple
B. axillary lymph nodes below axillary vein
C. pectoralis minor in most cases
2. lumpectomy witb low axillary nodes (for staging) followed by radiatioo- similar
statistics as for mod ified

173
QuesdoDl ued la che board review:
Items 1- 3
A. Infiltrating ductal carcinoma
B. Lobular carcinoma
C. Fibrocyslic change
D. Intraductal papilloma
E. Intraductal carcinoma
~ A 28-year-old woman during her brcllSl exam is noted to have a bloody discharge. No mll5SCS 8rc
palpable.
AM\\-er. 0
.". A 65 year old woman with a history of breast cancer in her mother i noted to have retraction of
skin in the right upper outer quadrant when abducting her right arm during her annual clinical
cxam. An indurated 3-cm mass is palpated directly benc:uh the skin retraction. Non-tender, finn
right a.xillary lymph nodes present in the lower 3.'(illtll')' chain .
Answer: A
r:r 1\ 26 year old woman without any fami ly history of breast cancer comp lains of pain and
"lumpiness" in her breasts that progress ively in creases throughout her menstrual cycle und is
relieved nfter menses is comple ted. Physica l exam revea ls a painfu l. ill -defined mass in the left
upper quadrant. No palpable ax.illary lymph nodes arc present.
Answer: C
cr A 58-year..ald woman with a 20-year history of smok ing has a history of breast cancer in her
mother. Her diet is poor in fiber and rich in saturated fats. Menarche occurred at 13 years of age
and menopause began at 52 years of age. She has had 3 children. She had a cervical conizalion at
28 years of age for severe cervical dysplasia and has since had normal cervical Pap smears. Which
of the fol lowing is her greatest risk factor for breast cancer?
A. Age
B. Family history
C. Smoking history
D. Severe cervical dysplasia
E. Low fiber, high salUnrted fat diet
B (first generation re latives overrides age)
QI" During her monthly self~b re DSt exam, a 19-year-old womnn notes tl moveable. finn, slightly tender
mass in the left upper outer quadrant. There is no skin retraction or nipple discharge. She is
currently loking birth control pill s. Her older sister was recenlly given 8 diagnosis of breast cancer
fo r which she opted for breast conserving therapy. You wou ld expect n fine needle aspiration of the
mass 10 reveal ...
A. an infillrating dUCUlI cancer
B. an intraductal papilloma
C. benign cyst fluid
D, a fibroadenoma
E. lobular cancer
o

174
The incidence ofbreasl cancer has been increasing in the Un ited Slales primarily due 10 ...
A. smoking
B. poor diet
C. mammography
D. breast self·exam
E. clinical breast uam
C
ar A 22 year old woman. gmv ida I, para 0, presents to the outpatient office for a routine prenatal visit
at 34 weeks' gestation. Her blood pressure is 150195 mm HG. On urine dipstick. she has 1+
glucose and 2+ albumin. Her blood pressure on the initial prenatal visit at 14 weeks' gest:uion was
l20n S mm j·IG. Her maternal grandfather has aduit--onset diabetes. Her mother and maternal
grandmother both have chronic hypertension. Which of the followi ng is the most likely
cxplo.mnion for the findings in Ihis patient?
A. PreeclAmpsia
B. Molnr pregnancy
C. Abruptio placenta
D. Gestational diabetes
E. Primary rena l disease
A
r:r A 17 year old girl presents with a sudden onset of abdominal pa in. Physical examination reveals a
tender mass in the left adnexa. A pregnancy test is negative. An x·ray exhibits a mass lesion of the
left ovary with focal areas of calcification. Which of the following best characterizes the ovarian
mass?
A. Benign surface.derived tumor
B. Malignant surface--derived tumor
C. Follicular cyst of the ovary
D. Benign germ cell tumor
E. Malignant germ cell rumor
o (cystic teratoma)
A 66 year old nulliparous woman pre5tnts with abdominal distention despite a poor appetite. She
underwent menopause 17 years ago. A rectal exam reveals induration in the rectal pouch of
Douglas. She has a nuid wave in the abdomen and a left~sided pleural effusion. A thoracentesis
shows clumps of malignant cells that stain positi\'e for CA 12.5. Wh ich of the following is the most
like ly diagnosis?
A. Meta.'l ttUic uterine leiomyosarcomas
B. Metastatic endometrial cancer
C. Metastatic ovarian cancer
D. Metastatic cervical cancer
E. Metastatic stomach cancer
c

175
A 28 year old woman, gravida I. para 0, at 12 weeks ' gestation presents with painless vaginal
bleeding. Her blood pressure is 160/95 mm Hg, there is 3+ proteinuria, and the uterus is large fo r
gestational age. Wh ic h of the following is the most likely diagnos is?
A. Gestational di abetes
B. Pl acenta previa
C. Abruptio placenta
D. Molar pregnancy
E. Twin placenta
D
r:r Place the following neoplasms of the femal e genital tract in decreasing incidence (morbidity) and
in decreasing mortality.
I. Cervi x
2. Ovary
3. Endometrium
Morb'd'
I my I'
MortalitY
A. 2 1 3 3, 2. 1
B. 1,3. 2 2,3, 1
C. 3,2. 1 2. 1, 3
D. 3.2. 1 3. 1, 2
E. 3, 2, 1 3.2,1
C
<3J" A 51 year old woman with a long smoking history is hospitalized for treatment of right
pyelonephritis. An intravenous pyelogram shows right-sided hydronephrosis and a dilated ureter.
During pelvic exam ination, you detect a malodorous vaginal discharge. A finn, ilTegular right
adnexaJ mass extends to the pe lvic side wall. The patient experiences vaginal bleeding after
examinatjon. What is the most likely diagnosis?
A. Perinephric abscess due to acule pyelonephritis
B. Endometrios is involving the right ovary
C. Invasive cervica l carcinoma
D. Invasive endometriaL.adenocarcinoma
E. invas ive ovarian adenocarcinoma
C
<7 A 22-year...old moderately obese woman presents with infertility problems. She has II long hi story
of oligomenolThea. Physica l exam reveals hirsutism on the face and anterior chest Pelvic exam
exhibits bilaterally enlllrged ovaries. Tbe following laboratory tests are availab le: serum LH
increased. serum FSH low, LHIFSH ralio :»11, serum tolal testosterone slightly increased, serum
free testosterone increased. serum DHEA-sulfate nonnal, serum prolactin normal, and the serum
TSH is nonnal. Based on these findings , you strongly suspect thal lhe patient ha~ ...
A. bilatera l a n drogen ~secre t ing tumors of the ovaries
B. defic iency of gonadolTopin-releasing hormone
C. polycystic ovarian syndrome
D. adrenal Cushing's syndrome
E. a gonadotropin-secreting pituitary rumor
c

176
Endocrine:

Exa mple.1 of stimuhttlon tests:


I. hypopituita rism : Gonadotropin releasing ho rm one (GRRH) stimulation test-
A, lack of an increase in gonadollopins over the baseline indi cates an anterior piruita,!
disorder
B. eventual rise in gonadotropins indicates a hypothalamic re leasi ng factor disorder
2. hypopituit:.lris m: growth hormone (GH) deficiency-
A, stimulation with sleep:
(I) best test
(2) GH and insu lin-like growth fa ctor 1 (somatomed in) are re leased around 5 Al\'1
(3) IGF- I is more sensitive lest
B. dopa and argin ine: lack of an increase in GH or somalomedins indicates an anterior
pituitary or hypothalamic disorder
3. hypopituita rism: thyrotropin releasing hormone (TRH) stimulation test-
A. lack of an increase in TSH indicates an ant erior piruilary disorder
B. eventual rise indicates a hypothalamic problem
4. hypopituitarism : pituitary/adrenal nxlS-
A. metyra pone test evaluates the integrity of the pituitary and adrenal axis for ACTH and
cortiso l:
(1) metyrapone blocks I I-hydroxylase enzyme in the adrenal cortex:
a. reduces synthesis of cortisol-+
b. stimulates ACTH-fo
c. proximal build-up of II-deoxycortisol

itt ACTH in pituitary


itt ll -deoxycortiso l
II-OHase bloc ked
by me[)lrapone 11-~'dro xy lase

.1..1..1. cortisol
d. lack of an increase in ACTH and deoxycorti so l indicates 8 hypothalamic.
anterior pituitary prob lem
e. increase in ACTH and decrease in deoxycorti so l indicates an ad renal cause
of hypocortisolism
B. insulin tolerance lest: hypoglycemia stimulates the release of both GH and ACTH
S. hypopituitarism: TRH s timulation lest for prolactin-
A. TRH is 8 potent stimulator of prolactin
8. good test for Sheehan's postpartUm necrosis
6. pO!llerior pituitary: diabetes insipidus-
A. differentilll for polyuria:
(I ) cenlTaUnephrogenic OI
(2) osmotic diuresis due to glucosuria in diabetes mell itus
(3) diuretics
(4) hypercalcemia: metastatic calcification of kidney rubules (nephrocalcinosis)
disrupts normal dilution and concenlration of urine
(S) psychogen ic polydipsia

177
8. water deprivation tes t for cemral or nephrogenic 01
( 1) in a normal person, water deprivation shou ld :
a. increase serum Na+ (POsm) to upper limit of normal
b. increase UOsm due to concentration of urine
(2) in a patient with 01, water deprivation will :
n. increase the POsm: i serum Na~ (POsm) = TBNa ·/.1.~ TBW, due to loss of
free water in the urine
b. decrease the UOsm due to loss of free water in the urine
(3) if intramuscular vasopressin (AOH) increases UOsm >50% from the baseline
UOsm, the patient has cemral 0 1
(4) if intramuscular vasopressin increases UOsm <45 % from the baseline. the
patient has nephrogenic 0 1
(5) in psyc hogenic polyuria (compuls ive water drin ker), water deprivation resembles
a norma l person
7. primary bypocortisolism (Addiso n's disease)-
A. short ACTH stimulation test :
(1) does not distingu ish hypop ituitsrismlhypothalamic cause of hypocor1isolism
from primary hypocortisolism (Addison's di sease)
(2) cortisol remains decreased in both
B. prolonged ACTB :
( I) stimulation (3 days) and measurement of urine 17-hydro=",ycort'icoids (17·0B)
(2) no increase in 17·0H and increase in plasma ACTH indicates Addi son's disease
(3) increase in 17-0H and decrease in plasma ACTH indicates a hypothalamic/
pi tu itary cause of hypocortisolism
Suppression tests:
1. majority of bormone excess conditions are due to benign ade nomas- primary endocrine
gland hyperp lasia or carcinoma are: sometimes responsib le for hormone access
2. in general, overactive endocrine disorders can not be s uppressed- exceplions include
A. prolact-inomas are suppressed with bromoc riptine (dopa analogue)
B. pituitary CushIng's suppressed with high dose dexamethasone
3. ora l glucose tolerance test for ncromegaly- lack of suppression of GH or IGF· } indicates <I
functioning pituita!)' adenoma
4. Cushing's syndro me-
A. hypercortisol ism:
(1) palient taking cort icosteroids
(2) pituitary Cushing's
(3) adrenal Cush ing's
(4) ectopic Cushing's
B. dexamethasone suppression test:
(1) dexamethasone is a cortisol analogue: should suppress ACTH in a nonnal person
and reduce cortisol
(2) low dose dexamethasone suppression lest: no suppress ion of a high baseline
cortisol in pituitary/adrena lfectop ic Cushing' s syndrome
(3) high dose dexamethasone suppression test suppresses conisol in pituitary
Cushi ng's but not ad renal or ectopic Cushing's
C. metyrapone test:
(1) normally blocks II·hydro=",ylase in adrenal cortex

178
(2) no change or inc rease in II -deoxycortiso llevels in pituitary Cushing ' s: indicates
that pitu itary tumor is responsive to a drop in cortisol induced by metyrapone
(3) no change in plasma ACTH o r ll -deoKycortisolleve!s in adrenal or ectopic
Cushing's .
5. phcoc hromoc),loma-
A. catecho lamine excess: usually a benign adenoma in adrena l medulla
8. c10nidi ne suppression test.:
( I ) su ppression tests are not usually performed
(2) clo nidine. an O,} agonist, normally decreases effe rent sympathetic outflow: does
not decrease catecbolamines in pi'leochrOmOC}10ma
6. primary aldosteronism (Cono '5 syndtome)-
A. minera locorticoid excess from benign adeno ma of zona glomeru losa
8. iSOlonic saline inrusion:
( I) isoto nic sa line should no rmal ly suppress aldosterone re lease by decrensing renin
re lease
(2) in primary aldostero nism, there is a lack of aldosterone suppression
Hypopituitarism :
I. adu lt-
A. non ~ functioning pituitary adenoma MCC of hypop ituitarism
( 1) MEN I (AD inheri tance) association:
a. pitu itary tumor
b. parathyroid adenoma
c. ZE syndro me
d. peptic ulcers
(2) o rder of succession of trophic ho rmone deficiency:
a. gonadolro p ins~
b. GH-+
c. TSH-+
d. ACTH-+
r. prola c t in~ ...
B. Sheeban 's postpart um nec rosis:
(t) sudden cessation of lactation
(2) hemo rrhagic infarction of pituitary usually rel ated 10 hypovolemic episode
during delivery
(3) nomally, pitu itary doubles in s ize during pregnancy
a. estrogen/progesterone inhibits prolac tin rel ease
b. delivery of placenta removes inhib ition and lactation begi ns
2. eh ildrcn-
A. craniopharyngioma MCC of bypopituitarnm
8. derives from Rathke's poucb
C. visual disturbances common: presses against optic chiasm
D. cystic and calcified
t:jf- Addison 's disease versus seco nda ry hypocortisolism:
I. main differences are in ACTB levels and electrolyte5-
A. Addison's disease (primary hypocortisolism):
( I) increased ACTH with hyperpigmentation
(2) mineralocorticoid deficiency from destruction of adrenal cortex:
a. severe hyponatremia

179
b. hyperkalemia
c. metabolic acidosis
d. no response to prolonged ACTH stimulation
(3) hypoglycem ia due to lack of gluconeogenic effect of cortisol
O. secondary bypocortisoliJm:
(1) decreased ACTH
(2) no mineralocorticoid deficiency
(3) mild hyponatremia due to mild inappropriate ADH from lad. of thyroxine and
cortisol effecl on inhibiting ADH
(4) hypoglycemia from loss of cortisol &nd GH
(5) adrenal response does occur to prolonged ACTH stimulation
Diabetes Insipidus:
I. central 01-
A. hypc mBt remia (increased POsm) with very low UOsm: lack of concentrat ion of urine
8. causes:
(1) CNS trauma with severance of piluitary stalk
(2) hypothalam ic disease:
a. Hand-Schuller-Christian di sease (hi stiocytos is X)
b. sarcoidosis
C. water deprivation: UOsm increases >50% with injection of ADH
2. nephrogenic DI-
A. same lab find ings as ccnual 0 1 except UOsm is <40% increased after ADH
administration
B. causes:
(I) drugs:
a. lithium
b. demeclocycline
(1) severe hypokaJemia: damages kidney tubules so they become refractory to ADH
(cal led vacuolar nephropathy)
C linicai llig ns and symptom-J and labonnory abaormalilies or hypopituitarism :
1. goaadotropins (FSH/LH) defic:iency-
A. adult women:
(1) secondary amenorrhea
(2) decreased libido
(3) diminished secondlU)' sex c haraeteristics
B. adult men :
(I) impotence (failure to sustain an erection)
(2) diminished secondary sex characteristics
C. children:
( 1) growth and sexual maturation retardation
(2) delayed fusion of epiphyses
D. laboratory:
( I) low FS H and LH
(1) 10westradioVtesrosterone
2. growth hormooe (Gn) deflcieocy-
A. adults: no effect on growth
B. children:
(I ) dwarfism

180
(2) delayed fusion of epiphyses
C. laboratory :
(I) low GH and IGF-I
D. insulin-like growth factor (JOF-I) is synthesized in the Ii,'er
b. GH stimulates IGF
(2) hypoglycemia: GH is a glucooeogenic honnone
(3) no increase in GH or fGF with oral glucose tolerance lest
3. tbyroid stimula ting hormone (TSB) deficiency-
A. adults:
( I) dry_brinl. hair
(2) cold intolerance
(3) weight gain
(4) periorbita l and facial puffiness
(5) impaired memory
(6) de layed Achilles' reflex. recovery
(7) musc le weakness (proximal musc le myopathy)
(8) ye llow ish skin : decreased conversion of p-Cl\rOlenC 10 reti noic ac id i ll imcslinnl
cell s
(9) constipation
B_ child:
( 1) mental retardation: brain requires thyroxine for maturation by 2 ys of age
(2) increased weight bUI decreased height
C. laboratory:
( I) low T.
(2) low TSH: key distinction from primary hypothyroidism
(3) increased CK (myopathy)
(4) increased LDL and cholesterol: decreased synthesis o f LOL receptors
(5) mild hyponatremia: mild inappropriate ADH. owing to the nonnal inhibitory
effect ofT, on ADH
.a. adrnocortic:otropic: hormone (ACTH) deficiency-
A. clinical: '
( 1) fatigue/weakness
(2) postural hypotension
B. laborotory:
(1) low ACTH : distinguishes it from Addison' s disense
(2) low cortisol
(3) fasting hypoglycemia : cortiso l is gluconeogenic
(4) mild hyponatremia: mild inappropriate ADH owing to the inhib itory en'ect of
con isol on ADH
(5) evcntual increase in 17-0H with prolonged ACTH stimulation test
(6) eosinophilia: due to the decrease in conjsol
5. prolacti a deficiency- sudden cessation of lactation in a woman with Shecha.n ' s postpanum
necrosis
6. aatidiuretlc bormont (ADH) defkiency- see previous discussion
r:r Pituitary apopltXy : refers to hemorrhage/infarction of pi1Uil.Cu,)' adenoma lead ing t\)
hypopituiwism
Qr Ntbon's syndromt: refers to enlargement of a preexisting pituitary adenoma secreting ACTH
after both adrenal glands are removed

181
African pygmies:
1. GR levels Dormal
2. low IGF-I
Laron dwarfism :
I. end·o rga n unres po nsiveness to GH effect-
2. normal to increased GB
3. low IGF·l
r:r G H excess:
I. GO funclions-
A. increase amino acid uptake in muscle
B. gl uconeogenesis
C. stimulates synthesis and release o r IGF· I in liver
2. IGF-I functions -
A. linear and lateral bone growth
B. chond rogenesis
C. growth of soft tissue
D. best screening test for hyper· or hypofunct ion states
3. giganlism- occurs in ch ildren where epiphyses have not fused
4. acromegaly-
A. occurs in adults
B. no linear growth in bones but lateral growth due to IGF·I :
( I) jaw w idens: gaps in teeth
(2) hand/feet enlarged
(3) enlarged fro nta l sinuses increases hat size
(4) visceromegaly
(5) cardiomyopathy: heart fai lure M e COD
(6) hyperglycemia
Galacforrheabr aginaJ bleeding in newborn girl : infl uence of matemnl estrogen
Prolac tino ma : ..
I. Me pituitary tumor
2. ctinicnl-
A. secondary ameno rrhea: prolactin inhibits Gn RH
B. ga lactorrhea
C. impotence in males : no galactorrhea
3. other causes of prolaclinemia-
A. drugs:
( I) birth co ntro l pills
(2) hydralazine
(3) H, blockers
B. primary hypothyro idism:
(1) low T~ increases T SH and TRH
(2) TRH is a stimu lator of pro lactin
4. R.'{ of prolactinoma-
A. bro moc.riptine in mOst cases: dopamine ana logue (i nh ibits pro lactin)
B. surgery if sella enlarged

182
ThYl"oid fUDction studies:
1. see schem:ltics
2. meas ul"ed total serum T~ (and T) -
A. renects ho nnone that is:
( I) bound to thyroid binding globulin (TBG)
(2) free. o r metabol ically active
B. changes in either the free hormone level (hypo- or hyperthyroidism) or TBG
concentrati on aln:r the total serum T~ without affecting free T~ level
(I) estrogen increases synthesis orTBG:
a. increased tOUlI T~
b. nonnal TSH
(2) androgens decrease synthesis of TBG:
a. decreased total T~
b. nonnal TSH
J. resin T) uptake (RTU)-
A. reneclS TSG concentration: reponed as a pe rcentage (25-35%)
B. preset amount of radioactive T ) is added to a sample of palient serum :
(1) radioactive T l binds to all available binding sites on TBG
(2) left over is bound to a resin and measured
(3) RTU is converted into a T~ bind ing ratio (T~ BR). by d ividing measured RTU by
reference serum mean RTU, which is 30%: e.g .• patient RTU 30%, T~ BR ""
30130~ I
(4) RTU and T~ BR arc low and high in hypo- and hyperthyroidism, respective ly
a. in hypothyroidism. more binding s ites available on TBO. hence less is left
over
b. in hyperthyroidism, less binding sites available on TBO (saturated with
T,,), hence more is left over
(5) RTU and T" BR are bolh increased when TBG is decreased: less binding siles
available on TBG. he nce more is left over
(6) RTU and T, BR are both decreased when TBO is increased: more binding si tes
available on T8G , hence less is left over
4. free T, index (IT~-I)-
A. ca lcul ated free T. hormone concemration
B. ca lcul aled by mUltiplying the total serum T. by the T4 BR
C. FT.-I is low and high in hypo- and hyperthyro idism, respective ly
D. FT~-I is normal when the TBG concentration is high : increased total T~ x decreased T.
BR = nonnal FT.-I
E. FTA-I is normal when the TBG concentration is low : decreased to tal T4 x increased
T.BR = nonna l FTl-1
5. serum TSH -
A. TSH has a negalive feedback relationsh ip with circu lating T. and T}
8. TSH is single best test for Ox of primary hypomyroidism and hypcnhyroidism
(1) primary hypothyroidism : TSH increased
(2) secondary hypothyroidism : TSH decreased
(3) hyperthy roidism: TSH decreased
C. TSH is always normal with alterations in TBG concentration since free T4 levels
nre no rmal

183
Normal

TIlG (Emoer. Ib)

•••
•••
Free T." 6
..• •.
~.~
~
Free T.. - 6
Total T.. - 12
Thyroid gla ud
Free T.. - 6
Total T. '" l2
TSO'" Dorma' Total T .. - 15
TSB - DormaJ
TSB - Dorma)

Primary Hypothyroidism Hyperthyroidism

TBG (AoaboUc .ucroids) TIlG TllG TIlG TllG

••• • •• • •••
•••
Fret T ..... 6
Fret T.. - 3
Total T .. a 5 • •••
Free T. - 8
Total T .. II: 20
Total T. - 9 TSB - bia:h
TSD - Dormal TSH "" S QPp~ed
6. r:,dioac live I'll uptnkc-
A. 1131 uptake is used 10:
( I) detect nodules
(2) prov ide an index of thyroid activity
B. thyroid honnonc is tyrosine with iodides attached 10 it:
( I) true hyperfunclio ning of the gland (e .g .. G raves ' disease) inc reases 1m uptake

C. hot nodules:
( 1) functionally active
!
(2) hypofunclioning gland has a decreased 11)1 uptake

(2) preferentially take up Ill : remainder of the gland is nOt visuali zed, si nce il is
suppressed
O. cold nodules:
( 1) inactive
(2) do nOI take up the I lll space); normally functioning gland lakes up the 1m
T hyr oglossa l duct CYSI : midline cystic mass
Branchia l cleft cyst: cyst in al1lero lateral neck
T hyroiditis:
1. :lculc Ihyroidilis-
A. in fec,tion caused by Staphylococcus oureus, SrreptocOCC/lS plleumoll ioe or
SrrepfOcoccllS pyogenes
B. signS/lnb of thyrotoxicosis but Illi uptake is decreased
2. !l ubuc ute gra nulomatous thyroiditis (de Quervain lbyro iditis)-
A. MCC ofpainfullhyro id gland
B. follows a viral infection (e.g., coxsackievirus)
C. granulomato us innammal'ion wi th mu ltinuclented giam cel ls
3. Hashimoto'! (by roidilis-
A. auto immune thyroiditis
B. pathogenesis:
(1) MCC hypothyroidism
(2) antibodies aga inst" peroxidase (antj-microsomal antibodies) and thyroglobulin
(nnti -thyroglobulin antibodies)
(3) cytotox ic T cell damage
(4) inhibitory autoantibodies against TSH receptor that prevent honnone synthesis
C. gross/micro:
( I) enlarged gland with heavy lymphocytic infiltrate. gennina l fo llicles
(2) predisposes to primary malignant lymphoma of thyroid
4. Reidl:!l's thy roiditis- intense fibro us tissue replacement of the gland
Thyroid horm o ne exccss:
1. hyperthyroidism-
A. excess synthes is of hormone: e.g., Graves' disease. loxic nodular go iter
2. th~' roloxicosis -
A. describes the end-organ effects excess thyro id hormone, regardless or the etiolog) :
e.g .• thyroiditi s, taking excess hormone

*" AI" ~.rwr:'> ¥/ ~.J ~6


j:,!-l ~.J. ~ do.
I .
J~ JI.'!j ,I"'.... ..
I ..:.. ttut-r.~.1J
184 "
r:r- Grn\'cs ' diselUc:
I. MCC of hypert hyr oidism a nd thyr otoxicos is-
A. female dominant autoimmune disease
8. due to synthesis of an autoantibody again st the T A receptor called th~roid
stimulating antibody (TSI): type D hypersensilivity
2. gross/micro-
A. symmetrical, nORtender thyromegaly
O. microscopic features of overactivity:
( 1) scant colloid
(2) papillary infolding orlbe glands
J. dinicalllab featu res uniq ue to Graves' diseasf-
A. infiltrative ophthalmopathy often associated with exophthalmos
8. pretibia l myxedema: excess glycosaminog lycan deposition
C. an lim icrosomal and thyroglobul in antibodies present
D. olher fea tures of thyrotoxicosis common to all types:
( 1) si nus tachycard ia
(2) ntri ul fibrill ation: a lways ord er TS H w hen this ar rhythm in Is present
{3} systo li c hypene ns ion
(4) anxiety
(5) weight loss
(6) heat intolerance:
(7) muscle weakness/increased reflexes activity
(8) diarrhea
(9) low cholesterol: increased synthesis of LOL receptors
(10) hypercalcemia: increased bone lumover
... la boratory profile- see table
S. R:I Or G ravH d isease-
A. J,l-blockers: block adrenergic effect ofth)Toid bonnones on target organs
8 . drugs that decrease hormone synthesis: propylthiouracil only one that can be used in
pregnancy ,
C. if above docs not correct in - I)T. radiation ab lalion is used
Toxic Dodu la r guiler (Plum mer 's d isease): develops in the seuing of It multinodu lar goiter
Hypolhyroid is m:
1. red uction in sce r etion or Ihyroid hormooe-
A. primary di sease: MCC i5 Has himolo's thyr oid itis
B. pituitaryl hypothalam us dys fu nclion
2. clin lcal-
A. muscle weakness:
( I ) M e symptom
(l) due to myopathy: iDcreased serum creatine kin ase
B. coarse skin
C. periorbital puffiness/pretibial myxedema: due to deposition of glycosaminoglytans
D. brin le hair
E. cORSlipation
F. cold intolerance
G. weight gain
H. delayed recovery ofthe Achilles reflex

185
I. diastolic hype n ension
J. yellow skin: increase in ~a rotenes fro m decreased conversion into relinoic acid
3. la b-
A. see table
B. hypercho lesterolemia
4. IU-
A. le\'othyroxine
B. follow patient with TSH levels until they become nonnal
Goite r:
I. enlargemeDt or lhyroid gland
2. du e to a D ab olute or reJalin deficiency or thYToid horm one- mOSl often associated with
iodide deficiency
J. rec urreal episodes orTSH s timulatio n (hyperplas ia) and iO\'olulion (colloid stage)
A. causes gland 10 enlarge:
( I) initially diffuse en largement
(2) eventually multinodular
B. hemorrhage into cyst is MeC of sudden gland en largement
Solitary thyroid nodule (usua lly cold nodule) :
1. ad ult wom9.n-
A. 60% due to cysts in goiter
O. 25% benign follicular adenoma
C. I S% malignant
2. adult maa/cblld - usually cancer
J. Hs: or radiation to head and oeck- cancer in majority of cases
Follicular adenoma: Me benign thyroid rumor
Papillary adeoocarcinoma:
1. Me Ibyroid cancer in adults and children
2. multirocaJ tumors-
A. papillary fronds lined..by empty appearing nuclei (Orphan Annie nudei)
B. psammoma bodies
C. lymphatic invasion with focal cervical lymph node involvement is the rule
3. ucelleol prognosis
Follicular carcinoma:
1. grols/mlcro-
A. we ll c ircumscribed encapsulated tumor with capsular andlor blood vesse l invas ion
O. invasive cancer without a capsule
C. blood vessel invasion without lymph node metastasis is the rule
D. more likely to have distant spread than papillaI)' cancer:
(I) lungs
(2) bone
2. commonly lllkes up l UI
Mtdull ary ca rcinomas :
I. sporadic (80%)
2. ramllial (20%)-
A. autosomaJ dom inant MEN lIa and lIb syndromes:
( I) ~fEN I)a .yndromt:

186
8. medullary carcinoma
b. primary hyperparathyroidi sm
c. pheochromocytoma
(2) MEN Db variant :
a. medu llary carc inoma
b. mucosal neuromas involving the lips and tongue
t. ph eochromocytoma
d. Marfanoid habitus
3. derive from parafoUitular C tells:
A. synlhes ize ca lcitonin
B. calcitonin is a tumor marker for the cancer
C. calcitonin is changed into amyloid in the tumor
Summary of thy roid profile and lUI uptake in thyroid disorders: bolded areas are shan cUIS to

rnak109 lh Dx
O

Disorder Total Serum RTUIT,BR FT4·1 TSH ri ll


T,
Grave's Increased Increased Increased SUDDrcsscd lncreased
Factitious thyrotoxicosis Increased Increased lncreased SUDoressed Decreased
Thvroid itis (ncute, subacute) lacreased increased Lncreased Suppressed Decreased
Pri mary hypothyroid ism Decreased Decreased Dec reased Increll5ed Decreased
(Hashimoto)
Secondary hypothyroidism Decreased Decreased Decreased Decreased Decreased
(hypopituitarism!
h'!o'Pothnlamic)
Increased TBG lncreased Decreased Normal Nonmd Normal
(increased estrogen)
Decreased TBG Decreased Increased Normal Nonuill Normal
(increased androgens) I
RTU - reS in T J uplake, FT~ -I 0

-
free T. °Index, T,BR - T4 binding rauo. TSH "" thyrOid stimulating
hormone. I 13 1 = radioacti ve io(line I 131 uptake, TBO = thyroid bind ing globulin
-
r::r Total serum calcium (see schematics):
1. mellSures calcium bound (40% a lbumin, 13% phosphate. citrate5) + free (47%, ionized
fraction , metabolically active fraction )
2. hypoalbuminemia-
A. decreases tola l calcium without altering the free ionized level
B. correct fo r hypoalbuminemia with formula: corrected ca lc ium .. (serum calcium -
serum albumin) + 4
3. alkalosis-
A. inc reases negative charges on albumin
B. binds more calcium: e.g., 60% bound vs normal of 40%
C. lowers ionized calc ium leve l without altering the total calcium
D. clinicaJ evidence of tetany
4. clinical fioding s witb hypocalcemia due to decreased ionized h!\'cls-
A. tetany
B. cl in ical:
(I) carpopedal spasm: Trousseau's sign with blood pressure c uff, thumb fle xes into
palm

187
(2) Chvo5tek's sign: facial twitch after tapping the facial nervo
Primary byptrplratbyroidlsm (HPJ 0):
1. MCC byperulcemll In tbe ambulatory population
2. MC due 10 a btoign parathyroid ade.aoma- composed of sheels of chief cells with no
inlervening fat
3. PTH rUDClions-
A. Increase reabsorption of calcium in kidneys: a-IeI' pump in early distaJ tubule an
conex
B. decreasc bicarbonate reclamation in proximal tubule: proximal r,,-nallubular acidosis
C. decreases phosphale reabsorption in proximal tubule
D. increase synthesis of 1-(1-hydroxylase enzyme in proximal tubule for second
hydroxylation of vitamin 0 : vitamin 0 reabsorbs calcium and phosphorous from the
small bowel
E. maintains ionized calci um levels:
(I) receptor on the osteoblast
(2) PTH causes release of IL-I (osteoclast activBting fac lor)
(3) IL- I activatcs the osteoclast t'O break bone down and ma in tain ionized calcium
leve ls
(4) estrogen has inhibitory effect on IL-I in women
(3) teslosterone has inhibitory effect on fL- ! in men
4. diaiul-
A. mosl cues are asymptomatic and are discovered incidenll1l1)
8. reaal.loaH Me symptomatic presentation:
( I) due to hypercalciuria
(2) more calcium is released from bone than is reabsorbed back by PTH in the
kidneys
C. peptic ulcer disease: hypercalcemia stimulates gastrin release
O. Icute pancreatitis: hypercalcemia activates phospholipase in pancreas
E. constipation
F. polyuria:
( 1) metastatic calcification of kidney tubules
(2) called nephrocalcinosis
C. short QT Interval
H. diastolic hypertension
I. bone discllSe :
(1) osteitis fibrosa cysticl1
(2) cystic lesion with hemorrhage (brown tumor)
(3) usually involves the jaw
5. lab-
A. eleVAted serum PTH
B. hypercalcemia
c. hypophosphatemia
D. nonnal anion gap metabolic acidosis: loss of bicarbonate in the unne is
counterbalanced by an equal gain in chloride
E. increase in serum alkaline phosphatB.Se: bone fonnation counteracts bone resorption
F. hypercalciurialhyperphosphaturia
G. chloride/phosphate ratio> 33

188
l\"1alignancy-induced hypercalcemia: pathogenes is includes
1. bo ne metastasis with activation of osteoclast!!:
A. MCC
B. sec retion of interleukin-I /prostaglandins by cancer cel ls
2. ectopic secretion of a PTB-like peptide:
A. primary squamous cell carcinoma of the lung.
B. rena l adenocarcinoma
C. breast cancer
O. PTH-Iike peptide increases calcium reabsorption in the kidneys (hypercalcemia) and
decreases phospho rous reabsorption (hypophosphatemi a)
3. hyperca lce mia suppresses the pafient 's PTB as It does with all the oth e r ca uses of
hyperca lce mia not related to primary KPTH
Other causes of hypercalcemia:
I. sarcoi d o~i5
2. thiuid es
3. vitami n D toxidt),
4. multipl e m yeloma
Secondary I:IPTR :
I. hyperplasiu of a ll of the parathyroid glands as compenution for hypocalcemia
2. MCC is hypovitaminosis 0 fro m ch ronic renal failure
Tertiary KPTH :
l. hy pe rcalcemia tha t has develo ped fro m secondary HPTB
2. usually anodated with chronic renal failure
Primary hypoparathyroidis m :
1. hypofunction of the parathyroid glands leading to hypocalcemia
2. causes-
A. previous thyroi d surgery MeC
B. autoimmune hypoparathyro idism: usuall y associated with pOlyendocrine de fi ciency
syndromes (mu}tiple endocrine gland de fi ciencies)
C. DiGeorge syndrome
3. dinical-
A. reduction in tOi al and ionized calcium level resul ts in tetany:
(I) circumoral paresthesias (numbness and tingling)
(2) positive Chvostek 's sign
(3) carpopedal spasm due to musc le spasms (Tro usseau's sign) when lak ing a blood
pressure
B. calcification of basal ganglia
4. lab-
A. low PTH
B. hypocalcemia
C. hyperphosphatemia
D. pro longed QT interval
Pseudohypopa.rathyroidism:
I. SXD inherited disease characlerized by end-orglln resis ta nce 10 PTH-
A. type I disease:
(I ) de fect proxim al to the generation of cAMP
(2) receptor pro blem

189
(3) no cAM"P response to infused PTH
B. type U disease:
(I) defect distal to the generation of cAMP
(2) posHeceptor problem
(3) cAM.P response to infused PTH
2. dinical-
A. mild mental retardation
B. =..
iI,Jiid Slll ..... ==
r pu
= ' (USMLE)
3. 11lb-
A. hypocalcemia
8. hype:rphosphatemia
C. normal to high PTH
Other causes hypocalcemia :
1. hypomagnesemia-
A. magnesium is a cofactor (or adenyhue cyclase, which generates cAMP (required for
PTH activation), hence inhibiting PTH act ivity
8. inhibits synthes is and release of PTH
c. causes hypomagnesemia:
(1) alcoholic
(2) diarrhea
(3) diuretics
(4) aminoglycosides
(5) cisplatin
2. acute pancreatitis- calcium used up in enzymatic fat necrosis
3. hypoalbuminemia- MCC of hypocalcemia
4. OiGeorge syndrome- see Immunopathology notes
A. failure of 3rd and 4th pharyngeal pouches to develop
8. hypoparathyroidism
C. absent thymus : T ce ll deficiency
O. truncus arteriosis: cyanotic
-, CHD
5. hypovitaminosis 0 -
A. cbronic. renal failure: MCC
B. malabsorption
C. cirrhosis
D. lack of sunlight
E. type] vitamin D-dependcot rickets:
(1) decreased Ia.-hydroxylase
(2) hypocalcemia and hypophosphatemia
(3) high PTH
(4) normal 2S(OH)D,
(5) low 1,25(OH),D,
f. type 0 vitamin D-depeodent ricke"':
(1) absent vitam in 0 receptor
(2) hypocalcemia and hypophosphatemia
(3) high PTH
(4) nonnal 2S(OH)D,
(5) high 1.25(OH),D,

190
NORMAL CONIlITION RYPOALBUMINEMIA ALKALOTIC STATE HYPOPARATHYROIDISM HYPERCALCEMIA

Alb umi n Album in Albumin Albumin


··..... ...
Albumin : -:
·· ..
····· .....
· .

0- 0- Oe Oe Oe
••••• •••••
TolBI Ca t + :: 10 Tota l Ca++= 8
•••
Total C.* ~ 10
•••
Total Co -H "'" 6
••••••
••••
Ionized Co...... = 5 Ionized Ca++= 3 Ionized Ca t.. = 3 Total Ca 1-' = 15
Ionized Co H- = 5
Ionized 0.++= 10
6. hypopar;&tb yroidism
or Su mma of calcium, hos horous and PTH related disorders:
Disorder Calcium Phos horous PTH
B\' erca1ce mia
Prima HPTH Increased Decreased Increased
MaJi anc ' PTH-Iike tide) Increased Decreased Decreased
H VDoca cemla
Primary HvooDnrathyro idism Detreased Increased Decreased I
Pseudohypoparatbyroidism Decreased increased Normal to
Increased
I Secondary HPTH (Malabsorption) Decreased Decreased Increased
Secondary HPTH (Renal Failure) Decreased Increased Increased
Hypoalbuminemin Decreased- normal ionized Normal Normal
calcium
Alkalosis Normal decreased ionized Normol Incrcased
calcium
..
HPTIi = hyperparathyroid ism
Graphic representa tion of PTHlcaicium disorders:

B C
PTH

I Norm~1
A D

Serum calcium
1. group A- ~
A. low calcium and low PTH
8. primal)' hypoparathyroidism
C. DiGeorge syndrome
D. hypomagnesemia
2. group 8 -
A. low ca lcium and high PTH
B. secondary hyperparathyroid ism:
( 1) chronic renal failure leading to hypovitaminosis 0 MeC
(2) malabsorption leading to hypovitami nosis D
J. group C-
A. high calcium and high PTH
B. primary hyperparathyroidism
4. group D-
A. high calcium and low PTlI
B. all other causes of hypercalcemia:
(1) malignancy induced
(2) sarcoidosis

191
Adrna l glllnd:
I. lona glome rulosa produces aldosterone-
A. activation of the renin-angiotensin-aldosterone system
8. enzymes present;
(I) 2 I-hydroxylase
(2) II-hydroxylase
(3) IS-hydroxylase
a. activated by angiotensin II
b. deoxycorticosterone con\'erted into aldosterone
c. enzyme not present in other zones
2. zona rasc.ic uillta produces glu cocortieoids-
A. dcoxycort isol
8. cortiso l
C. enzymes present:
(1) 17-hydroxy lase: not present in zona glomcnllosn
(2) 2 1-hyd roxylnse
(3) II-hydroxylase
3. lon" rellc ularis produces sex hormones-
A. estrogen:
(1) aramatization of androstenedione
(2) aramatization of testosterone
B. 3Ildrogens:
( 1) 17-ketosteroids DHEA and androstenedione
(2) testosterone
C. enzymes present: same as fasciculata
4. urine for t 7-bydroxyeorticoid5 (17-0H)- metabolites of II--deoxycortisol (compound S)
and con-isol
S. urine for 17-ketosleroids (17-KS)-
A. dehydroandrosterone (DHEA)
B. androstenedione
6. adrenal medulla- '
A. neura l crest origin
D. synthesizes the catecholamines epinephrine (EPI) Il1ld nore pinephrine (NOR)
C. EPI and NOR: metabo lized into biologica lly inact ive metabolites by monoamine
oxidnse (MAO) and catechol-O-methyltransferase (COMT). respec tively. and into the
urinary me tabo lites metanephrine and van illylmande lic acid (V MA), respectively
D. metabo lite o f do pamine is homovanil lic acid (HVA )
C us bing's syndrome:
l. causes-
A. long-Ienn glucocorticoid therapy MCC; zona fascicula ts and rcticularis atrophied
D. pituilary C ushing' s: sometimes called Cusbing's dluase
( I) rumor produces excess ACTH
(2) hyperplasia of zona fasciculata and reticularis
C. ad rena l C ushing's
(1) adenoma produces excess cortisol
(2) suppressed ACTH
(3) adrenal tissue surrounding the adenoma is atrophic and contralateral zona
fasciculatalreticularis is atrophied

192
Cholesterol (en)

...- choles terol ~ide chain cleavage enzyme

...... desmolase

17-hydro:a:ylase

.1.
Pregnenolone

3p-hydroXY5leroid dehydrogenase/i3omerase

t7-bydroI)'lase
...

1 1
17-nydroxypregnenolooe - -..~~ Dehydroepiandrosteront' (C I ,)

/17-k<IOSIUOid,

Progcslerone (ell) .., 17-lIydroxyproge..oUerone - -..~~ Androstenedione (Cit)


(pregn.nelriol)

1
+ol.idoreduclase

Il -hydrosylase ll -hydroxyla!c
aromala!l:e
Testosterone (C I , ) ... Estradiol (C I . )

II -deos')'curticoSlerone II -deoxycortisol (compo und S)


Sa - rt~ducta5e

r--~---------------------~L---------'I
I 'I - hydrOXYl... II -hYdrOXYI~ " 1'-bydrosycorticoids
Oihyd rotes lo!leronc

Cn rlico!'llerone Corlisol (C,,) I ~ ACTI.


(nega lh'e feedback on ACTJ-I)

ActhoaCt>d by ATU

Aldosterone (CII )
Schelll:l fic of Adrenal Gland lJio!'lynlh elic Pathways
D. ectopic Cushing's:
( 1) cancer ectopically sec reting ACTH
(2) l ona fasciculatalretic ularis exhibit hyperpl asia
2. pituitary Cusbing' s-
A. MC type: also caUed Cushing's disease
8. benign adenoma secreting ACTH
J. adrenal Cushing's-
A. excess production of cortisol secondary to neo plasia (adenoma, carcinoma) or
hyperplasia)
B. ACTH is suppressed
4. ectopic Cushing's-
A. sma ll ce ll carcinoma of lun g MeC
B. highest ACTH le.ve ls (hyperpigmentation)
S. clinical-
A. weight gain:
(1) fa t deposition in face ("moon faci es")
(2) upper back ("buffal o hump")
(3) trunk (truncal obesity) with sparing of the extremities
(4) due to hyperinsulinism
B. diastolic hypertension: due to increased synthesis of weak mineralocon icoids
C. glucose intolerance: due to increased gluconeogenesis
D. hi rsutism: due to increased 17-KS
E. purp le abdominal s[ria:
( I) ruptured blood vessels in stretch marks
(2) con isol weakens collagen
F. osteoporosis
G. thin extremities: due to excess cortisol breaking down muscle to get aJim ine fo r
gluconeogenesis
6. screening test5-
A. plasma ACTH '
8. low dose dexamethasone (cortisol ana logue) test: cannot suppress cortisol
C. 24 hour urine for free cortisol:
( 1) best screen
(2) measures excess unbound corti sol
7. confirmatory test-
A. high dose dexamethasone test: suppresses corti sol in pituitary Cushing's but not the
other types

193
cr Laboratory Slu d'.os I n C us b'102'' $ S
~YD d rome:

Laboralorv Test Pituitary Cushing's Adrena l C ushioe,'s Ectopic C usbin2.' s


Serum Cortisol Increased Increased Increased
Urine for free cortisol Increased Increased Increased
Low dose
dexamethasone
Cortisol not suppressed Cortisol not suppressed Cortisol not suppressed
I
High dose Cortisol s uppressed Conisol not suppressed Con isol not suppressed
dexamethasone
PlosmaACTH " Norma' ''' to Increased Low Markedh' Incrensed
Metyrapone Deoxycortisol nannal Deoxyconisol not Dcoxycortisol nOi
to increased- tumor is increased increased
not autonomous, hence
it can increase ACTH
synthesis in response to
,
hYPQCort isolism I
'Normnl " ., 8 plasma ACTH In the normal range IS not normal In the presence o f an Increase 10 serum
cortiso l.

Primal")' aldoslcroni.s m (Co nn 's synd rome): see Fluids and Hemodynamics notes
Pheocbromocytoma:
I. IlSsocialions-
A. neurofibromatosis
B, MEN II. and Ub,
C. von Hippel Lindau disease
2. clinic:al-
A. key features:
(1) unilateral
(2) benign adenomas
(3) majority arise in adrenal medulla
D. hypertension ;
C. episodic palpitations
D. drenching sweats
E. excessive anxiety
F. headache
J. screening lest.s-
A. 24 hour urine for vanilly lmandelic acid (VMA) and metanephrines (best overall
screen)
D. neutrophilic leUkocytosis: decreased adhesion molecule synthesis by catecholamines
C. hyperglycemia:
( I) glycogenolysis
(2) gluconeogenesis
4. ax by surgery-
A. phenoxYbenzamine an a-adrenergic blocker used preoperatively to control
hypertension
D. I3-blockers to control tachycardia after hypertension controlled
C. phenlolamine, an a-blocker used in hypertensive crisis

194
r::r Neuroblastoma:
1. malignant tumor of neural crest origlo-
A. Me site of origin is adrenal medulla
B. posterior mediastinum
2. deluio n or reunlngemeol o( tbe sbort arm of cbromo orne I luding 10 amplifiCD.tioD
of tbe N· m)·c oacogeoe
3. clinical-
A. "small cell" tumor composed ofneuroblasts and Homer. Wright roscnes
8. neuroblasts are Sloo tlntigen positive: contain dcnse corc neu~crtl ory granules
C. paJpable abdominal mass with abdominal distention
O. diastol ic hypertension
E. mewtasis: bone Me sile
4. lab- elevated urine VMA. metanephrines. HVA
S. prognosis-
A. age of the patient is the single most important factor detennining prognosis:
8. cure rate Is 85- 90'10 under t year of age
C. 15-40% cure rate in older ch ildren
Adrenal hYI}ofunctlon :
1. CIUSCS-
A. autoimmune dutTuclioD MCC chronic iDsufliciency (Addison's disease )
8. abrupt withdrawal of corticosteroids: MeC of acute adrenal insufficiency
C. miliary TB: MeC worldwide
O. disseminated meningococcemia;
(1) Wl'Ilerhouse·Friderichsen syndrome
(2) OIC with bilateral adrenal honorrhage
L congenital adrenal hyperplasia.: alias adrenogenital syndrome (see Genetics OOlC3)
2. diaical-
A. weakness
B. hypotension: salt loss
C. diffuse hyperpigmentation: increase in plasma ACTH
3. la b-
A. no detectable increase in urine 17·0H in both a short and prolonged ACTH stimulation
lest
B. elevated plnsma ACn-l
C. hyponatremia; hypertonic loss of sail in the urine
D. hyperkalemia: no ex.change of sodi um for potassium or hydrogen ions ror potassium
owing to hypoa ldosteronism
E. fasting hypoglycemia: loss of gluconeogenic activity of corti so l
F. nonnal anion gap metabolic acidosis: retain H' ions owing to loss of oldosterone and
its nonnal maintenance of the proton/IC ATPase pump in the co llecling tubules
G. eosinophilia: loss of the cortisol effect on eosinophils
f7 lslet celllu.mon :
I. lasulinoma-
A. benign tumor arising from the P islet cells thai produces fasting hypoglycemia
B. 80% howe MEN I syndrome
C. MC Is let tell tumor (70~.)
( I) secrete excess insulin and C·peptjdes

195
(2) fastin g hypog lycemia:
a. insulin inhibits gluconeogenesis
b. prolonged fast is the best test
D. clinica l:
(I) neuroglycopenia (brain without glucose) from fasting hypoglycemia
(2) forgetfulness
(3) menta l stalU5 abnonnali lies
E. labo ratory:
(I) hy pog lycemia in the presence of an increased insulin and C-peptidc level
(2) C-peptide is the best test for endoge nous iDsu lin release
F. factitious bypoglycemia from injection of insu lin :
( I) increased serum insulin and hypoglycemia
(2) decreased C-peptide : suppression of endoge nous insulin release b,
hypoglycemia
2. gaslrinoma (Zollinger-Ellison Syodrome)-
A. mali gnant is let ce ll tumor arising from G cells in pancreas Me s ite: excess o f gastrin
leads to hyperac idity and peptic ulcer disease
8. duodenum second MC location
C. associatio n with M EN I syndrome
D. c linical :
( I) s ing le ulcers in the usual locations for peptic ulcers in most cases
(2) multiple ulcers can also occur
(3) suspect ZE if any ulcer is in an unusual site
(4) abdominal pain from PUD
(5) diarrhea: malabsorption. since the enzymes cannot work in an acid pH
E. laboratory:
(1) basal acid output (BAO) is best sc reening test
a. marked ly increased BAO
b. ma."timal acid output (MAO) a lso increased and at its max.imum
(2) BAOIMAO ratio > 0.60
(3) intravenous secretin test is confinnato ry: see paradoxic.aJ increase in gastrin
(4) serum gastrin leve ls usually >600 pglmL
(5) other causes of hypergastrinemia :
a. Hl blockers (decreased acid, increases gastrin)
b. atrophic gastritis invo lving the body and fundus (decreased acid, increuses
gastrin)
c. pyloric obstruction (antral distention is a pote nt stimulu s for gastrin )
3. glucagonoma-
A. malignant tumor of islet ce lls (Cl cells) with excess secretio n o f g lucagon
B. clinical:
( 1) diabetes mell itus (glucagon is gluconeogenic)
(2) characteristic rash called necrolytic m igralory erythema
4. somntoslatiooma-
A. malignant tumor of islet cells (5 cells) secret ing excess somatostatin
B. cl inical :
( 1) achlorhydria: inhibits gastrin
(2) cholelithiasis : inh ibits cho lecyslok inin
(3) diabetes mellitus: inh ibits gastric inhibitory peptide, which nonnally stimu lates
insulin release

196
(4) steatorrhea: inhibits secretin and cholecystokinin
5. VIPoma or pancreatic cbolera or Verner Morrison syndrome-
A. mal ignant tumor of islets wi th excessive secret ion of vasoactive intestinal peptide
B. clinica l:
(I) severe secretory diarrhea: VIP acts by stimuhll ing cAMP similar to loxin in
cholera and toxigen ic E. coli
(2) hypokalemia and nonnal gap metabolic acidosis
(3) ac hlorhydria
Summary of type I and type n diabeles mellitus (DM) :
I. preva leoce-
A. typc I 5-IO%
O. type II 90-95%
2. age-
A. type I <20 years (80%)
B. type II >30 years of age
3. body habilus-
A. type 1 usually thin
O. type" maj ority are obese
4. family Ux-
A. type I family Hx uncommon : - 50% concordance rate with identical twins
B. type II family Hx is common:
(1) multifactoria l inheritance
(2) - 90% concordance rate with identical twins.
(3) increased in native Americans and African Americans
5. pilihogenesis-
A. type J insulin lack:
(1) presence ofHLA-DR3 and -O R4 (90-95 %):
a. l3-islet cell destruction by viruses: coxsackie B virus. mumps, EBV,
rube lla, rubeo la
b. p-i§let cell destrucrion by autoimmune disease: cytotoxic T cells producing
" insuliti s"
c. p-islet cell destruction by environmenta l factors:
• slreptozotocin (drug used in treating ma lignant islet ce ll tumors)
• alloxan
• pentamid ine
• children exposed to cow's milk (antibodie s against bovine albumin
cross-react against the is let cells)
(2) pancreas is devoid of Pcells
(3) islet cell antibodies in 80%
8. type II has no HLA re lationship:
(1) relative ins ulin deficiency:
a. not enough insulin to handle glucose load
b. peripheral tissue insulin res istance secondary to a receptor deficiency:
• direct relalionship wi th obesity
• more fat causes down-regu lation of insu lin receptors
c. postreCeplor defects :
• tyrosine kinase abnormalities

197
• prob lems with translocation of GLUT·4 receptors to fa ci litate
glucose absorption
(2) p ce ll s are fibrosed and often contain amy loid
6. initia l sy mptoms-
A. type I rapid onset of polydipsia. polyuria. weight loss
B. type II insidious onset, symptomatic or asymptomatic
7. ketoac.idosls-
A. type I may occur owing to insulin lack
B. type II has no ketoacidosis:
(1) susceptible to hypc:rosmolar nonkerotic coma~
(2) enough insulin to prevent ketosis but not hyperglycemia
(3) increllRd anion gap from lactic not ketoacidosis
8. trealmenl-
A. [)'pe I requires insulin
8. type II requ ires:
(I) dlo"
a. lo~ing weight most imponant Rx
b. lose adipose causes upregulation of insu lin receptors
(2) o ral glucose lowering agenlS.
(3) insulin necessary in some cases
Pathology or OM :
I. palbogeocsis-
A. nonenzymatic glycosylatioD (l'."EG):
(1) glucose combines with amino groups in proteins (e.g .. glycosylated Hb AlC)
(2) forms advanced glycosylation products (AGE) that alter vessel pt!nneabiliry and
increase atherogenesis
(3) hyaline arteriolosclerosis:
•. main cause of microvascular disease in OM
b. diabetic nephropathy
(4) macrovascular diS1ase:
•. coronary anery discase
b. periphend vascular disease
c. cerebrovascular disease
O. osmoclc damage:
(I) conve rsion of glucose into sorbitol and fructo se by aldose reductase and sorbitol
dehydrogenase, respective ly
(2) draw wnter inlo tissue leading to permanent damage
(3) periphera l neuropathy due to destruction of Schwann cells
(4) cataracts
(5) microaneurysms in diabet ic ret inopathy: damage 10 pericytes weakens the vessel
wali
2. duradoD aDd severity of disease-
A. key factors detennining organ damage
O. tight conlfOl of glucose reduces the onset and severity of complications related to
retinopathy. neuropathy and nephropathy in descending order
3. diabetic kelouidOJis (DKA)- see schematics
A. complication of [)'pC I OM

198
D. patboge nesis of byperglycemia:
(J ) reduced uptake of glucose by adipose and muscle
(2) increase in glycogenolysis due to glucagon and counterregul atory honnones
(3) in crease in gluconeogenes is:
0. primari ly a glucagon effect
b. most important factor
C. path ogenesis of ketone bod ies:
(1) increased lipolysis in adipose:
u. glucagon enhances honnone scns ili\'e lipasc-+
b. increased release of fatty acids :md glycerol-+
c. increased amount of fany acids for p-oxidalion-+
d. increased l3-oxidation of fatty ac ids leading to an excess of acetyl eoA--+
e. converted into acetoacetate and I3HB: AcAc also fonn s acetone gi vi ng
fru i[y odor to breath
D. severe vo lume depletion: due to osmmic diuresis and loss of sa lt in hypotonic
proportions
E. dilutional hyponatremia: osmotic effect of hyperglycem ia draws water imo the ECF
from the ICF
F. potassium and phosphate ion loss in the urine: osmotic diuresis
G. pathogenesis of hyperlipidem ia: reduced capillary lipoprotein lipase degradation of
chylomicrons and V LOL
R. fate of glycerol from lipolysis:
(1) converted in lhe liver to glycero l J phosphate: liver is the only organ lhat can
hand le glycerol since it has glycerol kinase
(2) glycerol 3P converted into dihydroxyncetone pbosphate
(3) OHAP is used as a substrate for gluconeogenesis
4. hypuosmolar Don-ketotic coma (HNK C)-
A. primari ly seen in type II OM:
B. presence of enough insulin to prevent ketogenesis but not hyperglycem ia
S. a cceleraled atberosclcr osis -
A. ischemic injury10 the extremities: DM is MCC non-trauma tic ampulalion of limbs
B. increased incidence of abdomina l aortic aneurysms
C. acute myocardial infarct ion: MC COD
D. atherosclerot ic strokes
6. dia betic nepbropathy- see rena l notes
7. r etinopa tby-
A. DM is tb e MCC of blindness in United Staies
B. microancurysms rupmre
C. neovascularization
8, per ipb eral neur opatby-
A. DM is the MCC of distal symmetrical polyneuropatb y in United States
B. peripberal ne uropa thy is the MCC of pressure ulcers on bOl1om of feet
9. Il ulonomic neu r opatby-
A. cardiac arrhythmias
8. gastroparesis
C. impotence
10. mucorm ycosis of the frontal sinu!'le.s lead ing 10 fronta l lobe bruin abscesses
II. m ali gnant external otitis from Pseudomonas aeruginosa

199
SUMMARY OF INSULIN ACTIVITY
GLYCOGENESISIFATTY ACID SYNTHESIS

1
Gl ucose
HMP shunt- DNA

GIUrOSe 6- P0
.
4< Glycogen synthesis
.. (glycogen synthase)
DHAP ~ Glycerol 3-P0 4 ~ VLDL (liver)

"
Pyruvate .....- - - - - Malate
malate dehydrogenase

te

OAA
:A: Acetyl CoA
citr'!!5..sjfflwse
Citrate
~~:====;~:;.:
L...

__ OAA
Citrat citrate lyase
~cetyl CoA - . Malonyl CoA ....~ Fatty acids
of inhibils ~-oxidation
Carnitine acyltransferase (mitochondria)
SUMMARY OF INSULIN ACTIVITY

LIPIDS

Chylomicrons VLDL

Capillary lipoproteill lipase/


(insulin enhaocesV

Glycero l + Fatty acids


liver glycerol killase ...
GlyceroI3-PO •
...
VLDL (stored in liver)

ADIPOSE

Glucose Glucose 6-P0 4 ··· ······~GlyceroI3-P04----. TG


(insulin +) ~
Hormone sensitive lipase
(insulin inhibits)
DIABETIC KETOACIDOSIS
HYPERGLYCEMIA

Muscle Adipose TG Glycogenolysis


(+ gluca/cat)
AI:nine
. hormonJ.ensitive lipase
~(+ glucagon/catecholamines)

1 Irallsamll1ase

Pyruvate
Glycerol + Fatty acids
L
liver glyc.ol kinase

.
Glycerol 3-PO•
DHAP
t
GLUCONEOGENESIS

HYPERGLYCEMIA
DIABETIC KETOACIDOSIS
KETOGENESIS

Adipose TG

1 hormone sensitive lipase


(+ glucagon/catecholamines)

Liver..,...- - Glycerol + Fatty acids

1
Camitille acyltrallsferase Mitochondria
-"-

~-Oxidation

,-
Acetyl eoA ...........• Ketogenesis (liver)
12. lab Dl of DM-
A. fasting glucose> 126 mgldL on 2 separate occasions
B. 2 hr blood glucose leve l >200 mgldL after 75 gm glucose c hallenge on 2 separate
occasions
C. random glucose > 200 mgldL with symptoms on 2 separate occas ions
13. home monitoring of blood glucose is ma ndatory
14. Hb Al e e,'a lua tcs loog-term glycemic cootrol-
A. represents mean glucose va lue fo r the last 3-4 mths
B. 6-7% is ideal
15. swectcoers- have no deleterious effect in OM
lmpa ired glucose tolerance: patient who does not fit the established criteria for OM but who does
have an increased risk for macrovascu lar disease and neuropathy
Gestational diabetes (GOM) :
I. refers to glucose intolerance tbat first develops during pregnancy- due to an :
A. increased placental size
B. anti-insulin effect of human pl acenta ll aclogen
2. screening-
A. all pregnant women between 24 -28 wks gestation
B. sc reened with a 50 gram glucose challenge followed by I hour gJucose leve l
C. >140 mgldL is a posi[ive screen
3. confirmatory test-
A. 3 hr 100 gram glucose tolerance lest
B. limits of the I'est set for highest sensitivity
4. newborns complications-
A. macrosomia: insul in increases adipose stores of fat and musc le mass
B. respiratory distress syndrome : insulin inh ibits surfactant producti on
C. open neural tube defects
D. neonatal hypoglycem ia:
(1) hyperglycemia in fetus causes fetal release of insu lin
(2) delivery of the baby decreases glucose level from mother but fetal insulin is still
present
E. transpos ition of the great vesse ls
Hypoglycemia:
1. types-
A. reactive type: adrenergic symptoms in fed slate hypoglycemia
(1) sweating
(2) trem bl ing
(3) anxiety
B. fasting type:
(1) neuroglycopen ic (brain w ithout glucose) in fasting state hypoglycemia:
a. di7.ziness
b. confusion
c. headache
d. inabi li ty to concentrate
2. patient on insu lin or o ral sulfonylurea compounds is MCC of hypoglycemia
3. alcobol-
A. primari ly prod uces a fast ing hypoglycemia

200
B. increased NADH produced in its metabolism converts pyruvate inlo lactate
C. less pyru\'ate for gluconeogenesis
O. decreased glycogen S10 res in severe liver disease
4. carnitinc deficicnc),-
A. camitine is a cofactor in the transport of fany acids (camitin" acyhransfcrase) into the
mitochondria
B. deficiency of camitine decreases the use of fatty acids for fuel
C. all tissues 3re glucose: dependent leading to hypoglycemia
O. absence of keto ne bodies. owing to reduced l3-oxidation of fany acids and lack of
acetyl CoA as substrate
5. ketotic hypog lycemia in ehildhood-
A. MCC of hypoglycemia from 18 months to mid-childhood
B. usually occ urs in fas ting State
C. mu ltiple etiologies:
(1) maple syrup urine disease
(2) ga lactosemia
(3) hereditary fructose intolerance
6. liver disease-
A. dec reased glycogen stores
8. decreased gluconeogenesis
7. "'OD Gie rke's glycoge n storage discue- see genetics notes

~ ..... darlne the boanl review:

Items I- 3
T. RT,Urr.BR IT.-index TSH l UI

A. lncreased Increased Increased Decreased Increased


B. lncreased Decreased Nonnal Normal Not indkated
C. Increased Increased Increased Decreased Decreased
I D. Decreased Increased Normal Nonnal Not indictlted
I E. Decreased Decreased Decreased Increased Not indicated
-
Cir A 45-year-o ld obese woman presents with a history of weight loss. weakness. and palpitations that
keep her awake a t night. She states that she has lost over 50 pounds the last 8 months while
anending a weight loss cl in ic. Physical exam reveals lid stare. a no n-palpable thyroid gland, brisk
deep tendon reflexes, systolic hypertension, and an irregu larly irregular pulse. She is nol taking any
prescriptio n medicatio ns other than the packet of pills given to her by the c linic.
Answer: C (taking excess honnone)
Qr
A 22-year-old woman complains of intennittent fluttering in her chest. Physictll exam reveals a
nonnal thyroid, no lid stare or exophthalmos. a regular hean rate of 108 beats/minute. normal deep
tendo n reflexes, and blood pressure of 100/80 mm Hg. A mid-systolic click and murmur is heard at
the apex that increases with expiration. She is currently taking binh contro l pills .
..o\.nswer: B (estrogen effect., MVP is incidenta l findin g)
Qr
A 28-year-old woman presents with complaints of chronic constipation and progressive weight
gain over the last 6 months in sp ite of being o n a pure vegan diet. She is currently on no
prescription or over-the-counter medications. Physical exam exhibits a pale young woman with

201
periorbital puffiness, dry, yellow-colored sk in, no nnal sclera, a nonnal cardiovascular and
respiratory exam, delayed deep tendon reflexes, and proxima l musc le weakness in her lower
eXTremities.
Answer: E (pri mary hypoth}To idism)
A .e Graves disease, D = Androgen effect
Items 4-5

B c
PTH

A
erum
• D
a cl u m

r:r A 55-year-old woman with d iasto lic hypertension presents with a sudden onset of right flank pain
with rad iation of pain into the right groin. A urina lysis reveals a posit'ive d ipstick for blood and a
negative dipstick ror nitrite and leukocyte esterase. Sed iment exam exhibits numerous RBCs and
sq uare crystals resembling the bac k o f an em'elope.
Answer; C (rena l stone in primary HPT H)
(ir A newborn presents with tetany. heart failure, and an absent thym ic shadow.
Answer: A (D iGeorge syndrome, could also be primary hypoparathyroid ism ). B = secondary
hyperparnthyroid ism, D = malignancy ind uced and a ll other types of hypercalcemia
r:r Wh ich of the fo llowing clin ical and laboratory abno nnalit ies occur in BOTH Add ison's disease and
panhypopituitarism?
A. Low I I-deoxycortisol post-metyrapone test
8. Nonna l urine ror 17-ketostero ids
C. Hypematremia
D. Hyperkalemia
E. LowTS H
A (choices B and C occur in neither condition: 17 KS would be low in both of them and both would
have hyponatremia; c ho ice D is only ~ present in Add ison's, choice E is on ly present in
panhypopiru itarism)
or Which of the fo llowing a lterations in lipid metabolism is expected in a pat ient with diabetic
ketoacidosis?
A. Activatio n of capil lary lipoprotein lipase
B. Increased ~x id atio n of fatty acids
C. Decreased production of acetyl-CoA
D. Increased fany acid synthes is
E. Inhibition o f hormone sensitive lipase
B (choice A occurs in the presence of insulin and is inhibited in DKA, choice C occurs when insulin is
present: acetyl CoA should be increased in DKA due to increased ~oxidation of FAs; choice D occurs
when insulin is present; it is inhibited in DKA, choice E occurs when insulin is present; it is acti vated in
DKA due to glucagon and catecho lam ines)

202
Cit" A 22.year.old woman has been amenorrheic for the last 6 months . She complains of (l mill..) '
discharge from her nipples that has been present for the last 7 mo nths. The pregnancy lest is
negative and the serum TSH is nonna l. Which of the following additional tests should you order on
this patient?
A. Serum prolactin
B. Mel)'rapone test
C. Serum T4
D. Serum conisol
E. Serum gonadotropins
A (prolactinoma)
r:;:- Which of the following tests distinguishes pituitary Cushings from both adrenal and ectopic
C ushings1
A. 24·ho ur urine for l7·ketosteroids
B. Low dose dexamethasone test
C. Serum conisollevel
D. High dose dexamethasone suppression test
E. 24·hour urine for free conisol
o (cortisol in pituitAry Cushing's is suppressed with the high dose dexumethasone Lest and nOI in other
types of Cushings; all the other choices are the same in all of them)

"'" A 3S.year.old pharma.cist presents to your office with recurrenl episodes of forgetfulness and
tiredness. A serum glucose is reponed to be 20 mgldL (70·110 mgldL). Additional studies on the
same sample reveal B high serum insulin and high C·pepide level. Based on these findings. you
suspect the patient hIlS .. .
A. a benign tumor involving p-islet cells
B. an early phase of type I diabetes mellitus
C. a benign tumor involving a-islet cells
D. ectopic secretion of an insulin-like factor
E. surreptit iously injected human insul in
A (high C.peptide proves it is an insulinoma. would be suppressed if choice E was cOlTect)
A 39 year old type I insulin dependent diabetic has a burning sensation around hi s ankles and on
the bottoms of both feel. Neurological e:<amination reveals depressed Achilles and knee jerk
reflexes bilaterally and decreased light touch sensation in both lower e:<tremit ies. The mechan ism
for this is most closely assoc:iat'ed with ...
A. osmotic damage
B. thiamine deficiency
C. pernicious anemia
D. syringomyelia
E. lumbar di sk disease
A (osmotic damage of Schwann cells produces a sensorimotor peripheral neuropathy)
" WBier deprlvat'ton test ~o r polYUria
I
POsm post waler UOsm post UOsm poSt
deprivation water deprivation vasopressin
I normaJ 292 750 760
A. 312 98 120
B. 319 110 550
C. 288 760 . ..780
A "" nephroge:nlC: dlllbclCS melll1us. c.. .. hthlUm, 8 · «ntBl dlabcte:s InsiPidus. e.g.. head InJury. C·
psychogenic: poi}'dipsjll

203
Musculoskeletal, soft tissue:

Synovial nu id ( F) analysis:
1. sec reted by syoo,'iocyte-
A. joint lubricant that is rich in hyaluronic acid
B. provides nourishment for aJ1icular cartilage
2. routine siudies-
A. gross appearance: e.g .. normally pale yellow
B. wac count and differential:
( I) "onnally <200 celisiuL
(2) neultophils <25% of the total count
C. crystal analysis
O. cu lture and gram stain ifinfeclion is suspected
J. crystal idcnllficalioo-
A. monosodium urale (MSU) crystals are needle shaped (monoc linic):
( 1) usi ng special red filter, s lide background becomes red
(2) crystals arc aJigned paral lel to the slow ray (axis) of the compensator
(3) yellow crysta l when para llel to the slow ray is a negatively birefringent MSU
crystal
yellow = negatively birefringent

'4 .. direction o f slow ray of compensator


B. calcium pyropbosphate crystals (epPO):
(1) can be needle shaped or chunk,,),
(2) blue crystal when parallel [0 the slow ray indicates a positively birefringent
ca lcium pyrophosphate crystals (C PPD)

blue::: POSilh'cly birefringent

.41-------.,.. direction of slow ray of compensator

OsleOllrlbrilis (OA):
1. Me rbeumatic disease and cau!e of join I disability in tbe United Siales
2. palbogCDcsls-
A. progressive degeneration of articular carti lage:
(1) age dependent process that is universal after 65 years orage
(2) 10 times more common in women than in men
(3) primarily targets weight bearing joints (hips. knees) and hands (distal
inlerphalangeal)
B. chondrocytes in articular cartilage requi re glucosamine in order to maintain the
macromo lecular structure of cartilage:
(I) glucosamine stimulates the synthesis of glycosaminoglycans (GAGS). which are
the pri mary components of cartilage
(2) most important factor predisposing a joint to OA is abnonnal load placed on a
weight bearing joint
C. articular surface reveals erosions and cleft fonnation: clefts penetrate into the
underlying subchondral bone (fibrillation)
O. bone rubs on bone: dense sclerotic bone resembling ivory (eburnation)

204
E. subchondral bone cysts develop beneath the articu lar surface
F. reactive: bone foonatio n occurs at the margins of the joints:
(1) produce osteophytes (bony spurs)
(2) "lipping" found in the vertebral bodies
(J) Heberdeo's nOOrs fo und at base oflhc: DtP joints of the fingers
(4) Boucbard 's node!; in the proximal interpha langeal cPtP)
J. seco ndary types due 10-
A. congenital hip dislocation
8. trauma
C. obe.sit)'
D. hemochromatosis
4. t'linical-
A. non-inflammatory joint disease
8. pain with passive motion of the joint : secondary synoviTis
C. join t stiffness/enlargemen t
O. narrow join t space
E. no joi nt fusion
5. no specific lab alterations
Neuropathic arthropathy (Cbarcot's joint):
1. joinl disuse that develoPJ secondary to a neurologic disease
2. causes-
A. diabetes mellitus (MCC)
B. syringomyelia
C. tabes dorsaJ is
Rheumatoid A.nbritis (RA) :
1. description- fema le-predominant ch ron ic-, systemic inflamrmuory disease occurring berween
30-50 ys of age
2. pathoge.ncsis-
A. microbial infections: "I Epstein-Barr virus
8. HLA-DR. '
C. inj ury-+
(I) influx of CO. T cell s/macrophages into synovial tissuc-+
(2) local stimulation of B cells to produce IgM autoantibodies directed against the
Fe receptor of IgG (rheumatoid factor- RF)-t
(3) RF aggregates into immunocomplexes that activate complement s),stcm-+
(4) chemota.xis of neutrophils into j oint space-+
(5) phagocytosis of irnmunocomple)(es (ragocytes)-.
(6) release of inflammatory mediators-+
(7) macro phages release interleukin-I (IL-I ) and tumor necros is fac tor (TNF)-o -+
(8) induce synovial cells ('0 re.letlSe inflammatory mediators that destroy connective
tissue. cartilage and bone-t
(9) chronically inflamed synovial tissue containing numerous pltlSma cells begins to
proliferate (pannus fo rmation) ond destroy the articular cartilage-t
(to) reactive fibrosis and fusion (ankylosis) of joint
3. c.linical- see table
4. lab- see table

205
r:ir Ju"enile Rheumatoid Arthritis (.IRA):
1. defioilion- chronic synovial inflammatory condition that frequents patients under 16 years
of age:
2. gi rls> boys
J. types of JRA-
A. Still's disease (20%): systemic disease with fever. rash. lymphadenopathy, polyanhrilis
B. polyarticular (40%): disabling anhritis predominates
C. p~uciarticular (40%):
(1) arthritis in few joi nts
(2) uveitis with the potential for blindness
4. lab- Rf negative (seronegative) in most cases
<if" Companson c hart 0 f osteoart h nllS . . (OA) an d r heumatot·d A rt h ntLS
·· (RA) :
I Characteristic Osteoartbritis Rheumatoid Arthritis
Classification Group I noninnammatory Group II inflammatory
Sex/Age Female dominant. Middle 10 late Female domi nanl . All uges.
decades of life.
HLA Poss ib le HLA AI. 8 8 liLA Dr4
relationships
Patho2enesis De2cnerati ve ImmunoloRic destruction
Initial silt of Articular carti lage Synovial tissue
invo lvement
Key abnonnal- Cani lage fib ril lation. subchondral lnnamed synovial tissue grows over articular I
ities bone cysts. osteophytes. secondary canilage (pannus) and releases degrad:u ivt
synovitis leading 10 reduced joint enzymes that degrade bone and cartilage.
mobility without fusion . Reactive fibrosis leads to fu sio n (ankylosis)
of the j oi nt and immobility.
Clinical Asymmetric involvement of weight Symmetric joint involvement . Involves
bearing joints and sm~11 joints of smaller joints and knee. Morning stiffness"> I
hands (DIP and PlP). Mjld morn ing hr. Targets Mep and PIP joint in hands.
stiffness. He berdeo', nodes (DlP) Ulnar deviation. Ba ker's cysts (synovial cyst)
nnd Bouchard 's nodes (PW). in popliteal fossa (confused with popl iteal
Compression neuropathies In artery aneurysms). Extraarticular disease: RA
vertebral column disease. ~'asculi' is (fingers. ankles: correlates with
high RF titers). subcutaneous (rheumatoid)
nodules (fibrinoid necrosis), pulmonary
disease (restrictive lun g disease). Caplan's
syndrome (coal worker's pneumoconiosis or
silicosis + RA lung disease), fibrinaus
pericarditis, hematologic disease (anemia
chronic disease, Iron deficiency anemia.
autoimmune hemolytic anemia). reactive
(secondary) amy loidosis , Sjogren's syndrome
eRA + dry eyes and dry mouth). Felty's
syndrome (RA + autoimmune neutropenia
and splenomegaly). uveitis. carpal [wmd
svndrome. I

206
I Characteristic Osteoarthritis Rheumatoid Arthri tis
Laboratory Slight elevati on of alkaline phospha-Posi tive rheumato id factor (RF; IgM antibody
tase from osteoph:,.1e fonnation. against IgG; positive in 70%). High RF titers
correlate with increased severity of disease.
rheumatoid nodules. greater frequency of
system ic complications. vasculitis, poorer
prognosis. Nonnal to increased serum
complement (e3). Increased erythroC)"1e
sedimentation rate (ES R). Polyc lonal
I 2ammo oathv, Positive serum A NA (30%).
X-ray Narrowing of j o int space. Narrowing of j oint space from deslructi on of
Osteophytes. Dense, sclerotic bone. articular cartilage. Marginal bone erosions.
I Subchondral bone Cysts .
..
MCP = metacarpophalangeal JOint PrP -
Fusion (ankylosis) of io int.
..
prOXimal Interphalangea l JOin t D[P d istal
interphalangeal joint

Gouty arthritis:
I. de fin ilion- male dominant disease with hyperuricemia. recurrent attacks of acute arthritis,
deposits of MSU (tophi) in soft ti ssue
2. types-
A. primary gout:
(1) disorder in uric acid metabolism
(2) underexcretion of uric acid MC
(3) overproduction less common
B. secondary gOUl:
(I) diabetes mellitus
(2) polycythemia vera
(3) leukemia
(4) diuretic therapy
(5) R.'( d isseminated carcinomas
3. initial IlUack-
A. usually invo lves metaLarSophalangeal joint (big toe): cal led podagra
B. inflammation due to an interaction of MSU crysta ls with mo nonuc lear phagocytes->
( I) stimulates macrophage production of interleukin 1 (lL- !)-Jo
(2) IL-I initiates inflammatory reaction (e.g .• fever)-+
(3) MSU crystals lyse neutrophils-t
(4) neutrophils release lysosomal enzymes and free radical s--t
(5) contribute to the inflammatory reacti on
4. chronic gout-
A. presence of tophi
B. tophi develop after 10 years of poorly contro lled disease
C. tophi represent deposits of MSU:
(1) M e occur around affected j oints
(2) granulomatous reaction with foreign body multinucleated giam cells surround ing
a central core of amorphous MSU crystals
S. compllcations-
A. defo nning arthritis with erosion of canilage and bone

207
B. renal d isease:
( I ) chron ic interstitia l nephritis
(2) uro lithiasis
6. lab-
A. demonstrat ion or MSU crystals in the SF is con firm atory. no t hyperuricemia
8. abso lute ne utrophil ic leukocytosis
7. Rx-
A. acute attacks: reduce infl ammation with :
(I) indomethacin (Me drug)
(2) colchicine:
B. inhibits release of l eukocyte~d e ri ved crysta l ind uced chemotactic facto r
b. c hemotactic facto r in itiates the inflammatory react ion in the joint when
neutrophils phagocytose MSU crysta ls
B. most are underexcretors: detennine this by n 24-hr urine co llection
( I) Rx with uricosuric agents like probenecid or sulfmpyrazone
(2) overproducers Rx witb allopurinol: inh ibits xanth ine ox idase
C. chronic disease: Rx with a llopurino l
Calcium pyrophosphate dibydrate crystal deposition artbropalby (CPPD disease), or
pseudogout :
I. defiDitioD- degenerative joint d isease w ith deposition of CPPD crysta ls in j o ints: knee MC
site
2. clinical-
A. crystals deposit in articu lar cartilage: caJled choDdrocalcinosis
B. linear densities on rad iographs
ADkylosiDg spondylitis (AS):
I. definition- seronegat'ive (rheumatoid factor negative) spondy loarthropathy thai targets
young men bern'cen the ages of 15- 30 years who are ID..A-B27 pos iti ve (95% or cases)
2. associatioDs-
A. Reiter syndrome
B. psoriasis ~
C. ulcerative co liti s
D. microbia l pathogens:
(I ) Yersinia species
(2 ) Shigella species
3. clinical-
A. targets the sacroiliac joint:
( I) sac ro iliitis is first mani festation
(2) stiffness upon wakening in AM
(3) subsides with activity
B. vertebral column becomes immobile:
(I > "bamboo spine"
(2) fusio n of the vertebra
C. aonhis: aortic regurgitat ion
D. uveil is: blurry vision
QI'" Reiter syndrome:
1. defiDition -
A. male domi nant seronegative spondyloarthropathy:

208
( 1) urethritis (usually Chlamydia rrachomalis)
(2) non-infectious conjunctivitis
(3) HLA B-27 positive arthritis
8. ... . . . .do. 111 .....rdo. of AchDles teadoe is c .......cterislk (U MLE)
Psoriatic ar'"l hriti.s :
1. ddin ition- HLA B27 positive seronegative spondylonnhropalhy thnl occurs In <10% of
p.u ients with psorias is:
2. penci l In cup derormi ty in fingers nOled on x-ray
[ nleropalhie a r1_hrilis:
1. definilion- HLA 827 posit ive arthritis associated with innllmmalory bowel disease:
2. IlJsocia lions- post-gastroenteritis with Shigella, Campy /obacter, So/molleI/o. or rersinia
species
cr Bone/soti tissue inreclions:
1. os teomyelitis In Hb S- Salmonello is MC in patients with sickle cell disease
2. oSleomyelilis-
A. metaphysis M e site: mosl vascular pan ofbolle
8. Slophylococcw; auretls MCC:
(1) neutroph ils enzymatically destroy bone and leave behind devitalized pon ions of
bone called sequeslra
(2) chronic disease has extensive reactive bone fonnalion in the periosteum cailed
in\'olu erum
(3) Brodie's abscess is a type of osteomyelius that becomes encapsu lated and
surrounded by dense sclerotic bone
C. due to h~malogenou s spread to bone
D. radionuc.lide bone sca n best for detection
E. compl icalions:
( 1) draining sinus tracts to the skin
(2) "'1_" cell carci...... d ...lopiag "Itbla doe ,I ... 'net (USMLE)
l. tuberculous oSleomyelitl5-
A. Me secondary to hematogenous extension from a primary focU5 in the lung
B. primarily tArgets the ver1~ bral column (caU~ POCl'S disease)
4. U8MLE. KftAdo: peript.eraJ Imeat. of lic.kIe celli In "atMnt wttll OIteomyeiltiJ.- >80%
of osteomye litis in sick le cell disease is due to Sa/mOlfe/la. not SI'PiI)I/ococclls aI/reus
S. 'lJSMLE ICeDario: dJabettc with wet .aagreDe or the (oot (onowed by . tttrealag of Ibe
Jaw••eck Itt"."" and Increased irritability and Increued hyperreOesla-1, dingnosis-
A. patient has tebna.
8. Rx:
( 1) tetanus immune globu lin
(2) ~nicillin
(3) give full immunization with toxoid when the patient recovers: infec tion does not
produce high enough titers of protective antibodies
6. -usMLE te:eaa&: ,.deat lte:pt OD aalI witJI .... bber Ihoes on .... Pstuc/omonas aeruginoso
infection may occur with underlying osteomyelitis
7. cat bile--
A. Pruleurt Jlo mU/foe.da infection likely:
8 . porential for septic anhritisltendinitis

209
8. disseminated gonococcemia-
A. d«reased CS-C9 :
(1) final common pathway complement components requircd for phagocytosis of the
organism
(2) GC Is Iht' MCC oheptic arthritis
8. knee Me large joint involved
C. pustules on wrists/feet
O. tenosynovitis wrists/feet
9. Lyme disease-
A. pathogeo- Borrelia burgdorjeri transmined by Ixodes lick
B. ea rly di.,Ua5e-
(1) erythema chronicum migrans:
(2) R., with doxycycline
C. lalt' diJelllft-
(1) bilateral Bell's palsy:
a. VU cranial nerve Me involved
b. . ilii;iI"BiU' i paIoy is.highly predictiv~orLym.~ '" ...
(2) arthritis
(3) myocardit is
(4) Rx with certriaxone
Osgood Scblalten:
1. ianamm.lioo of proximal tibialapophysls at insertion of patellar tcndon
2. propt.lUlry (or aclive boy.
3. permanent knobby appearing knees
Osteogenesis imperfect.:
I. usuaJly AD Inberilaoce-
A. brinle bone disease
B. Me genetic bone disease
2. patboguuis- defective synthesis of type I collagen
3. clioical- ,
A. pathologic fractures
8. ....... . . . . . . 10 riI-I'-!k!w 4!fClloroidal Yeial .Her lde:ra dft'Oid of collaceD
(tJIML~pIe....)
C. deafness
Achondrop lasia:
I. AD inherltaDce-
2. impaired enchondral ossification and premature dosure or tb e epiphyseal plates or long
boDes:
A. nonn31 sized head and venebral column
B. shonened arms and legs
Osteopetrosis:
1. AD diseue-
A. "marble bone disease "
B. secondary to an overgrowth and sclerosis of conical bone (" too much bone"): defeel in
olteadalts
2. clinical-
A. gradual replacement afthe marrow cavity by bone: severe anem ia

210
B. patho logic fractures
C. cranial nerve compression: visua l and hearing loss
D. hydrocephalus
3. Rx- bone morrow transplantation has been helpful uwing to the introducti on of normal
osteoc lasLS.
Osteoporosis:
l. MC metabolic abnormality of bone in tb e Uniled Sla tes
2. palhogenesis-
A. reduction in nonnal mineralized bone (decreased bone mass): bone subject to
pathologic frac tures
B. types:
( I) primary (e.g., postmenopausal)
(2) secondary to an underlying disease (e,g" hypercortisolism) or medication (e.g..
heparin)
3. postmenopausa l osteoporosis-
A. secondary to estrogen deficiency;
(I) estrogen serves to dam pen th e release of interleukin· 1 (osteoc last activating
factor) from osteoblasts
(2) deficiency leads to a greatcr breakdown of bone by osteoc lasIS Ihan formotion of
bone by osteoblasts
B. clinica l:
(I) Me see compress ion fractures of the vertebral bodies
(2) Colles fractu res of the distal radius
C. Ox of osteoporosis:
( 1) dual pbotOD absorptiometry (evaluates bone density)
(2) tests for bone turnover:
a. increased urinary concentration of osteocalcin

D. _110. b. increased pyridinium collagen crosslinks


or~.... ooteopo"- (USMLE):
( I) estrogen is gold standard fo r its prevention:
a. also a 50% reduction in incidence of ischemic heart disease
b. unopposed estrogen increases the incidence of endometria.l ca.rcinoma:
prevented by addit ion of progesterone
(2) calcium (1500 mg)
(3) vitam in D 400- 800 U
(~ weight bearing exercise: walking, not swimming (USML£)
E. Rx of postmenopausal osteoporosis:
( I) bisphosphonates: potent inhi bitors of bone resorption
(2) cnlcitonin·sal mon: inh ibits osteoc lasts
r::Ir Bone fractures:
I. ....oral ftftk rractare-
A. bleeds into the capsule
D. may compromise medial femoral circumnex a.rtery lead ing to avascular necrosis
(tjSMLEY
C. posterior dislocations are most dangerous
2. pelvic fracturH-
A. usually involve the symphysis, rami of the pubis, and ischium
S, bladder rupture and urethral tears are Me soft tissue inj ury

211
C. 3ssocin lC~d with the greatest amount of blood loss of all frac tures
3. scaphoid (na\; cular) bo ne-
A. M C fracturt of carpal bones
B. very susctptible to 1n'a5cular nec.ros is and non-union
4. Co llcs' fraclur~
A. person faUs on an outstretched hand
O. distal end of the radius at the suprastyloid level
C. fracture of styloid process of the ulnar
O. "dinner Co rk" d r formit)': radial fragment is displaced upward nnd backward
5. RIifMOMytar rnet1lft-
A. distal fracture of humerus
8. compromises brachial aner), with danger of Vol-••• '. ilcDeaic ~.lrKture of
foremn muscles (USM[.E)
C. median nerve injury
A"asc ula r (ou ptic) nec rosis of bo ne:
I. pa lhogenesi5- occu rs when the microci rculalion wi th in bone is disrupted lendi ng to bone
infarction
2. sites-
A. M C in II femora l head fracture in the elderly
B. SLE pat ients on long-term corticosteroid therapy
C. sickle cell discaR
3. J-ray-
A. bone has increased density on x-ray
B. MRI has highest sensitjvilY
Osteochondrosis:
1. aseptic oecrosis invoh'ing tb e o!l!ification center! of va rious bon es in children
2. Legg·Pertbe 's di.sean-
A. involves the femoral head
B. males > female5
C. 3-10 years of age ,
O. pain in knee or limp
E. secondary osteoarthritis common
Paget's disease of booe (osteitis deformans):
1. increued Ihickness or a bn orm a l bone
2. palhogencsis-
A. primarily occurs in e lderly males: unknown etiology (?s low vi rus)
B. in decreasing order il targets: pelvis > skull > femur
c. early phase of excessive osteoclastic resorption of bone
O. late phase of excessive bone formation :
(I> increased alkaline pbospbatase
(l) production of thick, weak bone (mosaic bone) without a normal lamellar
structure
3. clinlcaJ-
A. pathologic fractures
B. head enlargement
C. increased risk for osteogenic sarcomas
D. high output heart failure: arteriovenous connections in vascu lar bone

2\2
Fibrous dys plasia:
I. dclinhion-
A. lK'nign. nonneoplastic process of bone that primarily targets children and young adults
B. replacement of marrow by woven bone that lacks Ihe normal lamellar panem: subject
to pathologic frllcrures
2. si ngle bont (monostotic) type
3. polyo~lIolic type associated with A.lbrigbl's sy ndrom ~
A. abnormal skin pigmentat ion (cafe au lail spots)
8. precocious sexual development due 10 midline hamanoma in hypothalamus
C. sites: ribslfemur/cranial bones
Qr Fibrous cortical ddeetslnon ossify ing fibromas:
J. essentially tbe same d isease ucept for a difference in Jize-
A, fibrous con.ical defect 1-4 em
B. nOllossifying fibroma 5- 10 em
2. c llnle~l-
A. children
B. involve cortical aspect of metaphys is of long bones: fe mur, tib ia and fibula III
descending order of frequency
C. irregu lar, sharply demarcated rad iolucent defect in the metaphyseal cortex
CI'" Neoplastic disorders of bone:
1. MC primary bone tumors in order of increasing age- Ewing's sarcoma > osteogenic
sarcoma> chondrosarcoma > multiple myeloma
2. MC primary malignant tumors of bODe In dHCendlog order of frequency- multiple
myeloma > osteogenic sarcoma > chondrosarcoma > Ewing's sarcoma > giant cell tumor of
bone
3. metastasis is the Me malignaoey of boD ~ brellSt cancer is Ie primary site of origin
Ca rtilagiDous bone tumors:
I. osteo<:bnndroma-
A. children and adolescents
,
B. metaphys is of long bones
C. M e benign bone tumor;
( 1) lobul ated outgrowt h of bone (exostoses) capped by benign proliferating cnrtilage
(2) solitary
(3) mu ltip le ca lled osteochondromatosis
a. AD disease
b. chondrosarcoma risk greatest with multiple tumors
2. e.nchondroma-
A. children and adolescents
8 . tubular bones in hand
C. benign tumor:
( I) solitary
(2) multiple called cnchondromatosis (Oilier's disease); ehondrosarcom3 risk
greatest with multiple tumors
3. cbondrobilistoma-
A. 10-20 yrs old
B. epiphysis of distal end femur, proximallibia, humerus
C. benign rumor.

213
( 1) " popcom " appearance on radiographs
(2) extends into metaphysis
4. cho ndrosarcoma-
A. » 0 years old
8. pelvic bones > upper end femur and humerus
C. M e primary ma lignant cartilaginous tumor:
( 1) de novo
(2) secondary to osteochondromatosis or enchond romatos is
D. grade determ ines bio logic behavior
E. metastas ize to lungs
Bo ne fo nning tumors :
1. osteoma-
A. >40 years o ld
B. sites:
( 1) sinuses (M e)
(2) facial bones
C. benign tumor: association with Gardner' s syndrome
2. osteoid osteoma-
A. 5- 25 years
B. proximal fe mur: cortex of bone
C. benign tumor:
( 1) radiograph exhibits sma U radio lucent focus (nidus) surro unded by d e nse l~'
sd e rotk boae
(2) nocrurn a l pain r elieved by aspirin
3. osteobla.stoma -
A. <30 years o ld
B. vert'e brs > long bone
C. benign tumor:
( I ) hgiant osteoid o~teo ma "
(2) pain not worse at 'pighl and nOl re lieved by aspirin
4. osteoge nic sar co ma-
A. 10-25 years old
B. meta physis dista l end femu r > upper end tibia
(1) second Me primary malignant tumor of bone
(2) arises de no vo or secondary to other cond itions
(3) risk fac tors:
a. Paget 's
b. radiation
c. bone infarct
d. iaactivatio n ofRb suppressor gene o n cbro mosome 13
(4) destruction of metaphysis and invasion of subjacent soft tissue:
a. elevation of periosteum producing a Codm a n's triaogle o n x-ray
b. "s unbu rst" a ppea rance from calcified osteoid extending into soft tissue
(5) metastasize to lung
Ewing" ,a rcoma :
1. highly maligna nt prima ry bone tumor of ma rrow o rigin-
A. most frequentl y targets children in the fi rst and second decades of life
B. primitive neura l phenolype and is associated with B t( II :22) translocation

214
C. shes: femur, nat bones of pelvis and tibia
D. "round cell" tumors: differential diagnosis includes
( I ) metastatic neuroblastoma
(2) metastatic malignant lymphoma
(3) acute lymphoblastic leukem ia
2. clinical-
A. fever with locaJized heat over the tumor
B. anemia
C, onion s kinning noted around bone on x-rays
01"" Gillnt cell tum ors:
I. benign bone tumors that a rise in tbe epiphysis lind ex tend into tbe meta phys is of bone
2. fa\'ored locations in descending ord er- di stal end of the femur > proximal end or the tibia
3. patbology-
A. nonneoplastic component cons isting of mu lr.in ucleated giant cells
B. neoplastic mononuclear fib rob last-like ceHlnat determ ines the biologic behBvior or the
tumor
r:ir Mu scle disorders:
I. ry l,e I fib ers-
A. slow-twitch fiber s (red muscle)
B. rich in mitochondria and oxidative enzymes
C. capacity for long, sustained contractions without fatigue: e.g., soleus muscle
2. type 0 fi bers-
A. fast-twitch fibers (white muscle)
B. poor in mitochondria
C. gCllTed for anaerobic glycolysis
D. react to tra ini ng with hypertrophy: e.g., biceps muscle
3. muscle wea kness- causes include diseases inVOlving:
A. motor neuron pathways: e.g., amyotroph ic Interal sclerosis
B. neuromuscu lar synapse: e.g .. myasthenia gravis
C. muscle : e.g., Duchenne's musc ular dystrophy
4. neurogenic atropby- molor neuron or its axon degenerates leading to atrophy of both type I
and II fi bers
5. muscul ar dystrophy (MD)-
A. inherited progressive primary muscle disease th at Me presen L'i in early ch ildhood
o 1)<1 ' • 8 . Duchenne's :
~..r" "
'",
(I ) SXR (70%)
. (2) Me and severe MD .
(3) defi ciency of dystrophin: dystroph in nonnally anchors actin to the membrane
glycoprotein
(4) Becker' S type is milder: defective dystrophin (!!SMLE)
(5) progress ive degeneration of type I and II fi bers: progress ive fib rosis and
infi ltration of muscle tissue by fatt), tjssue (pseudohypertrophy of ca lf muscle)
(6) presents in second to fifth year of life
a. weakness and wllSti ng of proximal pelvic muscles
b. "waddling gait" with dtffi culty in standin g up
c. pillce hand on the knee to help stand (Gower's s ign)
d. death by t\lIenty years of age
e. may affect cardiac muscle

2 15
(7) laboratory:
n. antenatal diagnosis (dystrophin defect) usi ng recomb inant DN A
technology
h. serum CKlaldo lase increased at binh
• no c lin ical signs of disease
• serum CK declines as muscle tissue is progressively replaced by fat
and fibrous tissue
c. female carriers have elevated serum CK activity
C. yOlOaic: d)'ltrupby:
(I) AD disorder: Me adult MD
(2) triplet repeat 01" CTG on chromosome 19:
B. codes for a protein kinase
b. anticipation : disease worsens in future generations
(3) selective atrophy of type I fibers
(4) inilia l presentation in ado lescence with facia l weakness :
s. mouth hangs open
b. distal extrem ity muscle weakness
c. myotonia :
• inability to relax muscles (sustained grip)
• cannot release grip on a golf c lub(..USMLE)
d. frontal balding
e. cataracts
f. testicular atrophy
g. facia l muscle wasting
h. cardiac involvement
i. increased serum CK
6. co ngen ita l myopathies- primary non-progressive muscle diseases that present at birth with
poor muscle tone
A. central core disease
8. nema line rod myopathy
7. myaslbenia gravis (MGr '
A. auto immune disease: reductio n in acetylcboline-receptors due to
(1) presence of an autoantibody against the receptors
(2) type 11 bypersensitivity reaction
B. afflicts women in tbe second and third decades
C. a ffli cts men in the sixtb and seventb decades of life
D. -850/0 have thymic hyperplasia witb germinal follicles re presen ting B cell
proliferation: site of antibody synthesis
E. 15% may bave thymo mas
F. clinical:
(1) ptosis (firsl s ign)
(2) diplopia
(3) muscle weakness improved w ith rest
(4) dysphagia for solid s and liquids
G. Ox:
(1) Tensilon (edrophonium) lest:
a. inhibits acetylcbolinesterase. which increases acetylcholine
h. reverses muscle weakness
(2) assays fo r antiwacetylch oline receptor antibody

216
(3) anti-striated muscle antibodies
Il. Rx :
( I) anticholinesterase drugs : pyridostigmine
(2) surgical thymectomy (improvement in most cases)
(3) immunosuppression (corticosteroids)
(4) plasmapheresis to remove the antibody
'" Fibromatosi.s:
I. non-neoplastic. proliferatin con Dee-tin tissue duorders-
A. inliltrute tissue (usually muscle)
8. commonly rccur after surgical excision
2. DupuytreD 's cootraclure-
A. MC fibromatosis
n. invo lves pal ms of hands: contraction of fingers (usuilily the 4th and 5th)
C. MC in a lcoho lics
J. dellmoid tumor-
A. fib ro matosis of amerior abdominal \\lall in women
O. associated with :
( I) prev io us trauma
(2) multiple pregnancies
(3) Gardner's syndrome
c:r Soft tissue tumors:
l. lipo msu-
A. MC beDign soft tissue tumo r:
( I) arise in subcutaneous tissue
(2) uunk, neck. proximal extremities
B. no clinical significance
2. Iiposarcoma-
A. thigh, retroperitoneum: second Me sarcoma
8. Me occur in men >50 yellrS of age
C. lipob lasts ideDlified with fal stains
3. Obrosarcoma-
A. thigh, upper limb
n. may arise afte r radiation
4. dermaloflbroma-
A. lower extremities: elevated, red nodule that umbilicules (cen tral dimple) when
squeezed
B. benign nonencapsulated proliferation of spindle cel ls confined to the dennis
S. ma lign ani nbrous hisliocytoma-
A. Me adult sarcoma
B. Ihigh. retroperitoneum
C. associated with radiation therapy or surgical scan
6. rbabdomyomas-
A. heart tumors in childrt'n associated with tuberous sclerosis
8. also located in tongue and vagina
7. embryonal rhabdomyosucoma-
A. MC arcoma In cbildrtn
B. grape like masses protruding from the vagina or male urethra
(1) rbabdomyoblasts have cross-miations

217
(2) stain positive for desmin
8. leiomyoma-
A. M e benign rumor in women : myometrium of uterus
B. Me benign GI tumor: M e in stomach
C. uncommon outside the o rgans
9, Iciomyosa rcoma-
A. Me sa rcoma in the Gltraet and uterus
B. uncommon outs ide. organs
10. neurofibrosarcoma-
A. major nerve trunks (sciatic)
B. mOSI arise in conjunction with type I neurofib romatos is
Autoimmune diseases:
1. organ-specific-
A. Add ison's disease
8. pernicious anemia
C. Hashimoto's thyro iditis
2. syslemic-
A. SLE
B. rheumato id arthri tis (RA)
C. progressive systemic sclerosis (PSS)
Laboratory testing in autoimmune disease:
I. serum antinuclear antibody (..o..NA) test-
A. nuclear antibodies directed against:
( 1) DNA:
•. do uble-stranded Cds): specific for SLE
b. single-stranded (ss)
(2) hi stones : specific fo r drug-induced lupus
(3) acidic proteins:
a. anti-Smith (Sm):
, 100% specificity and PV' for SL E
b. anti-ri bonucleoprotein (RNP) : specific for mixed co nnective disease.
(4) nuc leo lar antigens
D. pattern of nuclear fluore scence:
(1) speckled
(2) rim : usually anti-ds DNA in SLE with glomerulonephritis
(3) homogeneous
(4) nucleo lar: spec ifi c for PSS
C. antibody titer > 1/80 is usually significant
2. lupus erythematosus (LE) ceU-
A. neutrophil with phagocytized DNA previous ly altered by IgG antibodi es
B. not specific for SLE
3. autoantibodies in autoimmune disease-
A. anti-acetylc.ho line receptor antibody: myasthenia gravis (90%)
B. anticentromere antibody:
(1) CREST syndrome (60%)
(2) PSS ( 10% )
C. anti-endomysia] antibody: cel iac disease
D. anti-gliadin antibody: celiac disease (95%)
E. anti-glomerular basement mem brane antibody:

218
( 1) Goodpasture' s syndrome
(2) against an epitope fro m type IV co llagen in the basemen! membnme of the
glomerulus and pulmonary capillaries
F, anti· insuHn and anti·islet cell anti-bodies:
( 1) anti·insulin antibodies develop in pre-type: I diabetes mellitus
(2) IUlli·jslet cell antibodies in type I diabc:les mell itus (60-90%)
(3) antibodies not present in type 1.1 OM
G. anti-microsomal antibody:
(I) Hash imoto's autoimmune th)Toid it is (97%)
(2) Graves disease
H, anti-mit ochondrial antibody: primary biliary cirrhosis (90- 100%)
I. anti-neutrophil c)1op lasmic an tibody (ANCA):
( 1) C (cytoplasmic) ANCA in Wegener' s granulo matos is (> 90%)
(2) P (perinuclear) ANCA in polynneritis nodosu (> 80%): antibody against
myeloperoxidase
J. anti-parieLaI cell and intrinsic factor antibodies:
(1) parie13.1 cell antibodies present in 90% ofpemicious anemia (PA)
(2) antibodies to intrinsic factor (IF) mo re spccilic fo r PA
K. anti·thyrog lobu lin antibody:
(I) Hashimoto's autoimmune thyroid itis (85%)
(2) Grave 's disease (30%)
Sys trmic lupus erythematosus (SLE):
1. dc: naitioD-
A. fema le dom inant disorder.
0) polyclonal aclivat'ion orB cells
(2) hormonal : increased estrogen activity
(3) environmental triggers :
a. sunlight
h. procainamide
2. clinlcsal-
A. skin:
(I) immunocomplex (Ie) deposilion of anti-DNA antibodies along the basement
membrane: ca lled band lest
(2) IC restricted to the areA of the rash in discoid lupus
(3) ICs in both normal and involved skin in SLE
(4) classic malar butterfly rash
B. joints:
(I) anhriti' or arthralgias involving small joints (c.g .. hands)
(2) Me presenting symptom
(3) morning stiffness resembling RA
c. .l'a.uular (lUeptic) necrosis of Ibe femora l head : complication of long-term
corticosteroid therapy
D. c.anliovlUcular:
( I) fibrinous pericarditis/e ffusion Me cardio\'RScular manifestation ofSLE
(2) LibmaD-Sacks endocarditis: sterile vegetations o n mitral valve
E. pulm onary disease :
(I) pleuriti s/effusion MC
(2) interstitial lung disease with a restrictive pattern

219
F. hematologic: auto immune hemo lyt ic ane mia/thrombocytopen ia/leukopenia
G. a nli-pbospbolipid antibodiC.!l :
( I) lupus anticoagulant and anti-card iolipin antibody
(2) mid-trimester feta l loss
(3) strokes
1:1. renal disease:
(1) diffuse proliferative GN Me rype
(2) correlates with presence of anti-dsDNA
(3) common COD
1. anti-SS-A (Ra) IlDtibodies in pregnancy: newborns with complete heart block
J. infection M e COD
3. laboratory-
A. positive serum ANA (99%)
B. anti-dsDNA (70%)
C. ami-Sm (30% sensitivity, 100% specificity)
O. anli- SS-A (Ro, 30%)
E. complement C3 levels are low in active disease
r. biologic false positive syphili s sero logy: due 10 anticardio lipin antibodies
4. drug-i nduced SLE-
A. Me due 10 procainamide: hydralazine also common offender
B. differs from SLE :
(I) abrupt rather than slo w onset
(2) very low incidence of renal/e NS involvement
(3) absent a n t i ~ d s DN AIanti~Sm antibodies
(4) norma l complement level s
(5) ~Ievated aDti~hi5toDe (95%) an tibodies
(6) disappears with d iscontinuance of the drug
Progres9ivc systemic sclerosi9 (PSS):
I. dcfloition-
A. fema le dom inant ~
B. initially involves smal l vessels
C. fo ll owed by excessive depos ition of no rmal co llagen in mUltiple o rgan systems
2. dinical-
A. skin:
( I) parc hment·like appearance
(2) swelling of fingerslhands
(3) tight facia l features
(4) multiple. punctate blood vesse l dilatations (telang iectas ias)
(5) Raynaud's phenomenon: Me initial sign of PSS
B. joints: polyarthritis
C. GI :
( I ) dysphagi a for so lids/liquids (lack of peristalsis)
(2) malabsorpt ion
(3) w ide~m ou thed diverticula
D. renal :
(1) glomerulonephritis
(2) severe hypertens ion
(3) renal fa ilure

220
E. pulmonary:
(1) diffuse interstitial pneumonitis with fibrosis
(2) MCCOD
F. heart : left ventricular dysfunction
J. CREST synd rome-
A. saJcinosis (fingertips) and anti-£cntromere antibod ies
B. Raynaud's phenomenon .
C. ~so phageal motility dysfuncti on
D. ~cleroda cty l )'
E. !elangiectasias
4, Illboratory-
A. positive ANA (70-90%)
D. 1IIti-8t1-10 170%): IOl!!!i!OIDCI'I5C I (I15ML!!)
C. anti-centromere antibodies (JO%)
Dermatomyositis (DM) and polymyos itis (PM):
1. dcfinilioo- both associated with an inflammatory myopathy with or without involvement of
the skin
2. clinical-
A. muscle pain and atrophy: shoulders common ly involved
B. dysphagia fo r solids/liqu ids: involves striated musc le of upper esophagus
C. puffy eyelids with a purple-red discoloration (heliotrope eyelids)
O. Gonron's patches over dorsum of PIP joints
E. increased risk for malignant neoplasms (15-20%): particu larly lung cancer
3. luboratory-
A. elevated serum creatine kinase
B. positive anti-Jo-I antibodies
C. Bx of muscle shows lym phocytic infiltrate
Mixcd connective tissue disease (MCTD):
1. features ofSLE, PSS, and polymyositis-
A. fo llows a more ben ign course than the autoimmune diseases
8. mrely involves the kidney
2. Dnti-ribonucleoprotein (RNP) antibodies are present in most cases
Sjog ren '5 syndrome :
I. definition-
A. fema le dominant di sease
B. associated with rheumatoid arthritis
2. elinical-
A. immune destruction of lacrimal/minor salivary glands:
( I) dry eyes (keratoconjunctivitis)
(2) dry mouth (xerostomia)
B. increased incidence of malignant lymphoma
C. renal : rubulointerstitial disease leads to distal renallubular acidosis (type I)
J. laboralory-
A. positive serum ANA (50-80%)
B. anti-SS-NRo (70·80%)
C. anti-SS-BILa (50-70%: more spec ific than anti Ro antibodies
D. positive rheumatoid ractor

221
E. confinnatory lesl: lip biopsy of a minor sa li vary glnnd demonst:rating lymphocytic
destruction of the glands
'Q iatIo.. ued duri .. tb. board ,"low:

c:r A sexually Ileti\'e 30 year o ld man presents with fe\'er and pain in his right big toe thllt woke him
up al night. He has been taking aspirin to relieve the pllin without relief. His mother has severe
ostwarthri tis. The right toe is swollen, hot and el(quisitely sensitive 10 louch. Laboratory studies
reveal II neutrophil ic:: leukocytosis and left sh..ift. A synoviallllp WIIS performed. Based on the above
hislory. his problem is most like ly related 10 •••
A. joint inflammation secondary to a positively birefringen t crystal
B. disseminated gonococeemia with septic anhritis
C. underexcretion of uric acid in lhe urine
D. osteomyelitis secondary to hematogenous spread of Sloph),/ocOCCIIS aureUJ
E. an ~[lA B27 positive spondyloanhropathy
C (gout, Me due to underexcrction rather than overproduction)
cr Which of the fo llowing characterizes joint disease associated wi lh rheumatoid an"hriti s rather than
osteoanhriti s?
A. Canllage fibrill ation
B. Subchondral bone cysts
C. Osteophytes
D. Ankylosis of the joint
E. Bouchard's nodes
o (all the other choices arc findings in OA)
r:r A 28 year old man who works in a summer camp in upstate ew York presents with bilateral facial
weakness involving both the upper and lower facial muscles. He also complains of joint pains in
both knees. He has a hislory a few weeks ago of having had a peculiar rash on his right thigh thai
was circular. red colored and had a centra l area of clearing. The patient'S disease is most likely
caused by a .. .
A. grum negative diplococcus
B. spirochcte
C. gram posi tive coccus
D. virus
E. immune complex reaction
B (classic Lyme di sease, due to the spirochete Borrelia bllrgdorjeri)
<7" A febrile. sexually active 23 year old woman presents with a hot, swollen right knee and pustular
lesions on the palm of her left hand. She recently returned from a campi ng t'fip in Co lorado. A
gram stain of the synovial fluid in the knee and from an aspirate o f one oflhe pustu les revea led a ...
A. gram positive diplococcus
B. gram neglltive rod
C. spirochete
D. gram negative diplOCOCCUS
E. intranuclear indus ion
o (dissemi nated gonococcemia due to Neissuia gono"hoeat, II gram negative diplococcus)

221
r:r Septic arthritis and tenosynovitis is most often associated with which one of the following
complement abnonnalities?
A. Cl esteruc inhibitor deficiency
B. C2 defICiency
C. C 3 doficiency
D. CSa deficiency
E. CS-C9 deficiencies
E (associated with disseminated gODococcemia)
<6" A veterinariao develops tenosynovitis within 24--hs of an animal bite. You suspect the offending
organism is ...
A. Yersinia ~$tl.s
B. EiU"./Ia corrodens
C. Fusoboc'<r/11111
D. Pasteurella muiloc/da
E. S,aplrylococcIU 0111"'"
D

223
Skin

r::r No rmal skin/dermatology lerml :


1. epidermls-
A. stratum basalis: actively dividing stem cells along the basement membrane
8. stratum spinosum: prominent desmosome attachments
C. stratum granulosum: granular layerw itb keratohyaline granules
D. stratum corneum: anucleate cells with keratin: site for superficial dermatophytes
2. dermi papillary and I1:ticular
J. dermalolon term5-
Term DeOa.idon Exall!l!!ts
Hyperkeratosis increased thickness of the stratum Chronic dermatitis, Iicben planus,
corneum. Clinically. it i. psoriasis, ichthyosis, verruca vulgaris
manifested as a scaly appearance (wan).
of the skin.
Acanthosis Increased ihickness of the See above examples.
epiderm is that is most commonly
due to hyperkeratosis.
Parakeratosis Persistence of nuclei io the Psoriasis.
stratum corneum layer.
Papillomatosis Spire. like proj ections from the Verruca vulgaris.
surface of the slrin often
associated with similar
projections downward into the
I papillary dermis.
Acantholysis Loss of cohesion between Pemphigus vulgaris.
squamous cells. Mosi: commonly
due to immunologic deS11Uction
of the intercellular bri.4ges.
Macule Pigmented Ar erythematous nat
lesion on the eoidermis.
Lentigo malign&." freckles (ephelides),
len!t&.o sirm?lex.
I
Papule Peaked or dome-shaped surface Acne vulgaris. pityriasis rosea.
elevation <10 mm in diameter.
Nodu le Elevated, deep dome-shaped Epidermal inc lusion eys~ basa l cell
lesion that extends into the carcinoma, erythema nodosum.
dermis.
Plaque Flanened, elevated area on the Psoriasis.
epidermis >5 mm that is often
associated with erythema and
sealing.
Vesicle Fluid filled blister <5 mm in Acute eczema, pemphigus.
diameter.
Bulla Fluid filled blister >5 mm io Bullous pemphigoid. ..aided skin
diameter. syndrome.
Pustule Fluid filled blister with Impetigo. folliculitis.
inflammatory cells.
Umbilication Central crater or indentation in a Molluscum contagiosum, basal cell
I papule or nodule. carcinoma.

224
r:7- [cze mlll:
1. ddinltion- skin lesions chamcterized by pruritus and distinctive gross and microscopic::
rutures
2. ac ule ecuma-
\\~ing. erythematous rash \\'ith ve.sicle fonnll ion
B. spongiosis (intercellular edema) in the epidennis
3. subacu te eczema-
A. crusts developing over ruptured vesicles. erythema and some scaling orlhe epidennis
D. hyperkeratos is with inclUKd thic kness or the strutum corneum
4. cbronic ec:r.ema-
lichenification: thickening due to hyperkeratosis from constant scratching
8 . scal ing
C. hy~rpigmentati on
5. atopic dumatitls-
A. type IlgE mediated disease
B. presents in nconates as a rash on the cheeks, trunk. and extensor surfaces: moves to the
nexor creases as the child grows older
6. conlAct dermlltilis-
A. innrunmatory disorder of sk.in associated with exposure 10 various antigens and
irritating substances
8. allergic- coouct dermatitis:
( I) cell mediated type IV bypt'rsensilivity reaction
(2) poison i ...),-oak-sumac
(3) nickel
(4) chemicals:
a. household cleaners
b. medications
(5) OllDlIla__ II : child de\felops a msh while running through bushes: poison
oa...___
ivy
C. Irritaot coata; t dermatitis:
(I) Me type
(2) nonimmunologic reaction due to a locall0xic effect of a chemical on the skin
(3) detergents present in soaps
O. contact photodermatltis:
( 1) type of allergic contact dennatit is that is dependent on ultraviolet (UV) light
reacting with drugs that have a photosensitizing effect
a. tetracycline
b. sulronamides
c. thiazides
(2)
L 1ItJ:JiIolIIiti __ is ... -.cyo1iJle ODd _ _ of nsI! is face, DOCk,
r......... 1IId lop (shortS)
b. 1liiie '1M'" aIkeiI wiih I bueball player (legs would not be im'olved)
E. contact urticaria:
( I) wheal and flare reaction:
(2) JgE med iated
(3) non-immunologic: reaction

225
S~ borrh~l c dermalltls:
1. definition - sca ly, often greasy type of dennatitis:
2. locutio ns-
A. scalp:
( I) dandruff
(2) cradle cap in infants •
n. face:
( I) eyebrows
(2) nasal creases
J. du e 10 a Pilyrospurum species (Malassui.afurfur) - disseminated in AlDS
S uperficia l mycoses (dermato phytosu) :
1. definition- group of fungi confined to the stratum corneum or its appendages
2. lin ea capi lis-
A. Me in children:
( I) circular, or ring-shaped patches ("ringworm") of alopecia (hllir loss) with
erythemll and sca ling
(2) MCC is Trichophyton lonsuram;: negative Wood's lamp, since the pathogen
infects the Inner hai r shaft pathogen
(J) Microsporum canis second Me palhogen: Wood's lamp positive, since the
pathogen infects the ouler hair shaft
J. other types of tinea-
A. MC caUSI! is Trichophyton rubrum:
8. KOH preparation for yeasts and hyphae
... linea venicolor-
A. caused by Molasst:lofurfur:
B. associated with areas of hyper- and hypopigmentalion after exposure 10 the sun:
scrapings reveal the classic "spagbetti (hypbae) aDd meatball (yeast)" a ppeara nce
5. RJ of dermalo pbytes-
A. topical imidazoles for body fungi
8. griseofulvin for hair/nails
Candida albicans:
I. commonly produces culaneous disease involving the skJ n-
2. MCC of diaper ras b
3. MCC of onyc bomycosis (nails)
Maculopapula r disorders:
I. dennlUon - combination of macules (nat, pigmented lesions) nnd papules (penked lesions <I
em)
2. human pa pillomaviru s (HPV)-
A. condy loma acuminata: venereal wans (HPV 6, II )
8. common warts: verruca vulgaris
J. moUUJcum contagiosum -
A. poxvirus family
8. bowl shaped lesion:
(I) ce:nlral areD. of umbilication filled with ke:ratin
(2) \·iral panicles in keratin debris
C. Me in children

226
D. disseminated in !-flV
4. measles (rubeola)-
A. clinical:
( I) feve r
(2) conjunctivitis
(3) coryza (excessive muco us productio n)
(4) Kopl ik spots in the mouth : fo llowed by maculopapul ar rash that begi ns at the
hai rline and extends down over the body
(5) not teratogenic
(6) Qliris media commo n
(7) pneumonia: Warthin·Finke ldey gian t cells
(8) subacute sc lerosing panencephal ili s
5. German measles (rubeJla)-
A. "three day measles"
8. feve r
C. arlhralgias (characteristic )
D. painfu l postauric ular lymphadenopathy (c haracteristic) 1 to 2 days prior to the onsel of
a maculo papular rash : beg ins on the head and spreads downward
E. teratogenic: sec genetics notes
6. parvovlrus 819-
A. erythema infectiosum (fifth disease):
(I) confluent macu lo papu lar rash :
(2) usuall y beginning on the cheeks ("s lapped face" appearance)
(3) extends centripeta lly ro involve the trunk
8. other parvovirus associatio ns:
( I ) pure RBe aplasia
(2) chronic arthritis
(3) repeated abortions
7. rost'Oia (exanthem subilum)-
A. caused by herpesvirus 6
8. sudden onset of tf high fever that fal ls by cris is on th e 3rd [ 0 4th day
C. followed by a macu lopapular rash that begins on the trunk and spreads centrifugally
8. ~ .liOok 'lJIdl'Ollle ('I'SJF;
A. due to a toxin producing strain of Slaphy/ococcu.r aureus : most frequent ly assoc iated
with tampon wearing menstruating women
B. 1-4 day prodrome:
( I) high feve r (>39°C)
(2) mental confusion
(3) diarrhea
(4) hypotension
(5) pharyngitis
(6) erythematous rash thai occurs during o r soon after menses:
a. diffuse, blanching, sunburned appearance
b. occurs predominantly on the hands/feel
c. reso lves with desquamation in 7- 10 d
C. R., with lIIfCillin (USMY;)

227
9. scarlet rcvcr-
A. SrrepIOCOCCIlS pyogenes that generates an erythrogenic toxin
B. erythematous rash (sandpaper consistency):
(I) begins on the trunk and limbs
(2) resolves with desquamation
C. circumoral pallor
D. tongue initially has a " white strawberry" appearance: followed by B " red strawberry"
appearance when desquamation occurs
E. association with post-streptococca l glomerulo nephritis
10. drug reoctiOD5-
A. MC skin lesion associated wi th drugs
B. e.g., penic illin . sulfa drugs
Papulosquamous disorders:
I. actinic (solur) keratol'lis-
A. pre-ma lignant ski n lesion:
(1) may progress to squamous ce ll carcinoma
(2) hyperkeratotic, pearly gray/wh ile appearance
B. induced by uve light damage on sun exposed areas like the face. hands, and forearms
C. also associated with arsenic poison ing
2. liche n planu.!l-
A. intensely pruritic, scaly. violaceous, flat-topped papules:
(1) wrists
(2) scalp
B. ora l mucosa (50%):
(1) fine while net-like appearance called Wickham's striae
(2) precursor for squamous cell carcinoma
3. psoriasis-
A. chronic disorder characterized by erythematous plaques secondary to an unregu lated
proliferation ofkeratinocytes; hy perplasia
B. pathogenesis:
(I) genetic factors
(2) environmental factors: e.g., infectio n due to streptococcal pharyngitis
(3) unregulated epidermal proliferation
(4) microci rculatory changes in the superficia l papillary demlis
(5) associated with destruct'ive arthritis ifpatiem is ffi..A-B27
C. plaques:
(1) located on scalp : Me location
(2) pressure areas like the e lbow
(3) well·demarcated, flat, e levated salmon colored lesions
(4) covered by silver-white scales: reveal pinpoint areas of bleed ing (Auspitz sign)
when picked off
(5) rash develops in areas of skin trauma : ca lled Koebner's phenomenon
(6) pilting orthe nails
D. microscopic:
(1) hyperkeratosis
(2) parakeratosis: persistent nuclei in the stratum corneum
(3) focal absence of granular layer

228
(4) regu lar panern of el ongation of the rete pegs: downward extens ions of the basa l
layer
(5) extension of papillary demli s c lose to the surface epithelium :
a. blood vesse ls located just below surface epithel ium
b. rupture when scales arc picked off (Auspitz sign)
(6) coll ections of neutrophi ls in the stratum corneum : ca ll ed Munro
microabscesses
E. R.x :
(1) topica l stero ids
(2) methotrexate in res istant cases
F. '"'l1SMLE lCe1J!!fo; pictu re of psoriatic plaques on elbow of Afri can American
4.,* pityriasis rosen-
A. presents initially as a single, oval shaped, sca ly, pink plaq ue on the trunk : cal led a
" herald patch"
B. followed in a fe w days to weeks by an eruption of papules on the trun k: rash fo llows
the lines of cleavage in a " C hristm ns tree" d istribution
r:ir Vesiculobullous disorders:
1. herpesvirus type I and n infections- see gy necology notes
2. cb.ickenpo:c: (,'a ricella)-
A. presents w ith a rash :
( 1) macule s~ vesic les -i- pustules
(2) lesions at different stages of deve lopmenl
(3) begins on the trunk and extend s centrifugally to involve the fa ce and extrem ities
B. Tzanck prep of vesicles has multinucleated squamous cell s with intranuclear
inclusions like Herpes
C. cercbel litis is commo n: self-limited
D. association with Reye's syndrome if patient takes aspirin
3. Herpes ZOSler (shi ngles)-
A. reappearance of the varicella-zoster vinls
B. remains donnam in sensory dorsa l rOOI gan gli a aft er the primary infection
C. prcsents as an" eruptio n of painful vesicles: rash follows along thc dennatomc of a
sensory nerve
D. Rx with acyclov ir
4. impetigo-
A. Streptococcus pyogf!lIes
8. rash begins on the face:
( 1) erythematous macules and progresses imo vesicles and pusrules thilt rupture to
fonn honey co lored, crusted lesions
(2) sometimes superinfected with Staphylococcus allreus: adds a bu llous component
to the lesions
C. associated with post-streptococcal glomerulonephritis
5. scalded skin syndrome-
A. neonatal disease
B. devel opme nt of bu lla (bl isters > 1 em) that rupture and leave large. red areas of
denuded skin
C. caused by a toxin produc ing Staphylococcus aureus

229
6. pemphigus \'ulgaris-
A. a utoimmune ski n disease:
(1) IgG antibodies directed against the intercellular attachment sites (desmosomcs)
between keralinocytes
(2) type D cytotoxic antibody hyperse nsitivi ty reaction
8. produces vesicles (blisters <S mm) and bulla:
(1) lesio ns occur on skin nnd on ornl mucosa
(2) vesicles arc in tracpidermal
(3) suprabasal location:
a. just above the basal cell layer
b. basal cells intact and look like a row ortombsto ncs
(4) detacbment of individua l keralinocytes from eacb otber:
a. called acant ho lysis
b. kerati nocytes are present within the vesicle fluid
(5) bulla exhibit the Nikolsky sign : outer epidermis separates eas ily from the basal
layer with minima l manual pressure
C. Rx with systemic corticosteroids
7. bullous pempbigoid-
A. autoimmune vesicular disease: IgG antibodies directed agai nst the basement membrane
8. vesicles arc in a s ubepid ermal location:
(1) negative Nikolsl..,), sign
(2) acantholysis is not present
8. dermatitis berpeti ro rmis-
A. autoimmune vesicular disease:
(I) IgA immunocomplexes (type til immunocomplex reaction) located al the tips of
the dermal papilla
(2) subepidermal vesicles with neutrophi ls
B. srrong association with celiac disease
C. anti·retic:ulin a ntibodies
9. erytbema muhiforme-
A. immunol ogic reaction )"
8. skin:
( I ) vesicles
(2) bullae with bull 's eye lesions: "target cell" o rlhe skin
C. involves mucous membranes:
(1) called Sieve D's Jobnson syndrome
(2) response to an inrection : e.g., Mycoplasma p"eumo11iae
(3) response to drugs:
a. penicillin
b. sulfa drugs
(4) response to various autoimmune diseases (S LE) o r pregnancy
l7" Pustula r disorders :
I. Staphylococcus aureus-
A. gram positive coccus associated with:
(\ ) furuncles (bo ils)
(2) carbuncles (furuncle with multiple sinuses)
(3) bullous type of impetigo
(4) scalded skin synd rome

230
(5) toxic shock syndrome
(6) hidradenitis suppurariva;
a. abscess of apocri ne glands
b. usually in the a.'ti lla
(1) paronychial infections
(8) postoperative wound or stitch abscesses
(9) postpartum breast abscesses
2. acne vulgaris-
A. chronic inflammatory diso rder involv ing the pilosebaceo us unit in the skin
B. obstructive type:
(1) closed comedones (wh iteheads)
(2) open comedones (blackheads)
C. inflammatory type:
( I) papuJesipustu leslnodu les!cysts/scars
(2) cand idate for isotretinoin acid
D. pathogenesis of obstructive type (comedones): plugging of the outlet of a hair follicle
by keratin debris
E. pathogenesis of inflammatory type:
(1) abnonnal keratinization of the fo ll icular epithelium
(2) increased sebum production (androgen controlled)
(3) bacterial lipase (Propionibacterium aClles): production of irritating fatt), acids
that produce an inflammatory reaction
F. aggravating factors :
(I) honnones (e.g., testosterone, progesterone. giucoconicoids)
(Z) drugs (e.g.. lithium)
(3) occupationa l facto rs (e .g., grease)
(4) dietary factors do nOl contribute to acne (e.g.. chocolate, nuts)
3. ac ne rosaeea- inflammatory disease of the pilosebaceous units on the face in middle aged
individuals
r:ir Urtkana:
t. definitioD- urticaril!(hives) refers to the presence of pruritic e levations of the skin
2. eauscs-
A. secondary to histamine release and other chemical mediators (type I JgE reaction) from
exposure to:
(J) foods (shell fis h, peanuts)
(2) d rugs (e.g., penici ll in)
(3) insect bites (e.g., bee sting
(4) rad iocontrast dyes
B. may be immunocomplex (rype nI) induced : serum sickness like syndrome in HBV
3. o ngioedema-
A. unlike unicaria, it is edema within the deeper subcutaneous tissue
B. produces diffuse swelling of the involved tissue:
(I) CI esterase inhibitor deficiency
(2) bradyk in in effect of ACE inh ibitors or IV vancomycin ("red man " syndrome)
cr Celluli tis:
I. cellulitis with Iympbangiitis (" red streaks")-
A. characteristic of Streptococcus pyogelles infections
B. elaboration of hyaluronidase: allows exudate to spread through the subc utaneous tissue

23 1
2. erysipelas-
A. due to Slr~prococclI.J pyogl!"f!.J :
B. raised. erythematous ("brawny edema"): hot cellul itis
C. usually on the face
Atro phy of s kin :
I. SLE a nd c bro nic d iscoid lupus-
A. produce skin lesions associated with epidermal atrophy
B. pathogenesis:
( I) anli-DNA antibodies directed against DNA trapped in the basement membrane
(planted antigen)
(2) DNA-anti-DNA immuDocomplues that deposit in the basement membrane of
the skin
Il. deposits noted in involved and uninvolved skin in LE
b. deposits only present in involved skin in chronic discoid lupus
C. microscopic:
( I) immune reaction leads to degeneration of the basal cells along the dermal-
epidemlill j unction and hair shafts: basi ~ of the band immun o nuo r l!SCenlles l
(2) lymphoid infiltratc in the same areus as we ll as around vessc ls in the papillary
derm is
(3) epiderm is atrophic
(4) follicular keratin plugs along the surface
Be nig n nodular a nd cystic diso rders :
I. s ubc utaneous myeoses-
A. acquired by puncrure wounds (traumatic implantation) oflhe skin
B. may also extend into the underlying bone
1. dilUloliln*-Jc.II-
A. verrucous (wan like) dermatitis associated with several l ~tH r.qi (USMLE}:
elicit a granulomatous reaction
8. commonly occurs in carpenters tba t get wood splinters
J. ~ -(1J MU)- ,
A~ caused by Sporolhri'f .Jchellclrii:
( I) acquired from traumatic implantation of the fungus growing in the soil
(2) commonly occurs in rose gardeners
B. clinical:
(1 ) Iymphocutaneous disease: chain of suppu rating subc utaneous nodul es
(2) R.x wi th ornl potass ium iod ide
4. Myco baclerium lep,Qt!-
A. Gcid fast o rganism:
( 1) cause of leprosy
(2) transm itted by direct contact or droplet infection
B. ruberc.uloid type :
( 1) inta ct ctllular Immunity: granulomas can develop
(2) positive lepromin skin tCst (test of cellular immunity)
(3) causes localized skin lesions with nerve involvement leading to·
a. sl..in anesthesia
b. muscle atrophy
c. aUloamputation of djgits
(4) no orga nis ms in lesions

232
C. lepromatous type:
(1) lacks cellular immunity
(2) negative lepromin skin test
(3) organisms easi Iy identifi ed in tissue within macrophages (lepra ce\ls)
(4) associated with the classic leonine facies
(5) skin biopsies:
a. narrow zone beneath the epidennis that is free of organisms called Gnu:
..,•• (USMLE)
b. underlying Grenz zone con tains foamy macrophages with organisms
(6) neural involvement is a late fea ture of the disease
S. keratoacantbom a-
A. rapid growth ofa craterifonn lesion
B. regresses and involutes with scarri ng
C. hi stologically confused with a well-differentiated sq uamous cell ca rcinoma
6. erythema Dodosum -
A. MCC of inflammation of subculaneous fal (pann iculitis)
B. raised, erythematous, painful Dod ules usually on the anterior port ioo of tbe shins
C. associations:
( I) coccidioidomycosislbistoplasmos is (USMLE)
(2) streptococcal infections
(3) sarcoidos is
(4) TBlleprosy
(5) drugs (e.g., sulfonam idcs)
Skin ca ncers associated with UV ligbt damage (excluding malignaot mela noma);
1. basal cell carcinoma (BCC)-
A. Me maligoant tumor oflbe skin
B. occurs on sun-elltposed, hair·bearing surfaces
C. locally aggressive, infiltrating cancer:
(1) arises from the basal ce ll layer of the epidennis
(2) muhifocal I-

(3) d o not metastasiz.e


(4) locations:
a. inner aspect of the nose
b. around the orbit
c. upper lip
D. appearance:
(1) raised nodules conta ining a central crater
(2) external surface of nodule is pearly-colored and has prominent vascular channels
visible beneath the surface
E. microscopic:
(I) cords of basophil ic shlining cells originating from multiple locations aJong the
basal cel l layer infiltrate into the underlying dennis
(2) fonn neatly arranged nests of malignant. basophil ic staining cells with peripheral
palisading
2. squamous cell carcinomas (SeC) of tb e s kin-
A. low but significant potential fo r metastasis:
B. located on the face : ears, nose, lower lip
C. predisposing causes:

233
( I) arsenic poisoning
(2) chronic skin ulcers
(3) sinfU tracts (panicularly chronic osteo myelitis. (lSMLE)
(4) sites of previous radiationlbum scars
(5) Me cance r associa ted witb immunosu pp ressh 'c th er a py
D. -USMLE aceD.rio: ulcer at o rifice o f chronica lly draining sinu s thai does nOI go away
with antibiotics. elc.
Epide rm al in clusio n cys t:
l. d~ rived fro m th e epiderm is of n hair follicle
2. conla ins lipid· r ic b deb r is inte rmixed w ith la mina ted kera ti n m a le n a l
Pih' T cyst (we n): scalp lesion similar 10 cpidennal inclusion cyst except for the absence of:
I. a stratum granulosum layer in the cyst wa ll
2. laminated keratin in the cyst
Mcla noc)'tic d iso rd ers:
1. vililico-
A. a Uloimmunc destructio n of melanocytes resulting in areAS of dcpigmemation
D. common in African·Amt!ricans
C. usually assoc iated with o the r autoimmune diseases
D. compare with a lbinism :
( I) me illnocytes present in a lbinism
(2) both have absence o f melanin pigment
2. seborrh eic keratosis-
A. benign epidermal tumor:
D. presents o n the skin of middle aged indi viduals
C. raised, pigmen ted lesions with a vcrruca·like surface
D. rapid increase in number called Leser-T rela l .5 ig n; possible gastr ic adenocarcinoma
3. Ilcu ntbosis nig n cll ns-
A. pigmented skin lesion commonly pre.sent in the ax illa
B. phenotypic ma rker fo r a n underlyin g a denocarcinoma of the stomach
4. freckles (epbelid es)- ;
A. pigmented macular les ions [hat occur in sun·exposed areas of the skin
S. not premalignant
C. normn l number of melanocytes along the basa l ce ll layer: increase in melanin within
indiv idual melanocytes
5. .lllo..iiWm....m. (\JSMLE)-
A. flat, hyperpigmented lesions on the fore head and cheeks
B. occur in women on oral contraceptives or who are pregnant; l~JlUk!.'
6. nevoce.l1ula r oevus-
A. benign tum or of neural cr esl.de rived cells : contain modified melanocytes of various
shapes (nevus cells)
B. begin in ea rly cbild hood as junctiona l nevi:
( I) neStS of pigmented nevus cells alo ng basa l celllllye r
(2) fla l, pigmented lesions
(3) junctional nevi deve lo p into compound nevi
C. compound nevus:
( I) nevus cells extend into underlying supe rficia l dennis
(2) both a junctional and intradennal component
(3) ra ised. pigmented, verruca-like lesio ns

234
(4) usually cbildreolad olesceoH
D. intradennaJ nevw; :
( I) develop around pubeny
(2) compound nevus loses its junctio nal component
(3) pigmented nevus cells only with in the dermis
(4) Me pevi in adulu
E. dysplastic nevus:
(I) mo re likely to develop in patients ..... ho have numero us nevi spread ove r the ent ire
body: called the d ysplastic lle\'U5 syndrome
(2) pred ispose to malignant melanoma
7. malignant melaoomas-
A. derive fro m melsoocytes
B. epidemiology:
(1) affect both sexes eq ua lly
(2) more common in w hites than African Americans
(3) predilection for fair skinned. blue eyed persons with red or blond hair
C. risk factors:
( 1) exposure 10 excessive sunlig ht lit a n early age s ingle most impo rta nt
predis posiog risk factor
(2) Hx of severe sunburn
(3) dysplastic nevus synd rome
(4) me lanoma in fi rst/second degree reliuive
(5) xeroderma pigmentosum: AR disease wilh 11 Inck of ON A repair enzymes
(6) congenital nevus
(7) lentigo ma ligna
D. radial g rowth phase:
(I) initia] phase where melanocytes proliferate laterally with in the epidermis, along
the dermoepidermaljunction. or within the papillaI)' dermis
(2) they do not metastasize w hile in tbis phase
E. ve rtica l growth phase: malignanl cells pc:netral'e inlo the underlying reticular dennis
f. superficial s prl!.adiog m el ano m a~
(I ) Me type (70%)
(2) primari ly affects women >50 years of age
(3) lower extremitieslback Me sites
(4) black, irregu lar, raised lesions
(5) foca l brown and/or red areas of discoloration
(6) foci of dep igmentation and ulceration
G. lentigo maligna melanoma (4-10%):
( I) extension of a lentigo maligna (inlraepidennallesion) inm dennis
(2) pri mari ly occur on the s u.o exposed face in eld erl)' people
H. nod ular melaoomas ( 15-30%);
( 1) lack radial growth phase
(2) directly invade into dermis
(3) poor prognosis
I. - " HP- __ (2- 8%):
( 1) located on the palms/soles/s ubungua l regioDs
(2) Me type in African Americans
(3) SPtfLE tried to pass off 8 picture of 8 subungual hematoma for an aeml
lentiginous melanoma

235
J. Breslo\\' system:
( 1) best staging system
(2) measures depth of invasion from the outermost granular layer 10 the deepest
margin of the tumor
(3) les ions with <0.76 10m of invilsion do nm metastasize
(4) les io ns with > 1.7 mm of invasion have the potential for lymph node metastasis
K. Clark system : subdivides invasions into leve ls J through V
L. overa ll 5 year surviva l regardless of type is - 80%
r::;r Acute intermittent porphyria (AlP): .\1 r' i' /'. (:/r,:r.L..
1. porphyrin s)'nthesis-
A. drugs produce SER hyperplasia lead to a dec rease in heme ( part o f cytochrome
system):
(1) barbiturates
(2) alcohol
B. heme has a negative feedback on acti vity of o-aminolevu l!n ic acid synthase (nue
limitin g e nzyme in porphyrin synthes is): see schematic
C. drugs lowering heme, cause an increase in ALA synthase activi ty
2. enzym e deficiency in AlP is uropo rphyrin ogcn synthase-
A. AD disease: uncommon fo r an enzyme deficiency
B. acc umu lation of porphobilinogen and 5-ALA prox imal to the enzyme block occurs:
preci pitates an acute porphyric attack
3. dinical-
A. abdominal pain simularing acute abdomen:
( 1) mi staken as a surgical abdomen
(2) end up w ilh a "bellyful of scars"
B. intermittent exacerbations of neurologic d ys function :
(I) psychosis
(2) neuropathies
(3) eventua l dement.,ia
4. lab- ,
A. increase in UriDe porphobilinogen :
( 1) must he ox id ized to porphobilin before wine-red color produced
(2) basis of the "window sill" lest: sunlight ox idizes porphobi linogen to porphobilin
B. increase in urine 5-aminolevulinic ucid
C. measure enzyme at My rime
5. RJC-
A. periodi c infusions of berne reduce the number of anacks
B. avoid drugs that rev up the cytochrome system
ar Porphyria cutan ea tarda (Pel):
1. acq uired disease-
A. decreased IIctivity of uroporphyrinogen decarboxylase: sec schemati c
B. net result :
(1) in crensed excretion of uroporphy rin J ( ur ine is wine-red co lor on void in g)
(2) slight increase in the formation o f coproporphyrins
(3) normal porphobilinogen levels
2. clinical-
A. photosensitive bullou.!i skin lesions
B. hyperpigmem8t ion

236
Glycine + succinyl CoA

~ ALA syothase (rare limiting)

b-Aminolevulinic acid

~ ALA dehydrase

Porpbobilinogen --.. Porpbobilin (colo r)


+
.--- 1
Uroporpbyrioogen I
Hydroxymethylbilane

Uroporpby rinogeo syntbase (J, in AlP)

+
Coproporpbyrinogco I Uroporphyrioo gen III

1 Uroporpbyrinogcn decarboxylase (,J. in


PCT)

Coproporpby rinogoo III

+
Protoporphyrioogcn IX

+
Protoporphyrin IX + iron

Heme
~ Ferrochelatase

... Hemo = +ALA syotbase activity


... Berne = • .6a.LA synthase activ ity
C. fragile skin
D. increased amounts of vellus type hair (hypertrichosis)
E. can be precipitated by alcohol
Alopecia:
1. gn elie prHiett r'min ation M CC- MC in males
2. It-lagro emuvium- when all the hairs enter the resting phase at once
A. postpartum
O. binh control pills
C. high fever
D. crosh diet
lrregul. r pi tches of bair loss In cbild:
1. trichotilloma nia
2. pullinG out of one's Own bair
3. rela tionsbip to tb e d ominant ba nd
111M(.! aeeaM'lO:· patient has hyperpigmented spots on the skin whic h when scratched become
imensely pruritic and red
I. pi llent his III"&icaria:pialU8DIOia
2. ~ au diI'.'t pressure causes the release of histamine sen ing up on urt icaria l reaction

~ ... _ dortaa do. boon! rnle..:

or A 52-year-old man with congeslive heart failUff develops cough and swelling in the deep
subcutaneous tissue. He is most likely laking ...
A. a thiazide diuretic
B. an ACE inhibitor
C. 8 phenothiazine
D. digilalis
E. a calcium channel blocker
B (mechanism is the release of bradykinin),
Qr The most common fungal cause ohinea capiris with a negative Wood's lamp is ...
A. Trichophyton ton.rurat/J
B. l\lncrosporum ca"is
C. Trlchophyto" rubrum
D. Cal/dido olhicons
E. Mo/assezio!ur!ur
A (M. co,,;s is Woods lamp positive, MCC o r tinea capitis)
~
A 43-year-old woman presentS with joint pains and a bunerfly. like rash on the race. She is most
likely taking ...
A. thiazides
B. oral contraceptives
C. hydralazine
D. doxycycline
E. barbiturates
C (procainamide is another drug that is associated with drug-induced SLE)

237
r:r A 42-year-old man has recurrent development of vesicular and bullous lesions in sun~posed
areas. He has had to avoid alcohol. because it seems to coincide with these episodes. You would
expect this patient to have .. .
A. a history of abdominal pain
B. an increase in 6-aminolcwlinic acid in his urine
C. an increase in porphobilinogen io his urine
D. colorless urine during these attaCks
E. a decrease in rod blood cell uropo<pbyrinosen decarl>uxylase
E (porphyria .utan.. wda)
r:r A 30-year-old female with a long history of alcohol and barbiturate abuse presents with diffuse
colicky abdominal pain. Examination reveals numerous surgical scars on the abdomen. There is no
history of photosensitivity. You suspect that her urine would ...
A. contain an increase in uroporphyrin
B. contain an increase in coproporphyrin
C. be positive for blood
O. have an excess amount of urobilinogen
E. tum a pon wine color after exposure to light
E (acute intenninent porphyria)

238
etalraJ Nervou. SYltemlSpecial Senses

In creand Intracran ial pr('.5.! u.re :


t. ca us('J-
A. noncommunicaring hydrocephalus (sec beloy,)
B. space occupying mass (e.g.• metaStatic Nmor)
C. cerebral edema:
<I) int:racellular (hyponatn'mia, ischemia)
(2) extracellular. increased vC'ssel pcrmeabilit) from tumor, inflammation,. trnuma.
or lead poisoning
2. clinlcal-
A. uncal herniation:
( I ) medial portion of the lemporallobe herniates through tenlorium cerebclii
(%) compression of midbrain: produces Durfl's hemorrha ges
(3) oculomotor nerve palsy:
A. eye down and out
b. pupil mydriatic
(4) compression of posterior cerebral 8r1ery: hemorrhagic infarction of the occi pita l
lobe
B. cerebellar tonsils inlo the foramen magnum: ca.rd iorespiralory arrest
l:lydroe:epbaJus:
1. denailioa- increase in CSF volume " rim distention of me ventricles
2. F fluid-
A. primarily produ~ by choroid plexus in laleral \'enlricles~
O. exits the fourth \'entricle through the foramina of lusc:hka and Magendie~
C. enters the subanchnoid space-fo
D. reabsorbed by the arachnoid granulations into the dural venous sinuses
3. co mmunlcatiag bydrocephalu.s-
A. open commun icltioq.ofCSF between ventricles and subarachnoid space
B. causes:
(1) increased production ofCSF: e.g.• cboroid plu us papilloma
(2) block in the reabsorption of CSF by tlle arachnoid granu lalions: e.g.. scarring
post-meningitis
4. nuncommunicatlng hydrocepbalus-
A. obstruction to CSF flow out of ventricles
B. ca uses: *
(1) stricture of the aqueduct ofSylyjus;·MCC 10 newborns
(1) tumors in the founh ventricle: e.g .• ependymoma
(3) Arnold-Chiari syndrome
(4) Dandy Walker syndrome
(5) blockage of CSF flow out of foramina of Luschka IUld Magendie: e.g...
inflammadon. blood clot
C. KI . .""
(I> gross of a colloid cyst in the third ventricle
(%) round object in third ventricle and c:mse of non-commun ic:.uing hydrocephalus
S. "UDical-
A. newborns: ventricles dilate and enlarge the bead circumference

239
B. Adults:
( 1) progress ive dement ia
(2) gait disturbances
(3) urinllry incontinence
6. bydrocepbalus U \'8C UO-
A. appearance of the venlric1es when the brain mass is decreased
B. e.g .• Alzheimer's disease
OpeD neural tube defects:
l. definitioo- fai lure of fusion of the lateral fo lds of the neural plate or to rupture of a
previous lY closed rube:
2. incrcas(' in maternal Q.-fe.loprote.in in serum or amniotic nuid - folic acid taken before
pregnancy protects the fetus from these defec ts
3. anencepbaly-
A. complel'e absence of th e brain
B. absence of the fe tal adrenal conex
C~ frog-like appearance
D. polyhydromnios
4. s pil.lll bifida-
A. fnilure o f the posterior vertebral arches to c lose on the 261h day of gestation
B. . pina blfida occulta: dimple or tuck of hair in the overlying skin of Ls-S,
C meningocele: spina bifida and a cystic mass containing dura and arachnoid.
O. meningomyelocele: cystic mass includi ng the spinal cord
Syringomyelia :
I. ddinhioo- fluid filled cavity (syrinx) within the cervical sinal cord
2. clinical-
A. may be acquired later in life from ischemia. inflammati on or tr3uma
B. cervicaJ cord enlargement
C. associated with Amold-Chiari syndrome
D. neurologic deficits:
( I) loS! of "Pain and temperature scosarion: disruption of the crossed lateral
spi notha lamic tracts
(2) atrophy of intrinsic muscles of the hands:
•. destruclion of anterior hom ce lls
b. confused with a myot'rophic late ral sclerosis (no sensory changes In
ALS)
(3) bUnI b••dlIaad do Dol MOl\' It (USMLE)
Arnold·Chlnri malformation: malfonnation associated with-
1. "-Iongadon of che medulla and crrebellar tonsils througb the foramen magnum
2. bydrocepbalus
3. platybasia
-*. meningomyeloc('le
S. syringomyelia
Dandy-Walker .syndrome: syndrome includes-
1. hypoplasia of the cerebellar vermis
2. cys tic diiatalioD of the fourth \'entricle
J. hydrocepbalus

240
Qr Phnkom aloses:
I. definilion-
A. neurocutaneous syndromes:
(1) disordered growth of ectodermal lis sue
(2) malfo rmations or rumors of the eNS
B. examples:
( 1) neurofibromatosis (see genetics nOles)
(2) Sturge-Weber (see cardiovascu lar notes)
(3) tubero us scleros is
2. tuberous sclerosis-
A. AD disease
B. clinica l:
( I) mental retardation
(2) skin lesions:
a. adenoma sebaceum
b. ~I!?". paltiillliAt arelijpoplpteated (USMLE)
c. periungual fibromas
d. hamanomas :
• prol iferations of astrocytes located in subepcndymaJ ponions of the
brain -+ "candlestick drippings" w ithin the ventricles
• angiomyoli pomas in kidneys
(3) ..........yo. . . . ~ ~ (lfSMLl:)
'" tofeerioG' of the CNS:
I, generul comments-
A. secondary to:
(1) hematogenous spread (MCC)
(2) traumatic implantation
(J) local extension from nearby infections
(4) ascent up the peripheral nerves: rabies
(5) ascent up o lfactory' bulb: herpes
B. types:
(I) leptomen ingil is:
•. Me type
b. hematogenous
(2) encephalitis: inflammation of brain
(3) cerebral abscess
C. leptomeningitis:
(1) acute purulent (bacterial)
(2) lymphocytic (viral)
(3) chronic granu lomatous types:
a. primary TB
b. systemic fungi
D. symptoms associated with leptomeningilis;
(I) fever
(2) headache
(3) nuchal rig idity: positive Brudzinski and Kemig's lest
(4) altered sensorium

24 1
.
E CSF fion d'I n 5 .In buc teraa ' I mcnmol IS : sec T a ble
. Uvlra
CSF cbaracleristics Bacterial Viral
TOJaI Cell Com11 1.000-20.000 ce llsiuL <1,000 ce lls/uL
Differential COli'" >90% polys First 24-48 hours, ncutroph ils, then switches
to !ympnocytes/monocytes after 48 hs.
CSF glucose Decreased Normal : except ions- mumps. herpes.
CSFprOlcill Inc reased lncreascd
Gram Slllin Frequent ly pos iti ve (sen· Negati ve
s itiv ity 75 to 80%)
F. SIS of encephahtls.
..
( 1) impainncnt o r mental status
(2) d ro ws iness
(3) intracytoplasmic Negri bodies in Purkinje cells in cerebe ll um in rabies
(4) intranuc lcar/ intracyt'Oplasmic inclus ion s:
a. CMV: Me eNS viral infection in AIDS
b. subacute scleros ing panencephatitis (SSPE) in meas les
(5) intranuclear incl usions:
Il. HSV 1 inre ctions: hemorrhagic nec rosis of the temporal lobes
b. H erpes :OSfer
c. progress ive multifocalleukoencephalopathy (PML)
G. cerebra l abscess:
( 1) adjacent focus of infectio n:
n. sinuses
b. middle ear
c. usually solitary
(2) hematogenous spread :
a. cyanotic hean d isease
b. bronchiectasis
c. usually multip le
2 summary 0 viruses associated wit
f . . h CNS'In ~ec ..IOD5 -
Pl'lIbogen Specific Or2a nism Disease/Com ments
Viruses Emerovjruses (115%): Meningitis : Vi ra l men ingitis peaks in lale summer
coxsackievirus (#1) . and early autumn. II most commonly invo lves
echovirllses. patients <40 years of age. CSF with in creased
polioviruses. I protein and Ivmphocytes. and oormalelucose .
Arborviruses Encepbalitis: Mosqui1ocs .re the vector and
vertebrates the narural host. Infections primari ly
occur in humans. St. Louis encephalitis is Me
encephalitis. English sparrows are the natural host.
Eastern equ ine e ncepha litis has lhe greatest
mortality. Wi ld birds are the natural host.
Herpes simp/ex rype 1 EncepbaJitis/MeoingitisfMcningoenccphalitis:
and /J HSV· I produces men ingoencepha litis with
bemo rrhagic necrosis of th e temporal lobes.
Intranuclear incl usions occur in neurons and glial
ce ll s. HSV·2 produces meningitis in adu lts and
encepha litis in in fants.

242
Pa thogen S pecific Orl!an is m D isease/Comm ents
VaricelJa/zOJler C roni a l nerve inn a mma tio n/ Me ningoencep ha litisl
Cerebella r infla mm ation: Herpe:> zoster (latent
virus) may invo lve the lri ge min al (V) nerve with
pai nful vesicul ar les ions (shingles) or Ihe VlI lth
nerve (Ramsay-Hum syndrome, le sions in outer ear
cana l). Va ri cella (chic kenpox) may produce self-
lim ited cerebel lar inflammation .
Rabies virus E ncepb a litis: In the United States, humans comrac t
rabies by the bite (v irus in sa li va) of a rabid skull!-
(Midwesl, South west. Califo rn ia ), fox (New
Engl and, Southwest, Alaska ), raccoon (East, New
Engla nd). o r bat (bite or aeroso lization). Rodent's
(e.g., rabbits, squirre ls) may contract rabies but not
transmit it to humans. In third world countries. most
cases are due to rabid dog bites (incl ude.s Mexican
border with USA). Viral incu bati on is 30 to 90 days.
Virus enters the eNS by ascending along peripheral
nerves.
C linica l: prodrome of fe ver. headache. exc itabi lity
and paresthes ias around the wound site. - 80% enter
into a eNS excitabi lity stage w ith pa in. convulsions,
autonomic excitability, frothi ng of the mouth.
Patien ts avo id water (hydrophobia). Eventua lly.
fl accid para lys is progresses to coma and respiratory
cenler fail ure . Neu rons (partic ul arly Purkinje ce lls)
contain intracytoplasmic Negri bod ies.
Diagnosis : serologic tests, culture of infected
material (bra in biopsy). or direct immunofl uorescem
, techniques on corneal smears, saliva. or skin .
Q uarantine dogs for 10 days (no signs of rabies, no
need to immunize the patie nt). All paliems with wild
animal bites (listed above) shoul d receive passive
(human rabies immune g lobuli n- 1/2 into wound
site) and active immunization (human diploid
vaccine). Wound must be thoro ughly , ...·sshed OUI
with q uaternary ammonium compound soaps.
Cytomegalovirus E nce phalitis: MC viral CNS infection in AIDS .
Intracytopl as mic and imranuc lear basophi lic
inc lus ions . Me congeni tal in fec tion in ch ildren
I (ence phalitis w ith periventricul ar calcifi cation).

243
Palboe:en Specific Oreanism Disease/Comments
Poliovirus MyelitislEncepbalitis: Enferovirus that specificaJ ly
anacks lower motor neurons, particularly [he
anterior hom cells of iht: spinal cord (rarely. il can
produce an encephalitis as well). Outbreaks OCCur in
... summer. Paralysis in children is less severe than
adults. Only permanent neurological residual is
lower motor neuron paralysis. Post-polio syndrome
can occur 25- 35 ys later. Sabin vaccine is nve.
SALK. is a ~ilJ ed vaccine. Cannot use live vaccine
in an immunocompromised host (AIDS), but SALK
vaccine can be used.
3. summarY of bacteria fUDllal and parasitic infections of tbe CNS-
Patboe:en S pecific On!.anis m Disease/Comments
Ba.cteria Neonatal meningitis « 1 Meningitis: All the infections are maternally
mth): • MCC is group B transmitted.. Group B streptococcus is MeC of
streptococcus (Streptococcus neonatal meningitis (E. coli #2, Listeria
aga/acNae), gram positive motlocyrogelles #3) . Group B strep may produce
coccus, • E. coli: gram infection in the first week or up to 4 months
negative rod, 2nd MeC, . later. ~b·O". kI::"l"o_id::. .~H~C:b~
LisrerJa monocytogenes. 3rd .~tenosil~
MCC
1 roth to 18 years or age: • Meni ngitis: Hemophilus Influenza was once the
Neisseria men;ngilidis- #1 cause before immunization against the
gram negative diplococcus, bacteria. N. meningir;dis now accounts for the
# 1 cause, • StrepJOCOCCUS majority of child/adolescent/young aduh
pneumoniae- gram positive meningitis. UsuaUy results in sepsis aod may
diplococcus, #2 MeC cause the Waterbouse-Frederickson syn-
:> 18 yea!} of agc! d rome. Petechial lesions are commonly present.
Streptococcus plleufflOhiae Panicularly common in crowded conditions
(boot camps). All close co ntacts m ust receive
rirampio to eradicate the car rier state,
Nfycobacterium wherculosls Meningitis/Mass lesion: TB men ingitis
(acid fast) generally involves the base of the b rain aod
produces an obliterative vasculitis with
associated cer ebral inrarctioD. Most commonly
a complication of prim ary TB in children.
Associated with post inflammatory scarring with
noncommun icating hydrocephalus. CSF has
<500 cens/uL (lymphocytes/mononuclear cells),
high p rotein (fonn s a pellicle), normal to low
glucose levels, and very low chlo ride levels.
T r eatmeDt: Syst!,:mic conicosteroids are
frequently used along with the anti-tuberculous
drugs to combat the vasculiti s and the potcmial
for strokes and hydrocephalus. A tuberculoma is
a granuloma that most commonly locales in the
cerebellum as a mass les ion.

244
P :uhol!en I Specific Ort!anism Disease/Comments
Treponema pallidum (spiro.- MyelitWEncepbalitWM,eningoencephalitis:
chete) Neurosyphilis is 'the tertiary stage of syphilis
(lOOft of patients) and is the Me overall
• manifestation of tertiary syphilis. Three typeS:
" meningovascular (25%). paretic. and tabes
dorsalis. Menlngo\'a5cular: chronic low grade
memngitis with vasculitis (increased plasma
cells around vessels) that frequently pre ents as a
sU"Oke in a young patient in the absence of
hypertension; no visible spirochetes. Paretic
form : has generalized atrophy (neuronal loss) of
the frontal cortex (organisms present), dementia,
and an ArgyJl~Robenson pupil. Tabes dorsalis:
spirochetes attack the posterior root ganglia or
their afferent sensory aKons between the
posterior rool ganglia and the cord producing
impaired joint position sense leading to a broad
based ataXia and a po iove Romberg's test
(lose balance with eyes closed. but restore
balance with eyes open), 105s of pain and
vibratiOb sensadobl Joint damage (Charcot
joints), sensory disturbances with "lightening
pains", absent deep iendon renexcs, Argyll·
Robemon pupil: pupils constrict with a near
stimulus (accommodates) but fail to react to
direct light. No spirochet'es present.
Laboratory findings in neurosyphilis: positive
VDRL (2S-50%. best initial test)~ positive CSF
FTA-ABS (80-95%), oligocionai bands
(demyelination), mild lymphocytosis, increased
Drotein. nomtal Rlucosc.
Systemic Cryptococcus neojormans Meningitis/Abscess: Me systemic fungal
Fungus infection in immunocompromise<i bost! (e.g.,
AIDS). Numerous budding yeasts (narrow based
bud) with absence of an inflammatory response,.
CSF india ink preparation (75% sensitivity).
May produce cystic lesions in the brain. Other
systemic fungi involve the CNS as we ll. Candida
(multiple abscesses), Histoplasma, Blastomyces.
Coccidioides, Mucormycosi (frontal lobe.
abscess in diabetic ketoacidosis).

245
Pathoecn SpeciJic Organism DiseaseJCommen is
Parasites Toxoplasma goudii (sporo- Encepbalitis: MCC of a space occupying CNS
zoan ) lesion in AIDS . Congenira l toxo plasmosis
produces basal ga nglia calcification .
Pia modium falciparum spo- Encephalitis: P. jalciparllm parasitizes large
rozoan) numbers of RBCs that block capillaries and
produce small necrotic and hemorrhagic foci ,
usually within the cortex and cerebellum.
Naegleria fowler i (free living ,MeDiDgoeacephalitis (USmE): Protozoal
amoebae) infection contracted by swimming in fre b
water lakes. Infects the frontal lobes through the
cribriform plate.
Trypanosoma brucei gamb- Encepbalitis: Transmitted b tsetse fl y. Diffus
ense or rhodesiel/se (hemo- neuronal degeneration . Produces sleeping
flagellate) sickness (die of starvation ). Trypanosome in the
blood . IgM elevated.
Taenia solium (tapewonn) Cyst formationfMeningitis : Cystieereos
(USMLE) is contracted when an indi idual
(intermediate host) ingests food or water
contaminated with the feces of a per on
(defmitive host) harboring the egg of T soliutn,
There is subsequent development of the
cysticerci (larva in host tissues (subc utaneous
tissue, brain eye. muscle with cyst formation .
Calcified cysts are frequent sites for Jacksonian
seizures. Hydroce~halus may oocur.
4 slow virus diseases of tbe CNS-
Type Disease Comments
Conventional Subacute sclerosing pan- SSPE is associated with the measles (nlbeola) virlls
Agent encephalitis (SSPE) either as a primary infection or as a complication of
immunization against measles (live attenuated
vaccine). Progressive disease with extensive brain
damage leading to death within one to two years.
Progressive multifocal PML, due (0 a papo avirus (JC virus. V40 virus)
leukoencephalopathy infects and destroys oligodendrocytes ( intranuclear
(PML) inclusions) leading to primary demyelination. It i
commonly observed in immunocompromised patients
(AIDS).

246
Type Disease Co mm en ts
Unconvent- Creutzfe ldt-Jakob (ell CJ presents as a r ap idly p rog ressive deme ntia. A
iona! Agent disease (subacute spong- subviral transmissible agent known as a prio n
iform encepha lopathy) (extracellular protein with no nucleic acid) has been
identified. Transmitted by cornea l tra nsp l:lnt'alion.
impro perly sterilized cortical electrodes (ncuro-
surgery), contaminated pituitary growth honnone (no
longer used), or eating human brains (called kuru :
confined to New Guinea head hunters). I< is not
transmitted perso.lLULP~ and is a health hazard to
those working with brains (ne u ro plltholog ists). A
recent outbreak occurred in England (iimad cow"
disease). Microscopically, it has a characteristic
"bubble and holes" spongifonn change in the cerebral
cortex with linle or no inflammato ry reacti on. Death
occurs within 1 'r.
9. CNS tra uRl a:
1. cer ebr a l conc ussion-
A. transieOl loss of consciousness immediately following a nonpenelrating blunt impact to
the head : e.g .. box ing
B. no histo logic evidence of damage to the brain
2. ce reb ral con tusiODS-
A. pennanem damage to small blood vessels and the surface of the brain :
(J) most commonly secondary to an acce leration-deceleration injury
(2) e.g., car acciden t
B. contusions occurring allhe site of impact are coup injuries
C. contusions occurring on the opposite side of the brain are called con trecoup inj uries:
more damage than coup injury
D. conttlsions and lacerations usually occur at lhe tips of the frontal and temporal lobes
3. diJruse axo na l inju ry- shearing ofaxons located in the white maller tracts in the brain
and/or brain stem
4. meningea l tean-
A. occur with basila r a nd orbita l skull fractures
B. frequently results in loss of CSF flu id through the nose (rhinorrhea) and/or ears
(otorrhea)
C. CSF bas a lower protein and glucose an d hig her chlo r ide tha n serum
5. acu te ep idura l he matoma-
A. definition:
(l ) fracture of tb e tempo ropa rieta l bone with severance of the m idd le meningea l
a rtery
(2) artery lies benveen the dura and inner table of bone
B, initially unconscious but then becomes lucid
C. arterial bleed ing creates a blood filled space bet\",een the calvarium and dura
D. intracranial pressure increases leading to herniation and death unless th,..
removed
6. s ubd ur al hema toma-
A. definition :
( I) co llectio n of venous blood between the dura and the arachnoid m

247
(2) most often the result of blunt trauma
(3) Me in patients with cerebral sirophy:
II. elderly patients
b. chronic alcoholics
(4) bleeding due 10 tea ring or bridgi ng veins between venous sinus located o\'er the
convexilics of the cerebral hemispheres
(5) organized blood clot OYer convexil)' of brain
B. nuctuating levels of consciousness
C. CT scans besl examination for demonstrating intracranial hemorrhages in the first 48
hs: MRJ is best after 48 hs
Vascull r Injuries in Ibe CN :
l. ne urons Ind neuroglial cells s usceptible 10 oxyge n d eprivation (hypoxia) or
hypogl)'cemia-
A. neurons most sensit ive in descending order:
( I) hippocampus
(2) cerebell um (Purkinje ce lls)
(3) layers 3, S. 6 in cerebral con ex
(4) neurons are permanently differenti ated
B. neurogl ial ce lls most sensitive in descending order:
( I) oligodendrocytes: synthesis of myelin (analogous 10 Schwann cell in peripheral
nervous system)
(2) astrocytes :
a. blood·brain-barrier
b. repair cell
c. analogous t'o fibroblast.. except collagen is not produced
(3) neuroglial cells are Stable cells and can divide
2. chronic iscbemia-
A. MC secondary to atherosclerosis involving internal carotid artery
B. laminar necrosis secondary to apoplosis of neurons in laycrs 3. S. and 6 of the cerebral
cortex: cerebral atrophy
,
C. wI\tershed inrarcts: j unctions between overlap of major arterial territories
D. cerebrovascu lar accidents (strokes)
Cerebrovascu la r Dcciduu (strokes):
1. types-
A. isc hemic (80%):
( I ) majority due to atherosclerosis
(2) infarction due to liquefactive necrosis
B. hemorrhagic: (20%):
( I) majority due to embolism (embolic stroke) from the left hean
(2) infarction due to liquefactive necrosis
C. intmcerebrol bleed (10%):
( I) intracerebral hemorrhage most ofte.n due 10 h),penen.sioQ
(2) intracerebral hematoma
D. subarachnoid hemorrhage (loo/.): Me due to a ruptured congenital berry aneul)"sm
2. a therosclerotic s trokcs-
A. +' MC due 10 platelct thrombus overlying an atheromatous plaque in the inteTTUlI carotid
near the bifurcation
8. 80% are pale Infarcts (liquefactive necrosis):

248
( I) usually in distribu tion of the middle cerebral artery (MeA)
(2) pale infarct because there is usually no reperfus ion
(3) hemorrhagic if they do reperfuse (20%)
c. microscopic:
(1) reactive astrocytes (gemistocytes) proliferate al the margins of infarci
(2) microglial cells (mncrophages of CNS) remove lipid debris
(3) res ponse 10 injury is called gliosis
(4) cystic area after 10 days 10 ) weeks
D. maj ori ty preceded by transient ischemic aHacks (TlAs):
( I) due to embolization or platelet m;. cbolesterol phlque ma terial infO the
peripheral vessels -
(2) results in transient neurologic deficiLSlasting <24 hours ~ I/I(~ f"S
(3! . defici~ nOl r.esolving within 24 hours are called strokes I*le/,i"rfv..l ,l?
E. cli nical mani fes tations of MeA occlusion: 'It!:.
itl/ ~IU . i" h"~/'1A
(1) contralaternl hemiparesis and sensory loss f (,/ j ->7,fr',h--'
(2) expressive aphasia if Broca ' s area is invo lved in the dominant (left) hemisphere
(3) amaurosis fugax:
a. neeling bl indness
b. cholesterol embolus secn in reti nal vessels: called Hollenhor.n plaque
(4) visual fic:ld defects :
a. contralateral inferior quadrantaJ'lopsia or
b. homonymous hem ianops ia
(5) deviation or tbe bead and eyes toward Ibe side or lhe lesion
F. vertebrobasilar arterial system strokes:
( I) vertigo
(2) ataxia
(3) ipsi lateral sensory loss in face and contralateral hemiparesis and sensory loss in
the trunk and limbs
G. maj ority of patients survive and recover some fu nction
3. embolic s troke-
A. thromboemboli originating in left heart or plaque material from internal carot id
B. M e associated with a hemorrhagic infarction in the distribudon of MeA:
( I ) area of infarction usually limited to the gray maller al the periphery oflhe brain
(2) hemorrhagic because ofreperfusion of the occluded vessel
(3) mortality -3 0%
4. intrllcerebral hemorrhage--
A. Me secondary to vascular changes related to hypertension:
(1) vascular changes occur in penetrating branches oflenticulolilriate vessels:
(2) fonnation of Charcot-Bouchard macroaneurys ms-t
(3) rupture of aneu rysms~
(4) intracerebral hemorrhage (c lot, not in f<!Lct)
8. locations: =
(1) basal ganglia (35- 50% in the putamen)
(2) thalamus ( 10%)
(3) pons ( 10%)
(4) cerebel lar hemispheres (10%)
C. )G-40% mortality
D. effective R.x of hype rtens ion has ils greatest benefit in preventing strokes

249
S. s ubarac hnoid bcm orrb ages-
A. majority secondary to ru pture of a co ngenita l be r ry a neurysm (80%): anerio\'cnous
malformations less common cause
B. patients (usually women) between the ages of 40-65 ys old
C. congenital aneurysms :
( 1) absence of internal elastic lamina and smooth muscle in the media of cerebral
vessels
(2) particularly occur at branching points in the vessels
(3) above histologic changes are prese nt in all peo pl e
(4) aneurysms are not prescnt at birth:
11.. develop with nonnal hemodynamic stress or
b. presence of hypenension:
• essentia l hypenension
• due to adult polycystic kidney disease
• due to increased pressure in a postductal c,oarctation of the aona
• due to renovascu lar hypertension
(4) M e aneurysm site is j unct ion of anterior communicating artery with ACA
D. clinical:
(I) sudden onset of severe occipital headache ("wors t headache I have ever ha d ")
fo llowed by a loss of consc iousness
(2) blood in subarachnoid space covers entire surface of brain
(3) - 25% die from firsl bleed
(4) addit ional 25-35% die by end of fi rst year owing to reb leeds
6. lacuna r infarets-
A. small «I cm) cystic space,s: represent areas of micro infarction secondary to:
(1) hyaline arteriolosclerosis due to hypertension (MeC)
(2) hyaline arteriolosc lerosis due to d iabetes me ll itus
8. pure m otor strokes with or without dysarthria: posterior limb of internal capsu le
C. pure se nsory l'IIrokes involve the thalamus
Demyelina ting disorders ef the eNS:
I. definition- demye linating disorders of the white mater refer only to those disorders thnt :
A. directly a ttack myelin synthesizi ng cells:
a. o ligodendrocytes in the CNS: c.g., v i rus~
b. Schwann cell s in the PNS: e.g., osmotic damage in diabetes me ll irus
B. mye lin sheet without destruction of the axon : e.g.. multiple sc leros is. Guillain-Barre
2. mul1 iple scle rosis (MS)-
A. Me demyelina ting disease: autoimm une destruction of lhe myelin sheath
8. epidemio logy:
( 1) sl ightly fe male predominant disease
(2) targets patients between the ages of20 and 40 ys
(3) d isease may be triggered at an early age by exposure to a virus : ? herpesvirus 6
(4) fILA-DR, haplotype
C. gross/micro:
(1) demyelinating plaques located in the white mater of brain/spinal cord: white
mater looks like gray mater
(2) favored locations:
a. surface of the optic nerves and optic chiasm
b. cerebellar peduncles

250
c. brain stem
d. angles of the ventricles
(3) microglia l ce lls with phagocytosed lipid
(4) reactive gliosis
(5) mild perivenular lymphoid {T cell)/plasma cell infiltrate
D. cl inica l:
( I) episodic course punctuated by ac ute relapses and remiss ions
(2) sensory and mOlOr dysfuncti on:
•. paresthes ias
b. weakness
(3) o ptic neuritis:
B. blurry vision or sudden 1055 of vision
b. MS is MCC of optic neuritis
(4) cerebellar incoordinat ion (ataxia)
(5) nystagmu s
(6) intention Lremor
(7) bilateral internuclear ophthalmoplegia: demye lination of medial longitud inal
fasciculus
(8) sconning speech: sound drunk
E. laboratory: CSF fluid findings:
(1) lymphocytes (T lymphocytes)
(2) elevated y-globulins
(J) nonnal glucose
(4) o ligoclonal bands:
a. small bands in the y-glo bulin region nOled o n high resolution
e lectrophores is
b. indicative of demyelinating disease
Dysmyelioatioo disorders (Ieukodystropbies):
I. definition-
A. genetic inborn errorspf metabolism
8. enzyme deficiencies thai result in the syn lhesis of abnormal m yelin (dysmye lination)
in the while maier
2. adrenoleukodystrophy-
A. SXR disease
8. enzyme deficiency:
(I) accumulation of long chain fany ac ids
(2) brain (dysmye lination)
(3) adrenal cortex (adrenal in sufficiency)
3. melachromatic leu kodystropby-
A. Me leukodystrophy
B_ AR di sease
C. deficiency of arylsulfarase A
4. Krabbe's diseue-
A. AR leukodystrophy
B. defic iency of galactocerebroside f3-galaclocerebrosidasc
C. accumu lation of galactocerebroside in large multinucleated, histiocytic cells (globoid
cells) in the eNS

251
Alzheimer', disease (AD):
l. MCC d ementia (-80%)-
A. muhi-infnrct dementia #2
8. Alzheimer's either presenile (<65 years) or senile (>65 years)
2. patbogc nesi5lusCK:ialjons-
A. reduced acetylcholine levels: acetylcholine plays a role in learning
B. aluminum toxicity
C. c hromoso me 21 coding for an Alzheimer pr«ursor proleln (APP), pan of \\hich is
amyloid-P (A-P) prOle in :
(J) toxic to neurons
(2) _.d. _ _ Dowol)'lldro..e (USMLE)
D. a polipoproleia geDe [, allele £4 located on c hro m o~o m e 19:
(1) produces a product that increases the neurotoxiciry of the A-13 protein
(2) cause of familial late-onset type
E. Ilbn ormulhies with chromosome 14: synthes izes 11 Tau microtubule-assoc iated
prote in located in neurofibri llary tangles
J. gross/micro-
A. cerebra l atrophy:
<I) neuronal loss in temporal, fron tal. parietal lobes
(2) neuronal loss in:
M. nucleus basalis of Meynen
b. amygdala
c. substantia nigra
d. brain stem
e.. hippocampus
(3) neurofibrillaty tangles: pairs of filaments coiled like a DNA helix
(4) senile plaques:
a.. most important lesion
b. contains a core of A-13 amyloid surrounded by neurites
(5) granulovJcuolar degeneration
4. clioieal-
A. general impairment of higher intellectu al fun ction without any focal neurologic
deficits
O. Me COD in patients with Down syndrome that reach age 40
C. clin ical diagnosis: confinna tion at autopsy
Id io pathic parkinsonism:
1. defiolll on-
A. movement disorder
8. degeneration and depigmentation of neurons in the substantia nigra and locus ceruleus
C. defi cirocy of dopam ine
D. generally occurs in patients over 45 ys of age
2. su bslanlia nigra-
A. pan of the striatal system involved in ,toluntary muscle move men, [;
( I) caudate
(2) putamen
(3) globus pallidus
(4) subthalamus
(5) thalamus
B. dopamine I! principal neurotransmitter of afferents in the nigroslrialal tract thlll
connects the substantia nigra with the caudate and putamen
C. neuronal loss with depigmentation and intracytoplasmic. eosinophilic bodies called
Lewy bod ies (example of u,biquinalion)
3. t'linica l-
A. u t ra pyram idal !l igns a Dd symptoms:
( I> cog·wheel rigidity
(%) resting tremor ("pill rolling.,
(3) bradykinesia : slowness of voluntary movement that is misinterpreted as
weakness
8. expressionless face
C. stooped posture
O. festi nating gait: progressively shortened accelerated steps
E. blepharospasm
4. Rx- replacement and prevention of reuptake and metabolism of dopamine
S. acquired cn uses of Parkinson's diseast-
A. postencephalilic Parkinsonism
O. d iscases th aI damage the basal ganglia:
(I ) isc hemia
(2) chron ic carbon monoxide poisoning
(3) Wilson 's disease
(4) drugs:
a. phenothiazines
b. reserpine
c. MPTP, a meperidine analog that damages dopaminergic neurons
Buntiag1on's disease (HD):
1. AD Inde with a delayed a ppearance or symploms uatil a mea n age or 3S-45 ~'un of
.go-
A. atrophy IUld loss of striatal neurons; )
(I) """audale nucleus (7-:' .r , _"",1<
(2) putamen
(3) fro nta l cortex
B. Iriplel repeats of CAG on the short ann of ehromoso me 4
C. decrease in:
( I) y·nminobutyric ac id (8 fa lse neurolransminer)
(2) acetylcholine
(3) substance P
2. clinlcal-
A. chorea
8. extrapyramidal signs
C. dementia
O. anticipation:
(1) progressively worse disease in future generations
(2) due 10 addition of triplet repeats
r:r Fried ~icb 's a taxia:
1. AR dlsease-
2. degencralion or-
A. spinocerebellar t:racts

253
B. posterior column s
C. pyramidal trllCts
D. periphera l nerves
3. association with triplet repeats: anticipation
4. retinitis pigmcntosum
A. tunnel vision
B. blindness
qr Amyotrophic lateral sclerosis (ALS. Lou Gehrig's disease):
l. 1055 of upper motor neurons (spasticity) and lower motor neurons (muscle atrophy.
muscle fasciculatioD.!l)-
A. some cases associated with a defect in the zinc/copper binding superoxide dismutase
(SO D1) on chromosome 21
B. oxyge n free rad ical destruction of motor neurons
2. clinical-
A. atrophy of the intrinsic musc les of the hand and forearm s with hand weakness : !'irst
sign
B. spast ic changes in the: lower legs
C. no sensory signs or bladder problems
D. ave:rnge surviva l 2- 3 ys : respiratory failure MC COD
3. Werdnig Hoffmann 's disease- childhood version of ALS
ar Wilson's di ~ease:
I. AR disease with a defect in copper excretion in bile
2. chronic li,'er disease at early age
3. decreased ceruloplasmi n:
4. f~ copper deposits in-
A. lentifonn nucleus (putamen and globus pallidus)
B Oescemer's membrane in the eye to prod uce Kayser Fleiscber ring
ar Vitamin B ll deficiency:
1. subacute combined degeneration of Ihe spinal cord
2. posterior column anti lateral corticospinnllract dem~' elinalion
Qr Alcohol abuse:
I. compl,i cations in CNS-
A. cortica l atrophy
B. Wem icke· Korsakoff's syndrome complex.
C. cerebellar atrophy: particularly Purkinje cells
D. central pOnline mye linolys is: demye lination di sease associated with rapid correction of
hyponatremia
2, Wernicke'S encephaJopatby-
A. thia.mine deficiency
8. clinical: acute onset of-
(1) confusion
(2) ata.."(ia
(3) nystagmus
(4) ophthalmoplegia
C. hemorrhages with assoc iated hemosiderin pigmentation:
(I) 'liiimillary bodiK(USMU:)
(2) third ventricle

254
(3) noor of the fourth ventric le
(") neuronal loss and gliosis involving these same areas
3. Korsakofrs psyc hosis-
A. more advanced stage of Wernicke's encephalopathy
D. targets the lim bic system
C. iO:l bility to form new mem ories (antegradc amnes ia) or recall old ones (retrograde
amnesia)
r:r Tumors of the CNS a nd PNS:
I. risk factor5-
A. Turcot syndrome: only AR hereditary polyposis syndrome
B. neurofibromatosis:
( I) optic gl iomas
(2) meningiomas
(3) acoustic neuromas (unilateral or bi lateral)
2. adult primary brain tumors-
A. 70% supratentorial
B. order of decreasing frequenc y:
(1) glioblastoma muhiforme (high gradel malignam astrocytoma)
(2) meningioma (benign)
(3) acoustic neuroma (schwannoma, benign)
(4) ependymoma in cauda equina
3. childhood primary brain tumon-
A. 70% are infratentoria l
B. in descending order of frequ ency they are:
(1) cerebellar astrocytoma (benign cystic tumor)
(2) medulloblastoma in cerebellum (malignant)
(3) brain stem gliomas
(4) ependymomas (fourth ventricle)
4. astroc)'tomlU-
A. -70% of all the neurogli al tumors
D. astrocytomas: grade I and U are benign
C. anaplaslic astrocytoma: grade, Ul and malignant
D. gliob lastoma multifonne:
(I) grade JV and malignanr
(2) 40-70 yr old
(3) '/If; frontal lobes
(4) commonly cross corpus callosum
(5) arise de novo, or by malignant transfonnation of a lower grade astrocytoma
(6) hemorrhagic tumors with rn.ultifocal areas oue.cms..is and c st;c de encration
(7) hemorrhagic necrosis with pseudopa\asading of neoplastic cells around areas of
necrosis and vascular channels
(8) seed the neura.'(is
(9) rarely metastasize outside the eNS
E. R.'t of astrocytoma: radiation
S. oligodendrogliomas-
A. benign tumor of oligodendrocyte
B. frontallobcs of adults
C. ~_ visible .a ....y1USMLE)

255
6. ependymomas-
A. benign IUmors: fourth ventricle in children
8. Me intraspinal tumor in adults :
( I ) myxopapillary panem
(2) lumbosacral portion of tbe spinal cord
(3) orientat'ion of ep~ ndym(l l cell s around vascu lD.r channels (perivascular
pseudorosenes) .
(4) ependymal cells contain blepharoplaslS
7. meduUoblastomlils-
A. Me primary malignanc,' of brain in children: second Me primary brain tumor in
ch ildren
8 . malignant primitive neuroectodennaJ tumors:
( I) arise in the midline of tbe cerebelJar vermis from the fetal extemal granular
layer
(2) seed neuraxis
(3) invade fourth ventricle
8. meningiomas-
A. Me benign tumor of brain in adults :
( I) arise from aracbnoidal cells
(2) second Me primary brain lumor in adults
(3) women > men
. . 10: I
B. associations:
( 1) 4 neurofibromatosis
(2) deletions on the long arm of chromosome 22
(3) previous Hx of radiation
c. locations:
(I) convexities of brain: parasaginal Me site
(2) olfactory groove
(3) lesser wing of the sphenoid
(4) spinal cord (thoracic segment)
O. gross/micro: ;
(1) finn tumors
(2) may indent (not invade) the surface of brain
(3) common cause of new onset seizure acth'iry in aduhs
(4) infiltrate over lying bone:
a. not a sign of malignancy
b. visible hyperostosis on a skull :<-ray '*
(5) swirling masses of meningotbc:lial cells encompass ing psa mm oma bodies
(calcified bodies) : _~ benign tumor 'rimpsammoma bodjes
9. eNS lym phoma5-
A. MCC are metastatic high grade non-Hodgkin's lymphomas of B lymphocyte origin :
meningeal spread and spare parenchyma
B. primary eNS lymphomas:
( I) muhifocal in their distribution
(2) most often associ ated with AIDS : HIV and [BV mediated
(3) also associated with immunosuppression in renal transplant patients
(4) incidence increased over the last decade due to AIDS

256
10. schwan nom as (neurilemomas)-
A. benign tumors derived from Schwann cells:
(I) cranial nerves
(2) spi na l nerve roots
(3) peripheral nerves
B. Me intracraoial s ile is the cerebelJoponfine angle:
( I ) invo lvement o f the V lIl th crania l nerve: called acoustic neuroma
C. acoustic neuroma:
(I) tinnitus (MC symptom)
(2) sensorineural deafness
(3) sensory changes in trigeminal nerve distribution
O. microscopic :
( 1) "zebra tumor"
(2) compact Antoni type A areas interspersed with loosely structured myxolllatous
appearing areas designated Antoni type B areas
E. MLE scenarios: CT with en larged internal acoust.ic meatu s in a patient with
tinnitus, nerve deafness
ll. mefastasis is Me eNS cancer-
A. lung cancer MC primary site
B. women non-smoker: consider b reasl
(jf" CNS and PNS abnormalities in AIDS:
I. HIV enters the eNS "ia macrophages/monocytes (reservoirs of mY) and astrocytes-
microgl ial ce lls are the reswoir for HfV in the eNS
2. * primary eNS Iymphoma- see previo us discuss ion
J. aseptic meningitis
4. eDcephalitis-
A. AIDS dementia complex (ADC):
(I) motor impainnent (e.g., spasticiry)
(2) cognitive deficits
(3) neurological deficits
, (e.g., memory loss)
(4) behavioral or neuropsychological impairment (e.g.. hallucinatio ns)
O. multinucleated microglial cells is a characteristic feature
5. vacuolar myelopathy-
A. subacute combined degeneration of spinal cord
B. similar to 8 11 deficiency but not due to B I, deficiency
6. peripheral neuropathy
7. opportunistic infectioDs-
A. CMV: Me CNS virus
B. toxoplasmosis:
( I ) Me panlSite
(2) MeC space occupying lesions
C. C. neojormans: Me funga l eNS infect ion
Peripheral neuropathies :
I. rypes-
A. demyelination (often segmental):
(1) sensory changes
(2) symmetrical "glove and stocking" distribution of sensory loss
(3) burning foot" syndrome

257
B. a.xona l degeneration : musc le atrophy and rasciculations
C. combinations of the two
2. C.Iluses-
A. dia betes mellitus:
(I ) MCC
(2) osmotic damage of Schwann cells
D. toxins:
( I ) alcohol
(2) heavy metals (e.g .. Pb)
(3) diphtheria
C. amyloidos is
D. drugs:
( 1) isoniazid (B6 deficiency)
(2) vincrlstme
E. nutri tional defic iencies:
(1) thiamine
(2) pyridoxine
3. G uill aiD ~ B a rre syn drome-
A. MCC llcute peripbe ra l ne uropathy: autoimmune demyelination syndrome
B. presenlS with an ascending or descending paralysis
C. high CSF protein leve ls + few lymphocytes
D. Rx: ~Iiilif'ph..... is <YS~}
4. C ba r cot-Ma n e-Tooth disease-
A. Me generic ( AD) peripheral neuropathy
8. peroneal nerve
C. atrophy of muscles of lower legs
D. inverted bonle appearance
5. WaUerian degener ation- see cell injury noles
6. id ioplltbic Bell 's pa lsy-
A. acute onset of unilatera l facial nerve (VUth nerve) pa ralysis
B. HSV I may be implicated
C. Ly me 's disease ir bilateral: Vll th nerve Me cronin I nerve involved
D. clinical:
(1) drooping of the comer of the mouth
(2) difficulty with speaking
(3) inabi lity to close the eye
(4) d rooling
Orbital cellulitis vs caverno us sin us thro mbosis:
l. botb bave proptosis o f tbe eye and pno r eye mO\'emenlS
2. caverno us sinus thro mbosis bas papilledema
T reatm ent of CMV r etinitis in AIDS if ganciclo\'ir does not wo rk:
1. fosca ru et (r enal toxicity)
2. CMV reti nitis MCC of blindness in AIDS
Mcnetrier d isease:
I. inc r eased e ndolr mph in inner ea r
2. diu.incss
3. vertigo
4. sensory hea nng loss

258
5. borizontal nystagmus
Benign positional vertigo:
1. MCC recurrent vertigo
2. no bearing los5 or tinnitus
3. nys tagmus
4. due to dislocation of otolitbs
ConjuDctlvitis:
I. bacleri:ll-
A. SiophylococCU5 OllreU$
B. fl. ifljlucllzae
C. Slrep'ococcu:; p"eumoniae
2. \'ir81-
A. adenovirus
B. HSV I: dendritic ulcers
3. IIlJergic- sensonal
r:r Gillucomll:
1. increase in intraocular pressure- nuid cannot exit cana l of chlcmm
2. clinical-
A. pa in
B. blurry vision
C. pupil fixed and mid-dilated
D. o ptic atrophy and blindness if not u eated
or S udden unilateral loss in vision:
1. amaurosis rugax-
A. embo lism of atherosclerotk plaque to retina l vesse l
B. "curtain down and then up"
2. retinal artery occlusion- embolism MCC
3. retinal vein occlusion- po lycythem ia MCC

I. innammation of uveal tract (cboroid, ciliar), body, iris)-


A. clinical:
(1) pain
(2) blurry vision
(3) severe photophobia
(4) congestion of ciliary vessels around cornea
(5) miotic pupils
(6) poor light reflex
(7) normal pressures
2. "'_'"'.-.''"''(USMLE)-
A. ankylosing spondylitis
B. rheumatoid arthritis
C. sarco idos is
D. j uvenile rheumatoid arthritis
Optic n~uritis:
l. innummlltion of optic nerve
2. causcs-

259
A. multiple sclerosis MCC
B. methyl alcohol poisoni ng
J. clin ical-
A. pain
B. blurry vision
C. name hemorrhages around nerve
D. danger of oplic atrophy:
(1) cupping of disk margin
(2) pale disk due to absence of disk vessels
Mac ulur degeneration: MCC of bl indness in elderly
Webe r Icst/Rinoe test:
I. Weber test reveals tuning fork sound IDteralizin g to affected ear in conduc tion
deafncss- Rinne test shows bone conduction > ai r conduction in thai ear
2. Weber lest reveals latera liu lion of so und 10 norm a l ear in sen!lorineu ral bearing loss-
Rinne test shows preservation of air > bone conduclion in both nonnal and abnonnal ear
whi ch means thai the opposite ear is affected ear
Presbycusis: MCC of sensorineura l hearing loss in elderly
O losclerosis:
I. MCC of cond uction deafn ess in elderly
2. middle ea r ouides fuse
O titis media:
1. Me conduction d eafness In children- pathogens in descend ing order:
A. S freptococcus pneumoniae
8. Hemophilus injluenzae
C. Moraxella catarrhalis
2. RJ: with amoxicillin
Qr Abseoce seizures:
l. abrupl onset or impaired consciousness (5lo re 1010 s pace)
2. 3-Hz spike and W5rVe activity on EEG
3. ethosuximide Rx of choice

QaestiODl ased durial tbe board review:

Cognitive abnom13lities, ataxia, nystagmus, ophthnlmoplegia, and foot drop are all associated
with . ..
A. folate deficiency
B. multiple sclerosis
C. Alzheimer's disease
D. alcoholism
E. Parkinson's disease
D (a ll of the SIS are due 10 thiamine deficiency and Wern icke's encephalopathy)

260
or A family history of chronic liver disease beginning 01 an early age and a movement disorder
developing later in lifc characterizes a disease associated wi th ...
A. a triplet repeal mutation
B. low ceruloplasmin levels
C. thiamine deficiency
D. excess alcohol intake
E. vitamin BI ~ deficiency
B (Wilson's disease)
.,. A 40·year..old neuropathologist developed a rapidly progressive dementia and died. The
pathogenesis of his disorder is related 10 .••
A. Ii decretlse in acetylc holine levels
B. a deficiency of dopamine
C. neuronal damage by amyloid
D. a slow virus disease: involving prions
E. subacute sclerosing panencephalitis
D (J·K diseAse)
r::r A 15·year-old boy develops feve r, nuchal rigidity, and petechial lesions. A spina l fluid revea ls
increased protein. decrcased glucose, increased neutrophils, and a pos iti ve gram sin in. You would
cxpecllhe CSF gram stain to reveal. ..
A. gram positive diplococci
B. gram negalive coccobacilli
C. gram positive rods
D. gram negative diplococci
E. gram positive cocci
D (Neisseria mtningitidis)

" Which of the followi ng represents a primary brain tumor that would more likely de\'elop in a child
rather than an adult?
A. Glioblastomn. mulliforme
B. Medulloblastoma
C. Malignant lymphoma ,
D. Acoustic neuroma
E. Meningioma
B
r:r An afebrile 52 year old smoker with weight loss, cough, and hemoptysis, develops a severe
headache associated with unilateral lid lag. ophthalmop legia, and mydriasis. An MRI reveals
mult iple densities in the cerebral cortex . The pathogenesis of th is patient'S neuro logic condition
most close ly relates 1'0 •• •
A. a demyelinating disease:
B. primary CNS tumor
C. embolic stroke
D. cerebral edema with uncal herniation
E. cerebellar hemiation into the foramen magnum
o (patient has metastatic lung cancer 10 the brain)

261
c;;r An autopsy is performed on a 40-year-old man with Down syndrome. Examination or the brain
revea ls atrophy of the frontal and temporoparietal lobes. The pathogenesis of the eNS disease in
this pat ient is most closely related to ...
A. JJ- amyloid protein
B. lacunar infarcts
C. decreased dopamine levels
D. a slow \.jrus disease
E. a triplet repeat disorder
A (class ic Down syndrome-Alzheimer's disease question)
<if" Recurrent episodes of blurry vision, scanning speech. nnd partsthesias is most close ly associated
with ...
A. multiple sclerosis
B. Wernicke's encephalopathy
C. slow virus disease
D. Hunt ington'S di sease
E. Wilson's disease
A
Cir An encapsulated mass is removed from the right cerebellopontine angle of a 32-year-old male with
right-sided sensorineural hearing loss and facial numbness in the distribution of the trigeminal
nerve. The palient most likely has alan .. .
A. ependymoma
B. medulloblastoma
C. demyelinating disease
D. meningioma
E. acoustic neuroma
E (sensory changes 8re due 10 V[Dlh nerve rumor in cerebellopontine angle impinging on the Vth eN)
(7 A 45-year-old woman with a history of mitral stenosis and 11 chronic arrhythmia died suddenly at
home. At autopsy, a hemorrhagic lesion was nOled at the periphery of the temporal lobe. The
ptlt'ient most likely has alan .. .
A. atherosclerotic stroke
B. intracerebral hematoma
C. embol ic stroke
D. glioblastoma multi forme
E. A V malformation
C (atrial fibrilhnion is the MC arrhythmia)
<if" A 62-year-old man complains of weakness in his left arm and tremor in his hands that prevents him
from writing legibly. He has a shuffiing gait when he enters the examining room. His deep tendon
reflexC's arc normal. The patient ' S clinical find ings most closely relate to ..
A. amyotrophic laleral sclerosis
B. atherosclerotic stroke
C. pure motor stroke
D. Parkinson's disease
E. post-poliomye litis syndrome
o

262
r:7' A 30·year.-old woman states that she burns her hands without feeling any pain. Phys ical exam
reveals decreased pain and temperature sensation in the upper extremities. atrophy of the intrinsic
muscles of his hand. and abnonnal deep tendon reflexes in the upper extremity. The patient most
likely has ...
A. S)Tingomyel ia
a. multiple sclerosis
C. amyotrophic lateral sclerosis
D. spinal cord tumor
E. au deficiency
A (classic question between syringomyelia and ALS)
r:r A febrile 2S·year..old man with AIDS and a CD4 T helper count of 50 celisiuL develops focal
epileptic seizures. A CT scan reveals multiple ring enhancing lesions in the brain. The
pathogenesis of this patient's CNS disorder is most closely related to ...
A. cytomegalovirus
a, cryptococcosis
C. toxoplasmosis
D. Epstein· Barr virus
E. disseminuted MAJ
c
c:r Which of the following is more often associated with Esc},ericl,,'u coli than Pseudomonru
aeruginosa1
A. Osteomyelitis from puncture wounds through rubber footwear
B, Malignant otitis externa in a diabetic patient
C. Pneumonia in an in tensive care unit
D, Death in a patient in a bum unit
E. Men ingitis in a newborn
E
r:r A SS y~ old man has broad·based ataxia and loss of pain and temperature sensation in the lower
extremities. Both pupils accommodate but do not constrict with direcll ight stimulation. You would
e;(ptct the spinal nuid to exhibit ...
A. encapsulated yeast with narrow·based buds
B. It neutrophil dominant cell count
C. It positive VORL
D. spirochetes
E, xanthochromia
C (tabes dorsalis)

263
nafomy Qu dollS:

I. Cap Junction: dye passes from one cell to the next


2. Oerintion of adrellRl medulla: neural crest origin (SI00 antigen positive); neuroblasts develop
into ganglia; know the layers of the adrenal gland from outside in: c-onex.: glomeruloS&. fuciculata,
rellcularis: medulla
3. Tibial aer n fuactlon: plantar flexiod of toes; injury: lou of plantar flexion, foot dorsinexed and
evened (caleaneovalJUS-Cavus). senJO<)' loss on sole of fOOl
4. Composition or lortic val\'e (also pulmoDic va lve): lined by ~dothelium and have abundant
fibroeJastic tissue plus. dense collagenous corcj a\·asc:uJar. MV and TV have a loose connective
tissue core which ~ Increased in mitralltricuspid valve prolapse (myxomatous degeneralioa)
S. Break bume.,.., wrist drop: radial nerve injury
6. Po l·radltal mastectomy-wi nced sapula: indicates Injury to the long tboracic aen'e; p&nllysis
or the serratuS anterior muscle
7. Pallenl with. aosebleed and rbinorrbea: fracture or cribriform plote in ethmoid sinus
8. Mediallongltudlaal (uciculus dC!myeJination ia M : bilateral internuclear ophthalmop legia
9. Par.thyrold derlya tlon: tJl ird Bnd fourth pharyngeal pouches
10. Aortic arch derlvl&UYea: review in embryology book or First Aid for Boards
11 . Wbat ru,OJ along the radial artery: median nerve
12. ,r1ery . ffected In remoral neck fracture: medial femoral circumflex artery- damage leads to
aseptic necrosis of the femoral head
13. EM of <&I: wh..., does sperm penetrate (zona pollucidal
14. \'Ybere is metll pbuc. n completed: in uterus
IS. Types of eoUacen : I (bone, tendon, skin; greata:t tensile stre.ngth~ m (initial collagen of wound
repair, replaced by type 1 [collagenase wilb Zn as a cofac"'rJ~ IV (buemenl mcmbmncl. X
(epiphyseal plate; picnue of bone on the exam and had to label wI1eto X was located)
16. Wallerila dtgeneration: Schwann cells begin to proUrmu:e and form a tube- that will JeI"W" to
guide axon tprouts in the regeoention process; regeneration of the nerve oceurs by the outgrO,.,.,th
or multiple axon sprouts from the proximal survivina sesmmt or the axon; sprouts are directed
distally (arowth "Ie of 1-3 mmlday) down the rube established by the proliferaling Schwann cells;
sprouts are remyelinated and reestablish continuity with the motor end plate oflhe muscle.
17. ClrelcofWU1lJ dlalram: nomcarteries
18. CT liver: show where the hepatic vein drains into the inferior vena cava
19. Nen'e inj ured in mldsbaf. bumtrus fracture: radial nerve
20. Know the layers or tbe gutric mucosa
21 . Ollgodendrocytes: myelinate In the e NS, while Schwrmn cells myellnatc In the PN
22. uage or elgs pos t~partum : meios is 1arrested in prophose
23. Eye closed cannot OpeD and eye deviated down aad out: oculomotor nerve palsy; eye down aad
In: trochlear nerve palsy
24. Child with. popslol. nick 1. blJ mouth [allJ dow. caUllog the pop.lolc stick 10 hit tb. bad<
or bis tbroal and develops ptosis aad meiosis of the ngbt e)'e: injury to the cervical sympathetic
pnglion
25. V.rdca1 diplopia lJ IWO<laled with: cranial n.....e IV palsy
26. Patleal wltb headacbe and pby icaJ findings of mydria.s In the rigbt eye io associatioD wftb
mild lid la ,and deviatioD or tbe eye d"",Ta and Ol&t: an aneurysm compressing cranial nerve m
(headache is \he giveaway for aneurysm)
27. Padfnl with I recUt history of bacterial muincids bu horizoDtAl diplopia in the I n c.yc..,
wbich is worst 00 pz.e 10 tbe left: cranial nerve VI palJy (lateral reclUS weakness from VI nerve
palsy)

264
28. Pltlent wftb bllat'tral lateral reCIUJ musde weakness: increase In intracranial pressure (classic
sign; papilledema usually present)
29. P.lraJy I.J of upward &U In an infant: hydrocephalus secondary to stenosis of the aqueduct of
Sylvius (thiJ iJ CAlled Parin.ud' s syndrome)
30. 1ultlpl ~ul.r motor n ne disord n: diabetes mellirus (conunon aanial nmoe palsies from
osmotic cIomIp 10 nerv )
31 . ' u n of tbe quadric pi mu.sde ud an abRnt Imet jerk reO x: herniated Lr t... disk
32. Paio la tbe blp and taleral qudriceps, numbness of the antuolate.raJ I~ and tbe . 'ebbed
'Pace M:cwH.a tbe gnl. toe, WHUeu of dorsiflexioa of tbe (OOl, aad aormal knee aad aakle
jnk ren.. : herni.ted L.-L,diJk(no.. the normal OTRs)
3] , umba J alon& LIIe late raJ ud posterior aspec-t of tbe calf aad plaatar aspect of tbe root,
aad an abKat lak! Jerk reno: herniated LrS, disk
34. Youae chUd falls on h.1s outst'retcbed arm and bas pain in the middle aDd lateral portioa or
hb davlcl ; upper utremlty tfmainJ in abduction I exteasion, and Inlernal rotation: nerve
injured is C,-C, (Erb-Ouchenne syndrome. or superior brachial plexus Injury due to II clavicular
fl'8cture; these Drc nJso the most common fracwres in newborns: C.- TI arc inferior brachial plexus
injures or Klurnpkc'. syndrome)
35. Palient has paralysIs of the oculomotor Derve arter a bud Injury: unc41 herniation with
compression of the Wrd nerve (ptosis of eye, mydriasis)
36. Nu.mba s of tbe tbettar aspect of the bud: median nerve (carpal tUMel)
37. Wrist bone wit.b Jffattstladdeau of aseptic aecrosls: naviculzu bone (1CIpbold)
31. ttpracoadyll r frac:tttrt: injury to brachiaJ artery .nd median nerve; danger of ischemic
contracture! in forearm mUleI" (Volkmann's ischemic CODtncture)
39. Know tbe fttll circ.ulado.a: duc:tus venosus and umbilical vein have !he hipest oxygen content
40. EM of.l ad with maCTOphage, type 0 p ..eDm~a {lamenar bodlq ( "rrattaneD
41. EM of man bowel wltb mlc.ro\;JIj on the surface
42. Rbtolock ItC-tJon of seml.alfefOus tubu.le: identify cclilhat mak sex hormoae bindina globulin
(Scnoll cell); Senoli cell abo synthesizes inhibin (n.gativ. reedback with F H)
43. Kao"" tbe b.adJ la b-Ietal muscle: A band has myosin ATPase
44. Raplratory bronchiole: last airway structure with cilia
45. Termlaal broacbJol : cUia but n6 goblet cells; site of obstruction in asthma, Cf. chronic
bronchitis
46. EM: know nonna l orpnellcJ in a cell-see picture: in First AID for Boards
47. Know normal . tructure CT exams of the cbest, abdomen,lee, arm, CNS
48. Voice hoarseness poSt thyroid l urgery: injury to laryngea l nerve
49, MRl of orbit: nnd superior oblique muscle
so. MRI of abdonlt n: Identiry splenic artery above the pancreas
51. Chest x· ray: Ouid In the coSlOphren ic sulcus in eHF
52. Barium ' Iudy or .mall IDtettlae: need to know bow to separate from la.rge bowel; superior
mesenteric artery supplies small bo\\'el
53. X· ray . bowIJl& e.a.Jarcunut of the postuior heart: enlarged left atrium in I pltient with mitral
stenosiJ: IIlOS1 pomriorly Ioeo.ed clwnber in lb. heen; see hest with tranJeSOpboseoJ ullrlSoUnd
54. "",,,".01: emphuiJ on blood supply and lesions of the bralnstem ond eonex, .11 of the
erani.1 nerves and their functions
55. DU or the carotfd. with occlusion of tile anterior ~ert.bral artt-ry: 'would rfTea the
c:on1r1llo.eral Ie.
56. h......... In juc"l.r r.... m... : weakness of pllale/lOSS gal renexllasyngul parolysis (X),
IJ1IpeziusimmocleidonwlOid (XI), loss ..... sensation posterior third of IOn",' (IX)
57. Weber syndrome: lesioo of oculomotor ne:rve and UMN signs with. mjdline, midbrain lesion

265
58. .Borner's syndrome : diagram of vertebra and sympathetic IlUnk; pick cervical sympathetic
ganglion
59. Pupillighl re.Oexes with eye diagrams of pupils: ODe set is an oculomotor nerve lesion
60. Gro or brainstem a nterior view-find area of oculomot'or nen 'e
61 . FroDtal lobe lesioo: affects personality
62. Bilemponll bemiaDopla: lesion at the optic chiasm; commonly a cran iopharyngioma (derived
from Rathke'. pouch) :
63. lnrerior q uadranta nopia: defect in the superior fibers in the parietal lobe
64. Loss of unsatloD in the bands, his lory of burns without knowing it: syriogomyelill in cervical
cord and lavolvemenl of ero ed ' pi.aothaiamks: syringomyelia
65. CllraJisection of fasciculus grac.ilis: effectS vibration and fine touch of lower extremity only
66. C m aJld UMN lie115 on op posite s ide: midline midbrain lesion
67. Lo of pain a nd temperature aDd UMN sigas ODOp'pOJlle ,'de: mid pons lesion
68. Horne.r ', syndrome wi.lb locaJiution of lesion on • djagram: latcral medullary syndrome with
associoted cranial nerve palsy in the medulla and hypothalamus with associated temperature
regulAtion problems
69. Pllrklnson's disease: MPTP drug of abuse association
70. C rosJ-seetion of 8.D embryo: identify neural crest tissue
7 1. Blood production .prior 10 birth: liver, bone marrow
72. Boe.bdale.k he.rnia tn posterolateral part of diaph ragm on left: present early in life; visceral
contents extend into the chest cavity causing severe respiratory distre$$ at birth: parasternal
diaphragmatic hernias exte.nd lhrough the foramen of Morgagni beneath the sternum and do not
usually develop symptoms until later in life.
73. Artery assoclated ",'lth forteut: celiac; midgut: superior mesenteric; hindgut: inferior mesenteric
74. Damage 10 bearing in • rock and roll bud player: injury to cochlea
75. Know Dormal bbtoloelc a ppearance of small bowel a nd stomach
76. Argyll-RObertso n pupil: accommodates when patient follows nnger moving towards the nose but
does nOl react to direct light; neurosyphilis
77. t.1RJ of linr: bad to identify the bepatic vein- vessel emptying into the inferior vena cava
78. Pathoguesls ofbypospadias: faUlty closure of urethral folds
79. PatbogtDHls of epispad ias: defect in genital tubercle, associated with exstrophy of bladder
80. Feces draining from umbilicus lD 4 day old: persistent umbilical (vitelline) sinus
81. Uriae dralalag from umbilic.us in 4 day old: persistenl uracbaJ sinus
82. Femoral atC-k fra t ture: bleeds into the capsule, may compromise medial femoral circumflex
artery leading to avascu lar necrosis, posterior dislocations are most dangerous

266
'Mkrobloloey Qu tlou :

I. Mtchaabm of .dioD of SrrtplOCOCCMS aga/llcllat (aroup B Inptococc.u,): inhibits


pha8OC)llOl1s
2. _use of aD ImmuDodt.Od aey with. high Ig1\1: defect in iJoIype twitching
3. Cydcwporlae necessary In Wt.atic.al rwiD tnlDsplant-?why: there is still some dirrermCt: in
MHC liltS (rom crossover of chromosomes duridg meiosis
4. 1ajor driff with ian ut.D2A : major changes in the reauonment of genome pieces indicating a need
for I new vaccine: only protects .galMt innuenza A; ega based vaccine; killed virus vaccine
S. Receptor (or HIV: CD. molecule on helper T cells (also macropbagts. dendritic cells)
6. T.,..., birth: DiGeorae syndrom., a pure T «II d.ficlency due to absence of tho thinl and
r..nII pIwyn uI pouches (3 n1 lnferior parathyroids + thymU$; 4th ,uperior patathyroids); absen,
thymic shadow
7. Oran ve rS&l) bost ruction : common in T cell deficiencies: must irradiate the donor blood to
destroy lymphocyte. (also de" roy, CMV)
8. C D common to boih Band T cells: CD..s is present in all leukocytes
9. Complement nxatlon rcoellon!: read Jawitz section In Med ical Microbiology on Immunologic
tcsts
10. Hanta ,'lruI: carried in rodcnts (deer mice); ARDS, hemorrhage. renal f'ailure; vira.l RNA in lung
tiuue (PCR lest)
11. laph)'lococcus GlUt-us: protein A attaches to Fe receptor or macrophage!, bence blocking
opsonl ..tioa ofbacteria
12. PrDtl!US ",lrabJlis: moves with nageil., urease producer
ll. C. diphllr.rI.: toXio iohibits <Iongation _ 2 by ADP'ribosylatlon, hea« blocking pro<ein
synthesis
14. MYaJpllUmu plfl!umDlfia~: requi~s sterols
15. Paeuma.ltl; (larval traumigratioa) + oya ia stools: Ascaris or hookwonn; nol Strongyloides
(rbabdltiform larvae in the .toolllOl eggs)
16. La tH' aulutlaat10e rtactioas: antibody to capsular au1icens Is an.lc.hed to the beads
11. Loc:.tlo.. or ..lis I. Iym pb .od.: B (follicles), T (pa.-noxJ, hlltloeyrcs (sin....)
18. Cells th•• attack protozoaas: CD4 T cells
19. Wbat Ch' baduia their sbape: peptidoglycan layer in the cell waH
20. T umbllJlI motillty, Cram po hivt. rod : Listeria nronocy'og,nls: invades mononuclear cells.
beta·hemolysls in blood agar. transplacental infection In felUS or occurs in renal tmnsplant patients...
mainly contracted from eallng unpasteurized cheese (goal'S milk cheese)
21. Gray membrane chal bleeds when removed: Diphtheria
22. Denl ue: transmitted by mosquito (Aedes aegypti; same mosquito as in yellow fever); "breakbone
fever"; may be hemorrhagic
23. In(Ktioa UJoelaltd wltb prtmature ruptu_1'e of Ihe membrane: group 8 StJ'CplOCOCCUS (5:
agoloeJloa); CAMP 'e"
24. JL.I f.actloD OD 8 Iyatpbocyt : activates B cells
n . Locadoa ror Slap"J'ltJalCClIS tJ.llreus carriers: anterior nares
26. Rh'" bU.d. : Oncbocen:a, bite ofblacldly, do skin biopSY, l _ t in
27. Rhlaovlnu : common cold occurs men. often in fall and winter; person to person droplel infection
and contamination of hands; acid·labile (does not uuse gast.roenteritis because oflhis)
2a. Vinas mpon Ible (or a cold la spnDI ud summer: adenovirus
29. Lac.obacUlus ia valla a: responsible for the acid pH
30. I.n .......etl • • : killed; <iII'based
31 . PruritiC' kia I loa I. Cul rWar nten.o: cutaneous leishmaniasis due to bite or sandfly

267
32. Bi b vaccine: antibody against capsular polysaccharide
33. (n addition to the norm al ~hlldboo<l immunization , wbat additional immunizatio os are
reco mme.nd ed 10 siC-kle celJ disease and cynle Obro is: Pneumococcus and influenza
(pneumovax is given after 2 years of age)
34. Wbich Uve va.ccine un be given to a patient witb AID : MMR (MMR is givcn only because the
natural Infection for measles is worse than the one that potentially could happen with the
attenuated virus) .:
35. List the live vaccines: MMR., varicella. OPV. BCG. smallpox, and yellow fever
36. List Ihe polys8ccllaride vaccines: Pneumococcus and Hlb (meningococcal vatcine is another
example)
31. List the killed virus va,ccioes: influenza. rabies, SALK vaccine
38. List th e im.munb:adons (bat are eontraindicated In patJenu with anapbylactic reactions
IgalD t eggs: MMR, influenza, yellow fever
39. Un immunWations thal are contraindicated in paUeolS with ana pbylactic reactiOD! against
neomycin: M.MR, varicella (neomycin is used as a preservative)
40. Verrueold lesion in lower extremity in a patient returning from outh Amuica: South
American blastomycosis, yeast with a ships wheel appeatanccj North American b.lastomyeos is has
wide based buds
41. AID paUentwlcb 2 peaks in tb e naturaJ bJstory of his diJease: p24 antigen
42. ELISA ' esl for HIV: anti-gp 120 IUltibodies, confinn with western blot
43. Best test ror detecting mv viral burd en ID blood: 1!lV RNA by PeR
44. ewhofD baby ID mv positive mo,ber. oewhom has o.tl·SP 120 in the serum (IgG antibody);
preve.nt fUV in newborn by giving mother AZT.
4S. Animal usooa'tio'n witb toxoplasmoaiJ: cat
46. Most common c:au e of diarrhea La children: rotavirus
47. 'E. coil: .~hes to che urogcniw epi.theliu~ bmc:c its til SUlUlS for urinaly traCt infections
48. Bruloa's agammllglobullaemia: SXR; defect in pre-B to B teUS; prone to respinllory infectio,ns;
.eed lV gommo globulin
48. cm: first immunodeficiency treated with gene therapy (replacement ofadenosine deaminase)
SO. Wl.Jkon Ald.neb: SXR; mad of sinopulmonary infections, ect.e:ma. thrombocytopenia; B and T
celt deficiency; increased imHdence of leukemia/lymphoma
5 I. Part of a vaccine that is antigeniC: polysaccharide capsu.le
52. Penoa worki ng with animal bid es develops lung diJease : Bacillus anrhrax (woolsorter's
disease)
53. Cryptococcus: evades host destrUction via its polysaccharide capsule
54. Child with anemia and diarrbea-? parasite: hookwonn produces iron deficiency
55. Lym phocutaneous nodules in a rose gardener: sporotrichosis; treat with pol8ss lum iodide
56. Elderly male smoker with non-productive cough, bacteriA falls to grow OD ordinary media.
(mus. be supplemented with Iron and cysteine), Deed Dieterle sUver stain t'o Ideotify:
LegionellB; treat with erythromycin
57. Matropboge attivadoD: r-in,erferon seemed from helper T coli
58. Know examples or all the bypene ..itivlty reac'lo ..: see immunopathology chapter
59. TcA deficiency: sinopulmonarydisease; most common genetic Immunoddicieoey
60. AID : most common acquired immunodeficiency
61: Bl.A sy tem coded on chromosome 6
62. Hyperacute rejectioD or a tran.splant: ABO incompadbility or patient had anti ~ HLA antibodies
against an HLA antigen in the graft, type n bypersensitivlty reOlCtion
63. HLA·~ B, C code for class I andgeas: CD, cytotoxic T cells recognize
64. HLA-J> loci code (or clau n antigens: CD.. helper T celts, maerophages recognize

268
65 . G ran VI bosl ruction : NK cell mediated; common in bone marrow and liver trWlsplants; rash.
jaundice (necrosis of bi le ducts). diarrhea, danger in T cell deficient patients
66. R~eptor for EBV: COl i on B ce lls: polytlonal stimulator causing incrtased synthesis of
immunoglobulins:- reason for hypergammaglobulinemia in AIDS; increased di\'isions increase risk
fort;8: 14 and Burkin's lymphoma
67. CD rype for bistiocyta: CO l: positive in patients ""i lh hislioc)1osis X (Hand-Schulkr..christian:
Lenerer~Siwe . eosinophilic granuloma) ~
68. Delt roy C. difficl/~ In bedpaa: au toc lave
69. Ranta vtrw : carried in rodents (decr mice); ARDS. hemorrhage. renal failure; viral RNA in lung
tiJsuc (PeR test)
70. 8 1astomycosls: spores usociated with beaver dams and inland ~.ter ways; broad.bascd-bud;
shipwhcelappeanog yeast is South American blastomycosis
71 . Impedeo: most commonly caused by group A .streptococcus
72. C DJ : antigen recognition site for T cells
73. Antigen Ibat bindl to CD. on T celb In B1V: gp 120
74 . Ril k fo r HIV If accidental needle nick (rom an RIV posllin padent: 11330; R:< with trip le
thera py fo r 6 months n.nd gel ELISA test al repeated intervn.ls
75. Fever In mAlaria : coincides with rupture of RBCs
76. Malaria: l.real with primaqu ine nod de ... elop acute intra ...ascu lar hemO lysis In 06PD deficiency

269
Biocbemlstry Qutliloas:

1. Rate Umhiag mcdoa in cbol terol syatbesis: HMG Co reductase


2. FiadiDgs In PKU: mousy odor; tyrosine missing, bence it must be supplied in the diet.. can
diagnose by amniocentesIs and finding the abnormal gene; eliminate phenylalanine from diet
(NutrUwe.. is upII1ale and phenylalanine. so cannot ... Il)
3. I ..II distast: inability to phosphorYlate the mannooe re ldues of potendel I)"sosomal enzymes
loc:ated in 0018i apparalUS, benee they unDo! be taken up by th. Iysosom to degrode eomplex
substrates
4. or
umber &Iucosa DUeuary 10 build palmitic acid a 16 carboD compouad: 4 glucoses. each
&lucOK run produces 2 .cetyl CoA. the Janer containing '2 carbons each
S. I.a alin tac.k In DKA: d ctt':tsed glycolysis. glycogenesis, ratty Kid synthesis, sconge of fat in
adipose
6. Dcouplllli ageots (e.g.. alcohol, saJiC)'lales): produces brown rat from Increased heat from
re.ac:tiOflJ; trying to Increase the generation of more prolOns ('0 make ATP
7. Vo n Gierke'.: decreased glucose 6~p hosphatase (gluconeogenlc enzyme) with decrease In glucose
(fasting hypoglycemia) Dnd increase in glucose 6-phosphate with production of nonnal glycogen in
the liver and kid neY1i stimu lation tests with glucagon, fruccose. etc. cannot increase the glucose
levels ow Ina to the missing enzyme
8. Biochemical prOctS5e!l In both cytosol aDd mlloc.boDdria : urea cycle, heme synthesis,
gluconeoge.nesis
9. Female wllb pbeoc.hromocytoma: wh81 diewy aheration: probably d rease phenylalanine
(essential AA) and cyrosine (not an essential AA) in the diet
10. Preg...1 remlle wllh P-'CU: what diet low in phenylalanine and high in iyTOSine; 8\'oid
utrlJ""'ttl since it contains upartate and phenylalanine
11. Lesch , .... : SXR with absent HGPRT, self~mublalion, hyptrurieemia, mental retardatioo
12.. C luc:okJaue: only In liver, high Vm and high Km. not inhibittd by glucose 6.phosphate;
hexokinase: in all tissues; inhibited by glucose 6-pbospbare; low Vm and low Km
I1. B....c..td (balo Imillo acldJ ud maple synp uriae diKue: only mutcle can metabolize
branched chain amino acids
14. GlycOl Dolysls: review biochemistry
15. Koy <azymela , Iacoo"",oa Is: fructose 1,6 bispbosph..... (catalyzes the oon .... ion offrue_
1,6-bisphospil.oteto fruClOSc 6-phosphate)
16. Loc:.tlollJ of &luCOM 6-pbo.pbat.ase (glucoatOgealc hormone): liver, kidney, Intestinml
epithe lium (lesser extent thlln otherS); absent in von Gierke's disease
17. Sbuttlu and what they carry l camitine (even chained fatty acids), malate (NAOH)
18. Funcllon!l or LDL: vilam in 0 synthesis, oiher steroid synthesis, cell membranes. synthesis of bile
sal t51acids
19. Acetyl eoA usa: how many times used in FA synthesis, CH synthesis. ketone body synthesis; not
a substrate for gluconeogenesis
20. Km aad Vmu: Llacwelver Burke:. competitive vs non-QQmpelilivc inhibilO~ competitive (e,g..
alcohol dehydrogenue binding ethanol, methanol. ethylene glycol at the lame binding site) has no
change in Vm but an increuc: in K.m (lower affmity for ethanol, increasing ethanol reverses the
inh ibitioo), non~pedti"" Inhibitor (e.g.. organophosphates and effect on I<Otylchollnest<rase,
does nOl bind to the same sice as acetylcholine, binds to another site on the cnl)11le that decreases
ve locity oftbe reaction) has I decrease in the Vm. but the Km remains the same, since the substrlllt
still binds to Ibe nonnal site
21. QuadoD OD rany add lealth aad e.aergy production
22. Urea cycle: method or eliminating ammonia. located in the hepatocyte

270
23 . Epinephrine given and only s mall branc bed chaios (ouod : debrancher deficiency
24, Know c bolesterol syntbesis: review Harvey/Champe, note that first few steps are similar to ketone
body synthes is except HMO eoA lyase is used instead of FlMG CoA reductase
25. Origin or a polipoproteins 100 (liver) an d 48 (i.ntesti ne)
26. Rale limiting Jtep in glycoge nolysis: glycogen pbospborylase
21. Reason why liver can aot use ketones ror ruel: liver cannot activate acetoacetate in the
mitochondria, which requires 5uccinyl CoA.: a~etoacetate CoA transferase (8 thiouansferase
enzyme) in order to convert AcAc into acetoacetyl CoA..
28. McArd lea dtseasr: absent muscle phosphorylase; increased glycogen in muscle; no in cre~ in
lactic acid after exercise
29. Preg·nant woman is a beer drinker-wbat s upplements does she need : still needs folate because
alcohol increases Joss of folate in urine and stool whioh offsets the amouot of folale present in
beer; funhennore, she would probably be taken off beer because of fetal alcohol syndrome and
would need folate; iron is not affected by beer drinking
30. lnsulin : key honnoDe of the fed stat·e ; glucagon: key hormone of me fas ling state
31, Mannose 6-pbosphate: inyolved in trans fer of dolichol (lipid) in the RER in [he synthes is of O·
linked glycosides
32. Know all tbe lysosomal s torage diseases: see Genetics notes; twO SXR diseases ore Fabry' s and
.Hunter's syndrome
33. Know aU tbe reactions lavolYing NADINADB and NADPINAD"PH: see Nutrition notes
34. l\1ajor source of NADPH: HM.P shunt; malate dehydrogenase reactlon (0 a lesser extent; NADPH
supplies redUCing equiYalentS
35. MUlation cbanges an a mino acid seq uenc.~wbic:.b one would have the greates1 effect on
migration 10 a serum protein e.lectropboruis: one with the most negative charges (most aCidic)-
glutamine; One that would ~main closest to the anode (. pole) is the most basic amino acid-
arginine
36. Mechaobm or ketoacidosis in D.KA: increased fl.oxidation of fauy acids and produ~ion of acetyl
CoA, which is used by the liver to synthesize ketone bodies
31, Promote.r location: lincar gene drawing with labels: pick upstream location
38. Energy source for protei n synthesis: GTP
39. boenzyme wltb 1 Ce. .., 4 l ubun lts;.LDH isoenzymes; 5 isotypes: LLLL, LLLH. LLHH, LHHB,
HHHH
40. Seco nd mweoGen: atrial natriuretic peptide: cGMP, insulin: tyrosine kinase. nicotinic: ion
channels; sec KlIlZUnWTreyor phannncology for more second messenger relationships
4J . Best method or detecting relatedness of a ne.w bacteria : restriction fra.gm ent length
polymorphism
42. Enzyme u ed for peR: DNA polymerase
43. Source of glucose in I brain in stantatioD: alanine (transaminated into pyruvate)
44. Glutamine: most abundant amino acid, carries ammonia in nontoxic fonn
45. Tyrosine: thyroid honnone, synthesis of melanin, synthesis of neurotransmitters (dopamine, Nor.
Epi), decreased in P.KU (must supply tyrosine in diet)
46. Glycine: inhibitory neurotransmitter blocked by telMUS toxin, synthesis of 6 aminolevulinic acid
in beme synthesis, synthesis of bile acids/salts
47. Biocbemistry of be patlc cncepbaJopatby: increase in aromatic amino acids- phenylalanine,
tyrosine, nyptophan (mnemonic- P'IT) leads to increased ~thesis of false neurotransmitters
(GADA, octopamine) in hepatic encephalopathy; branched chain amino acids inhibit synthesis of
false neurotransminers- reason why they are given for Rx

271
Ph........coIOCY Quatlo .. ,

l. Pbarmac.olocy &ueral! heavy mpbasis on antimicrobials and mechanisms or action and


ruiJtan<e 10 lIIe drua. clltdi<ww:ular drup, anhma dru", NSAIDI, endocrine drup, drup;
know !he cb.u ofdrua. mechanism ofoctlon, and signiftcalll.1de e/Tecu
'2. TrnllDe.at of bt.azodLuepam onrd fiumnC'nil
). nlcbldoo.ic add metabollsm: teYi~ inflammation chapter
4. CaUH r co uChJ1olioedema i.a padnl on aD ACE inhibitor: bradykinin
S. ohpl. pbrlae la . bode 1:1000 dilution subcutaneOUSly
6. D... lrolu.: ...... in !he ....unentof ntllignanl bypenhennla.nor haJoIhane
7. TrauD a. of IUUmiuophu ov rdose: acetylc:ysteine 10 replace UJCd up GSH (neutralizes
ace..minophen ftee rodicals formed in !he liver cycoehrome 1)'SIC1D)
I. M h.a .. of loop diuretic: blocks a·K·2CI cotnInspet1 pump in !he IIIlck ucendin, 11mb in
the rt.1'lI1 medulla; .110 blocks calcium reabsorption
9. CyaDIde polsonlD. treatmeat: review in environmental pathology chapter; amyl nitrite and
thiosulrate
10. Thrush: may be. complication ora con.ieostcroid inhaler
11 . b otretlnoln : alway, do a pregpanc)' test in femoles; put them on acp
12. Mecbaal.m of prop)'lthlourac.ll: blocks iodination or the. tyrosine residues of thyroglobulin; also
blocks coupllna of DIT and MIT; nnly drug iliac can be used In ,,",snancy bUI may prnduce goiter
In !he newborn one! noll defec..
13. P450 fYlt 00 I. lIIeliver: makes drugs wacer soluble
14. A.CI..........d reoal fallu~ ACE Inhibitor relationship
IS. t .. huls .. of ..do. of ...""olc .cld: behaves like I ...... Id in lllat it binds 10 .......,.. in !he
nueleut willi aub quen, IrInscription of genes; _ins produced by chi. acdon are imponanl in
srowlll. diff..... liotion. rq>rnduction, and embryonic developmenr, also ...... In .........1 of acnre
prosnnulocytlc: leukemia
16. liop.rtlol ..1I0. I. purill.sy.tIaesIs: blocks xanlbJne oxidue (ltbematic of purine mecabolism
..... provided and had 10 identify !he reaaioos (bypoxanllline 10 _lIIine and aanlhine 10 uric
acid]~ m<rCIpcopurino degndadon Is inhibiced u well
11. Moo co ....o. . .llblolle ..... to prev..1 e.doeardllis I. palle. willi val..1ar dl~·.....o:
arnoxictllin Is lIIe drug of choice; aU valvular di........cepl uymplOntltie MVP and all
consenllli h..... disease excepl uympLOmatic ASD)
18. eUow coloradon of tbe skla Iba'i c.an be ml' bbn ror Jaundlce-'1 drug: quinacrine;
chlorpronu,zine and arsenic produce a blue-gray color to the skin
19. Diffuse erytb ema followed by uparadoD of the . kln (.el lded 'ki n syndrom e or tolie
epidermal necrolysl.)-?dru&: barbiturates, sulfonamldes, phenytoIn , NSAIDs
20. n llr lOll 10 I womaa-1drug: oral contraceptives (predictable . Ide effect; e trOgen causes hair to
be II same ""e of developm.... may also occur Wr dellvecy)
21. El")'tbem••ou~ bypnplcmea1ed p1aqH:--like lesioa. tbat rKurs It the samt Jile eYtry time.:
fIXed dru& eruptlon (phenolphlllalein. NSAIDI, Ie....cydin., 8ac%rIm. and batbhumes are !he most
CODUDOQ cause ofthcsc: reactioM)
22. Group of dAp !u.s tbt bieba-. usocia:lioa with urticarial aod maulopapuJ.r leslo :
arnoxicfllin, TMP MX, ampicillin/penicillin (rash.. are !he MOST COMMON adverse reaction
to drup, willi maculopapular rub.. leading lIIe lis~ mOSI druB 1Uct10lll involving akin are OT
type 1Icypenensitivicy biscamine-relaled)
23. Eld rt WOOl ..... Ikiazid Is ....I . 1 risk for d."dopl.C; pI1
24. TanlI.. d 1d ...1o, lDaUc... 1 I)'lldrome (....ea""l. b ~p<rpyruJa, ..lo ....1c I..... illey):
newoleptlca

272
25. Aadpsycbotlc drug requiring vis ual examination: thioridazine (also produces hean conduction
derects)
26. epbroacnic diabetes insipidus-? drug: lithium for bipolar disturbances
21. Drue contraindicated witbMAO inhibitors: epinephrine
21. ae o( phentolamine: non~selectivc a·blocker that lowers blood pressure during surgery for a
pheochromocytoma
29. Meehani m o( AZT: inhibits reverse transcrlp ase; produces a macrocytic anemia unrelated to
rol.,eIBI2
30. TrealmeD1 ofPb poisoning: BALand EOTA
31. Drugs Involved In (ola te metabolism: phenytoin blocks intestinal conjugasc (polyglut.amarc 10
monoglutamate. BCP blocks uptake of monoglutamate, methotrexateITMP·SMX block
dibydrofolate reductase
32. Cromolyo sodium: stabilizes mast cell membrane preventing re lease of preformed mediators and
release of prostaglandinslleukotrienes after the release reaction
33. MethOlrexate: blocks dihydrofolate reductase and the cOllversion of dihydrofolate to
letrahydrofolBtc
34. Glucurouyl transferase In liver renders compounds water soluble
35. Cyclooxygenase: aspirin Inhibits irreversible, NSAIDs reversible
36. Thrombo:cane Al! synthesized in platelet; vasoconstrictor and increases plate lei aggregation. also
a bronchoconstrictor
37. tcr~et o( protOD blackers: blocks W~K·-ATPase prolon pump In parietal cell: nOl a receptor
mediated event; 8 1 blacken: blocks Hz receptor, which nonnaHy activates adenylate cyclase
producing cAMP which stimulates protein kinase; uetylchoUne: activates cholinergic receptor
Clusing the release of calcium, which stimulates proteiD kinase; misoprostol: blocks the
prostaglandin receptor, which nonnaBy inhibits adenylAte cyc lase and cAMP production
38. Treatmen t of schistosomiasis and clonorchiasis: praziquantel
39. 1-fold membrane spanning proteln-? drug: propranolol a ~b l ocke r
40. Pb ... 3 eilDltai trials: doubl. blind
41 . Ticlopidint: substitute for aspirin in preventing strokes, CAD if the patient is allergic 10 aspirin;
causes neutropenia
42. Drug induced LE: procainamide and hydraJazine
43. Overdose of succinylcholine: use acetylcholine blockers
44. Fl nasterfde: blocks S--a reductase. which converts testOsterone inlo dihydrotestosteronc; hence
testosterone would incrense proximal to the block and dihydrotestosterone would decrease;
inereases hair growth
45. Flu'amlde, cyproterone, spironolactone: block androgen reoeptor, hence testosterone!
dihydroteSlosteronc increase but have not physio,logic effect
46. 'KetMoDllZole: inhibits testosterone synthesis (suppresses adrenal steroid synthesis)
41. LeuproUde: GnRH analogue, which when given in sustained fas hion, inhibits FSH and LH. hence
lowering testosterone and estrogen levels
48. ACE Inbl bitors: increase in renin and An.. but a decrease in ATII and aldosterone; aldosterone
eventuAlly increases, hence the addition of spironolactone to keep aJdosterone suppressedG
increases longevity in CHF
49. An eDic poi oDing: dimercaprol
SO. Cbloroqulne In treatment of malaria-malaria recurred·why?: exoerythrocytic/heplltic stage
(e.g., P. \livca, l'. ova/e); drug kills active disease but does not eradicate hepatic stage
5I. 'Prfmaqulae in treatment of malaria: not good in the active stage but does IOU the hepatic stage
of P. vlvox and ova/e

273
52. Daatrolene: reduces the relellSc of calcium from the san::oplasmic reticulum of skeletal musclej
Ilf\tispumodic drug; a~ used in treating malignant hyperthennla
53. MelbaaGI: Increased anion gap metabolic acidosis due to conversion of methanol into formic acid;
optic nerve degeneration and blindness; treat with alcohol infusion to bloc:k metabolism of
methanol by alcohol dehydrogen15<
54. Botulis m toxla: blocks the release of acetylcholiM (diagram of neurotransmlner synthesis and
must locate the block); good diagram i~ KatzungfTrevor phannacololY book: used to treat LES
spasm in Dchalasla
55. RJbavirio: used in severe RSV infections in children
56. Asthma: albuterol (J.\rseJecuve agonist; broncbodilator); albuterol may taUK hypokalemia- drives-
K· In", ceU.
S1. c.e1ylcboUat brukdo a: OCCUJ'$ in the synapse into choline and acewe by acetylc.boliocsteraK
in the cleft; producu ~ rec:y<:led and not =mod
58. Kaow tb crapb of ORlEPJJisoprotereaol aad tlTecu 00 blood p ressure aad btart rate, plus
tbe e.rrec1 of o,pbloc.ke rs: see graph in Katzung!Trevor book
59. Amphotericin: disrupts ce ll membrane pe:nneabiJ ity; binds to ergosterol in the membrane
60. Kelocoaazole: inhibits the mClIlbolism of nonscdating antihistamines like eldanc leading to
cardiac arrhythm ias
61 . Be very ramilla r witb second messengers: see Harvcy/Champe bloohem book page 80
62. Oploid toxicity: n410xone (opioid receptor antagonist)
63. OpioidJ: no tolerance to miosis and constipation
64. Codetafl metabolized Inlo morphine in small amounts owing to .ignlficant first pass metAbolism
of morphine in the liver
65. 0 Uria,m tre.mea : use benzodiazepines
66. Lovutadn: inhibbs HMO CoA reduaase
61. Pale.., ductus an t riosUJ: keep open with PGE
68. PuklUln: O-Iactam antibiotic that inhibits cell wall synthesis; bind to specific receptors in
cytoplasmic membran~ Inhibit transpeptidase enzymes that crou-link linear peptidoglycan chains
that fom part of lb. cell wall; activate autolytic enzymes In the cell wall
69. Cepbalolpori .. + amlaOjly<.,ld..: synergistic effect of enhancing oephrotoxi<ity
70. £ rytb .....yd.: Intera<: .. w.ith the SOS subunit of bocterial ribosomes luding to Inhibitioo of
protein syntb is; inhibits the formation of the initiation complex and interferes with tnnslocarloo
reactions; rellJtan« develops secondary to plasmid·medisted fonnalion of enzymes that methylate
me receptor thaI erythromycin binds with; colifonns produce a tntn missible plasmid that produces
an esternsc thnl hydrolyus the lactone ring of erythromycin
11. Cblorampbeolcoh gray baby syndrome (dose related aplastic anemi a)
72. Sulfonamide., dap. onf: may precipitate hemolysis in G6PD dc.f101oncy
13. TMP: Inhibits dlhydrofolate reductase; plasmjd.mediated resistance; TMP + SMX has a synergetic
effect from the seque.ntial blockade of folatc synthesis, sinu MX blocks dihydropr:erate
synthase. which il an enzyme that CQDvens paraam inobenzoic aeid into djbydrofotic acid
74. Metroaiwole: produces a disulfiram-like reaction
75. Flucoaazote: treatment of esophageal candidiasis in AIDS
76. Gaaciclovir: first drua used in uuting CMV retinitis, the most common cause of blindness in
AIDS; foscamet is used if gancitlovir does Do t work
17. Cbollaomlm edc used ia trutiae opt-a au,le claacoma: pilocarpine, pby,ol tigmiAe
18. DobutamlDfI: inotropic (increase contractility) vasodjlator (decreases aft.erloed) that activates 01
and ~1 > f)2 activity without much ebroootropic effect:: used in the treatment of shock
(canliogcnio) woelated with hypctensioo, hypotension associated with mill failun: or CHF

274
79. Dopamine: stimulates cardiac r31 receptors. peripheral a·receptors, and dopamincrgiC: receptors in
vessels in the renal and splanchnic bed; at low. doses, it is primarily a vasodilator that increases
renal and splanchnic blood now; at high doses, it increases cardiac contracti lity (inotrQpic) and
cardiac outpul via its activation of cardiac ~ 1 receptors
80. 8eDzodiazepines: via its own receptors in the thaJamus, limbic structures, and cerebra conex
which arc part of the GABA receptor-chloride ion channel macromolecular complex,
benzodiuepines facilitate the inhibitory action Df GABA via increased conductance in the chJoride
ion channels (tlumazenil blocks this effect by blocking the receptor for benzodiazepines and is the
treaunenl of choice for benzodiazepam overdose)
81 . An elderly woman on thiazides is most at risk fo r developing
A. diabetes melliws
B. gaUl
C. IIrenal SlaDe
D. metabolic acidosis
E. hyperkaJemia
82. A child who ingests 30 adult' aspirins will most likely develop
A. a nonnal anion gap metllbolic acidosis
B. a mixed respiratory and metabolic acidosis
C. profound respiratory acidosis
D. an increased an ion gap metabolic acidosis
E. metabolic alkaJosis from vomiting
Answer: 0 (children, unlike adults, do not conunon ly develop a mixed metabolic acidosis and respiratory
alkalosis. Rlt is to puroon gastric lavage and add activated charcoal and to produce an alkaline urine for
increased excretion of the acid)
83. Open angle glaucoma is best treated with
A. a Ji-adrenergic blocking agent
B. topical corticosteroids
C. topical antihistamines
D. furosemide
t . intravenous acetazolamide •
Answer: A (timolol is a favored agent; pjlocarpine may also be used; most common type of glaucoma;
produces gradual loss of peri pheral vision (tunnel vision) and optic- atrophy)
84. Trazodooe: second generation anti~epressant; inhi.bition of serotonin reuptake; sedation; may
cause priapism
85. Thioridazine: retinitis pigmentosum may be a complication
86. Tamoxifen: anti..eslrogen; estrogen receptor partial agonist that blocks the binding of estrogen to
their receptors in ERA-positive breast cancer cells; also protects against osteoporosis and CAD;
risk faclor for endometrial cancer; can be used in treating progestin-resistant endometria l cancer
87. Ace[aminop hen: ana lgesic and antipyretic but not an anti· inflnmmatory agent; inhibits
prostaglandin synthesis in the CNS; very weak cyclooxygenase inh ibitor; most common drug
causing acute fulm inant hepatitis; converted into free radicaJs in the liver; glutathione inactivates
the FRs (acerylcystoinc ueaunent replaces GSH)
88. Digitalis toxldly: tr••t with d igoxin antibodies (FAB fragme nts)
89. Cephalosporias: first generation: drug of choice for surgical prophylaxis in many cases; second
genernlion: sinusitis (cefuroxime)~ mixed anaerobic infections; third generation: men ingitis~ GC
(cefiriaxone)
90. Cromolya sodium in asthma: only for prophylaxis; blocks early and lare asthmatic responses to
allergens

275
91 . Omt!'prazole: inhibits the proton pump located on the luminal membrane oflhe purietlll cel l; useful
in the t:reaunent of ZE syndrome, GERD, and H. pylo,.i inrections
92. C lozapiD~: blocks 0 4 (dopamine) and 5-HT:4 receptors moreso than Dl receptors
93. Fever ia I pa lien l on a loop diuretic: do not use any type of NSAlD (incl uding aspirin) since it
blocks renal synthesis of prostaglandin, which vasodiJlLtes the afferent arteriole. Use
acetaminophen to lower fever, since it does not interfere with prostaglMd in synthesis. Loop
diuretics lead [0 volume depletion. hence angiotensin n will be elevated (vDSOConstricts effere nt
arterio le). Loss of prostaglandin effect predisposes the pat ient to renal railure.
94. PatboleaesiJ of cough aad angioedema in ACE inblbltors: increase in bradykinin
95. Rrd maa Iyadrome: IV vancomycin
96. Drug for bacterial carrier slala (S. au,.ew. N. meningitidis. H. infiut ll:.oe): rifampin
97. Single dose drug for GC aad C bJamydia: azithromyc:in
98. Rx of maUgaaat hypertension : nitroprusside
99. Palienl 00 Lasix ... bo develops fever and needs utipyretic: use acetaminophen. do nOt use
NSA IDs since they block prostaglandin production in the kidneys
100. Antihypertensive and aotlarrhymic drug that lowers blood pressure and increases heart rate:
calc ium channel blocker, e.g., niredipine

276
fbYliology QutsUoal:

NOTE: a lot of graphs are used; know the classic cardiac cycle graph very well; review Yarrow diagrams
on fluid aherations in my fluids and hemodynamics chapter
I. M--.)or site Of water rubsorpllon in the Gl iratI; in descending order: jejunum. ileum, colon
2. Most effecti ve aepb_roa site for acid ext-retioa: proximal wbule Of the kidney
J. epbroa sit for ADD drec.r: eoJlecting tubul ..
4. epbrOD slle for g'r ealesl ae.neratioD of free water/most s usceptJble (0 ischemia: thick
ascending limb in the medulla
5. CompnssioD of the tborax UDder water: produces a resuicdve pauem on PFTs
6. Constriction of tbe elrert-ac arteriole.: increases the GFR and decreases renal plasma flow
7. Know larlln, tqualion for GFR mUmln: GFR - Kf [(Poc • Pas) • noc]. where Kf - hydraulic
conductance in mlImln . mm Hg or filtraJion coefficient in mVmin . mm Hg. Poe - hydrostatic
pressure in the glomerular capillary in mm Hg (pushes fl uid out or the capillary; e.g., +4S mm Hg.;
it is increased If the efferent arteriole is constricted and decreased if the afferent arteriole is
constricted). PBS - hydrostatic pressure in Bowman' s space in mm Hg (pusbes fl uid Into the
capillary; e.g" · 10 mm Hg), and 1'tQc = the oncotic pressure in the glomerular capillary In mm Hg
(brings flu id back into the capil lary; e,g., · 19 mm S a); the net fi ltration is the algebraic sum of the
above 3 paramcters (in the above example, the net pressure would be·l O+ 45 · 19 - + 16 mm Hg)-
fluid should move OUt or the glomerular capillaries; question: what larllng force changes to
produce a net uJtrafiltration or zero (answer. 1tcx. which becomes increased [pulls fluid back into
the glom rulu capillary; using the above example- 10 T 4S - ll - 0 net pressure]i It is the only
Cactor that does not remain constant along the length of the capiJIary owing to differences in
protein concmb'8tion; the high ltoc at the end oC the glomerular capillary extends into the
peritubular capillaries that sunoo.nds the nephrons and allows ror the reabsorption of solutes).
I. FUtradoD (radioa ia tbe &lomuuJus: FF = GFRJRPF; constrict afferent arteriole. no change in
FF (~ GPRlJ. RPF). ....triet efferenl arteriole increases FF (rt GFR/J. RPF). Increase plasma
protein concentration, hence increasing ltQc, decreases the FF (.1. GFRlno cha.nge RPF); decreasing
plasma protein concentration. hence decreasing Xoc, increases the FF (i GFRlno cbange RPF);
constricting the ureter, decreases PBS, hence decreasing GFR and the FF without affecting RPF (
GFR/no change RPF) -
9. egative cba",e of COM: heparan sulfate
10. 1 beart ouod correlata ,.,lIb C wave in tbe jugular veDOUS pulstl (C wave is dOJ'ure of the
tricuspid valve); know the jugular veno us pulses: sec box in cardiOVAscular Chapter on physical
diagnosis
11. Carotid massage: decreases heart rate and increases vasodilatalion (carotid sinus baroreceptor
Innervoled by the DC and Xth nervej impulses generated in th is receptor inhibit tonic discharge oC
the vasoconstrictor nerves and excite vagal innervation of the heart producing vasodilatation.
vcnodilation. drop in blood pressure and hcart-rate, and a d«reasc In cardiac output)
12. 3% Bypenoaic. salloe .ad dfect OD POsm aad ADD levels: 3% hypmonic sal ine incn:ascs
POsm. lncreased POsm stimulates the release of ADH (increues). Atrial natriuretic peptide is also
stimulated and Donnally does inhibit ADH release, however, hypertonicity overrides ANP.
13. PadeDt walking briskly oa • bot d.y: question was put in a cban (onn; no increase in body
temperarure. vasodi latl.uon o(vessels in skin
14. Mara.hoD ru.DD.e r OD • bot day: question was put in a chart Corm; increase in body te:mpera~,
vasodilalauon of vessels in skin

277

Vous aimerez peut-être aussi